Você está na página 1de 356

Caros alunos e professores

Desde a sua primeira edio em 2005 o Banco de Questes (BQ)


mostrou ser um material motivante para alunos e professores. O seu
objetivo divulgar nas escolas pblicas problemas de olimpadas nacionais e internacionais, por isso grande parte do BQ no de questes
originais. A nossa idia que o BQ seja um material informal, no modelo apostila, que entusiasme alunos e professores. No h qualquer
preocupao com uniformidade de contedos ou de nveis de diculdade dos problemas. A nossa principal meta : problemas interessantes que despertem o prazer de raciocinar.
A publicao do BQ completou 5 anos em 2009, perfazendo 678 problemas e suas solues, e 50 desaos. Esse material preparado nesses
anos serve agora para compor o BQ-2010, que apresentamos dividido
em 3 nveis. Os problemas foram agrupados em nveis apenas por uma
questo de organizao, encorajo todos os alunos a passearem pelos
3 nveis.

Banco
de
Questes

2005

2006

Voc pode ou no conseguir resolver os problemas, mas vale a pena


tentar... no desista na primeira vez, nem na segunda, nem na terceira...! Persistncia essencial e bom humor fundamental! assim
que se aprende Matemtica.

2007

Nesses 5 anos o BQ tem servido para timos bate-papos com alunos e


professores, que gentilmente me escrevem apontando erros de digitao
ou nas respostas. Espero continuar recebendo essas valiosas crticas e
sugestes.

2008

Um convite: se voc tem solues diferentes das apresentadas no BQ


envie para famf@impa.br que ns as publicaremos em nosso site.

2009

Agradeo a Eduardo Brietzke (UFRGS) e Claus Doering (UFRGS)


o trabalho dedicado e competente de reviso e, principalmente, de
enriquecimento de todo o material do BQ-2010.

2010

Suely Druck
Diretora Acadmica da OBMEP

OBMEP 2010

Texto j revisado pela nova ortograa.

ii

OBMEP 2010

Contedo
Nvel 1

Nvel 2

35

Nvel 3

69

Desaos

105

Solues do Nvel 1

114

Solues do Nvel 2

182

Solues do Nvel 3

259

Solues dos Desaos

347

Nvel 1

Nvel 1
1. Qual o nmero? Quando Joana entrou em sua
sala de aula, a professora estava apagando o quadro
negro, mas ela ainda pde ver algo escrito, conforme
mostra a gura. Qual o nmero que foi apagado?
(a) 8

(b) 9

(c) 11

(d) 12

(e) 13

2. Muro em 15 dias Um pedreiro capaz de assentar 8 metros de muro por dia.


Quantos metros de muro esse pedreiro consegue assentar em 15 dias?
(a) 104

(b) 110

(c) 120

(d) 128

(e) 112

3. Medindo pilhas de papel Numa papelaria, so armazenados pacotes de papel em


pilhas de 60 pacotes. Cada pacote tem 500 folhas de papel e cada folha de papel tem
uma espessura de 0,1 mm. Ignorando a espessura do papel utilizado para embrulhar os
pacotes, podemos armar que a altura de uma pilha de 60 pacotes aproximadamente
igual altura de:
(a) uma pessoa adulta;

(d) um prdio de 10 andares;

(b) um beb de um ano;

(e) uma sala de aula.

(c) uma mesa comum;


4. Quanto pesa? A balana da gura est em equilbrio com bolas e saquinhos de
areia em cada um de seus pratos. As bolas so todas iguais e os saquinhos tambm.
O peso de um saquinho de areia igual ao peso de quantas bolas?
(a) 1
(b) 2
(c) 3
(d) 5
(e) 6
5. Calcule a diferena Considere dois nmeros naturais, cada um deles com trs
algarismos diferentes. O maior s tem algarismos pares e o menor s tem algarismos
mpares. Se a diferena entre eles a maior possvel, qual essa diferena?
(a) 997

(b) 777

(c) 507

(d) 531

(e) 729

6. Qual o volume? Trs frascos, todos com capacidade


igual a um litro, contm quantidades diferentes de um mesmo
lquido, conforme ilustrao. Qual das alternativas abaixo
melhor expressa, aproximadamente, o volume do lquido contido nos frascos A, B e C, nessa ordem?
(a)

3 4 2
; ;
7 9 5

(b)

2 1 1
; ;
3 2 4

(c)

2 4 2
; ;
3 6 4

OBMEP 2010

(d)

2 4 3
; ;
3 7 4

(e)

3 4 2
; ;
3 5 3
1

Nvel 1
7. Descontos e descontos Uma farmcia d desconto de 30% sobre o preo de tabela
de todos os medicamentos que vende. Ao adquirir um remdio cujo preo de tabela
R$ 120,00, quanto reais uma pessoa ir pagar?
(a) 36

(b) 84

(c) 64

(d) Mais do que 116

(e) 94

8. O carro de Maria Um litro de lcool custa R$ 0,75. O carro de Maria percorre


25 km com 3 litros de lcool. Quantos reais Maria gastar com o lcool necessrio para
percorrer 600 km?
(a) 54

(b) 72

(c) 50

(d) 52

(e) 45

9. Calculando distncias As quatro cidades A, B, C e D


foram construdas beira de uma rodovia reta, conforme a
ilustrao.

C D

A distncia entre A e C de 50 km e a distncia entre B e D de 45 km. Alm


disso, sabe-se que a distncia entre a primeira e a ltima cidade de 80 km. Qual a
distncia, em quilmetros, entre as cidades B e C?
(a) 15

(c) 20

(c) 25

(d) 5

(e) 10

10. Pesando caixas Num armazm foram empilhadas algumas caixas que
formaram o monte mostrado na gura. Se cada caixa pesa 25 kg, quantos
quilogramas pesa o monte com todas as caixas?
(a) 300

(b) 325

(c) 350

(d) 375

(e) 400

11. Consumo de gua Na tabela a seguir vemos o consumo mensal de gua de uma
famlia, durante os cinco primeiros meses de 2004.
Qual o consumo mensal mdio de janeiro a maio
dessa famlia, em m3 ?
(a) 11,3

(c) 12,7

(b) 11,7

(e) 317,5

(d) 63,5

Meses
Janeiro
Fevereiro
Maro
Abril
Maio

Consumo (m3 )
12,5
13,8
13,7
11,4
12,1

12. Folheando um livro Um livro de cem pginas tem suas pginas numeradas de 1 a
100. Quantas folhas desse livro possuem o algarismo 5 em sua numerao?
(Ateno: uma folha tem duas pginas.)
(a) 13

(b) 14

(c) 15

(d) 16

(e) 17

13. Calculando a soma Escreva os nmeros de 0 a 9 nos crculos


ao lado, de forma que eles cresam no sentido anti-horrio. Em
seguida, subtraia uma unidade dos nmeros mpares e some uma
unidade aos nmeros pares. Escolhendo trs crculos consecutivos,
qual a maior soma que se pode obter?
(a) 19
2

(b) 21

(c) 23

(d) 24
OBMEP 2010

(e) 25

Nvel 1
14. Desenhando o cubo A gura ao lado foi desenhada em
cartolina e dobrada de modo a formar um cubo.
Qual das alternativas mostra o cubo assim formado?

(a)

(b)

(c)

(d)

(e)

15. Crculos concntricos Na malha quadriculada a seguir, todas as circunferncias


tm o mesmo centro. Pode-se concluir que a rea da regio cinza destacada igual a
(a) dois quintos da rea do crculo maior;
(b) trs stimos da rea do crculo maior;
(c) metade da rea do crculo maior;
(d) quatro stimos da rea do crculo maior;
(e) trs quintos da rea do crculo maior.

16. Brincando com engrenagens Jos colou uma bandeirinha em cada um dos dois
discos dentados que formam uma engrenagem, como mostra a gura.
Os dois discos so exatamente iguais, inclusive os dentes em
cada um deles. Jos girou a engrenagem e claro que as
bandeirinhas mudaram de posio. Qual a nova posio
das duas bandeirinhas?

(a)

(b)

(d)

(c)

(e)

17. Troca de garrafas A prefeitura de uma certa cidade fez uma campanha que permite
trocar quatro garrafas de 1 litro vazias por uma garrafa de 1 litro cheia de leite. Quantos
litros de leite pode obter uma pessoa que possua 43 garrafas vazias de 1 litro fazendo
vrias dessas trocas?
(a) 11

(b) 12

(c) 13

(d) 14
OBMEP 2010

(e) 15
3

Nvel 1
18. Retngulo e quadrados A gura dada representa
um gramado retangular em que foram marcados sete
quadrados numerados de 1 a 7. Se a rea do menor
desses quadrados 1 m2 , a rea total do gramado, em
m2 , igual a
(a) 42

(b) 44

(c) 45

(d) 48

2
3
1

5
6

(e) 49

19. Quantas fatias de bolo? Nove amigos compraram trs bolos, cada um deles
cortado em oito fatias. Todos comeram bolo e no sobrou nenhum pedao. Sabendo
que cada um s comeu fatias inteiras do bolo, podemos ter certeza de que:
(a) algum comeu quatro fatias;
(b) um deles comeu somente uma fatia;
(c) todos comeram duas fatias, pelo menos;
(d) uns comeram duas fatias e os demais comeram
trs fatias;
(e) um deles comeu, no mnimo, trs fatias.
20. Mosaicos quadrados Uma sequncia de mosaicos quadrados construda com
azulejos quadrados pretos e brancos, todos do mesmo tamanho, sendo o primeiro formado por um azulejo branco cercado por azulejos pretos, o segundo por quatro azulejos
brancos cercados por azulejos pretos e assim, sucessivamente, como indica a gura. Se
numa sequncia de mosaicos formada de acordo com esta regra forem usados 80 azulejos
pretos, quantos sero os azulejos brancos utilizados?
(a) 55

(d) 85

(b) 65

(e) 100

(c) 75
21. Quanto custa? Ester vai a uma papelaria para comprar cadernos e canetas. Nessa
papelaria, todos os cadernos custam R$ 6,00. Se ela comprar trs cadernos, sobram
R$ 4,00. Se, em vez disso, seu irmo lhe emprestar R$ 4,00 adicionais, ela conseguir
comprar dois cadernos e sete canetas, todas iguais.
(a) Quanto custa cada caneta?
(b) Se ela comprar dois cadernos e no pedir dinheiro emprestado, quantas canetas
Ester poder comprar?
22. Encontre o nmero O nmero da casa de Jlia tem exatamente trs algarismos,
cuja soma 24. Encontre todos os possveis nmeros da casa de Jlia, em cada uma
das situaes seguintes.
(a) Os trs algarismos so iguais.
(b) Apenas dois algarismos so iguais.
(c) Os algarismos so todos diferentes.
4

OBMEP 2010

Nvel 1
23. Campeonato de futebol No ltimo campeonato de futebol do bairro em que moro
participaram seis equipes, denominadas A, B, C, D, E e F. Cada equipe disputou, com
cada uma das outras, exatamente uma partida.
V E D GP GC
Na tabela de classicao do campeonato, ao lado,
A 4
1
0
6
2
V indica o nmero de vitrias, E o nmero de emB
2
1
2
6
6
pates, D o nmero de derrotas, GP o nmero de gols
C
0
3
2
2
6
marcados e GC o nmero de gols sofridos de cada
D 1
1
y
3
6
equipe.
E
0
1
4
1
5
F
x
1
0
z
3
(a) Quantas partidas foram disputadas?
(b) A tabela est incompleta. Determine a quantidade de vitrias da equipe F, a
quantidade de derrotas da equipe D e a quantidade de gols marcados pela equipe F,
representados por x, y e z na tabela.

24. Dividindo o paraleleppedo Um bloco de madeira na


forma de um paraleleppedo retngulo tem 320 cm de comprimento, 60 cm de largura e 75 cm de altura. O bloco
cortado vrias vezes, com cortes paralelos s suas faces,
de modo a subdividi-lo em blocos menores, todos na forma
de paraleleppedos retngulo de 80 cm de comprimento por
30 cm de largura por 15 cm de altura.

60
75

320

(a) Quantas peas foram obtidas?


(b) Um metro cbico dessa madeira pesa aproximadamente 900 kg. Qual o peso de
cada uma dessas peas?
25. Uma calculadora Uma calculadora possui duas teclas especiais:
a tecla A, que duplica o nmero que aparece no visor; e

a tecla B, que acrescenta uma unidade ao nmero que aparece no


visor.

Por exemplo, se o nmero 45 estiver no visor e for apertada a tecla B, o visor mostrar
o nmero 46. Se, em seguida, apertarmos a tecla A, o visor mostrar o nmero 92.
Nesse exemplo, apertamos ao todo duas vezes as teclas A e B: uma vez a tecla B e
depois uma vez a tecla A, para, a partir de 45, chegar ao 92. Suponha que no visor
esteja o nmero 1. Indique uma maneira de obter o nmero:
(a) 10 apertando um total de quatro vezes as teclas A e B;
(b) 15 apertando um total de seis vezes as teclas A e B;
(c) 100 apertando um total de oito vezes as teclas A e B.
26. Ano bissexto Um ano comum tem 365 dias e um ano bissexto, 366 dias. O ano
bissexto, quando o ms de fevereiro tem 29 dias, ocorre a cada quatro anos.
(a) Com frequncia dizemos Um ano comum tem 52 semanas. Ser correta essa
armao? E para um ano bissexto? Justique suas respostas.
(b) Se um ano comum inicia numa tera-feira, ento o ano seguinte iniciar em qual
dia da semana?
OBMEP 2010

Nvel 1
(c) Responda a pergunta anterior para um ano bissexto.
27. Nmeros triangulares O famoso matemtico grego Pitgoras denominou os nmeros obtidos pela soma dos primeiros nmeros inteiros positivos de nmeros triangulares.
Por exemplo, 1, 3, 6 e 10 so nmeros triangulares.
1 =
3 =
6 =
10 =

1
1+2
1+2+3
1+2+3+4
1

1+2=3

1+2+3=6

1 + 2 + 3 + 4 = 10

A gura ilustra a motivao para o nome dos nmeros triangulares. A sequncia de


nmeros triangulares continua com 1 + 2 + 3 + 4 + 5 = 15, 1 + 2 + 3 + 4 + 5 + 6 = 21
etc. Quantos so os nmeros triangulares menores do que 100?
28. Livros separados Uma bibliotecria recebe 130 livros de Matemtica e 195 livros
de Portugus. Ela quer arrum-los em estantes, colocando igual quantidade de livros
em cada estante, sem misturar livros de Matemtica e de Portugus na mesma estante. Quantos livros ela deve colocar em cada estante para que o nmero de estantes
utilizadas seja o menor possvel?
29. Alunos com culos A sexta parte dos alunos de uma classe usam culos. Dentre
os que usam culos, uma tera parte so meninas; alm disso, quatro meninos usam
culos. Quantos so os alunos dessa classe?
30. Quadrado mgico Complete as casas em branco da tabela ao
lado com fraes, de tal modo que a soma dos trs nmeros de
qualquer linha, qualquer coluna e das duas diagonais seja sempre a
mesma.

3/5
0,4

1/2
0,5

31. Trs algarismos Sejam A, B e C algarismos diferentes de zero tais que (AB)2 =
CAB, isto , o nmero de dois algarismos AB elevado ao quadrado d o nmero de
trs algarismos CAB. Determine o valor de A + B + C.
32. Pintando quadradinhos Uma faixa quadriculada tem 5 quadradinhos na largura
e 250 quadradinhos no comprimento. Alguns quadradinhos sero pintados de cinza,
comeando da esquerda, conforme o modelo ilustrado na gura, e continuando com
este padro at chegar ao nal da faixa, direita.
Quantos quadradinhos no sero pintados?

250 quadradinhos

OBMEP 2010

Nvel 1
33. A cisterna do Joo Joo tem, em seu jardim, uma cisterna na qual ele armazena
gua de chuva e tira gua para regar suas ores. meia-noite do dia 31 de dezembro
de 2005, a cisterna continha 156 litros de gua. Joo tem o hbito de anotar em um
quadro, todo dia, o nmero de litros de gua que ele gasta para regar as ores e o de
gua recolhida da chuva.
Ao lado vemos parte do quadro referente aos primeiros 8
dias de janeiro de 2006. Quantos litros de gua havia na
cisterna do Joo meia noite do dia 8 de janeiro de 2006?

Jan
1
2
3
4
5
6
7
8

ores(l)
6
9
0
4
9
0
11
0

chuva(l)
2,5
0
5
0
3
0
4,5
0

34. O mltiplo de 13 Da igualdade 9 174 532 13 = 119 268 916 pode-se concluir que
um dos nmeros a seguir divisvel por 13. Qual esse nmero?
(a) 119 268 903

(c) 119 268 911

(e) 119 268 923

(b) 119 268 907

(d) 119 268 913

35. Um bilho Arnaldo armou que um bilho o mesmo que um milho de milhes.
O professor Piraldo o corrigiu e disse, corretamente, que um bilho o mesmo que
mil milhes. Qual a diferena entre o valor correto de um bilho e a armao de
Arnaldo?
(a) 1 000

(b) 999 000

(c) 1 000 000

(d) 999 000 000

(e) 999 000 000 000

36. Energia de abelha Com a energia fornecida por um litro de mel, uma abelha
consegue voar 7 000 quilmetros. Quantas abelhas conseguiriam voar um quilmetro,
cada uma, com a energia fornecida por 10 litros de mel?
(a) 7 000

(b) 70 000

(c) 700 000

(d) 7 000 000

(e) 70 000 000

37. Perda de safra Um agricultor esperava receber cerca de R$ 100 000,00 pela venda
de sua safra. Entretanto, a falta de chuva provocou uma perda da safra avaliada entre
uma quinta parte e uma quarta parte do total previsto. Qual dos valores a seguir pode
representar a perda do agricultor, em reais?
(a) 21 987,53

(b) 34 900,00

(c) 44 999,99

(d) 51 987,53

(e) 60 000,00

38. Placa decorativa Uma placa decorativa consiste num


quadrado branco de quatro metros de lado, pintado de forma
simtrica com partes em cinza, conforme a gura. Qual a
frao da rea da placa que foi pintada?
(a)

1
2

(b)

1
3

(c)

3
8

(d)

6
13

(e)

7
11

39. O suco do Diamantino Diamantino colocou trs litros de gua e um litro de


refresco num recipiente. O refresco composto de 20% de suco de laranja e 80% de
OBMEP 2010

Nvel 1
gua. Depois de misturar tudo, que porcentagem do volume nal representa o suco de
laranja?
(a) 5%

(b) 7%

(c) 8%

(d) 20%

(e) 60%

40. Uma eleio Trs candidatos concorreram eleio de representante de uma turma
de escola: Joo, Rosa e Marcos. Joo obteve 2/7 dos votos e Rosa 2/5 dos votos. Quem
ganhou a eleio?
41. Soma de potncias Qual o valor de 26 + 26 + 26 + 26 44 ?
(a) 0

(b) 2

(d) 42

(c) 4

(e) 44

42. Seis retngulos Com seis retngulos idnticos formamos um retngulo maior, com um dos lados medindo
21 cm, como na gura. Qual a rea do retngulo maior,
em cm2 ?
(a) 210

(b) 280

(c) 430

(d) 504

21 cm

(e) 588

43. Duas populaes H trs anos, a populao de Pirajussara era igual populao
que Tucupira tem hoje. De l para c, a populao de Pirajussara no mudou, mas a
populao de Tucupira cresceu 50%. Hoje, a soma das populaes das duas cidades
de 9 000 habitantes. Qual era a soma dessas duas populaes h trs anos?
(a) 3 600

(b) 4 500

(c) 5 000

(d) 7 200

(e) 7 500

44. Trs balanas As balanas (1) e (2) da gura dada esto em equilbrio. Sabe-se
que todos os tringulos tm o mesmo peso, bem como todos os quadrados e tambm
todos os crculos. Quantos quadrados devem ser colocados no prato direito da balana
(3) para que ela tambm que equilibrada?

(2)

(1)
(a) 7

(b) 8

(c) 9

(d) 10

(3)
(e) 12

45. Poucos domingos Em um ano, no mximo quantos meses tm cinco domingos?


(a) 3

(b) 4

(c) 5

(d) 6

(e) 7

46. Metade de potncia Qual a metade do nmero 212 + 3 210 ?


(a) 26 + 3 25

(b) 26 + 3 210

(c) 21 1 + 3 25

(d) 211 7

(e) 29 7

47. Minutos demais Neste momento, so 18 horas e 27 minutos. Qual era o horrio
2 880 717 minutos mais cedo?
(a) 6h22min
8

(b) 6h24min

(c) 6h27min
OBMEP 2010

(d) 6h30min

(e) 6h32min

Nvel 1
48. Dois nibus Os alunos de uma escola participaram de uma excurso, para a qual
foram contratados dois nibus. Quando os nibus chegaram, 57 alunos entraram no
primeiro nibus e apenas 31 no segundo. Quantos alunos devem passar do primeiro
para o segundo nibus para que seja transportada a mesma quantidade de alunos nos
dois nibus?
(a) 8

(b) 13

(c) 16

(d) 26

(e) 31

49. Cubo de papelo Em qual das alternativas abaixo aparecem dois pedaos de papelo
com os quais pode-se construir um cubo, dobrando pelas linhas tracejadas e colando
pelas linhas contnuas?

(a)

(b)

(d)

(e)

(c)

50. Algarismo das unidades Qual o algarismo das unidades do nmero


1 3 5 97 99?
(a) 1

(b) 3

(c) 5

(d) 7

(e) 9

51. Regio sombreada A gura mostra um retngulo


formado por 18 quadrados iguais com algumas partes
sombreadas.
Qual a frao da rea do retngulo que est sombreada?
(a)

7
18

(b)

4
9

(c)

1
3

(d)

5
9

(e)

1
2

52. Colorindo um mapa A gura mostra o mapa de um pas


(imaginrio) constitudo por cinco estados. Deseja-se colorir esse
mapa com as cores verde, azul e amarelo, de modo que dois
estados vizinhos no possuam a mesma cor. De quantas maneiras
diferentes o mapa pode ser pintado?
(a) 12

(b) 6

(c) 10

(d) 24

(e) 120

53. Pintando um tabuleiro As nove casas de um tabuleiro 3 3 devem ser pintadas


de forma que em cada coluna, cada linha e cada uma das duas diagonais no haja duas
casas de mesma cor. Qual o menor nmero de cores necessrias para isso?
(a) 3

(b) 4

(c) 5

(d) 6

(e) 7

OBMEP 2010

Nvel 1
54. Nmero X,Y Considere um nmero escrito na forma decimal X, Y, onde X e Y
so algarismos diferentes de 0. Determine esse nmero, sabendo que X, Y igual a
3
(X + Y ).
10
55. Construo de casas Em um mesmo lado de uma rua sero construdas seis
casas vizinhas. As casas podem ser de alvenaria ou de madeira, mas como medida de
segurana contra incndio, duas casas de madeira no podem ser vizinhas. De quantas
maneiras se pode planejar a construo dessas casas?
56. Comparao de grandezas Qual o maior dos nmeros dados?
(a) 1 000 + 0,01

(c) 1 000/0,01

(b) 1 000 0,01

(e) 1 000 0,01

(d) 0,01/1 000

57. Maior nmero de seis algarismos Qual o maior nmero de seis algarismos
que se pode encontrar suprimindo-se nove algarismos do nmero 778 157 260 669 103,
sem mudar a ordem de seus algarismos?
(a) 778 152

(b) 781 569

(c) 879 103

58. Qual o numerador? Se


(a) 5

(b) 6

(c) 7

(d) 986 103

(e) 987 776

n
1 1
um nmero entre e , quem n?
24
6 4
(d) 8
(e) 9

59. Correndo menos Correndo a uma velocidade de 10 km/h, Joo completa um certo
percurso em seis minutos. Com qual velocidade, em km/h, ele pode completar o mesmo
percurso em oito minutos?
(a) 7,5

(b) 7,75

(c) 8

(d) 8,25

(e) 8,5

60. Cinco vizinhas As vizinhas Elza, Sueli, Patrcia, Helosa e Cludia chegam juntas
do trabalho e comeam a subir as escadas do prdio de cinco andares onde moram.
Cada uma mora num andar diferente. Helosa chega a seu andar depois de Elza, mas
antes de Cludia. Quando Sueli chega ao seu andar, Helosa ainda tem dois andares
para subir e o mesmo ocorre com Patrcia quando Elza chega ao seu andar. Sueli no
mora no primeiro andar. Em qual andar mora cada uma delas?
61. Potncias de 9 Qual o valor da soma 920 + 920 + 920 ?
(a) 920

(b) 366

(c) 923

(d) 341

(e) 323

62. Dois nmeros Miguel escolheu um nmero de trs algarismos e outro de dois. Qual
a soma desses nmeros se sua diferena 989?
(a) 1 000

(b) 1 001

(c) 1 009

(d) 1 010

(e) 2 005

63. Menor natural Qual o menor nmero natural n para o qual 10n 1 um mltiplo
de 37?
(a) 6
10

(b) 5

(c) 4

(d) 3

(e) 2

OBMEP 2010

Nvel 1
64. Imunes gripes Num certo pas com 14 milhes de habitantes, 0,15% da populao
contraiu uma certa gripe. Quantos habitantes no contraram essa gripe?
(a) 13 979 000

(b) 1 397 900

(c) 139 790

(d) 13 979

(e) 139 790 000

65. O cdigo secreto O cdigo secreto de um grupo de alunos um nmero de trs


algarismos distintos diferentes de 0. Descubra o cdigo utilizando as informaes a
seguir.
1
4
6
5
8
(a) 137

2
5
1
4
4

Nenhum algarismo correto.


S um algarismo correto na posio certa.
S um algarismo correto, mas na posio errada.
S um algarismo correto, mas na posio errada.
S um algarismo correto na posio certa.

3
6
2
7
3

(b) 876

(c) 768

(d) 678

(e) 576

66. Parnteses, colchetes e chaves Qual o valor de 2 2 2 2 2 2(4 2)


(a) 0

(b) 2

(c) 2

(d) 4

(e) 10

4 4 4 3 6
67. Ordenando fraes Qual a ordem crescente correta das fraes , , , , e
3 5 6 5 5
2
?
5
2
3
4
4
6
4
2
3
4
4
6
4
(a)
< < < < <
(d)
< < < < <
5
5
6
5
5
3
5
5
5
6
5
3
4
2
3
4
6
2
3
4
4
4
6
4
< < < < <
(e)
< < < < <
(b)
3
6
5
5
5
5
5
5
5
3
6
5
2
3
4
4
4
6
(c)
< < < < <
5
5
5
6
3
5
68. Nmeros de trs algarismos Quantos nmeros de trs algarismos maiores do que
200 podem ser escritos, usando-se apenas os algarismos 1, 3 e 5?
(a) 10

(b) 12

(c) 14

(d) 15

(e) 18

69. Velocidade de maratona Uma maratona de 42 km comeou s 11h30min e o vencedor terminou s 13h45min do mesmo dia. Qual foi a velocidade mdia do vencedor,
em km/h?
(a) 18,6

(b) 25,3

(c) 29

(d) 32,5

(e) 38

70. Bilhetinhos com nmeros Cinco alunas escreveram cada uma um nmero num
papel. Os nmeros s podiam ser 1 ou 2 ou 4. Qual pode ser o produto dos cinco
nmeros escritos?
(a) 100

(b) 120

(c) 256

(d) 768

OBMEP 2010

(e) 2 048
11

Nvel 1
71. Produto de fraes Qual o valor do produto 1
(a)

119
120

(b)

5
7

(c) 2

43
60

1
5

(d)

1
2

1
3

1
4

1
?
5

1
120

(e)

72. Produto mximo A soma de dois nmeros naturais 11. Qual o maior produto
possvel que se pode obter com esses nmeros?
(a) 30

(b) 22

(c) 66

(d) 24

(e) 28

73. Quem o cubo? Se m um nmero natural tal que 3m = 81, quanto vale m3 ?
(a) 813

(b) 381

(c) 64

(d) 24

(e) 48

74. Qual o maior? Se a 1 = b + 2 = c 3 = d + 4, qual o maior dentre os


nmeros a, b, c e d?
(a) a

(b) b

(c) c

(d) d

(e) So todos iguais

75. Quatro formiguinhas Quatro formigas atravessam o piso de uma sala coberto de
lajotas retangulares, segundo os trajetos indicados na gura. Qual o comprimento
do trajeto percorrido por Biloca?
(a) 30 dm
Trajeto de Pipoca = 25 dm

(b) 43 dm
(c) 55 dm

Trajeto de Tonica = 37 dm

(d) 24 dm

Trajeto de Cotinha = 32 dm

(e) 48 dm

Trajeto de Biloca =

76. Trocando gurinhas Clia quer trocar com Guilherme gurinhas de um lbum
sobre animais brasileiros. Clia quer trocar quatro gurinhas de borboleta, cinco de
tubaro, trs de cobra, seis de periquito e seis de macaco. Todas as gurinhas de
Guilherme so de aranha. Eles sabem que
(a) uma gurinha de borboleta vale trs gurinhas de tubaro;
(b) uma gurinha de cobra vale trs gurinhas de periquito;
(c) uma gurinha de macaco vale quatro gurinhas de aranha;
(d) uma gurinha de periquito vale trs gurinhas de aranha;
(e) uma gurinha de tubaro vale duas gurinhas de periquito.
Quantas gurinhas Clia poder receber se ela trocar todas que quiser?
77. Soma de fraes Qual o valor da soma
(a) 1
12

(b) 20

(c) 30

10 + 20 + 30 + 40
10
+
?
10
10 + 20 + 30 + 40

(d) 10,1
OBMEP 2010

(e) 1,01

Nvel 1
78. Geometria com palitos A gura dada formada por um
tringulo e um retngulo, usando-se 60 palitos iguais. Para cada
lado do tringulo so necessrios seis palitos. Se cada palito mede
5 cm de comprimento, qual a rea (em cm2 ) do retngulo da
gura?
(a) 1 200

(c) 2 700

(b) 1 800

(e) 4 500

(d) 3 600

79. Um incndio e o bombeiro Uma casa pega fogo. Um bombeiro se mantm no


degrau do meio de uma escada, jogando gua sobre o incndio. As chamas diminuem
e ele sobe cinco degraus. O vento sopra e o bombeiro desce sete degraus. Um pouco
depois, ele sobe oito degraus e ca l at acabar o incndio. Ento, ele sobe os ltimos
sete degraus e entra na casa. Quantos degraus tem a escada do bombeiro?
(a) 25

(b) 26

(c) 27

(d) 28

(e) 29

80. rvore genealgica A gura mostra a rvore genealgica de uma famlia. Cada
echa vai do pai em direo ao seu lho. Quem o irmo do pai do irmo do pai de
Evaristo?
Ado

(a) Francisco

Lus

Andr

(b) Jos
(c) Andr

Felipe

(d) Felipe

Cristvo

(e) Simo

Jean

Jos

Evaristo

Francisco

81. Colcha quadrada Uma colcha quadrada em branco e cinza


feita com quadrados e tringulos retngulos issceles. A parte em
cinza representa qual percentagem da colcha?
(a) 36%

(c) 45%

(b) 40%

(e) 60%

(d) 50%

82. Falsas igualdades Considere as igualdades a seguir.


(i) 3 106 + 5 102 = 8 108

(iii) 5 8 + 7 = 75

(ii) 23 + 23 = 20

(iv) 5 + 5 5 = 2

Qual delas est correta?


(a) (i)

(b) (ii)

(c) (iii)

(d) (iv)

(e) Nenhuma

83. Menor valor da soma Se a, b e c so nmeros inteiros positivos tais que


3a = 4b = 7c, qual o menor valor possvel de a + b + c?
(a) 84

(b) 36

(c) 61

(d) 56
OBMEP 2010

(e) 42
13

Nvel 1
84. Procurando um quadrado perfeito Um nmero um quadrado perfeito se igual
a um nmero inteiro elevado ao quadrado. Por exemplo, 25 = 52 , 49 = 72 e 125 = 252
so quadrados perfeitos. Qual o menor nmero pelo qual devemos multiplicar 120
para obter um quadrado perfeito?
(a) 10

(b) 15

(c) 20

(d) 30

(e) 35

85. Visitas num museu A mquina que registra o nmero de visitantes de um museu
marca 1 879 564. Note que esse nmero tem todos os algarismos distintos. Qual o
menor nmero de visitantes que so necessrios para que a mquina registre um outro
nmero que tambm tenha todos os seus algarismos distintos?
(a) 35

(b) 36

(c) 38

(d) 47

(e) 52

86. Ligando nmeros por echas Os nmeros de 0 a 2 000 foram ligados por echas;
a gura dada mostra o comeo do processo.

Qual a sucesso de echas que liga o nmero 1 997 ao nmero 2 000?


(a)

(b)

(c)

(d)

(e)

87. Mltiplos de 9 Encontre o menor mltiplo positivo de 9 que pode ser escrito apenas
com os algarismos: (a) 0 e 1;
(b) 1 e 2.
88. A orista Uma orista colheu 49 kg de ores do campo. O quilograma das ores
pode ser vendido imediatamente a R$ 1,25 ou, mais tarde, com as ores desidratadas,
a R$ 3,25. O processo de desidratao faz as ores perderem 5/7 de seu peso. Qual
o tipo de venda mais lucrativo para a orista?
89. Divisores Seja N o menor nmero que tem 378 divisores e da forma 2a 3b 5c 7d .
Quanto vale cada um desses expoentes?
90. O produto dos algarismos Denotemos por P (n) o produto dos algarismos do
nmero n. Por exemplo, P (58) = 5 8 = 40 e P (319) = 3 1 9 = 27.
(a) Dentre os nmeros de 1 a 999, quais so os que tm produto dos algarismos igual
a 12, isto , quais so os inteiros n tais que 1 n < 1 000 e P (n) = 12?

(b) Quantos nmeros inteiros existem entre 0 e 200 cujo produto dos algarismos seja
igual a 0, isto , quantos inteiros n existem tais que 1 n < 200 e P (n) = 0?
(c) Quais so os nmeros inteiros 1 n < 200 tais que 37 < P (n) < 45?

(d) Dentre todos os inteiros de 1 a 250, qual o nmero cujo produto dos algarismos
o maior possvel?
14

OBMEP 2010

Nvel 1
91. Suco de laranja Davi vai a um armazm que vende uma garrafa de suco de laranja
por R$ 2,80 e uma caixa com seis dessas garrafas por R$ 15,00. Ele precisa comprar
22 garrafas para seu aniversrio. Quanto ele gastar, no mnimo?
92. A casa da Rosa A gura mostra a planta da casa da Rosa. O quarto e o quintal
so quadrados. Qual a rea da cozinha?
..........................................................................................................
.
.
.........................................................................................................
.
.
.
.
.
.
.
.
.
.
.
.
.
.
.
.
.
.
.
.
.
.
.
.
.
.
.
.
.
.
.
.
.
.
.
.
.
.
.
.
.
.
.
.
.
.
.
.
.
.
.
.
.
.
.
.
.
.
.
.
.
.
.
.
.
.
.
.
.
.
.
.
.
.
.
.
.
.
.
.
.
.
.
.
.
.
.
.
.
.
.
.
.
.
.
.
.
.
.
.
.
.
.
.
.
2 .
2
.
.
.
.
.
.
.
.
.
.
.
.
.
.
.
.
.
.
.
.
.
.
.
.
.
.
.
.
.
.
.
.
.
.
.
.
.
.
.
.
.
.
.
.
.
.
.
..........................................................................................................
.........................................................................................................
.
.
.
.
.
.
.
.
.
.
.
.
.
.
.
.
.
.
.
.
.
.
.
.
.
.
.
.
.
.
.
.
.
.
.
.
. Quintal .
.
.
.
.
.
.
.
.
.
.
.
.
.
.
.
.
.
.
2.
.
.
.
.
.
.
.
.
.
.
.
.
.
.
.
.
.
.
.
.
.
..........................................................................................................
.
.
.........................................................................................................
.
.

Sala
24 m

Quarto
16 m

Cozinha

4m

93. O passeio do Matias Matias passeia de bicicleta nas ruas em volta de quatro quarteires perto de sua casa, dispostos como na gura. O seu passeio consiste em fazer o maior percurso possvel, mas ele inventou uma regra para se divertir
mais, a saber: ele pode passar vrias vezes pelos cruzamentos das ..................................................................................................................................................................................................................
. .
. .
. .
. .
. .
. .
. .
.
. .
. .
. .
. .
. .
. .
. .
. .
. .
. .
. .
. .
. .
ruas, mas ele no pode passar mais do que uma vez pela mesma rua, ............ ............
. .
. .
. .
. .
. .
. .
. .
. .
. .
. .
. .
. .
. .
. .
. .
. .
.
. .
. .
. .
. .
. .
. .
. .
. .
. .
. .
. .
. .
devendo terminar seu passeio de bicicleta quando no puder mais ..... .......................................... ................................................ ...........
.
.
.
.
.
.
respeitar essa condio. Os quatro quarteires so quadrados, cada ........... ......................................................... ......................................................... ...........
. .
. .
. .
. .
. .
. .
. .
. .
. .
. .
. .
. .
. .
. .
. .
. .
. .
. .
. .
. .
. .
um com 100 metros de lado e a largura das ruas no considerada ........... ...........
. .
. .
. .
. .
. .
. .
. .
. .
. .
. .
. .................... .................... .
. .
. .
. ................... ................... .
. .
.
.
relevante. Partindo do ponto indicado por P na gura, qual o ..............................................................r.............................................................
.
P
maior desses percursos de bicicleta que Matias pode fazer?
94. O adesivo dos carros ociais O prefeito de uma cidade decidiu colocar adesivo em
cada carro ocial. O adesivo ter uma forma retangular, com seis quadrados dispostos
em 2 3 e com trs cores: um quadrado azul, dois quadrados amarelos e trs quadrados
verdes. Dentre quantos tipos diferentes de adesivo o prefeito poder escolher?
95. Adio de nmeros Qual o algarismo a em a 000 + a 998 + a 999 = 22 997?
96. Cubo perfeito e divisibilidade Quais so os cubos perfeitos que dividem 94 ?
97. Localizao de um ponto Qual o ponto indicado na gura?

18

19

20

98. Clculo de porcentagem Se voc acerta 58/84 das 84 questes de um teste, qual
o seu percentual de acertos?
99. Comparao de algarismos Dizemos que um nmero ascendente se cada um
de seus algarismos for maior do que o algarismo que est sua esquerda. Por exemplo,
2 568 ascendente e 175 no . Quantos nmeros ascendentes existem entre 400 e 600?
OBMEP 2010

15

Nvel 1
100. Muro colorido Um muro deve ser construdo conforme a gura com 14 tijolos
coloridos, disponveis em amarelo, azul e vermelho, cujos preos esto dados na tabela.
Se dois tijolos quaisquer que se toquem devem ser de cores diferentes, qual o menor
valor que se gastar na compra desses 14 tijolos?
tijolo
amarelo
azul
vermelho

............................................................................
.
.
............................................................................
.
.
.
.
.
.
.
.
.
.
.
.
.
.
.
.
.
.
.
.
.
.
.
.
.
.
.
.
.
.
.
.
.
.
.
.
.
.
.
.
.
.....................................................................................................
.....................................................................................................
.
.
.
.
.
.
.
.
.
.
.
.
.
.
.
.
.
.
.
.
.
.
.
.
.
.
.
.
.
.
.
.
.
.
.
.
.
.
.
.
.
.
.
.
.
.
.
............................................................................
.....................................................................................................
.
..........................
.
.
.
.
.
.
.
.
.
.
.
.
.
.
.
.
.
.
.
.
.
.
.
.
.
.
.
.
.
.
.
.
.
.
.
.
.
.
.
.
.
.
.
.
......................................................................................................
.
.
.
.
.....................................................................................................
.
.
.
.
.
.
.
.
.
.
.
.
.
.
.
.
.
.
.
.
.
.
.
.
.
.
.
.
.
.
.
.
.
.
.
.
.
.
.
.
.
.
.....................................................................................................
.
.
.
.....................................................................................................
.
.

R$
6
7
8

101. Divisores e fatorao Decomponha 96 em dois fatores inteiros positivos cuja soma
dos quadrados seja 208.
102. O retngulo do Lus Lus desenhou um retngulo de 6 10 cm e quer dividi-lo
em quatro partes. As reas das 4 partes devem medir 8, 12, 16 e 24 cm2 . Desenhe
como ele pode fazer essa diviso.
103. Comparao de nmeros Escreva em ordem crescente os nmeros

3
4
121,
729
e
38 416 .
104. As moedas Uma brincadeira comea com sete moedas alinhadas em cima de uma
mesa, todas com a face coroa virada para cima. Para ganhar a brincadeira, preciso
virar algumas moedas, de tal modo que, no nal, duas moedas vizinhas estejam sempre
com faces diferentes viradas para cima. A regra da brincadeira virar duas moedas
vizinhas em cada jogada. Quantas jogadas so necessrias, no mnimo, para ganhar a
brincadeira?
......
......
......
......
......
......
.......
.......
.......
.......
.......
.................
.................
.................
..... .......
..... .......
..... .......
..... .......
..... .......
..... ........
..... .......
..... .......
..... .......
..... .......
..... .......
...
.. ....
...
..
.. ....
.. ....
.. ....
.. ....
.. ....
.. ....
..
.. ...
.. ...
.. ...
.. ...
.. ...
..
. .
..
.
. .
. .
. .
. .
. .
. .
.
.
.
.
.
.
.
. ..
. ..
. ..
. ..
. ..
. ..
.
..
..
..
..
.
..
..
.
..
..
..
..
.
..
..
.
..
..
..
..
.
..
..
.
.
.
.
.
.
.
.
.
.
. .
. .
. .
. .
. .
. .
.
.
..
..
..
..
..
..
.
.
. .
. .
. .
. .
. .
. .
.
.
. .
. .
. .
. .
. .
. .
..
.
. ..
. ..
. ..
. ..
. ..
. ..
..
...
...
...
...
...
...
...
..
.. ....
.. ....
.. ....
.. ....
.. ....
.. ....
...
..
..
..
..
..
..
..
.................
.................
.................
.................
.................
.................
.................
................
................
................
................
................
................
................

coroa coroa coroa coroa coroa coroa coroa

105. O preo do frango O preo do quilograma de frango era R$ 1,00 em janeiro de


2000, quando comeou a triplicar a cada 6 meses. Em quanto tempo o preo atingir
R$ 81,00?
(a) 1 ano

(b) 2 anos

(c) 2

1
anos
2

(d) 13 anos

(e) 13

1
anos
2

106. Excurses a Foz do Iguau Em 2005, uma agncia de turismo programou uma
excurso para Foz do Iguau, distribuindo as pessoas em nibus de 27 lugares, tendo
sido necessrio formar um nibus incompleto, com 19 lugares ocupados. Em 2006,
aumentou em 53 o nmero de participantes e a agncia continuou a utilizar nibus
de 27 lugares. Quantos nibus a mais foram necessrios e quantas pessoas caram no
nibus incompleto em 2006?
107. As fraes de Laura Laura desenhou cinco crculos, dentro dos
quais ela quer colocar nmeros inteiros positivos, de tal modo que
formem uma igualdade entre uma frao e seu valor inteiro.
16

OBMEP 2010

j+ j j
+
j

............................................
............................................

= j

Nvel 1
De quantas maneiras pode Laura colocar os nmeros 2, 3, 5, 6 e 11 dentro dos cinco
crculos para que a igualdade seja verdadeira?
108. Clculo da unidade Qual o algarismo da unidade do produto
(5 + 1)(53 + 1)(56 + 1)(512 + 1)?
(a) 0

(b) 1

(c) 2

(d) 5

(e) 6

109. Nmeros cruzados Francisco escreveu 28 algarismos numa tabela 66 e pintou de


preto as demais casas, como nas palavras cruzadas. Ele fez a lista de todos os nmeros,
28
175
885

45
51
289
632
5 647 5 873

57
746
7 592

72
752
8 764

88
805

em ordem crescente, que podem ser lidos horizontal ou verticalmente, excluindo os nmeros de um s algarismo. Preencha a
tabela escrevendo de volta os nmeros de Francisco. Um algarismo j foi colocado.
2
110. Ovos e mas Num certo armazm, uma dzia de ovos e 10 mas tinham o
mesmo preo. Depois de uma semana, o preo dos ovos caiu 10% e o da ma subiu
2%. Quanto se gastar a mais na compra de uma dzia de ovos e 10 mas?
(a) 2%

(b) 4%

(c) 10%

(d) 12%

(e) 12,2%

111. Dividindo nmeros decimais Sabendo que 144 177 = 25 488, podemos concluir
que 254,88 0,177 igual a:
(a) 1 440;

(b) 14,4;

(c) 1,44;

(d) 0,144;

112. Almoo dos amigos Jlio e Denise almoaram num


restaurante que oferece trs tipos de prato e trs tipos
de vitamina, cujos preos esto na tabela ao lado. Cada
um escolheu um prato e uma vitamina. Jlio gastou 6
reais a mais do que Denise. Quanto Denise gastou?

(e) 144.

prato simples
prato com carne
prato com peixe
vitamina de leite
vitamina de frutas
vitamina especial

R$
7
11
14
6
7
9

113. Somas de trs em trs Encontre quatro nmeros inteiros positivos que, somados
de trs em trs, do somas 6, 7, 8 e 9.
s
....
.
.
.
..
.
.
.
.
.
.
114. O passeio do Jorge Jorge passeia por um caminho em forma P.........................s............................s............................s..................................s
...
.
.
.
.
.
.
.
.
.
.
.
.
.
.
.
.
.
.
.
.
de retngulo, onde esto dispostas 12 rvores, brincando de tocar ......s
.
.
.
s
.
.
.
.
.
.
.
.
.
.
.
.
.
cada rvore durante seu passeio. Primeiro ele toca a rvore do ............
.
.
.
.
.
.
..............................s .............................
s
s
s
................s............................................
.
...
...
.
canto, assinalada com P na gura, e percorre 32 metros num
mesmo sentido do percurso; a ele volta 18 metros e depois retorna ao sentido inicial
OBMEP 2010

17

Nvel 1
por mais 22 metros. Entre duas rvores consecutivas, a distncia de 5 m. Em quantas
rvores ele tocou?
115. A descoberta do algarismo Os quadrados dos nmeros naturais de 1 a 99 foram
escritos um aps o outro, formando o nmero 14916253649. . . . Qual o algarismo que
ocupa a 100a posio? (As posies so contadas da esquerda para a direita, portanto,
a 1a posio a do 1, a 2a a do 4, e assim por diante.)
116. OBMEP Cada um dos sete discos X, Z, O, B, M, E e P da gura tem um peso
diferente, que varia de 1 a 7 g. Em algumas intersees de dois discos, indicamos a
soma dos pesos desses dois discos. Qual a soma dos pesos dos cinco discos O, B, M, E
e P?
.................
.................
.....................
.....................
.....
.....
....
....
....
....
...
...
....
....
...
...
..
..
..
..
..
..
..
..
..
..
..
..
..
..
..
..
..
..
.
.
.
.
.
.
.
.
.
.
.
.
.
.
.
.
.
.
.
.
.
.
.
.
.
.
.
.
.
.
.
..............
..............
..............
.
...............
...............
...............
.
... .
...
... .
... .
.
....
....
.... ......... .
....
....
.... ......... .
.
.
..... .
..... .
..... .
..... .
.
.
.
.
...
...
... ..
.
.
...
...
......
.
.
.
.
...
...
...
...
.
.
.
.
.
..
.. ..
..
..
.
..
.. ...
..
..
.
.
....
..
..
..
.....
..
..
.. .
..
.. .
..
..
.
.
..
..
. .
.
..
..
..
..
.
. .
.
...
...
.
.
.
..
...
...
.
.
. .
.
.
..
. .
.
.
..
....
....
...
...
.
.
....
....
...
...
.
.
.
.
.
.
.
.
......... ...........
........ . ...........
.....................
.....................
.
..
.
.
.. .
.
.
.
.
.
. ..
. ..
.
.
.
.
.
.
.
...... . ........
...... . .........
.
...... .. .........
.
...... . .........
.
.
.
.
.
..
..
.
.
.
...
...
...
...
.
..
.
. ..
.
....
....
...
...
.
..
.
. .
.
.
.
..
..
. .
.
.
..
..
..
..
..
.
. ..
..
..
..
..
.. .
..
.
. ...
..
..
..
..
..
..
...
..
..
.. ..
.
..
..
..
..
..
.. ...
.
.
.
.
.
.
.
.
..
..
..
...
.
.
.
.
...
...
..
...
.
.
...
... .......
....
.
.
.
...
...
... .......
..... .
..... .
...
..... .
.
.
.
.
......................
.
.
.
.
......................
.....................
....................
....................
...................
.
.
.
.
.
.
.
.
.
.
.
.
.
.
.
.
.
.
.
.
.
.
.
.
.
.
.
.
.
.
.
.
.
..
..
..
..
.
.
..
..
..
..
..
..
..
..
...
...
..
..
...
...
..
..
....
....
....
....
....
....
....
....
........ ..........
........ ..........
....................
....................
...
...

13

117. Prdio misterioso As guras mostram as plantas de dois andares de um prdio que
guarda segredos muito valiosos. H nove elevadores que atendem esses dois andares,
representados por letras. Qual o caminho mais curto entre a entrada indicada de um
andar e a sada indicada do outro?
qqqqqqqqqqqqqqqqqqqqqqqqqqqqqqqqqqqqqqqqqqqqqqqqqqqqqqqqqqqqqqq
q
q
A B qqqq C qqqq
qqqqqqqqqqqqqqqqq
qqqqqqqqqqqqqqqqqqqqqqqqqqqqqqqqqqqqqqqqqqqqqqqqqqq
qq
qq
qq
q
qq
qq
qq E qqq
qq D
qq
qq
qq
qq
qq
qq
qq
qq
qq
qq
qq
qq
qqqqqqqqqqqqqqqqqqqqqqqqqqqqqqq
qq
q
qq
qq
qq
qq
qq F qq G qqq H qqqq
qqqqqqqqqqqqqqqqqqqqqqqqqqqqqqqqqqqqqqq
qqqqqqqqqqqqqqqq
qq
qq
qq
qq
qq
qq J
q
qq
qqqqqqqqqqqqqqqqqqqqqqqqqqqqqqqqqqqqqqqqqqqqqqqqqqqqqqqqqqqqqqqqq

qqqqqqqqqqqqqqqqqqqqqqqqqqqqqqqqqqqqqqqqqqqqqqqqqqqqqqqqqqqqqqqqqqqq
A q B C qqq
qq
qqqqqqqqqqqqqqqqqqqqqqqqqqqqqqqqqqqqq
qqqqqqqqqqqqqqqqq
qqq
q
qq D
E qqqqq
qq
qq
qq
q
qqqqqqqqqqqqqqqqqqqqqqqqqqqqqqqqqqqqqqqqqqqqqq
qq
q
qq
qq
qq
qq
qq F qq G q H qqq
qq
qq
qqqqqqqqqqqqqqqqq
qq
q
qq
qq
qqq
qq J
q
sada
qqqqqqqqqqqqqqqqqqqqqqqqqqqqqqqqqqqqqqqqqqqqqqqqqqqqqqqqqqqqqqqq

entrada

118. Soma de fraes Qual o valor de

1
1
1
1

?
10 100 1 000 10 000

27
do
119. Biblioteca A biblioteca de uma escola comprou 140 novos livros, cando com
25
nmero de livros que tinha antes da compra. O nmero de livros antes dessa compra
era:
(a) 1 750;

(b) 2 500;

(c) 2 780;

(d) 2 140;

(e) 1 140.

120. Comparao de fraes Existem quantas fraes menores do que 1, nas quais o
numerador e o denominador so nmeros inteiros positivos de um s algarismo?
18

OBMEP 2010

Nvel 1
121. Diviso com resto Quais so os nmeros que deixam resto 5 ao dividir 2 007?
122. Panelas Uma panela pesa 645 g e uma outra 237 g. Jos divide 1 kg de carne entre
as duas panelas, de modo que as duas, com seus contedos, cam com o mesmo peso.
Quanto de carne ele colocou em cada panela?
123. Domins Juliana representou uma multiplicao com 5 domins. Seu irmo Bruno
trocou dois domins de posio e agora a multiplicao cou errada. Troque de volta
a posio de dois domins para que a multiplicao que novamente correta.
...........................................
.
.
.
.
. s .
.
.
.
.
.
.
. s s.
.
.
.
.
.
...........................................
.
.
.
.....s................
..
.
.
. s .
.
.
.
.
.
.
...................s..
..
........................................... . s s.
. s s. .
. s s.
.
.
. s . . s s.
.
.
. .
.
.
.
. s s.
. s s. . s s.
. .
.
.
. .
.
.
.......................................... ......................
.
.
.
.

...........................................
.
.
.
.
. s s. s .
.
.
.
.
.
.
.
.
.
s .
.s
.
.
...........................................
.
.
...........................................
.
.
.
.
.
. s . s s s.
.
.
.
.
.
. s s s.
.
.
.
.
.
...........................................
.
.

3212
3
16456

...........................
...........................

124. Cdigo secreto Antnio precisa descobrir um cdigo de 3 algarismos diferentes


A, B e C. Ele sabe que B maior que A, que A menor do que C e tambm que valem
as igualdades seguintes.
.
.
.......................
.
.......................
.
.
.
.
.
.
.
.
.
.
.
.
.
.
.
.
.
.
.
.
.
.
.
.
.
.......................
.
.
.......................
.
.

.
.
.......................
.
.......................
.
.
.
.
.
.

.
.
.......................
.
.......................
.
.
.
.
.
.

.
.
...................................
.
.
..................................
.
.
.
.
.
.
.
.

B B + ..............A........................A.............. + ..............C........................C.............. = ..................2..........................4..........................2................


.....
....
.....
....
.....
....
.....
....
...
..
...
..
...
..

............
.
.
.
.
............
.
.
.
.
.
.
.
.
.
.
.
.
.
.
.
.
.
............
............
.

............
.
.
.
.
............

............
.
.
.
.
............

...................................
.
.
.
.
.
.
.
.
..................................

.
.
.
.
.
.
.
.
B ................A............... .................C.............. = .................3..........................6.........................0................
...
..
...
..
...
...
.....
....
.....
....

Qual o cdigo que Antnio procura?


125. Os doze pontos Doze pontos esto marcados numa folha de papel
quadriculado, conforme mostra a gura. Qual o nmero mximo de
quadrados que podem ser formados unindo quatro desses pontos?

126. Relgio O grande relgio de parede da escola marca a data (dia,


ms e ano) e as horas (horas e minutos), como na gura. Em que
dia, ms e ano voltaro a aparecer juntos no relgio esses mesmos
10 algarismos pela primeira vez?

r r
r r r r
r r r r
r r

.
...............
...........................
........
.....
.....
....
.....
....
....
...
....
...
...
..
...
..
..
..
..
..
.
..
..
..
..
..
..
..
..
.
.
.
.
.
.
.
.
.
.
.
.
.
.
.
.
.
.
.
.
.
.
.
.
.
.
.
.
.
.
.
.
.
.
..............
.............
............
............
.
.
.
.
.
.
.
.
.
.
.
.
.
.
.
.
.
.
.
.
.
.
.
.
.
.
.
.
.
.
.
.
.
.
.
.
.
.
.
.
.
.
.
.
.
..
.
.
.
.
.. .
.
.
.
.
.
.
.
.
.
.
.
.
.
.. .
.
.
.
.
.. ............
.
............. ...
.
.............. ...
.
.. ............
.. .
.
..
..
...
..
...
....
...
...
....
.
.....
...
.....
....
....
.......
......... .............
...................
......

28 05 94
14 h 00

127. Lpis Setenta e quatro lpis foram embalados em 13 caixas. Se a capacidade mxima
de cada caixa de seis lpis, qual o nmero mnimo de lpis que pode haver em uma
caixa?
(a) 1

(b) 2

(c) 3

(d) 4

(e) 6

128. Contagem Se o algarismo 1 aparece 171 vezes na numerao das pginas de um


livro, quantas pginas tem o livro?
OBMEP 2010

19

Nvel 1
129. Viagem a Recife Quando fui receber a medalha de ouro que conquistei na OBMEP,
apareceram as seguintes informaes nas telas da cabine de passageiros do meu voo
para Recife:
Velocidade mdia: 864 km/h
Distncia do local de partida: 1 222 km
Tempo de chegada a Recife: 1h20min
Se o avio manteve a mesma velocidade, ento qual a distncia, aproximadamente,
em quilmetros, entre Recife e a cidade em que comeou meu voo?
(a) 2 300

(b) 2 400

(c) 2 500

(d) 2 600

(e) 2 700

130. Praa Maria e Joo do uma volta completa na praa, juntos, contando as casas que
cam em volta da praa. Eles comearam a contar as casas em pontos diferentes. A
quinta casa da Maria a dcima segunda do Joo e a quinta casa do Joo a trigsima
da Maria. Quantas casas h em volta da praa?
131. Sequncia de guras As guras , , , , e 2 so repetidas indenidamente
na sequncia
, , , , , 2, , , , , , 2, . . .
(a) Que gura aparecer na 1 000a posio da sequncia?
(b) Em qual posio aparece o milsimo ?
132. A brincadeira com o quadrado Um quadrado de 1 m de lado foi cortado, com
cortes paralelos aos seus lados, em quadradinhos de 1 mm de lado. Colocando-se lado a
lado os quadradinhos, sem superposio, formou-se um retngulo de 1 mm de largura.
Qual o comprimento desse retngulo?
133. O cdigo da Arca do Tesouro Simo precisa descobrir
um nmero escondido na tabela fornecida, que o cdigo da
Arca do Tesouro.
Para descobrir o cdigo, ele precisa formar todos os grupos de
trs algarismos que estejam em casas sucessivas, na horizontal
ou na vertical, e cuja soma seja 14. Retirados da tabela todos
os possveis nmeros desses grupos, o cdigo a soma dos
nmeros que restam na tabela. Qual esse cdigo?
134. Operaes com decimais Efetue a diviso

5
6
8
7
2
5

9
3
2
4
7
2

4
7
4
5
6
3

9
3
2
7
1
6

4
4
5
5
2
7

1
8
5
2
8
1

(0,2)3 + 1
.
0,2 + 1

135. Fatores inteiros Decompor 96 em dois fatores inteiros cuja soma dos quadrados
seja 208.
136. Divisibilidade No nmero 6a78b, a denota o algarismo da unidade de milhar e b
denota o algarismo da unidade. Se 6a78b for divisvel por 45, ento o valor de a + b :
(a) 5;
20

(b) 6;

(c) 7;

(d) 8;
OBMEP 2010

(e) 9.

Nvel 1
137. Nmero simples Digamos que um nmero inteiro positivo seja simples se ele tiver
apenas os algarismos 1 ou 2 (ou ambos). Quantos nmeros menores do que 1 milho
so simples?
138. Venda de TV O gerente de uma loja foi vericar qual tinha sido o preo de venda de
uma televiso da marca VejoTudo em 2006. Ele encontrou uma fatura meio apagada,
em que se podia ler lote de 72 TVs da VejoTudo vendido por R$ _ 6.79_ , 00, mas o
algarismo da dezena de milhar e o da unidade do preo pago pelo lote estavam ilegveis.
Qual foi o preo de venda de cada uma dessas televises em 2006?
139. Chocolate Henrique comprou barras de chocolate por R$ 1,35 cada uma. Ele pagou
com uma nota de R$ 10,00 e recebeu um troco inferior a R$ 1,00. Quantas barras ele
comprou?
140. O quadradinho Qual o valor de

em

6 400 000
= 1,6
400

141. Dois nmeros O produto de dois nmeros de dois algarismos cada 1 728. Se o
mximo divisor comum (MDC) deles 12, quais so esses nmeros?
142. As idades dos irmos No dia de seu aniversrio de 7 anos, em 13 de maro de
2007, uma tera-feira, Carlos disse a seu irmo: A contar de hoje, faltam 2 000 dias
para voc completar 15 anos. Em que dia da semana vai cair o aniversrio do irmo
de Carlos? Quantos anos ter Carlos nesse dia?
143. A mistura de concreto Uma certa mistura de concreto feita de cimento, areia e
terra, na razo de 1 : 3 : 5 por quilo. Quantos quilos dessa mistura podem ser feitos
com 5 quilos de cimento?
(a) 13

1
3

(b) 15

(c) 25

(d) 40

(e) 45

144. Ponto na escala A que nmero corresponde o ponto P indicado na escala dada?
12,62

12,44
P

145. O pomar do Francisco O pomar do Francisco tem macieiras, pereiras, laranjeiras,


limoeiros e tangerineiras, dispostas em cinco las paralelas, cada uma com uma nica
variedade de rvores, da seguinte maneira:
(a) as laranjeiras esto do lado dos limoeiros;
(b) as pereiras no esto do lado das laranjeiras nem dos limoeiros;
(c) as macieiras esto do lado das pereiras, mas no das laranjeiras, nem dos limoeiros.
Em que la esto as tangerineiras?
(a) 1a

(b) 2a

(c) 3a

(d) 4a
OBMEP 2010

(e) 5a
21

Nvel 1
146. Quatro quadrados Quatro quadrados iguais esto superpostos formando a gura
dada. Se cada um dos quatro quadrados tem uma rea de 3 cm2 , qual a rea dessa
gura?
...........................................
.
.
..........................................
.
.
.
.
.
.
.
.
.
.
.
.
.
.
.
.
.
.
.
.
.
.
.
...........................................
.
.
.
...........................................
.
.
..........................................
.
.
.
..........................................
.
.
.
.
.
.
.
.
.
.
.
.
.
.
.
.
.
.
.
.
.
.
.
.
.
.
.
.
.
.
.
.
.
.
.
.
.
.
.
.
.
.
.
.
.
.
.
.
.
.
.
.
.
.
.
.
.
.
.
.
.
.
.
.
.
.
.
.
.
.
.
.
.
.
.
.
.
.
.
.
.
.
.
.
.
.
.
.
.
.
.
.
.
.
.
.
.
.
.
.
.
.
.
.
.
.
.
.
.
.
.
.
.
.
.
.
.
.
.
.
.
.
.
.
.
.
.
.
.
.
.
.
.
.
.
.
.
.
.
.
.
.
.
.
.
.
.
.
.
.
.
.
.
.
.
.
.
.
.
...........................................
.
.
.
.
.
.
.
..........................................
.
.
.
.
.
.
.
.
.
.
.
.
.
.
.
.
.
.
.
.
.
.
.
.
.
.
.
.
.
.
.
.
.
.
.
.
.
.
.
.
.
.
.
.
.
.
.
.
.
.
.
.
.
.
.
.
.
.
.
.
.
.
.
.
.
.
.
.
.
.
.
.
.
.
.
.
...........................................
................................
.
.
.
.
.
.
.
.
.
.
.
.
.
...........
.
.
.
.
.
.
.
.
.
.
.
.
.
.
.
.
.
.
.
.
.
.
.
.
.
.
.
.
.
.
.
.
.
.
.
.
.
.
.
.
.
.
.
.
.
.
.
.
.
.
.
.
.
.
.
.
.
.
.
.
.
.
.
.
.
.
..........................................
.
.
..........................................
.
.
.
.
.
.
.
.
...........................................
.
...........................................
.
.
.
.
.
.
.
.
.
.
.
.
.
.
.
.
.
.
.
.
.
.
.
.
.
.
.
.
.
.
.
.
.
.
.
.
.
.
.
.
.
.
.
.
.
.
.
.
.
.
.
.
..........................................
...........................................
.

147. O o de arame Ernesto formou a gura abaixo com um o de arame, em que cada
segmento de reta tem o comprimento do dimetro dos semicrculos.
q

.....
.....
.........
.........
... ....
... ....
..
..
.
.
...........
...........
...........
...........
...........
...........
.
.
.
.
...........
.
............
..
... ....
.........
....

Qual das guras abaixo ele pode formar com esse mesmo o de arame, cortando-o ou
no, mas sem dobr-lo ou desdobr-lo?
(a)

...
........
.... .....
.
..
.
.
.................................
................................
.
.
.
.
.
.
..
.
..
... ...
... ....
.........
.........
.....
....

(b)

............
.
...
............
.
... ...
.
...
.
.
.
.
.
.
.
.
.
.
.
.
... ...
.... .. .
... ... ...
.
............
.
...

...
...
...
........
..........
..........
.... .....
.... .....
.... .....
.
..
.
.
.
............
............
.
...........
...........
.
...........
...........
.
.
.
.
.
.
.
..
... ...
.........
.....

(c)

(d)

.......
.........
.. ...
..
.
.
...........
.
.
...........
.
.
.
.
.
. ..... .
.
. ....... .
.
..
.
.
..
.
.... ....
.
.
...........
.
.
...........
.
.
.
.
..
.
... ....
........
....

(e)

.......
.........
.. ...
..
.
.
...........
.
.
...........
..
.
..
. .. ....
.
. ..........
.
. ....... .
.
..
.
.
...
.
.... ....
.
.
.
...........
.
.
.
...........
.
.
.
.
..
.
... ....
........
....

148. Sequncia de fsforos Quantos fsforos so necessrios para formar o oitavo termo
da sequncia cujos trs primeiros termos esto dados?
rr

r
r

r
r

.
..................
...................
. ..
.
.
.
.
.
. ....
.
. ......
.
.
..
.
.
.
.
.
. ......
.
.
....
.
.
.
.
.
. .....
....................
.
.
.
.................
.

(a) 21

(b) 24

rr

rr
r

r
r

r
r

.
..............................
. .
...............................
.
.
.
.
.
.
.
.
.
. ......
.
.
. ........
.
.
.
.
.
.
.
.
.
.
.
.
.
. ......
.
.
.
.
.
.
.
.
. ......
.
.
.................................
.
.
.
.
.
..............................
.
.

(c) 27

rr

rr
r

rr
r

r
r

r
r

..........................................
.
...........................................
.
.
. .
.
.
.
. ..
.
.
.
. ...
.
.
.
. ...
.
.
.
.
....
.
.
.
.
..
.
.
.
.
.
.
.
.
.
.
.
. .....
.
.
.
. ......
.
.
.
. .... .
.
.
.
.
.
.
.
.
.
.
.
.
............................................
.
.
..........................................
.

(d) 30

qqq

(e) 34

149. O trajeto das formiguinhas As formiguinhas Maricota e Nandinha passeiam numa varanda cujo cho
formado por lajotas retangulares de 4 cm de largura por
6 cm de comprimento, conforme indicado na gura.
Maricota parte do ponto M, Nandinha parte do N e,
ambas, andam apenas pelos lados das lajotas, percorrendo o trajeto no sentido indicado na gura.

............................
...........................
.
.
.
.
.
.
........................................
.........................................
.
.
.
.
.
.
.
.
.
.
.
.
.
.
.
.
............................
.
...........................
.
.
.
.
.
.
.
.
.
..............
.
..............
.
.
.
.
.
.
.
.
.
.
.
.
.
.
.
.
.........................................
........................................
.
.
.
.
.
.
.
.
.
.
.
.
.
.
.
.
.
.
.
.
.
.
.
.
.
.
.
.
.
...........................
.
.
............................
.
.
.
.
.
.
.
.
.
.
.
.
..............
.
.
..............
.
.

(a) As duas formiguinhas se encontram depois de andarem uma mesma distncia.


Qual foi essa distncia?
(b) Em que ponto elas se encontraram?
150. A soma 100 A soma de trs nmeros 100, dois so primos e um a soma dos
outros dois.
(a) Qual o maior dos trs nmeros?
22

OBMEP 2010

Nvel 1
(b) D um exemplo de tais trs nmeros.
(c) Quantas solues existem para esse problema?
151. Cdigo de barras Um servio postal usa barras curtas e barras longas para representar seu Cdigo de Endereamento Postal (CEP) composto por oito algarismos, em
que a barra curta corresponde ao 0 (zero) e a longa ao 1. A primeira e a ltima barras
no fazem parte do cdigo e a converso do cdigo dada como segue.
11000
00011
01010
00101
00110

=
=
=
=
=

0
1
2
3
4

01100
10100
00001
10001
10010

=
=
=
=
=

5
6
7
8
9

(a) Escreva o CEP 36470130 na forma de cdigo de barras.


(b) Identique o CEP que representa o cdigo de barras seguinte.

||||||||||||||||||||||||||||||||||||||||||
152. Atletas da escola Numa escola, um quarto dos alunos joga somente vlei, um
tero joga somente futebol, 300 praticam os dois esportes e 1/12 nenhum desses dois
esportes.
(a)
(b)
(c)
(d)

Quantos
Quantos
Quantos
Quantos

alunos
alunos
alunos
alunos

tem a escola?
jogam somente futebol?
jogam futebol?
praticam pelo menos um dos dois esportes?

153. Dzima peridica Obtenha o algarismo da 1 997a casa decimal de cada uma das
1
1
fraes seguintes. (a)
(b)
22
27
154. Ana na corrida Para ganhar uma corrida, Ana precisa completar os ltimos cinco
quilmetros em menos de 20 minutos. Qual deve ser sua velocidade mnima, em km/h?
155. Quadradinhos e o buraco Quantos quadradinhos foram retirados do tabuleiro de
10 20 quadradinhos da gura? Se o lado de cada quadradinho mede 1 cm, qual a
rea e qual o permetro do buraco?

OBMEP 2010

23

Nvel 1
156. Quadrados perfeitos no retngulo Complete as seis casas da tabela dada, colocando um algarismo em cada uma, de modo que os dois nmeros de trs algarismos
formados na horizontal e os trs nmeros de dois algarismos formados na vertical sejam
quadrados perfeitos.
(a) Quais so os nmeros?
(b) Quantas solues existem?
157. Aula de diviso Na aula sobre diviso, a professora pediu que seus alunos colocassem nmeros no lugar das estrelas. Quais so esses nmeros?
.

.
.
38 ............................
......
......
(a)

.
.
75 .....................12......
.......
......
(b)

.
.
.....................3..............
...
..
(c)
7

(d)

.
42 .....................................
...
..

158. Linhas de nibus No ponto de nibus perto de sua casa, Quinzinho pode pegar
os nibus de duas linhas para ir escola. Os nibus de uma linha passam de 15 em 15
minutos e os da outra de 25 em 25 minutos, sendo que s 7h30m da manh os nibus
das duas linhas passam juntos.
(a) A que horas passaro juntos novamente?
(b) Entre as 7h30min da manh e a meia noite, quais so os horrios em que os nibus
passam juntos nesse ponto perto da casa de Quinzinho?
159. Quadrados dentro de um retngulo O retngulo da gura est
dividido em oito quadrados. O lado do menor quadrado mede 1 cm.
(a) Quanto mede os lados dos outros quadrados?
(b) Qual o permetro desse retngulo?

........................................
.
.
.
.
.
.
.
.
.
.
.
.
.
.
.
.
.
.
.
.
.
........................................
. .
. . . .
. . . .
. . . .
. . ............
. .
........................................
.

160. Festa na escola A professora Ana foi comprar po de queijo para homenagear os
alunos premiados na OBMEP, sendo que
cada 100 gramas de po de queijo custam R$ 3,20 e correspondem a 10 pes de
queijo; e
cada pessoa come, em mdia, 5 pes de queijo.
Alm da professora, estaro presentes festa 16 alunos, um monitor e 5 pais de alunos.
A preciso da balana da padaria de 100 gramas.
(a) Quantos gramas de po de queijo ela deve comprar para que cada pessoa possa
comer, pelo menos, cinco pes?
(b) Nesse caso, quanto a professora gastar?
(c) Se cada pessoa comer cinco pes de queijo, sobrar algum po de queijo?

24

OBMEP 2010

Nvel 1
161. Ai que fome Maria est olhando a tabela seguinte.
Salgados
Empada: R$ 3,90
Sanduche: R$ 2,20
Pastel: R$ 2,00

Bebidas
Refrigerante: R$ 1,90
Suco: R$ 1,20
Refresco: R$ 1,00

Doces
Sorvete: R$ 1,00
Bombom: R$ 0,50
Cocada: R$ 0,40

Maria possui 5 moedas de 50 centavos, 7 moedas de 25 centavos, 4 moedas de 10


centavos e 5 moedas de 5 centavos.
(a) Quantos reais Maria possui?
(b) Se Maria precisa guardar 90 centavos para a passagem de nibus, quais os possveis
lanches que incluam um salgado, uma bebida e um doce ela poder pedir?
162. Advinhe Tenho alguns nmeros naturais cujo nico divisor comum 1. Se eu somar
50 a cada um deles, encontro nmeros de dois algarismos. Se eu subtrair 32 de cada um
deles, tambm encontro nmeros naturais de dois algarismos. Quais so os nmeros
que eu tenho?
163. Produto de consecutivos Dentre os nmeros 712, 1 262 e 1 680, qual o nico que
pode ser escrito como um produto de quatro nmeros naturais consecutivos?
164. Palndromos O ano de 2002 um palndromo porque
no se altera quando for lido da direita para a esquerda.

373 e 1 221
foram anos palndromos.

(a) Qual ser o prximo ano palndromo depois de 2002?


(b) O ltimo ano palndromo, 1991, foi mpar. Quando ser o prximo ano palndromo
mpar?
(c) O ltimo ano palndromo primo ocorreu h mais de 1 000 anos, em 929. Quando
ocorrer o prximo ano palndromo primo?
165. O maior MDC Quais so os seis nmeros de dois algarismos cujo mximo divisor
comum o maior possvel?
166. Quantidade de gua na Terra A Terra tem, aproximadamente, um volume de
1 360 000 000 km3 de gua, que se distribui entre os oceanos, os mares, as geleiras, as
regies subterrneas (os aquferos), os lagos, os rios e a atmosfera. Somente a gua
encontrada nesses trs ltimos itens oferece um acesso fcil ao consumo humano. Com
estes dados, complete a tabela a seguir.
Especicaes
gua salgada
gua doce
Gelo
gua subterrnea
Lagos e rios
Vapor de gua

Volume de gua (km3 )

Percentual
97%

Forma decimal do percentual

40 000 000
1,8%
0,00960
250 000
0,00001
OBMEP 2010

25

Nvel 1
167. Balas De quantas formas podemos repartir 14 balas idnticas entre trs crianas de
modo que cada criana receba, no mnimo, trs balas?
168. Minutos Uma prova de Matemtica comea s 12h35min e tem uma durao de
5
4 horas. A que horas termina a prova?
6
169. Menor nmero Qual o menor nmero de cinco algarismos divisvel por 4 que se
pode formar com os algarismos 1, 2, 3, 4 e 9?
170. Contas do papagaio Antnio tem um papagaio que faz contas fantsticas com
nmeros inteiros. Quando Antnio sopra certos nmeros em seu ouvido, o papagaio
multiplica esse nmero por 5, depois soma 14, da divide o resultado por 6 e, nalmente,
subtrai 1, gritando o resultado em seguida. Entretanto, o papagaio no sabe nada sobre
decimais, de modo que, s vezes, ca sem poder gritar resposta alguma.
(a) Se Antnio soprar o nmero 8, qual nmero o papagaio gritar?
(b) Se o papagaio gritou 3, qual foi o nmero que Antnio soprou em seu ouvido?
(c) Porque o papagaio nunca grita o nmero 7?
(d) Quais so os nmeros que, soprados por Antnio, provocam uma resposta do
papagaio?
171. Soma maior do que 34 Quantos nmeros de quatro algarismos existem cuja soma
dos algarismos seja maior do que 34?
172. Nenhum 1 Roberto quer escrever o nmero 111 111 como um produto de dois
nmeros, nenhum dos quais terminado em 1. Isso possvel? Por qu?
173. Nmeros equilibrados Um nmero dito equilibrado se um de seus algarismos
a mdia aritmtica dos outros. Por exemplo, 132, 246 e 777 so equilibrados. Quantos
nmeros equilibrados de trs algarismos existem?
174. Nmeros primos Quais so os nmeros cujos triplos somados com 1 do um
nmero primo entre 70 e 110?
175. Quadro moderno Para fazer um quadro bem moderno para
sua escola, Roberto divide uma tela quadrada em oito partes com
quatro faixas de mesma largura e uma diagonal, como na gura. Ele
pinta o quadro de azul e verde, de modo que duas partes vizinhas
sempre tenham cores diferentes. No nal, ele repara que usou mais
verde do que azul. Que frao do quadro foi pintada de azul?
176. Encontro de amigos Embora eu tenha certeza de que meu relgio est 5 minutos
adiantado, na realidade ele est 10 minutos atrasado. Por outro lado, o relgio do meu
amigo est realmente 5 minutos adiantado, embora ele pense que seu relgio esteja
certo. Ns marcamos um encontro para as 10 horas e planejamos chegar pontualmente.
Quem chegar em primeiro lugar? Depois de quanto tempo chegar o outro?
26

OBMEP 2010

Nvel 1
177. Trabalho comunitrio Uma classe tem 22 alunos e 18 alunas. Durante as frias,
60% dos alunos dessa classe foram prestar trabalho comunitrio. No mnimo, quantas
alunas participaram desse trabalho?
(a) 1

(b) 2

(c) 4

(d) 6

(e) 8

178. rea de trapzios Unindo quatro trapzios


idnticos, que tm lados no paralelos iguais e
bases medindo 50 e 30 cm, como o da gura dada,
podemos formar um quadrado de 2 500 cm2 de
rea, que tem um buraco quadrado no meio.
Qual a rea, em cm2 , de cada um dos quatro
trapzios?
(a) 200

(b) 250

(c) 300

(d) 350

(e) 400

179. Adivinhao Pensei em dois nmeros de dois algarismos, que no possuem algarismos em comum, sendo um o dobro do outro. Alm disso, os algarismos do menor
nmero so a soma e a diferena dos algarismos do maior nmero. Quais so os dois
nmeros?
180. Dezoito nmeros consecutivos Escreva dezoito nmeros consecutivos de trs
algarismos e verique que pelo menos um deles divisvel pela soma de seus algarismos.
Isso sempre verdade. Ou seja, se voc escrever dezoito nmeros consecutivos de trs
algarismos, ento pelo menos um deles ser divisvel pela soma de seus algarismos.
Mostre esse fato.
181. Completar uma tabela Descubra a regra utilizada para as casas j preenchidas e
complete a tabela. Qual o valor de A?
0
1
2
3
4

1
2

2
5

3
10

182. Procurando mltiplos de 9 Consideremos um conjunto formado por dez nmeros


naturais diferentes. Se calcularmos todas as diferenas entre esses nmeros, pelo menos
uma dessas diferenas ser um mltiplo de 9?
183. Correndo numa praa Um atleta costuma correr
15,5 km ao redor de uma grande praa retangular de
900 600 m. Ele inicia a corrida sempre do ponto P, situado
a 550 m de um dos vrtices, correndo no sentido horrio, como
mostra a gura. Em que ponto da praa ele para?
184. Ovos para um bolo Uma doceira foi ao mercado comprar ovos para fazer 43 bolos,
todos com a mesma receita, que requer menos do que nove ovos. O vendedor repara
OBMEP 2010

27

Nvel 1
que se tentar embrulhar os ovos que a doceira comprou em grupos de dois, ou de trs,
quatro, cinco, ou seis ovos, sempre sobra um ovo. Quantos ovos ela usa em cada bolo?
Qual o menor nmero de ovos que a doceira vai gastar para fazer os 43 bolos?
185. Cortando uma cartolina Uma folha retangular de cartolina foi cortada ao longo
de sua diagonal. Num dos pedaos obtidos, foram feitos dois cortes paralelos aos dois
lados menores, pelos pontos mdios desses lados. Ao nal, sobrou um retngulo de
129 cm de permetro. O desenho dado indica a sequncia de cortes.
.
...........................................................
...........................................................
.
.
.
.
.
.
.
.
.
.
.
.
.
.
.
.
.
.
.
.
.
.
.
.
.
.
.
.
.
.
.
.
.
.
.
.
.
.
.
.
.
.
.
.
.
.
.
.
.
.
.
.
.
.
.
.
.
.
.
.
.
.
.
.
.
.
.
.
.
.
.
.
.
.
.
.
.
.
.
...........................................................
.
...........................................................
.

.
.
.....
.....
.
.... .
.... .
.
.
.
....
....
.
.
....
....
.
.
.
....
....
.
..
..
.
.
.
....
....
.
..
..
.
.
....
.
....
.
..
..
.
.
....
....
.
.
..
..
.
.
....
....
.
..
.
..
.
.
....
....
.
..
.
..
.
....
.
....
.
..
..
.
.
....
.
....
.
.
..
.
...........................................................
............................................................
..
..

.
..............................
.
.
..............................
.
.
.
.
.
.
.
.
.
.
.
.
.
.
.
.
.
.
.
.
.
.
.
.
.
.
.
.
.
.
.
.
.
.
..............................
.
..............................
.

Qual era o permetro da folha antes do corte?


186. A soma errada A soma ao lado est incorreta. Para corrigi-la, basta
substituir um certo algarismo em todos os lugares que ele aparece na conta
por um outro algarismo. Qual o algarismo errado e qual seu substituto
correto?

742586
+829430
1212016

187. Nmero de cinco algarismos Os algarismos 1, 2, 3, 4 e 5 foram usados, cada um


uma nica vez, para escrever um certo nmero a b c d e de cinco algarismos tal que a b c
divisvel por 4, b c d divisvel por 5 e c d e divisvel por 3. Encontre esse nmero.
188. Tabela misteriosa Complete a tabela 6 6 de tal
modo que, em cada linha e cada coluna, apaream
apenas mltiplos de um dos nmeros

32

40
49
22
15

2, 3, 4, 5, 6, 7, 8, 9, 10, 11 e 12.

24

Alm disso, permitido repetir apenas um nmero


na tabela.

42

189. Habitantes e esporte Numa certa cidade com quase trinta mil habitantes, exatamente dois nonos dos habitantes so homens que praticam esporte somente nos nais
de semana e dois quinze avos so mulheres que praticam esporte somente nos nais
de semana. O nmero de habitantes que no pratica esporte o quntuplo dos que
praticam esporte regularmente. Com esses dados, complete a tabela dada.

No praticam esporte
fem.
8 563

28

masc.
7 582

Praticam esporte
somente nos ns
de semana
fem.
masc.

OBMEP 2010

Praticam
esporte
regularmente
fem. masc.
1 252

Populao
total

Nvel 1
190. Botes luminosos No mecanismo luminoso da gura,
cada um dos oito botes pode acender nas cores verde ou
azul. O mecanismo funciona do seguinte modo: ao ser ligado,
todos os botes acendem a luz azul, e se apertamos um boto,
esse boto e seus dois vizinhos trocam de cor. Se ligarmos o
mecanismo e apertarmos sucessivamente os botes 1, 3 e 5,
qual ser o nmero de luzes verdes que estaro acesas no nal?
(a) 3

(b) 4

(c) 5

(d) 6

1
2

7
4

6
5

(e) 7

191. Qual o nmero? Um nmero de seis algarismos comea por 1. Se deslocamos


esse algarismo 1 da primeira para a ltima posio direita, obtemos um novo nmero
de seis algarismos, que o triplo do nmero de partida. Qual esse nmero?
192. Jardim variado Um jardim retangular de 120 por 80 m foi dividido em seis regies,
conforme indicado na gura, em que N, M e P so pontos mdios dos lados e R divide
o comprimento do lado na razo 1/3. Em cada regio ser plantado um dos seguintes
tipos de or: rosa, margarida, cravo, bem-me-quer, violeta e bromlia, cujos preos,
por m2 , esto indicados na tabela. Quais so as possveis escolhas das ores em cada
regio, de modo a se gastar o mnimo possvel?
Tipo

rosa
margarida
cravo
bem-me-quer
violeta
bromlia

1
4

P
6

Preo por m2
3,50
1,20
2,20
0,80
1,70
3,00

193. O algarismo 3 Luis escreveu a sequncia dos nmeros naturais, ou seja,


1, 2, 3, 4, 5, 6, 7, 8, 9, 10, 11, 12, . . . .
Quando ele escreveu o algarismo 3 pela 25a vez?
194. Soma de potncias Ser o nmero 3444 + 4333 divisvel por 5?
195. Telefonemas Joo mora em Salvador e seus pais em Recife. Para matar a saudade,
ele telefona para seus pais a cada trs dias. O primeiro telefonema foi feito num
domingo, o segundo telefonema na quarta feira seguinte, o terceiro telefonema no
sbado, e assim por diante. Em qual dia da semana Joo telefonou para seus pais
pela centsima vez?
196. O maior produto Com os algarismos de 1 a 5 e um sinal de
multiplicao, Clara forma o produto de dois nmeros, com o sinal
entre eles. Como Clara deve colocar os cartes para obter o maior
produto possvel?
OBMEP 2010

5
29

Nvel 1
197. O caminho da Joaninha Dona Joaninha quer atravessar um ptio ladrilhado com azulejos quadrados numerados, como mostra a gura dada. Ela vai partir do
ponto P e quer chegar ao ponto C, andando somente
ao longo dos lados dos azulejos. Dona Joaninha no
quer ter nmeros primos imediatamente sua direita ao
longo de todo o percurso. Qual o menor percurso que
ela pode fazer?

198. O lugar dos amigos Sete amigos traaram um tringulo, um


quadrado e um crculo. Cada um marcou seu lugar com um nmero
e pronunciou uma frase.
Ana:
Bento:
Celina:
Diana:
Elisa:
Fbio:
Guilherme:

Eu
Eu
Eu
Eu
Eu
Eu
Eu

.
................
........ ...........
..
....
...
..
..
..
..
.
..
.
..
..........................
..
.
.........................
.
.
.
.
.
.
.
.
.
.
.
.
.
.
.
.
.
.
.
.
.
. ..
.
. ....
.
.
.
.
.
.
.
.
.
.
.
.. ..
.
.
.
.... ...
.
.
.
.
.
.
.. .
.
.
.
..
.. ..
.
.
..
..
..
.
.
.. .
.
..
.. ..
.
.. .
.
.. .
.
.
..
..
..
.
.
.. ... .
.
..
.. .
...
.
... .. .
... . .
.
.
.
.
.
...... . ........ ...
...... . ........ ..
.
.
.. ..................... .......
....
.. .............................
.
.
.... .
.. .
...
..
..
..
.
..
.....................................
.....................................
...
...
.
.

2
7
3
4 6
5

no falo coisa alguma.


estou dentro de uma nica gura.
estou dentro das trs guras.
estou dentro do tringulo mas no do quadrado.
estou dentro do tringulo e do crculo.
no estou dentro de um polgono.
estou dentro do crculo.

Encontre o lugar de cada um.


199. Quadrado perfeito? Cada um dos cinco nmeros abaixo tem cem algarismos e
formado pela repetio de um ou dois algarismos, como segue.
N1 = 333333 . . . 3
N2 = 666666 . . . 6
N3 = 151515 . . . 15
N4 = 212121 . . . 21
N5 = 272727 . . . 27
Algum desses nmeros um quadrado perfeito?
200. Preenchendo quadradinhos Complete os quadradinhos com os nmeros 1, 2, 3,
5 e 6.
+

= 4

201. Os trs nmeros Soa brinca de escrever todos os nmeros de quatro algarismos
diferentes que se pode escrever com os algarismos 1, 2, 4 e 7. Ela soma trs desses
nmeros todos diferentes e obtm 13 983. Quais so esses trs nmeros?
202. Preencher uma tabela Jandira deve terminar de preencher uma
tabela 44 que j tem duas casas preenchidas com os nmeros 1 e 2,
conforme indicado na gura. Duas casas so consideradas vizinhas
se tm um vrtice ou um lado em comum. As regras que ela precisa
respeitar so:
30

OBMEP 2010

Nvel 1
uma casa s pode ser preenchida se alguma de suas casas vizinhas j contiver
algum nmero;
ao preencher uma casa, deve-se colocar a soma de todos os nmeros que j constam
em suas casas vizinhas.
Qual o maior nmero que possvel escrever na tabela?
203. Olimpada de Pequim Na Olimpada de Pequim sentaram-se a uma mesa quadrada,
conforme indicado a seguir, as mulheres Maria e Tnia e os homens Juan e David. Todos so atletas e cada um deles pratica um esporte diferente: natao, vlei, ginstica
e atletismo.
(a) Quem pratica a natao estava esquerda de Maria.
(b) Quem pratica ginstica estava em frente a Juan.
(c) Tnia e David sentaram-se lado a lado.
(d) Uma mulher sentou-se ao lado de quem pratica volei.
Qual dos atletas pratica atletismo?
204. Culturas diferentes Jorge, que mora em Recife, se corresponde com seu amigo
ingls Ralph, que mora na Inglaterra. Os dois se compreendem muito bem nas duas
lnguas, mas tm um problema com as datas pois, no Brasil, a data 08/10 signica 08
de outubro e, na Inglaterra, 10 de agosto. Por causa disso, os dois combinaram no se
escrever nos dias em que a data for ambgua. Eles preferem datas como 25/03, que s
pode signicar 25 de maro.
(a) Em quais das datas a seguir Jorge e Ralph no podem se escrever?
(i) 3 de dezembro

(ii) 18 de agosto

(iii) 5 de maio

(b) Quando ocorrem os maiores perodos em que os dois amigos no podem se escrever?
205. Uma liquidao Na liquidao da loja SUPER-SUPER todos os produtos esto
50% mais baratos e, aos sbados, existe ainda um desconto adicional de 20%. Carla
comprou uma cala antes da liquidao, e agora ela se lamenta: No sbado eu teria
economizado R$ 50,40 na cala. Qual era o preo da cala antes da liquidao?
206. Nmero com muitos zeros Se a representa o nmero 0, 000 . . . 000 1, ento qual
das expresses a seguir representa o maior nmero?
(a) 3 + a

(b) 3 a

(c) 3a

(d) 3/a

2009 zeros

(e) a/3

207. Corrida das tartarugas Cinco tartarugas apostaram uma corrida em linha reta e
na chegada a situao foi a seguinte: Sininha estava 10 m atrs de Olguinha e 25 m
frente de Rosinha, que estava 5 m atrs de Elzinha, que estava 25 m atrs de Pulinha.
Qual foi a ordem de chegada?
OBMEP 2010

31

Nvel 1
208. Que memria... Esquecinaldo tem pssima memria para guardar nmeros, mas
tima para lembrar sequncias de operaes. Por isso, para lembrar do seu cdigo
bancrio de cinco algarismos, ele consegue se lembrar que o cdigo no tem algarismos
repetidos, nenhum dos algarismos zero, os dois primeiros algarismos formam uma
potncia de 5, os dois ltimos formam uma potncia de 2, o do meio um mltiplo
de 3 e a soma de todos os algarismos um nmero mpar. Agora ele no precisa mais
decorar o nmero, porque ele sabe que seu cdigo o maior nmero que satisfaz essas
condies. Qual esse cdigo?
209. Uma frao irredutvel Encontre uma frao irredutvel tal que o produto de seu
numerador pelo denominador seja 2 3 4 5 10. Quantas dessas fraes
irredutveis existem?
210. Transformar em decimal Escreva o resultado das seguintes expresses na forma
decimal.
(a) 7

2
5
+ 16
3
12

(b) 5 2

5
3

(c) 1 +
1+

3
1+4

211. Uma sequncia especial Escrevendo sucessivamente os nmeros naturais, obtemos


a sequncia
1 2 3 4 5 6 7 8 9 10 11 12 13 14 15 16 17 18 19 20 21 22 . . .
Qual o algarismo que est na 2 009a posio dessa sequncia?
212. Cortar um retngulo Como podemos cortar um retngulo de 13 por 7 cm em
treze retngulos diferentes sem deixar sobras?
213. Medida de ngulo Na gura dada, AOD e B OY so ngulos retos e a medida de
DOY est entre 40 e 50 . Alm disso, os pontos C e Y esto sobre a reta r, enquanto
D e E esto sobre a reta s. O possvel valor para a medida de AOC est entre
(a)
(b)
(c)
(d)
(e)

30 e 40 ;
40 e 50 ;
50 e 60 ;
40 e 60 ou
no pode ser determinado.

D s

C
O

Y
r

214. Permetros e reas Um quadrado tem 3 + 3 cm de lado e as dimenses de um

retngulo, em centmetros, so 2 e 72 + 3 6. Qual dos dois tem maior rea? E


maior permetro?
A

215. Clculo de ngulo Encontre a medida do ngulo B AD, sabendo que DAC = 39 , AB = AC e
AD = BD.
32

OBMEP 2010

..
......
.
.... ....
..... ......
.... .. ......
...
.... .
...
... ..
....
....
.... .
.... .
.
....
....
.
....
.
....
.
....
.
.
....
.
....
....
.
....
.
....
.
.
....
....
.
....
....
.
.
....
.
....
...
.
....
.
..
.
....
....
.
...
.
...
.
.
....
.
....
.
....
.
....
....
.
....
.
.
....
..
.
....
.
.
..
........................................................................................
.........................................................................................
.. .
....
.
...
..

Nvel 1
216. O caminho da formiga Uma formiga sai de um ponto A, anda 7 cm para a
esquerda, 5 cm para cima, 3 cm para a direita, 2 cm para baixo, 9 cm para a direita,
2 cm para baixo, 1 cm para a esquerda e 1 cm para baixo, chegando no ponto B. Qual
a distncia, em cm, entre A e B?
(a) 0

(b) 1

(c) 4

(d) 5

(e) 7

217. Menino mentiroso Joozinho sempre mente nas teras-feiras, quintas-feiras e sbados e, no restante dos dias da semana, sempre fala a verdade. Um dia, Pedrinho
encontra Joozinho e ocorre o dilogo seguinte.
Pedrinho pergunta: Que dia hoje?
Joozinho responde: Sbado.

Pedrinho pergunta: E que dia ser amanh?


Joozinho responde: Quarta-feira.

Em que dia da semana Pedrinho encontrou Joozinho?


218. Encontre quatro nmeros Observe que os nmeros 1, 2, 3 e 6 tm uma propriedade
notvel: a soma de trs quaisquer deles divisvel pelo quarto nmero. Encontre quatro
nmeros distintos de trs algarismos com essa mesma propriedade notvel.
219. Colando seis tringulos Construa uma gura com seis
tringulos equilteros adjacentes, o primeiro com lado de comprimento 1 cm e os tringulos seguintes com lado igual
metade do lado do tringulo anterior, como indicado na gura
dada. Qual o permetro dessa gura?
220. Os livros da Elisa Elisa tem 24 livros de cincias e outros de matemtica e literatura. Se Elisa tivesse um livro a mais de matemtica, ento um nono de seus livros
seria de matemtica e um quarto de literatura. Se Elisa tem menos do que 100 livros,
quantos livros de matemtica ela possui?
221. Substituindo pela soma Comeando com nmero natural, Mrcio substitui esse
nmero pela soma de seus algarismos, obtendo um novo nmero, com o qual ele repete
o processo, at chegar, nalmente, num nmero de apenas um algarismo. Por exemplo,
Mrcio substitui 1 784 102 por 23 e, em seguida, por 8. Ele tambm aplica esse processo
a listas de N nmeros naturais, substituindo cada nmero da lista pela soma de seus
algarismos, obtendo, assim, uma nova lista de N nmeros, com a qual ele repete o
processo, at chegar numa lista nal de N nmeros, cada um de apenas um algarismo.
(a) Comeando com 32 009 , qual o nmero nal de apenas um algarismo?
(b) Comeando com 172 009 , qual o nmero nal de apenas um algarismo?
(c) Comeando com a lista dos primeiros 20 092 009 nmeros naturais, a lista nal
tem mais algarismos 4 ou 5? Quantos 9 tem a lista nal?

OBMEP 2010

33

Nvel 1
222. Uma brincadeira em sala de aula A professora Raquel inventou a seguinte
brincadeira: escrevendo um nmero inteiro positivo no quadro, acrescente trs unidades
ao nmero se ele for mpar e divida o nmero por dois se ele for par. Essa operao pode
ser feita diversas vezes. A professora est interessada em obter, ao nal, o nmero 1 e
perguntou para a classe: Como obter o nmero 1 aps trs operaes? E aps quatro
operaes? E aps cinco operaes?
223. Calcule a idade Laura e sua av Ana acabaram de descobrir que, no ano passado,
suas idades eram divisveis por 8 e que, no prximo ano, sero divisveis por 7. Vov
Ana ainda no centenria. Qual a idade de Laura?
224. Divises e restos O dobro de um nmero dividido por 5 deixa resto 1. Qual o
resto da diviso desse nmero por 5?
225. Preenchendo o crculo Cada um dos sinais , , , e representa um nmero
de um algarismo. Descubra quais so esses nmeros e complete o nmero que falta no
crculo em branco.
 

 

47 423 282

 

34

1 448

 

OBMEP 2010




Nvel 2

Nvel 2
1. Populao Em 1998, a populao do Canad era de 30,3 milhes. Qual das opes
abaixo representa a populao do Canad em 1998?
(a) 30 300 000

(b) 303 000 000

(c) 30 300

(d) 303 300

(e) 30 300 000 000

2. Rguas em 15 minutos Uma certa mquina capaz de produzir oito rguas por
minuto. Quantas rguas essa mquina consegue produzir em 15 minutos?
(a) 104

(b) 110

(c) 112

(d) 128

(e) 120

3. Alturas iguais Luza, Maria, Antnio e Jlio so irmos. Dois deles tm a mesma
altura. Sabe-se que
Luza maior que Antnio;
Antnio maior do que Jlio;

Maria menor que Luza;


Jlio menor do que Maria.

Quais deles tm a mesma altura?


(a) Maria e Jlio

(c) Antnio e Luza

(b) Jlio e Luza

(e) Antnio e Maria

(d) Antnio e Jlio

4. Unidade O algarismo da unidade do nmero 1 3 5 79 97 113


(a) 1

(b) 3

(c) 5

(d) 7

(e) 9

5. Em que o? A, B, C, D, E, F, G e H so os os de apoio que uma aranha usa para


construir sua teia, conforme mostra a gura. A aranha continua seu trabalho. Sobre
qual o de apoio estar o nmero 118?
(a) B
(b) D
(c) E
(d) G
(e) H

Para resolver as duas prximas questes, utilize as informaes da tabela dada, que
mostra o desempenho das selees do grupo A da Copa do Mundo de 2002. Nessas
partidas de futebol, a equipe vencedora ganha trs pontos e a perdedora no ganha
nem perde pontos; em caso de empate, as duas ganham um ponto.
Seleo
Dinamarca
Senegal
Uruguai
Frana

J
3
3
3
3

V
2
1
0
0

E
1
2
2
1

D
0
0
1
2

OBMEP 2010

GP
5
5
4
0

GC
2
4
?
3

P
7
?
2
1
35

Nvel 2
Legenda: J jogos, V vitrias, E empates, D derrotas, GP gols marcados,
GC gols sofridos, P pontos.
6. Pontos ganhos Quantos pontos obteve a seleo do Senegal?
(a) 3

(b) 4

(c) 5

(d) 6

(e) 7

7. Gols sofridos Quantos gols sofreu a seleo do Uruguai?


(a) 2

(b) 3

(c) 4

(d) 5

(e) 6

8. Qual o ngulo? Na gura, temos B = 50 , sendo AD e


CD as bissetrizes dos ngulos A e C, respectivamente.
Qual a medida do ngulo ADC?

B
50

(a) 90
(b) 100
(c) 115

(d) 122,5

(e) 125

9. Basquete O grco mostra o nmero de pontos


que cada jogador da seleo de basquete da escola
marcou no ltimo jogo. O nmero total de pontos
marcados pela equipe foi
(a) 54
(b) 8
(c) 12
(d) 58
(e) 46
10. Telefone Geni cliente de uma companhia telefnica que oferece o seguinte plano:
tarifa mensal xa de R$ 18,00;

gratuidade em 10 horas de ligaes por ms;

R$ 0,03 por minuto que exceder as 10 horas gratuitas.


Em janeiro, Geni usou seu telefone por 15 horas e 17 minutos e, em fevereiro, por 9
horas e 55 minutos. Qual foi a despesa de Geni com telefone nesses dois meses, em
reais?
(a) 45,51

(b) 131,10

(c) 455,10

(d) 13,11

(e) 4,55

11. rea Na gura dada, temos dois quadrados. O lado do maior mede a + b e o do
menor a. Qual a rea da regio cinza destacada?
36

OBMEP 2010

Nvel 2
(a) b
(b) a + b
(c) a2 + 2ab

a+b

(d) b2
(e) 2ab + b2

12. Comprando sorvete Veja as promoes de dois supermercados:


Supermercado A
6 latas de 3 litros do sorvete QUENTE
R$ 24,00

Supermercado B
Sorvete QUENTE lata de 3 litros
4 latas s R$ 14,00

Joana quer comprar 12 latas de sorvete para a festa de seu aniversrio. Em qual
supermercado ela deve comprar e por qu?
(a) No A, pois economizar R$ 7,00 em relao ao B.
(b) No A, pois economizar R$ 6,00 em relao ao B.
(c) No B, pois economizar R$ 8,00 em relao ao A.
(d) No B, pois economizar R$ 6,00 em relao ao A.
(e) Tanto faz, porque o preo o mesmo nos dois supermercados.
13. Cartolina e barbante Passa-se um barbante atravs dos seis furos de
uma cartolina. A frente da cartolina, com o barbante, mostrada na gura.
Qual das guras a seguir no pode ser o verso dessa cartolina?

(a)

(b)

(c)

(d)

(e)

14. Amigos e fraes Adriano, Bruno, Csar e Daniel so quatro bons amigos. Daniel
no tinha dinheiro, mas os outros tinham. Adriano deu a Daniel um quinto do seu
dinheiro, Bruno deu um quarto do seu dinheiro e Csar deu um tero do seu dinheiro.
Cada um deu a Daniel a mesma quantia. A quantia que Daniel possui agora representa
que frao da quantia total que seus trs amigos juntos possuam inicialmente?
(a)

1
10

(b)

1
4

(c)

1
3

(d)

2
5

(e)

1
2

15. Escolhendo sorvetes Paulo quer comprar um sorvete com quatro bolas em uma
sorveteria que dispe de trs sabores: aa, baunilha e caj. De quantos modos diferentes ele pode fazer essa compra?
(a) 6

(b) 9

(c) 12

(d) 15

OBMEP 2010

(e) 18

37

Nvel 2
16. Peas de um quadrado Pedro montou um quadrado com quatro das cinco peas
abaixo. Qual a pea que ele no usou?

(a)

(b)

(d)

(c)

(e)

17. Paradas de nibus Uma linha de nibus possui 12 paradas numa rua em linha
reta. A distncia entre duas paradas consecutivas sempre a mesma. Sabe-se que a
distncia entre a terceira e a sexta paradas de 3 300 metros. Qual a distncia, em
quilmetros, entre a primeira e a ltima parada?
(a) 8,4

(b) 12,1

(c) 9,9

(d) 13,2

(e) 9,075

18. Desenho Qual dos seguintes desenhos no pode ser feito sem tirar o lpis do
papel e passando apenas uma vez ao longo de cada linha?

(a)

(b)

(c)

(d)

(e)

19. Qual o cubo? Cortamos um canto de um cubo, conforme mostra


a gura.
Qual das representaes a seguir corresponde ao que restou do cubo?

(a)

(b)

(d)

(c)

(e)

20. Quadrado mgico Dizemos que o quadrado abaixo um quadrado


mgico porque a soma dos nmeros de cada linha, de cada coluna e
de cada diagonal sempre a mesma. No caso do quadrado mgico da
gura, essa soma 15.

38

OBMEP 2010

Nvel 2
Complete os cinco nmeros que faltam no quadrado abaixo para que ele seja um
quadrado mgico.

21. Torneio Sete equipes, divididas em dois grupos, participaram do torneio de futebol
do meu bairro. O Grupo 1 foi formado pelas equipes Avaqui, Botgua e Corinense. O
Grupo 2 foi formado pelas equipes Dinossauros, Esquisitos, Flurinthians e Guaran.
Na primeira rodada do torneio, cada equipe enfrentou cada uma das equipes do seu
grupo exatamente uma vez. Na segunda rodada do torneio, cada equipe enfrentou
cada uma das equipes do outro grupo exatamente uma vez.
(a) Quantas partidas foram disputadas na primeira rodada no Grupo 1?
(b) Quantas partidas foram disputadas na primeira rodada no Grupo 2?
(c) Quantas partidas foram disputadas na segunda rodada?
22. Truque numrico Voc j viu um truque numrico? Aqui vo os passos de um
truque numrico:
(i) Escolha um nmero qualquer.
(ii) Multiplique-o por 6.
(iii) Do resultado subtraia 21.
(iv) Divida esse novo resultado por 3.
(v) Desse ltimo resultado subtraia o dobro do nmero que voc escolheu.
(a) Experimente essa sequncia de cinco passos trs vezes, iniciando cada vez com
um nmero diferente. Qual foi o resultado de seu experimento?
(b) A seguir, usando a letra x para representar o nmero que voc escolheu no primeiro
passo, mostre por que os resultados do item (a) no so apenas uma coincidncia,
mas sim um fato matemtico.
23. Jogando sinuca Na gura abaixo vemos uma mesa de sinuca quadriculada e parte
da trajetria de uma bola, tacada a partir de um canto da mesa, de modo que, sempre
que a bola bater em uma das beiradas da mesa, ela segue seu movimento formando
ngulos de 45 com a beirada.
(a) Em qual das quatro caapas a bola cair?
(b) Quantas vezes a bola bater nas beiradas da mesa
antes de cair na caapa?
(c) A bola seguir pela diagonal de quantos desses
quadrados durante sua trajetria?
OBMEP 2010

39

Nvel 2
A

24. Tringulo issceles Na gura, o tringulo ABC issceles,


com B AC = 20 .

20

Sabendo que BC = BD = BE, determine a medida do ngulo


B DE.

E
D
B

25. Pesando moedas So dadas quatro moedas aparentemente iguais, das quais trs
so verdadeiras e uma falsa. As trs verdadeiras tm o mesmo peso e a falsa tem um
peso diferente das verdadeiras, mas no se sabe se a moeda falsa mais leve ou mais
pesada do que as verdadeiras.
Mostre que possvel determinar a moeda falsa empregando somente duas pesagens
em uma balana de dois pratos.
Observao: Numa balana de dois pratos s podemos comparar os pesos colocados
nos dois pratos: a balana s pode car equilibrada ou, ento, pender para o lado mais
pesado.
26. Nmeros binomiais Os quadrados em branco da gura devem ser preenchidos com
nmeros de tal modo que cada nmero, a partir da segunda linha, seja igual soma
dos dois nmeros vizinhos da linha imediatamente superior. Por exemplo, o nmero
da primeira casa da segunda linha 11, porque 11 = 5 + 6. Qual o nmero que vai
aparecer no quadrado indicado com ?
(a) 4

(b) 6

11

(c) 9
(d) 15

60

(e) 10

27. Costuras da bola Uma bola de futebol feita com 32 peas de couro. Dessas peas
12 so pentgonos regulares idnticos e as outras 20 so hexgonos, tambm regulares
e idnticos. Os lados dos pentgonos so iguais aos lados dos hexgonos. Para unir
dois lados de duas dessas peas necessria uma costura. Quantas so as costuras
necessrias para fazer uma bola?
(a) 60

(b) 64

(c) 90

(d) 120

(e) 180

28. Razo de reas A gura ao lado mostra uma grade formada por
quadrados de 1 cm de lado. Qual a razo entre a rea sombreada
e a no sombreada?
1
1
2
2
1
(b)
(c)
(d)
(e)
(a)
4
5
6
5
7
40

OBMEP 2010

Nvel 2
29. S sorvete Em um quente dia de vero, 64 crianas comeram, cada uma, um
sorvete pela manh e outro tarde. Os sorvetes eram de quatro sabores, abacaxi,
banana, chocolate e doce de leite. A tabela dada mostra quantas crianas consumiram
um desses sabores pela manh e outro tarde. Por exemplo, o nico nmero 7 que
aparece na tabela indica que sete crianas tomaram sorvete de banana pela manh e
de chocolate tarde.

Quantas crianas tomaram sorvetes de sabores diferentes nesse dia?


(a) 58

(b) 59

(c) 60

(d) 61

(e) 62

30. Brincando com tabuleiro Camila e Lara tm, cada uma, um tabuleiro 4 4.
Comeando com ambos tabuleiros em branco, elas fazem uma brincadeira com o desdobramento seguinte.
Camila, escondida de Lara, pinta de preto algumas casas de seu tabuleiro.

Ainda em seu tabuleiro, Camila escreve em cada casa o nmero de casas vizinhas
que esto pintadas de preto (duas casas distintas so vizinhas se possuem um lado
ou um vrtice em comum).
Camila copia os nmeros escritos em seu tabuleiro no tabuleiro de Lara.

Lara deve adivinhar, a partir dos nmeros escritos em seu tabuleiro, quantas so
as casas pretas do tabuleiro de Camila.

Por exemplo, se Camila pintar seu tabuleiro como o da gura esquerda, ento ela
coloca os nmeros no tabuleiro de Lara como na gura direita.

Quantas foram as casas que Camila pintou se o tabuleiro de Lara tiver os nmeros do
tabuleiro a seguir?
(a) 3

(d) 6

(b) 4

(e) 7

(c) 5

OBMEP 2010

41

Nvel 2
31. Cartes numerados Larissa e Jorge esto jogando com cartes numerados de 1 a
6 que devem ser colocados nas casas do tabuleiro a seguir de tal modo que formem um
nmero de seis algarismos.

Jorge coloca o primeiro carto e, a seguir, as jogadas so alternadas entre os dois.


O objetivo de Larissa obter o maior nmero possvel e o de Jorge obter o menor
nmero possvel. Larissa tem os cartes com os algarismos 1, 3 e 5 e Jorge tem os
cartes com os algarismos 2, 4 e 6. Se os dois jogadores forem espertos, qual o
nmero que aparecer ao nal do jogo?
(a) 254 361

(b) 253 416

(c) 251 634

(d) 256 134

(e) 251 346

32. Faltam balas Uma professora tem 237 balas para dar a seus 31 alunos. Qual o
nmero mnimo de balas a mais que ela precisa conseguir para que todos seus alunos
recebam a mesma quantidade de balas, sem sobrar nenhuma?
(a) 11

(b) 20

(c) 21

(d) 31

(e) 41

33. Artesos de braceletes Um arteso comea a trabalhar s 08h e produz seis


braceletes a cada 20 minutos; j seu auxiliar comea a trabalhar uma hora depois e
produz oito braceletes do mesmo tipo a cada meia hora. O arteso para de trabalhar s
12h, mas avisa ao seu auxiliar que dever continuar trabalhando at produzir o mesmo
nmero de braceletes que ele. A que horas o auxiliar ir parar de trabalhar?
(a) 12h

(b) 12h30min

(c) 13h

(d) 13h30min

(e) 14h30min

34. Girando um pentgono Qual gura ser obtida se girarmos no sentido


horrio o pentgono regular por um ngulo de 252 em torno do seu centro?
Observao: o sentido horrio o sentido em que giram os ponteiros de um relgio; no
caso do pentgono, isso est indicado pela seta no desenho.

(a)

(b)

(c)

(d)

(e)

35. rea em funo da diagonal O permetro de um retngulo mede 100 cm e a


diagonal mede x cm. Qual a rea desse retngulo em funo de x?
(a) 625 x2

(b) 625

x2
2

(c) 1 250

x2
2

(d) 225

x2
2

(e) 2 500

x2
2

36. Valor de uma quadrtica Se x + y = 8 e xy = 15, qual o valor de x2 + 6xy + y 2 ?


(a) 64

(b) 109

(c) 120

(d) 124

(e) 154

37. ngulos em funo de x Na gura esto indicadas, em graus, as medidas de


alguns ngulos em funo de x. Quanto vale x?
42

OBMEP 2010

Nvel 2
(a) 6

5x

(b) 12
(c) 18
(d) 20
(e) 24

2x

3x

6x
4x

38. Operao diferente Se m e n so inteiros maiores do que zero e se m < n,


denimos mn como a soma dos inteiros entre m e n, incluindo m e n. Por exemplo,
2226
58 = 5 + 6 + 7 + 8 = 26. Qual o valor de
?
46
(a) 4
(b) 6
(c) 8
(d) 10
(e) 12
39. Taxi caro O preo de uma corrida de txi de R$ 2,50 xos (a bandeirada)
mais R$ 0,10 por 100 metros rodados. Tenho apenas R$ 10,00 no bolso. Logo, tenho
dinheiro para uma corrida de, no mximo, quantos quilmetros?
(a) 2,5

(b) 5,0

(c) 7,5

(d) 10,0

(e) 12,5

40. Mltiplos de 3 ou 4 Quantos nmeros entre 1 e 601 so mltiplos de 3 ou mltiplos


de 4?
(a) 100

(b) 150

(c) 250

(d) 300

(e) 430

41. Lados de um paraleleppedo Se x e y so nmeros inteiros positivos tais que


xyz = 240, xy + z = 46 e x + yz = 64, qual o valor de x + y + z?
(a) 19

(b) 20

(c) 21

(d) 24

(e) 36

42. Pontos da reta Na reta dada esto representados os seis nmeros a, b, m, n, p e q,


1
alm dos nmeros 0, , 1 e 2.
2

1
2a b

m
1

n
2

Ento os nmeros que melhor representam a + b, a b e ab so, respectivamente:


(a) m, p e q;

(b) m, q e p;

(c) n, q e p;

(d) n, p e q;

(e) q, m e p.

43. Velocidades Numa corrida de carros, um piloto percorreu trs trechos: um de


240 km, um de 300 km e um de 400 km. O piloto sabe que as velocidades mdias
nesses trechos foram 40 km/h, 75 km/h e 80 km/h, mas no se lembra qual dessas
velocidades corresponde a cada um desses trechos. Podemos garantir que o tempo
total em horas gasto pelo piloto para percorrer os trs trechos foi:
(a) menor do que ou igual a 13 horas;
OBMEP 2010

43

Nvel 2
(b) maior do que ou igual a 13 horas e menor do que ou igual a 16 horas;
(c) maior do que ou igual a 16 horas e menor do que ou igual a 17 horas;
(d) maior do que ou igual a 15 horas e menor do que ou igual a 18 horas;
(e) maior do que ou igual a 18 horas.
44. Comprimento de diagonal Do quadrado ABCD foram
cortados os tringulos issceles sombreados, como na gura,
restando o retngulo P QRS. Sabendo que a rea total do que
foi cortado mede 200 cm2 , qual o comprimento de P R, em
cm?

(a) 200 (b) 200 (c) 800 (d) 25


(e) 88

B
Q

S
D

45. Diviso de nmeros grandes Determine o valor de 123 456 123 456 10 000 001.
46. Refrigerante no cinema Toda vez que Joozinho vai ao cinema, ele toma dois
refrigerantes. Ele gastou toda sua mesada de R$ 50,00 indo ao cinema seis vezes e
tomando um total de 20 refrigerantes, incluindo os que ele tomou quando foi ao cinema.
Se Joozinho tivesse tomado s um refrigerante cada vez que foi ao cinema, com essa
economia ele poderia ter ido ao cinema mais uma vez, tomando um refrigerante tambm
nessa ocasio. Em relao ao preo do ingresso do cinema e o preo do refrigerante,
podemos armar que
(a) o preo do ingresso o triplo do preo do refrigerante;
(b) o preo do ingresso o qudruplo do preo do refrigerante;
(c) o preo do ingresso o quntuplo do preo do refrigerante;
(d) o ingresso R$ 6,00 mais caro do que o refrigerante;
(e) o ingresso R$ 5,00 mais caro do que o refrigerante.
47. Diviso de potncias Qual o quociente de 5050 por 2525 ?
(a) 2525

(b) 1025

(c) 10025

(d) 225

(e) 2 2525

48. Palitos de dois tamanhos Voc possui apenas palitos com 6 e 7 cm de comprimento. Qual o nmero mnimo desses palitos que so necessrios para cobrir um
segmento de reta com 2 metros?
(a) 29

(b) 30

(c) 31

(d) 32

(e) 33

49. Maior raiz Qual a maior raiz da equao (x 37)2 169 = 0?


(a) 39

(b) 43

(c) 47

(d) 50

(e) 53

50. Mquina com visor Uma certa mquina tem um visor, onde aparece um nmero
inteiro x, e duas teclas, A e B. Quando se aperta a tecla A, o nmero x do visor
substitudo por 2x + 1. Quando se aperta a tecla B, o nmero x do visor substitudo
por 3x 1. Qual o maior nmero de dois algarismos que pode ser obtido apertando
44

OBMEP 2010

Nvel 2
alguma sequncia das teclas A e B a partir do nmero 5 no visor?
(a) 85

(b) 87

(c) 92

(d) 95

(e) 96

51. Quadrado mgico parcial Num quadrado mgico, a soma dos


trs nmeros de cada linha, coluna ou diagonal sempre a mesma.
Dado o quadrado mgico ao lado, parcialmente preenchido, qual
deve ser o valor de x?
(a) 20
(b) 22
(c) 23
(d) 25
(e) 27
52. rea do retngulo Um retngulo ABCD est dividido
em quatro retngulos menores. As reas de trs deles esto
indicadas na gura dada. Qual a rea do retngulo ABCD?
(a) 80

(b) 84

(c) 86

(d) 88

1
26

14

x
13

A
16

(e) 91

12

27
C

53. Lado do quadrado Quatro peas iguais, em forma de tringulo retngulo, foram
dispostas de dois modos diferentes, como mostram as guras dadas. Os quadrados
ABCD e EF GH tm lados respectivamente iguais a 3 cm e 9 cm. Determine a
medida do lado do quadrado IJKL.
H
I
D

A
B

54. Maior nmero Qual o maior dentre os nmeros dados?


(a) 2 0 2 006

(b) 2 0 + 6

(c) 2 + 0 2 006

(d) 2 (0 + 6)

(e) 2 006 0 + 0 6

55. Operao O smbolo representa uma operao especial com nmeros; alguns
exemplos so 2 4 = 10, 3 8 = 27, 4 27 = 112 e 5 1 = 10. Quanto vale 4 (8 7)?
(a) 19

(b) 39

(c) 120

(d) 240

(e) 260

56. Terceiro lado Se dois lados de um tringulo medem 5 e 7 cm, ento o terceiro lado
no pode medir quantos centmetros?
(a) 11

(b) 10

(c) 6

(d) 3

(e) 1

3 2
1

= , quanto vale ?
24 8 3
6
(b) 21
(c) 23
(d) 25
(e) 29

57. Asterisco Se
(a) 20

OBMEP 2010

45

Nvel 2
a

58. Expresses algbricas O que representam, geometricamente, na gura dada, as expresses


a2 + 1,5 a

1,5

e
4a + 3.
59. Faixa decorativa A gura dada composta de tringulos retngulos issceles, todos
congruentes. Qual a rea, em cm2 , da parte sombreada?
(a) 20

(d) 45

(b) 25

(e) 50
30 cm

(c) 35

60. Bicicleta e chocolate Se eu der duas barras de chocolate para Tio, ele me empresta
sua bicicleta por 3 horas. Se eu lhe der 12 bombons, ele me empresta a bicicleta por 2
horas. Amanh, eu lhe darei uma barra de chocolate e 3 bombons. Por quantas horas
ele me emprestar a bicicleta?
(a) 1/2

(b) 1

(c) 2

(d) 3

(e) 4

61. Retas paralelas? Na gura dada, as retas EC e F D sero paralelas?


A
62

E
F
42
48

C
28

62. Menor nmero Se x > 5, ento qual dos nmeros dados o menor?
(a) 5/x

(b) 5/(x + 1)

(c) 5/(x 1)

(d) x/5

(e) (x + 1)/5

63. rea de quadrado O quadrado ST U V formado de um


quadrado limitado por 4 retngulos iguais. O permetro de cada
retngulo 40 cm. Qual a rea, em cm2 , do quadrado ST U V ?
(b) 200

46

(c) 160

(a) 400

(e) 80

(d) 100

OBMEP 2010

Nvel 2
64. Operando fraes
1 1 1 1 1 1 1 1 1
,
,
,
e .
2 2 3 3 4 4 5 5 6
1 1
1
1
1
(b) Deduza de (a) o valor da soma + +
+
+
.
2 6 12 20 30
1
1
1
1
1
1 1
+
+
+
+ +
.
(c) Calcule a soma + +
2 6 12 20 30 42
999 000
(a) Calcule as diferenas 1

65. ngulos e permetro Calcule os ngulos que no esto indicados e o permetro


da gura, sabendo que BD = BC e
DBC = B CD.

B
26

120 m
26
30
E

C
115 m

70

226 m

66. Desigualdade racional Quais so os valores de x que satisfazem a desigualdade


1
< 4?
x2
9
9
(c) x < 2 ou x >
(e) x < 2
(a) x >
4
4
9
(b) 2 < x e x <
(d) x < 2
4
67. Desigualdade dupla Quantos nmeros inteiros e positivos satisfazem a dupla inequao 2 000 < n(n 1) < 2 005?
(a) 1

(b) 2

(c) 3

(d) 4

(e) 5

68. Dimetro do crculo Na gura, O o centro do crculo


e AB = 5 cm. Qual o dimetro desse crculo?

B
O
A

69. Falta um ngulo Na gura dada, T U = SV.


Quanto vale o ngulo S V U, em graus?
(a) 30

(d) 65

(b) 50

(e) 70

5
4

U
30
T

75

(c) 55

50
S

70. Caf, bolo e gato Dez minutos antes de colocar o bolo no forno, coloquei meu
gato para fora da casa. O bolo deve cozinhar por 35 minutos, portanto coloquei o
despertador para tocar 35 minutos aps colocar o bolo no forno. De imediato z um
caf para mim, o que me tomou 6 minutos. Trs minutos antes de acabar de beber
o caf, meu gato entrou em casa. Isso foi 5 minutos antes do despertador tocar. O
telefone tocou no meio do tempo entre eu acabar de fazer o caf e o gato entrar em
casa. Falei ao telefone por 5 minutos e desliguei. Eram, ento, 3h59min da tarde.
OBMEP 2010

47

Nvel 2
(a) A que horas coloquei meu gato para fora?
(b) O despertador tocou quantos minutos depois de colocar o gato para fora?
(c) Por quanto tempo o gato j estava fora de casa quando o telefone tocou?
71. Muitos ngulos Quais guras esto corretas?

Figura I

72
112
18

95

63

Figura II

45

62

Figura III

44

29

72. Sinal de produto e de quociente a, b, c e d so quatro nmeros no nulos tais que


os quocientes
b
11
18
a
,
,
e
5
7a
abc
abcd
so todos positivos. Determine os sinais de a, b, c e d.
73. Sinais e radicais Quais dos nmeros dados so negativos?

(a) 10 3 11
(c) 18 5 13
(e) 10 26 51

(b) 3 11 10
(d) 51 10 26
74. ngulos entre retas Sabe-se que as retas r e s so paralelas. Encontre os ngulos
x e y.
60

x
y
80

48

OBMEP 2010

Nvel 2
75. Variao de temperatura A tabela dada
mostra as temperaturas mximas e mnimas em
centgrados durante cinco dias seguidos em certa
cidade. Em qual dia ocorreu a maior variao de
temperatura?

Dia
a
2 feira
3a feira
4a feira
5a feira
6a feira

Mx.(o C)
7
0
2
9
13

Mn.(o C)
12
11
15
8
7

76. Ordenando fraes Qual dos nmeros ca entre 2/5 e 3/4?


(a) 1/6

(b) 4/3

(c) 5/2

(d) 4/7

(e) 1/4

77. Frao de rea A gura mostra um retngulo maior


dividido em 18 retngulos menores, todos com a mesma
largura. Qual a frao do retngulo maior que representa a parte em cinza?
78. Uma a mais! Dentre as nove fraes
5
,
4

17
,
6

5
,
4

10
,
7

2
,
3

14
,
8

1
,
3

5
3

3
2

temos oito com as propriedades seguintes.


2 fraes cuja soma

2 fraes cuja diferena

2
5

2 fraes cujo produto

2
5

2
5

2 fraes cujo quociente

2
5

Encontre a frao que est sobrando.


K

79. Qual o ngulo? No tringulo KLM temos


KL = KM, KT = KS e LKS = 30 . Qual a
medida do ngulo x = T SM ?
(a) 10

(b) 15

(c) 20

(d) 25 (e) 30

T
L

x
S

80. Operao circular Dentro dos crculos, escreva os


nmeros inteiros que tornam correta a sucesso de operaes.
81. Pratos e copos Iara possui R$ 50,00 para comprar copos e pratos. Cada copo
custa R$ 2,50 e cada prato, R$ 7,00. Ela quer comprar, no mnimo, 4 pratos e 6 copos.
O que ela pode comprar?
82. Desigualdades de inteiros Quantos so os nmeros inteiros x tais que
5 < x 1 5?
(a) 8

(b) 9

(c) 10

(d) 11
OBMEP 2010

(e) 12
49

Nvel 2
83. Nove quadrados A gura dada mostra nove quadrados. A rea do quadrado A mede 1 cm2 e a do quadrado
B 81 cm2 . Qual a rea, em cm2 , do quadrado I?
(a) 196

(d) 324

(b) 256

(e) 361

(c) 289

84. Muitas medalhas Andr, Bruno, Celina e Dalva ganharam, juntos, 21 medalhas
num concurso. Andr foi o que mais ganhou medalhas, Bruno ganhou o dobro de
Celina e Dalva ganhou trs a mais do que Bruno. Quantas medalhas cada um pode
ter ganhado?
85. As somas so quadrados Escreva numa linha os nmeros de 1 a 15 de tal modo
que a soma de quaisquer dois nmeros adjacentes nessa linha seja um quadrado perfeito.
86. rea de uma regio Um retngulo est dividido em
trs regies, conforme indicado na gura. Se as reas de
duas delas medem 24 e 13 cm2 , qual a rea da terceira
regio?
0,00001 (0,01)2 1 000
:
87. Potncias de 10 O valor de
0,001
(a) 101 ;

(b) 102 ;

(c) 103 ;

(d) 104 ;

(e) 1.

88. Diferena de quadrados Se (x + y)2 (x y)2 = 20, ento xy igual a:


(a) 0;

(b) 1;

(c) 2;

(d) 5;

(e) 10.

89. Um quadriltero O quadriltero ABCD da gura um paralelogramo?


.
.
.
.
.
.
.
.
. ..
......
....
....
....
.
..... .
.......
.... ..
.
.
.... .
.
.
....
....
.
.
.
.
....
.
.... 45 .
.
.
....
.
.
....
.
.
....
....
.
.
.
.
....
.
.
....
.
.
....
.
.
....
.
.
....
.
....
.
.
.
....
.
....
.
...
.
.
.
.
.......
.......
.
....
.
.
............ .
............ .
....
... 45..
. .. ....
............
.
.
. ...... ..................
. ......
.
. .... ..............
.
...............
.
.
........
.
.
. .
.
. ..

.
.
.
.
....
.
.
... 115
.
.
.
.
.
.
.
.
.
.
.
.
.
.
.
.
.
.
.
.
......
......

.
..
............
............
....
............
. ..65 .................
. ..
. . ............
..................
.
............

90. Sexta-feira treze Qual o nmero mximo de sexta-feiras treze que podem ocorrer
num ano que no bissexto? Nesse caso, em que dia da semana cai o dcimo dia do
ano?
91. Tringulos com lados inteiros Quantos tringulos existem cujos lados so nmeros inteiros e o permetro mede 12 unidades?
(a) 1
50

(b) 3

(c) 5

(d) 7

(e) 9

OBMEP 2010

Nvel 2
92. Festa de aniversrio Para comemorar seu aniversrio, Ana vai preparar tortas de
pera e de ma. No mercado, uma ma pesa 300 g e uma pera, 200 g. A sacola de
Ana aguenta um peso mximo de 7 kg. Qual o nmero mximo de frutas que ela
pode comprar para poder fazer tortas das duas frutas?
93. Os dois quadrados As medidas em centmetros dos lados de cada um
dos dois quadrados da gura so nmeros inteiros. Se o menor quadrado
tivesse 2 001 cm2 a mais de rea, as reas dos dois quadrados seriam
iguais. Quanto pode medir o lado do maior quadrado?

.
......................................
.
......................................
.
.
.
.
.
.
.
.
.
.
.
.
.
.
.
.
.
.
.
.
.
.
.
.
.
.
.
.
.
.
.
.
.
.
.
.
.
.
.
.
.
.
.
.
.
.
.
.
.
.
.
.
.
.
.
.
.
.
.............
.
.
.
.
.............
.
.
.
.
.
.
.
.
.
.
.
.
.
.
.
.
.
.
.
.
.
.
.
.
.
.
.
.
.
.
.
.
.
.
.
.
.
.
.
.
.
.
.
.
......................................
..........................
.............
.

94. A multiplicao Jlio faz multiplicaes usando apenas os quadrados dos nmeros.
Ele tem que calcular o produto 85 135. Para isso, ele desenha um retngulo de
85 135 mm e, nesse retngulo, traa o maior quadrado possvel; faz o mesmo no
retngulo restante e assim sucessivamente. Dessa maneira, ele obtm oito quadrados
que ele, ento, soma. Desenhe a gura feita por Jlio e escreva 85 135 como a soma
de oito quadrados:
85 135 = 852 + . . . .
95. Expresso fracionria Se
(a) 1;

1
(b) ;
2

(c)

x
xy
= 2, ento
igual a:
y
x
1
;
2

(d) 1;

(e) 2;

96. Diferena e soma de quadrados Calcule:


(a) 1 6782 1 6772

(b) 1 0012 + 1 0002

(c) 19 9992

(d) 2 0012 + 2 0022 + 2 0032

97. Um queijo triangular Osvaldo comprou um queijo em forma de um tringulo


equiltero. Ele quer dividir o queijo igualmente entre ele e seus quatro primos. Faa
um desenho indicando como ele deve fazer essa diviso.
98. Notas de Matemtica Joo e Cludia receberam de volta suas provas de matemtica em que os algarismos das notas foram substitudos por smbolos. A nota de
Joo foi e a de Cludia . Juntos, eles obtiveram
. Alm disso, Cludia
obteve 13 pontos a mais do que Joo. Qual foi a nota de cada um?

99. Operao com raiz quadrada O nmero ( 6 + 2)( 3 2)


3 + 2 igual a:

(a) 3;
(b) 2;
(c) 2;
(d) 1;
(e) 2.
100. Para a escola de bicicleta Ctia sai da escola todos os dias no mesmo horrio
e volta para casa de bicicleta. Quando ela pedala a 20 km/h, ela chega em casa s
16h30m. Se ela pedalar a 10 km/h, ela chega em casa s 17h15m. A que velocidade
ela deve pedalar para chegar em casa s 17h?
101. Distncia na reta Cinco pontos esto sobre uma mesma reta. Quando listamos
as 10 distncias entre dois desses pontos, da menor para a maior, encontramos 2, 4, 5,
7, 8, k, 13, 15, 17 e 19. Qual o valor de k?
OBMEP 2010

51

Nvel 2
102. Nmero mpar Se n um nmero inteiro qualquer, qual dos seguintes um nmero
mpar?
(a) n2 n + 2

(b) n2 + n + 2

(c) n2 + n + 5

(d) n2 + 5

(e) n3 + 5

103. Quatro nmeros inteiros Se quatro inteiros positivos distintos m, n, p e q satisfazem a equao (7 m)(7 n)(7 p)(7 q) = 4, ento a soma m + n + p + q igual
a:
(a) 10;

(b) 21;

(c) 24;

(d) 26;

(e) 28.

104. As pginas do dicionrio Para numerar as pginas de um dicionrio, imprimiu-se


1 988 vezes o algarismo 1. Quantas pginas tem esse dicionrio?
105. Soma de potncias de 2 Determine um valor de n para o qual o nmero 28 +211 +2n
seja um quadrado perfeito.
106. Reverso de um nmero O reverso de um nmero inteiro de dois algarismos
o nmero que se obtm invertendo a ordem de seus algarismos. Por exemplo, 34 o
reverso de 43. Quantos nmeros existem que, somados ao seu reverso, do um quadrado
perfeito?
107. ngulos externos de um tringulo Dados os ngulos de 150 e 160 indicados na gura, calcule os valores dos ngulos x, y e z.

160

150

150
160

108. Uma brincadeira feita uma brincadeira com quatro nmeros inteiros da seguinte
maneira: some trs desses nmeros, divida essa soma por 3 e o resultado some com o
quarto nmero. Existem quatro formas de fazer esta brincadeira, obtendo os seguintes
resultados: 17, 21, 23 e 29. Qual o maior dos quatro nmeros?
109. Ovos e mas Num armazm, uma dzia de ovos e 10 mas tinham o mesmo
preo. Depois de uma semana, o preo dos ovos caiu 2% e o da ma subiu 10%.
Quanto se gastar a mais na compra de uma dzia de ovos e de 10 maas?
(a) 2%

(b) 4%

(c) 10%

(d) 12%

(e) 12,2%

110. Dividir um cubo Se dividirmos um cubo de 1 m de aresta em cubinhos de


1 mm de aresta, que altura ter uma coluna formada por todos os cubinhos, dispostos
sucessivamente um em cima do outro?
(a) 1 m
52

(b) 1 km

(c) 10 km

(d) 100 km

OBMEP 2010

(e) 1 000 km

Nvel 2
111. Uma expresso A expresso
(a)

a3
;
2

(b)

2
;
a3

(c)

a2
4a
1 3 , em que a = 0, igual a:
a5
(2 a)

1
;
2a3

(d)

a5
;
2

(e)

2
.
a5

112. Uma igualdade Os nmeros a e b so inteiros positivos que satisfazem 96a2 = b3 .


Qual o menor valor possvel de a?
113. Somas de trs em trs Encontre quatro nmeros inteiros positivos que, somados
de trs em trs, do somas 6, 7, 8 e 9.
114. O retngulo do Lus Lus desenhou um retngulo de 6 10 cm e quer dividi-lo
em quatro partes. As reas das 4 partes devem medir 8, 12, 16 e 24 cm2 . Desenhe
como ele pode fazer essa diviso.
115. Uma fbrica de blusas Uma fbrica produz blusas a um custo de R$ 2,00 por
unidade, alm de uma parte xa de R$ 500,00. Se cada unidade produzida comercializada a R$ 2,50, a partir de quantas unidades produzidas a fbrica obtm lucro?
(a) 250

(b) 500

(c) 1 000

(d) 1 200

(e) 1 500

116. Existncia de tringulos Qual dos seguintes tringulos no pode existir?


(a) tringulo agudo issceles
(b) tringulo retngulo issceles
(c) tringulo retngulo obtusngulo
(d) tringulo retngulo escaleno
(e) tringulo escaleno obtusngulo
117. Os doze pontos Doze pontos esto marcados numa folha de papel
quadriculado, conforme mostra a gura. Qual o nmero mximo de
quadrados que podem ser formados unindo quatro desses pontos?

r r
r r r r
r r r r
r r

118. O colar Um colar composto de prolas grandes e prolas pequenas, num total de
menos do que 500 prolas.
(a) Se substituirmos 70% das prolas grandes por pequenas, o peso do colar diminui
60%.
(b) Se substituirmos 60% das prolas pequenas por grandes, o peso do colar aumenta
70%.
Quantas prolas tem o colar?
OBMEP 2010

53

Nvel 2
119. Mulheres votantes Numa certa cidade, 40% de todas as mulheres so votantes
e 52% da populao de mulheres. Qual o percentual da populao formado por
mulheres votantes?
(a) 18,1%

(b) 20,8%

(c) 26,4%

(d) 40%

(e) 52%

120. Amigos do sculo XX Dois amigos nasceram no mesmo ms e ano do sculo


XX, com uma semana de intervalo. Escrevendo as datas dos dois aniversrios da
esquerda para a direita, comeando com o (ou os) algarismo(s) do dia, depois o (ou
os) algarismo(s) do ms e, por ltimo, os dois ltimos algarismos do ano, obtemos dois
nmeros. No colocando o algarismo 0 na frente dos nove primeiros dias do ms nem
dos nove primeiros meses do ano e sabendo que um desses nmeros o sxtuplo do
outro, qual a data de nascimento do amigo mais velho?
121. Operao em uma frao Que nmero se deve somar aos dois termos de uma
frao para se obter o inverso dessa mesma frao?
122. O nmero 119 O nmero 119 tem as propriedades seguintes:
(a) a diviso por 2 deixa resto 1;
(b) a diviso por 3 deixa resto 2;
(c) a diviso por 4 deixa resto 3;
(d) a diviso por 5 deixa resto 4;
(e) a diviso por 6 deixa resto 5.
Quantos inteiros positivos menores que 2 007 satisfazem essas propriedades?
123. Fonte com trs torneiras Slvia vai encher seus 10 garrafes numa fonte que tem
trs torneiras. Um dos garrafes demora um minuto para encher, outro dois minutos,
outro trs minutos e assim por diante. Como Slvia dever distribuir os 10 garrafes
pelas trs torneiras de modo a gastar o menor tempo possvel? Qual esse tempo?
124. A sequncia xyz Quais so os valores provveis de x, y e z na sequncia
1 5 3 7
, , , , x, y, z?
2 8 4 8
125. A mesa circular J existem N pessoas sentadas em volta de uma mesa circular
com 60 cadeiras. Qual o menor valor possvel para N se a prxima pessoa a se sentar
vai ter que se sentar ao lado de algum?
126. Nmeros proporcionais Se
(a)

54

x2
;
9

(b) x3 z;

x
3
= , ento 9y 2 igual a:
y
z

(c) 3x2 ;

(d) x2 z 2 ;

OBMEP 2010

(e)

1 2 2
xz .
9

Nvel 2
127. Esportistas de uma escola Em um grupo de 40 estudantes, 20 jogam futebol, 19
jogam vlei e 15 jogam exatamente um desses dois esportes. Quantos estudantes no
praticam nem futebol nem vlei?
(a) 7

(b) 5

(c) 13

(d) 9

(e) 10

128. Vamos ao teatro Na campanha Vamos ao teatro, 5 ingressos podem ser adquiridos pelo preo usual de 3 ingressos. Mrio comprou 5 ingressos nessa campanha. A
economia que Mrio fez representa que percentual sobre o preo usual dos ingressos?
(a) 20%

(b) 33

1
%
3

(c) 40%

(d) 60%

129. Uma desigualdade Os valores de x que satisfazem


(a) x < 2;

(b) x > 1;

(c) 1 < x < 2;

2
%
3

(e) 66

1
> 1 so dados por:
x1

(d) x < 1;

(e) x > 2.

130. A sala do Professor Newton O professor Newton dividiu seus alunos em grupos
de 4 e sobraram 2. Ele dividiu seus alunos em grupos de 5 e um aluno cou de fora.
Se 15 alunos so mulheres e tem mais mulheres do que homens, o nmero de alunos
homens :
(a) 7;

(b) 8;

(c) 9;

(d) 10;

(e) 11.

131. Um jardim retangular Na gura, o retngulo ABCD


representa um terreno retangular cuja largura mede 3/5 do
comprimento. O retngulo ABEF representa um jardim retangular cuja largura tambm mede 3/5 do comprimento.

D...............................................................................F.........................................A
...
..
.
....
.
.
.
..
.
.
.
.
.
.
.
.
.
.
.
.
.
.
.
.
.
.
.
.
.
.
.
.
.
.
.
.
.
.
.
.
.
.
.
.
.
.
.
.
.
.
.
.
.
.
.
.
.
.
.
.
.
.
.
.
.
.
.
.
.
.
.
.
.
.
.
.
.
.
.
.
.
.
.
.
.
.
.
.
.
.
.
.
.
.
.
.
.
.
.
.
.
.
.
.
.
.
.
.
.
.
.
.
.
.
.
.
.
.
.
.
.
.
.
.
.
.
.
.
.
...............................................................
................................................................
.

Qual a razo entre a rea do jardim e a rea total do terreno?


(a) 30%

(b) 36%

(c) 40%

(d) 45%

(e) 50%

132. Nmeros decrescentes Escreva em ordem decrescente os nmeros

1 3
1 1
5
3 , 32/3 , 32 ,
e
.
3
3
133. Os bombons misturados Marta e Carmem ganharam, cada uma, muitos bombons.
Elas misturaram todos os bombons e, agora, no sabem mais qual foi o nmero de
bombons que cada uma ganhou. Vamos ajud-las a descobrir esses nmeros? Sabe-se
que:
(a) juntas, elas ganharam 200 bombons;
(b) Marta se lembra que ganhou menos do que 100 bombons, mas mais do que 4/5
do que ganhou Carmem; e
(c) o nmero de bombons que cada uma ganhou um mltiplo de 8.
134. Jantar aos sbado Trs casais jantam todo sbado num mesmo restaurante, sempre
mesma mesa. A mesa redonda e os casais combinaram que
OBMEP 2010

55

Nvel 2
(a) jamais marido e mulher sentam mesa como vizinhos; e
(b) a disposio dos seis mesa diferente a cada sbado.
Desconsiderando rotaes nas disposies mesa, durante quantos sbados esses trs
casais podero ir a esse restaurante sem repetir sua disposio mesa?
135. Expresso com radicais O valor de
(a)

2+

3;

(b)

1
2

1+

1+

(c) 1 + 2 3;

7+3 5 ;

:
(d) 3;

(e) 3 + 2 2.

136. Possveis tringulos Os lados de um tringulo tm comprimentos a, a + 2 e a + 5,


sendo a > 0. Determine todos os possveis valores de a.
137. Uma diferena O valor de
(a) 3,3 ;

(b) 4,7 ;

0,001 400
0,036 0,4

:
0,25
0,4

(c) 4,9 ;

(d) 3,8 ;

(e) 7,5 .

138. A Terra A superfcie do globo terrestre consiste em 70% de gua e 30% de terra.
Dois quintos da terra so desertos ou cobertos por gelo e um tero da terra pastagem,
oresta ou montanha; o resto da terra cultivado. Qual o percentual da superfcie
total do globo terrestre que cultivada?
139. Uma frao Na gura dada, determine o valor da frao
AN
.
AC

A
.....
..
. ..
. .
. ..
. ..
. .
. .
. .
.........N
.
M.........
.
....
.
.
.
.
.
.
............................
.
.... C
B.

140. Cculo de ngulo Na gura dada, a reta P Q paralela reta RS e T U = T V. Se


o ngulo T W S mede 110 , o ngulo QU V mede:
(a) 135 ;

(b) 130 ;

(c) 125 ;

(d) 115 ;

U
110

T
V
P

56

OBMEP 2010

(e) 110 .

Nvel 2
141. Uma loja de brinquedos Uma loja estava vendendo cada unidade de um brinquedo
a R$ 13,00. Para conseguir vender todo o seu estoque, que no era superior a 100
unidades, a gerncia da loja resolveu baixar o preo por um nmero inteiro de reais,
obtendo R$ 781,00 por todo o estoque. Qual foi a reduo do preo, por unidade?
1

142. Frao de frao Qual o valor de 1 +

1+

1+

1
2

143. Potncias de 3 Se 3a = 2, quanto vale 27 2 a ?


144. Aumento de preo Se o preo de um produto passou de 5,00 para 5,55 reais, qual
foi o percentual do aumento?
145. Roseiras em la Jorge ganhou 15 roseiras para seu jardim e quer plant-las em
6 las de 5 roseiras cada uma. Isso possvel? Em caso armativo, faa um desenho
indicando como Jorge pode plantar suas roseiras.
146. Calculadora diferente Uma fbrica produziu uma calculadora original que efetua
duas operaes,
(a) a adio usual, denotada por + e
(b) uma operao denotada por .
Sabemos que, para todo nmero natural a, valem
(i) a a = a

(ii) a 0 = 2a

e, para quaisquer quatro nmeros naturais a, b, c e d, vale


(iii) (a b) + (c d) = (a + c) (b + d) .
Quais so os resultados das operaes (2 + 3) (0 + 3) e 1 024 48?
147. Dois quadrados Na gura dada, o lado do quadrado maior mede
10 cm e o lado do menor mede 4 cm. As diagonais do quadrado
maior contm as diagonais do quadrado menor. Quanto mede a
rea da regio tracejada?
148. Paralelismo Sendo o segmento IL paralelo ao segmento EU e o segmento RE paralelo ao segmento N I,
determine o valor da frao

..
..
...............................................
...
..............................................
....
....
.
.. .
. ..
.. .
. . ..
.. .
. . ..
.. .
.
. . ..
.
. . .
.. .
.. .
. . ....
.. .
. . ....
..
.
. . . ..
..
.
. . . . ..
.
. . . . ...
..
.
. . . . ...........................
..
.
. . . . . ..
.
.
. . . . . ..
....
.
. . . . . ..
..
.
. . . . . ......................
.
. . . . . . ..
.
. . . . . . ...
.. .
.
.. .
.
. . . . . . ..
.. .
.. .
.
. . . . . . ..
. .
.
. . . . . . .. ... .
.
. . . . . . . ... .
.
......
.. ... .
.
......
.. ..
.
.
......
.
....
.
.....
..
.
.
......
.
..
.
......
.
.
......
.
.
. . . . . . ....... .
.. ... .
.
. . . . . . ... ... .
.
...... .
.. .
.
...... .
.. .
.. .
.
.
. . . . . . ..
.
. . . . . . ....
.. .
.
. . . . . ..
...
.
. . . . . .. .
..
.
.
. . . . . .....................
.
. . . . . .......................
..
.
. . . . .....
.. ..
. ..
.
.
. . . . ..
.
.
. . . . ..
.
. . . ..
..
..
.
. . . ...
.
. . ..
..
..
.
. . ...
..
.
.
. . ..
. . ..
.. .
.. .
. . ..
.. .
.
.. .
. .....
. ..
.. .
.. .
. ..
.
...
..
...............................................
...
.
. ..
..
..............................................

......
......
......
......
......
......
......
......
. . ....
... .......
.
.. .. ...........
.. .. ............
.. ... .. ........
..
..
.. ... .. ........
......
..
..
......
..
..
....
.
......
....
......
..
..
......
..
.
..
.
......
..
......
.. ..
..
.....
.. ..
..
..
.
.
.
..
....
.......
..
.. ..
..
.. ...
.
.
.
.............
.............
.
..
..
.......................
..
..
.............
.
.......
..
..
..
................
.. ...........................
.
.
......... ....
..............
.
.............
.............

FN FU
.
FR FL

OBMEP 2010

U L

57

Nvel 2
149. Um subconjunto O conjunto {1, 2, 3, . . . , 3 000} contm um subconjunto de 2 000
elementos em que nenhum elemento o dobro do outro?
s

150. Tringulos retngulos Determine os valores de ..........................


. .. w
.
.
v, w, x, y e z na gura dada, em que j esto marcados 8 ....................r.......... ............... 9
z
. ...
.................s.................................................................s
..........................................s .......................................
s
.
.
.... .
.
. r.
.
. .
..
...
. .
.
...
.......
...
trs ngulos retos e os comprimentos de trs segmentos.
.
v ..............
.......
.
...
...
.
.
...
...
.
.
...
.
...
.
.

..
..

.
..
.
..
.
.
...
...
.
.
.
...
...
.
.
...
...
.
.
...
.
...
.
.
..
.
...
..
.
...
.
...
..
.
.. .
...
.. ..... ......
.
.. ..... .....
.
.. . ......
.. . .
.
... . ......
.. . ...
.....
.
....
.

y ..............

20

r
s

151. Uma desigualdade especial Que valores de x satisfazem x2 < |x| + 2?


(a) x < 1 ou x > 1

(b) x > 1

(c) 2 < x < 2

(d) x < 2

(e) x < 0

152. Sapo Cururu Cururu um sapo estranho, que se desloca apenas com dois tipos de
saltos, o de
Tipo I: 10 cm para o Leste e 30 cm para o Norte e o de;
Tipo II: 20 cm para Oeste e 40 cm para o Sul.
20 cm

30 cm

10 cm

Tipo I

40 cm

Tipo II

(a) Como Cururu faz para chegar a um ponto situado a 190 cm para o Leste e
950 cm para o Norte de sua casa?
(b) possvel Cururu chegar a um ponto situado a 180 cm para o Leste e 950 cm
para o Norte de sua casa?
153. Distribuindo algarismos em linhas Joana escreveu uma sequncia em 10 linhas
usando os algarismos de 0 a 9, seguindo o padro seguinte.
0
1 1 0
2 2 2 1 1 0
3 3 3 3 2 2 2 1 1 0
.
...
.
.
Qual foi o algarismo mais usado? Quantas vezes esse algarismo foi utilizado?
154. Ser que existe? Existe algum nmero inteiro N tal que valha
2 008 N = 222 . . . 2?
58

OBMEP 2010

Nvel 2
155. Conferindo uma desigualdade Ser verdade que

1
1
1
1
+ 3+ 3 <
?
3
4
5
6
12

156. Parte inteira A parte inteira de um nmero real x o maior inteiro que menor
do que ou igual a x. Denotamos a parte inteira de x por [x]. Por exemplo, [2,9] = 2,
[0,88] = 0 e [1,7] = 2.
1,7

2,9

0,88

Calcule as partes inteiras seguintes.


(a)

12

(b)

28 756
12 777

(c)

2 007
2 008

(d)

111

157. Soma nove Quantos nmeros inteiros entre 10 e 999 tm a soma de seus algarismos
igual a 9?
158. Retngulos As medidas dos lados de um retngulo so nmeros pares. Quantos
retngulos desses existem com rea igual a 96?
159. Nmero de retas Sabemos que dois pontos distintos determinam uma
nica reta. Quantas retas so determinadas por dois quaisquer dos nove
pontos marcados no quadriculado dado?
160. Cubo Pedro quer pintar uma caixa de formato cbico de tal maneira que as faces que
tenham uma aresta em comum sejam pintadas em cores diferentes. Calcule o nmero
mnimo de cores que sero necessrias para pintar a caixa dessa maneira.
161. rea Um lote retangular foi divido em quatro terrenos,
todos retangulares. As reas de trs deles esto dadas na
gura, em km2 . Qual a rea do lote?

27

18

72

162. Inteiro mais prximo Determine o nmero inteiro mais prximo de


(a)

19 19
+
15
3

(b)

85 43 29 15
+
+
+
42 21 14
7

(c)

11 1 7 2
+
10 2 5 3

163. Brincando com nmeros mpares Beatriz adora nmeros mpares. Quantos
nmeros entre 0 e 1 000 ela pode escrever usando apenas algarismos mpares?
164. gua no jarro Joo e Maria tm, cada um, um jarro grande com um litro de gua.
No primeiro dia, Joo coloca 1 ml da gua do seu jarro no jarro da Maria. No segundo
dia, Maria coloca 2 ml da gua do seu jarro no jarro do Joo. No terceiro dia, Joo
coloca 3 ml da gua do seu jarro no jarro da Maria, e assim por diante. Depois de 200
dias, quantos mililitros de gua tem no jarro de Maria?
OBMEP 2010

59

Nvel 2
165. Formiga no cubo Uma formiga parte de um vrtice de um cubo, andando somente
ao longo das arestas, at voltar ao vrtice inicial, no passando duas vezes por nenhum
vrtice. Qual o passeio de maior comprimento que essa formiga pode fazer?
166. Promoo Em uma promoo, Joana comprou blusas por R$ 15,00 cada uma e
calas por R$ 17,00 cada uma, gastando, ao todo, R$ 143,00. Quantas blusas e calas
Joana comprou?
167. Soma de cubos Se x + y = 1 e x2 + y 2 = 2, calcule x3 + y 3 .
168. O revezamento em uma corrida Numa competio de revezamento, em que
cada equipe tem dois atletas, cada atleta corre 21 km e o segundo atleta s inicia a
corrida quando o primeiro atleta termina a sua parte e lhe passa o basto. O recorde
dessa competio de 2 horas e 48 minutos. Na equipe de Joo e Carlos, Joo inicia
a corrida e corre a sua parte com uma velocidade de 12 km/h. Para bater o recorde,
qual deve ser a velocidade de Carlos?
169. Produtos consecutivos Divida os nmeros 2, 3, 5, 7, 11, 13 e 17 em dois grupos
de tal forma que, multiplicando todos os nmeros de um grupo e todos do outro,
encontremos nmeros consecutivos.
170. Distraindo na la Vivi, Tnia e Rosa esto em la, no necessariamente nessa
ordem, e gritam sucessivamente, cada uma, um mltiplo de 3.
3
12
.
.
.

6
15
.
.
.

9
18
.
.
.

Vivi foi a primeira a gritar um nmero maior que 2 003 e Rosa a primeira a gritar um
nmero de quatro algarismos. Quem gritou o nmero 666? E o 888?
171. Nmero e o dobro Um nmero menor do que 200 formado por trs algarismos
diferentes e o dobro desse nmero tambm tem todos os algarismos diferentes. Ainda,
o nmero e seu dobro no tm algarismos em comum. Qual esse nmero? Quantas
solues tm esse problema?
172. Invertendo os algarismos Entre 10 e 99, quantos nmeros existem tais que,
invertendo a ordem de seus algarismos, obtemos um nmero maior do que o nmero
original?
173. Razo entre segmentos Na gura, O o centro do semicrculo de dimetro P Q, R um ponto sobre o semicrculo e RM

perpendicular a P Q. Se a medida do arco P R o dobro da

medida do arco RQ, qual a razo entre P M e M Q?

174. Tringulos Quantos tringulos existem que tenham um permetro de 15 unidades


e lados medindo nmeros inteiros?
60

OBMEP 2010

Nvel 2
175. Nmero interessante O nmero 119 muito interessante porque deixa resto 1 ao
ser dividido por 2, deixa resto 2 ao ser dividido por 3, deixa resto 3 ao ser dividido por
4, deixa resto 4 ao ser dividido por 5 e, nalmente, deixa resto 5 ao ser dividido por 6.
Existem outros nmeros de trs algarismos com essas propriedades?
176. Time vencedor Um time de futebol ganhou 60% das 45 partidas j disputadas.
Qual o nmero mnimo de partidas que esse time ainda precisa vencer para atingir
uma porcentagem de 75% de vitrias?
177. Brincando com dados Dois dados so lanados. Qual a probabilidade de o
produto dos nmeros obtidos nos dois dados ser divisvel por 6?
178. Contando solues Quantos so os pares de nmeros inteiros positivos (x, y) tais
que
xy
= 144 ?
x+y
179. Crculos tangentes Os vrtices de um tringulo cujos lados medem 3, 4 e 5 cm,
so centros de trs crculos que so dois a dois tangentes exteriormente. Qual a soma
das reas desses trs crculos?
180. Grupo de amigos Joo, Jorge, Jos e Jnio so bons amigos. Certa vez, Joo
estava sem dinheiro, mas seus amigos tinham algum. Ento Jorge deu a Joo um
quinto de seu dinheiro, Jos deu um quarto de seu dinheiro e Jnio deu um tero de
seu dinheiro. Se todos eles deram a mesma quantidade de dinheiro para Joo, que
frao do dinheiro do grupo cou com Joo?
181. Um trapzio A gura dada representa um trapzio ABCD
em que AB paralelo a CD e as diagonais AC e BD cortam-se
no ponto P. Se as reas dos tringulos AP B e CP D medem
4 e 9 cm2 , respectivamente, qual a rea do tringulo P CB?

B
P

182. Vista ruim Numa classe, 40% dos alunos no enxergam bem. Desses, 70% usam
culos e os 30% restantes usam lentes de contato. Sabendo que 21 alunos usam culos,
quantos alunos tem nessa classe?
183. Idade mdia da populao de Campo Verde A razo entre o nmero de homens
e o de mulheres na cidade de Campo Verde de 2/3. A idade mdia dos homens
37 anos e a das mulheres 42 anos. Qual a idade mdia dos habitantes de Campo
Verde?
184. rea de tringulo Se AC = 1 cm e AD = 4
cm, qual a relao entre as reas dos tringulos
ABC e CBD?

OBMEP 2010

.
.. .
....... .
....... ..
..
......... ...
.......... ...
...... ...
..
...... ....
..
...... ...
...... ....
..
..
...... ......
...... ......
..
..
....
.....
...
..
...
......
......
..
....
......
..
....
......
.
..
.
..
...
..
....
......
....
..
......
.
.
..
...
..
..
....
......
....
......
.
.
..
...
..
..
....
......
....
......
.
.
..
...
..
..
....
......
....
......
.
..
..
. ..
. ...
...
....................................................................................................
.....................................................................................................
. .. .
......
..
.
.

61

Nvel 2
185. Construindo quadrados perfeitos Observe as igualdades a seguir.

1234+1 =
2345+1 =
.
.
.

25 = 52
121 = 112

10 11 12 13 + 1 = 17.161 = 1312

.
.
.

Ser que isso sempre verdadeiro? Isto , ser sempre um quadrado perfeito o produto
de quatro nmeros inteiros consecutivos, mais 1?
186. Feira de Cincias Na Feira de Cincias
de uma escola, observou-se que metade dos
alunos do ensino fundamental e um quarto
dos alunos do ensino mdio presentes nesse
evento compraram um adesivo cada.

FEIRA DE CINCIAS
Preo dos Adesivos (unidade)
R$ 0,30 alunos do ensino fundamental
R$ 0,50
alunos do ensino mdio

Notou-se tambm que o nmero de alunos do ensino mdio presentes que no compraram adesivos foi o dobro do nmero de alunos do ensino fundamental que no
compraram adesivos. Sabendo-se que foram arrecadados R$ 38,00 na venda de adesivos para os alunos desses dois nveis, quantos alunos de cada nvel participaram da
feira?
187. Par perfeito Dizemos que dois nmeros naturais formam um par perfeito quando
a soma e o produto desses dois nmeros so quadrados perfeitos. Por exemplo, 5 e 20
formam um par perfeito, pois 5 + 20 = 25 = 52 e 5 20 = 100 = 102 . Ser que 122
forma um par perfeito com algum outro nmero natural?
188. Um trapzio No trapzio da gura dada, AB paralelo a DC, AD = AB = BC =
1 cm e DC = 2 cm. Quanto mede o ngulo DAC?
(a) 30

(b) 45

(c) 60
(d) 90

........................................
..........................................
..
..
..
..
.
..
..
.
..
..
..
.
..
.
..
..
..
.
..
.
..
..
..
..
..
..
.
.
..
.
.
..
.
..
.
.
..
..
.
.
..
.
.
..
.
..
.
..
..
.
..
.
..
.
.
.
.
..
...............................................................................
................................................................................
.
.
.

(e) 120

189. Mistrio das bolas Henrique tm duas urnas. A primeira urna contm somente
bolas pretas e a segunda somente bolas brancas. Henrique retirou um certo nmero de
bolas da primeira urna e as colocou na segunda. Em seguida, retirou o mesmo nmero
de bolas da segunda urna e as colocou na primeira. Depois disso, o nmero de bolas
brancas na primeira urna maior do que, menor do que ou igual ao nmero de bolas
pretas na segunda urna?
62

OBMEP 2010

Nvel 2
190. Contando a palavra BRASIL Quantas vezes
aparece a palavra BRASIL na gura dada? S vale ler
a palavra emendando letras que esto escritas em quadradinhos adjacentes.
B

B
R

B
R
A

B
R
A
S

B
R
A
S
I

B
R
A
S
I
L

191. Quais so os nmeros? Descubra quais so os nmeros inteiros positivos x e y


que satisfazem a equao x4 = y 2 + 71.
192. No jogo Aldo, Bernardo e Carlos jogam baralho. No incio, a quantia em dinheiro
que eles tinham, na ordem Aldo : Bernardo : Carlos, estava na proporo 7 : 6 : 5.
No nal do jogo, na mesma ordem, a proporo era de 6 : 5 : 4. Se um dos jogadores
ganhou 12 reais, qual foi a quantidade de dinheiro com que cou cada jogador, no nal
da partida?
193. Um nmero inteiro Mostre que M =

5+2

5 2 um nmero inteiro.

194. rea de tringulos A rea do quadrado ABCD mede 300 cm2 .


Na gura, M o ponto mdio de DC e o ponto F pertence reta
que passa por B e C.

(a) Qual a rea do tringulo ABF ?

(b) Qual rea do tringulo AF D?

195. Um quadriculado Observe que o retngulo quadriculado na


gura ao lado constitudo de 31 segmentos e compreende doze
quadrados.
Numa folha retangular de 21 por 29,7 cm, quadriculada com quadrados de lados
medindo 0,5 cm, Rosa desenhou um grande retngulo quadriculado, constitudo de
1 997 segmentos. Quantos quadrados tem esse retngulo?
196. Inteiros de quatro algarismos Determine o valor do nmero natural a, sabendo
4
que 4a2 e a3 so nmeros inteiros de quatro algarismos.
3
197. Pares positivos Quantos pares (x, y) de inteiros positivos so solues da equao
3x + 5y = 501?
198. Diferena de quadrados A diferena dos quadrados de dois nmeros inteiros
consecutivos 2 000.
(a) Os dois inteiros so menores do que 100.
(b) Os dois inteiros so menores do que 1 000, porm maiores do que 99.
(c) Os dois inteiros so menores do que 10 000, porm maiores do que 999.
(d) Os dois inteiros so menores do que 100 000, porm maiores do que 9 999.
OBMEP 2010

63

Nvel 2
(e) No existem esses dois nmeros.
199. Clculo de ngulos Calcule o valor do ngulo x em cada uma das guras a seguir,
sabendo que os segmentos AB e ED so paralelos.

A.........................................................................................................................................................B
.

A........................................................................................................................................................B
.
.
.

.
..
..
..
...

..
..
..
...
..
..
...
...
...
..
...
...
...
...
...
..
...
..
...
...
...
...
..
..
.
x ..............
.
..
.
.
..
..
..
.
..
..
...
...
..
...
..
...
..
...
o ....
..
150 ....
..
..............................................................................
.............................................................................

.
25o ......
..
...

160o

.
..
..
..
...
...
...
..
...
..
...
..
...
..
...
..
...
..
... x
...
..
...
..
...
..
...
..
...
..
...
...
...
..
...
..
...
.
..
..
o ....
..
55
..
.............................................................................
...........................................................................
..
.

200. Tabela Na tabela ao lado, com seis colunas e diversas


linhas, esto escritos, ordenadamente, os nmeros 1, 2,
3, 4, . . . Qual a posio do nmero 1 000 nessa tabela?

1
7
13

2
8
14

3
9

4
10

5
11

6
12

.
.
.

.
.
.

.
.
.

.
.
.

.
.
.

.
.
.

201. Entre 1 e 2 Encontre todos os inteiros positivos a e b tais que a/5 e b/7 sejam
menores do que 1 e valha a condio
1<

a b
+ < 2.
5 7

202. Triatlon Maria est planejando participar do Triatlon-Brasil que consta de 800 m
de nado, seguido de 20 km de bicicleta e, nalmente, 4 km de corrida. Maria corre a
uma velocidade constante que o triplo da velocidade com que nada e pedala 2,5 vezes
mais rpido do que corre. Para terminar a prova em, no mximo, 1 hora e 20 minutos,
qual deve ser sua velocidade mnima em cada uma das trs modalidades?
203. Foto de formatura O diretor de certa escola decidiu tirar uma foto dos formandos
de 2008. Ele colocou os alunos em las paralelas, todas com o mesmo nmero de alunos,
mas essa disposio era muito larga para o campo de viso de sua mquina fotogrca.
Para resolver esse problema, o diretor resolveu tirar um aluno por la, colocando-os
numa nova la. Essa disposio no agradou o diretor porque a nova la tinha quatro
alunos a menos do que as outras. Ele decide, ento, tirar mais um aluno de cada la
original, colocando-os na nova la recm criada, e constata que, agora, todas as las
caram com o mesmo nmero de alunos e nalmente tira sua foto. Quantos alunos
apareceram na foto?
204. Circunferncias tangentes Desenhe duas circunferncias de mesmo centro, uma
de raio medindo 1 cm e a outra de raio medindo 3 cm. Na regio exterior circunferncia de 1 cm de raio e interior de 3 cm de raio, desenhe circunferncias que sejam,
simultaneamente, tangentes s duas circunferncias, como mostrado na gura dada.
64

OBMEP 2010

Nvel 2
..
....................
... .
.
.......................
........
........
.. ....
.. ....
.... .
.... ..
...
.
...
...
. ..........
..
.
.....
.
..
.
..............
..
.
.
..
.
..
..
..
..
..
..
..
..
..
..
...
..
.
.... ....
.
.
..
..
...
.
... .. .
.
.
..
.
...........
..
.
....... ......
.
.
.
..
.
..
..
.
.
.. .
..
..
.
.
...
.
.
.
.............. .
.
.
.
. ........... .
.
.
.
.
.
.
.
.
.
.
.
.
..
.
.
..
.
.
.
.
.
..
...
.
.... .....
.
.
.
.
..........
.
.....
.
..
..
.
..
..
.
..
..
..
..
..
..
...
...
..
...
....
....
...
....
.....
.....
.....
.....
....................
....................

(a) Qual deve ser o raio dessas circunferncias?


(b) Qual o nmero mximo dessas circunferncias que podem
ser desenhadas, sem que elas se sobreponham?

205. Festa na escola Para a festa de aniversrio da escola, Ana, Pedro, Miriam e Fbio
levaram um total de 90 docinhos. A professora deles observou que
se Ana tivesse levado dois docinhos a mais;

se Pedro tivesse levado dois docinhos a menos;


se Miriam tivesse levado o dobro e
se Fbio tivesse levado a metade,

os quatro amigos teriam levado todos o mesmo nmero de docinhos. Quantos docinhos
levou cada um dos amigos?
206. Inao Mrcia est numa loja comprando um gravador que ela queria h muito
tempo. Quando o caixa registra o preo ela exclama: No possvel, voc deve ter
registrado o nmero ao contrrio e trocou a ordem de dois algarismos, pois lembro que,
na semana passada, custava menos do que 50 reais! Responde o caixa: Sinto muito,
mas ontem todos os nossos artigos foram aumentados em 20%. Qual o novo preo
desse gravador?
207. Gatos no condomnio Num certo condomnio moram 29 famlias, cada uma das
quais possui ou um, ou trs, ou cinco gatos. O nmero de famlias que possuem apenas
um gato o mesmo que o de famlias que possuem cinco gatos. Quantos gatos tem
nesse condomnio?
208. Soma constante Preencha as cinco casas em branco da tabela 3 3
dada com os nmeros de 3 a 8, sem repeti-los, de modo que as somas dos
quatro nmeros escritos nas quatro subtabelas formadas por quadrados
2 2 seja sempre a mesma.

2
9

209. Qual o nmero? Na adio ao lado, letras iguais representam o


mesmo algarismo e letras diferentes, algarismos diferentes. Encontre o
nmero A B C D E.

A B C D
B C D
C D
D

E
E
E
E
E
A A A A A

210. Proporo triangular Num tringulo ABC, o ponto F


est sobre o lado AC e F C = 2AF. Se G o ponto mdio do
segmento BF e E o ponto de interseo da reta passando por
A e G com o segmento BC, calcule a razo EC/BE.

F
G
B

211. Nmeros primos entre si Encontre todos os pares de inteiros positivos (x, y) tais
x y
+
que x < y, x e y so primos entre si e 2 000
seja um nmero inteiro mpar.
y x
OBMEP 2010

65

Nvel 2
212. Fique atento Determine todas as solues da equao

x = x 2.

213. Solues inteiras Determine todos os nmeros inteiros x e y tais que


1
1 1
+ =
.
x y
19
214. No ponto de nibus Um certo nmero de meninos e meninas aguardam pelo
nibus. No primeiro nibus que passa no ponto em que se encontram, embarcam
somente quinze meninas e cam dois meninos para cada menina no ponto de nibus.
No segundo nibus que passa, embarcam somente 45 meninos e cam cinco meninas
para cada menino no ponto de nibus. Determine o nmero de meninos e meninas que
estavam no ponto antes de passar o primeiro nibus.
215. Contorno circular A gura a seguir formada por quatro crculos tangentes de raio
a. Determine o comprimento do contorno externo, que est com o traado destacado.

216. Um quadriltero especial Dois lados consecutivos de um quadriltero medem 10 e


15 cm. Se cada diagonal divide o quadriltero em duas regies de mesma rea, calcule
seu permetro.
217. Nmero curioso O nmero 81 tem a seguinte propriedade: ele divisvel pela
soma de seus algarismos, 8+1=9. Quantos nmeros de dois algarismos cumprem essa
propriedade?
218. Nmero premiado Um nmero de seis algarismos premiado se a soma de seus
primeiros trs algarismos for igual soma de seus trs ltimos algarismos. Por exemplo,
342 531 premiado, pois 3 + 4 + 2 = 5 + 3 + 1.
(a) Quais so o maior e o menor nmero premiado com seis algarismos distintos?
(b) Mostre que a soma de todos os nmeros premiados com seis algarismos distintos
divisvel por 13.
219. Altura versus lado Seja ABC um tringulo tal que a altura relativa ao lado BC
no menor do que o lado BC e a altura relativa ao lado AB no menor do que o
lado AB. Determine as medidas dos ngulos deste tringulo.
66

OBMEP 2010

Nvel 2
220. Fraes egpcias Determine todos os nmeros inteiros positivos distintos x e y tais
que
1 1
2
+ = .
x y
7
221. Tabuleiro de xadrez De quantas maneiras podemos colocar dois bispos de mesma
cor num tabuleiro de xadrez em las, colunas e casas de cores distintas?
222. Quem menor? Sem usar calculadora, decida qual dentre os nmeros 3312 , 6310
e 1278 o menor.
223. Brincando com nmeros A soma 1 + 1 + 4 dos algarismos do nmero 114 divide o prprio nmero. Qual o maior nmero menor do que 900 que satisfaz essa
propriedade?
224. Cortando papis No incio de uma brincadeira, Andr tinha sete pedaos de papel.
Na primeira rodada da brincadeira, ele pegou alguns destes pedaos e cortou cada um
deles em sete pedaos, que foram misturados aos pedaos de papel que no foram cortados nessa rodada. Na segunda rodada, ele novamente pegou alguns pedaos e cortou
cada um deles em sete pedaos, que foram misturados aos demais papis. Continuando
dessa maneira, ao nal de alguma rodada, Andr poder ter exatamente 2 009 pedaos
de papel?
225. Um trapzio especial A base AD de um trapzio ABCD mede 30 cm. Suponhamos que exista um ponto E na base AD tal que os tringulos ABE, BCE e
CDE tenham permetros iguais. Determine o comprimento de BC.
226. Uma estrela Na estrela ABCDE da gura dada, sabe-se que GBF = 20o e GHI =
130o . Qual o valor do ngulo J EI?

E
I

H
C

227. Nmero palndromo Um nmero dito palndromo se sua leitura da direita para
a esquerda for igual da esquerda para a direita. Por exemplo, os nmeros 23 432
e 18 781 so palndromos. Quantos nmeros palndromos de quatro algarismos so
divisveis por 9?

OBMEP 2010

67

Nvel 2
228. Multiplicao com letras Na operao dada, as letras a, b e c representam algarismos distintos e diferentes de 1. Determine os valores de a, b e c.
abb
c
bcb1
229. Nmeros sortudos Digamos que um nmero sortudo se a soma de seus algarismos
for divisvel por sete. Por exemplo, 7, 25 e 849 so nmeros sortudos. Os dois menores
nmeros sortudos so 7 e 16.
(a) Encontre oito nmeros consecutivos, dos quais dois so nmeros sortudos.
(b) Encontre doze nmeros consecutivos, tais que nenhum seja sortudo.
(c) Mostre que qualquer sequncia de treze nmeros consecutivos contm, pelo menos,
um nmero sortudo.
230. Uma sequncia especial Na sequncia 1, 3, 2, . . . cada termo depois dos dois
primeiros igual ao termo precedente, subtrado do termo que o precede, ou seja,
se n > 2, ento an = an1 an2 . Qual a soma dos cem primeiros termos dessa
sequncia?
231. Tringulos e ngulos. . . Determine os ngulos e dados na gura.
130

70

45

68

OBMEP 2010

Nvel 3

Nvel 3
1. Usando velas Uma cidade ainda no tem iluminao eltrica, portanto, nas casas
usam-se velas noite. Na casa de Joo, usa-se uma vela por noite, sem queim-la
totalmente, e com quatro desses tocos de velas, Joo fabrica uma nova vela. Durante
quantas noites Joo poder iluminar sua casa dispondo de 43 velas?
(a) 43

(b) 53

(c) 56

(d) 57

(e) 60

2. Rodas e bandeiras Juliano encaixou duas rodas dentadas


iguais, cada uma com uma bandeirinha igual desenhada, como
mostra a gura.
Ento ele girou a roda da esquerda um pouco. Qual das alternativas abaixo pode
representar a posio nal das rodas?

(a)

(b)

(d)

(c)

(e)

3. Nmero de latas Uma fbrica embala latas de palmito em caixas de papelo de


formato cbico de 20 cm de lado. Em cada caixa so colocadas 8 latas e as caixas
so colocadas, sem deixar espaos vazios, em caixotes de madeira de 80 cm de largura
por 120 cm de comprimento e 60 cm de altura. Qual o nmero mximo de latas de
palmito em cada caixote?
(a) 576

(b) 4 608

(c) 2 304

(d) 720

(e) 144

4. Qual a menor frao? Quantas fraes da forma


7/9, sabendo que n um nmero inteiro positivo?
(a) 1

(b) 2

(c) 3

(d) 4

n
so menores do que
n+1

(e) 5

5. Pistas de corrida Um atleta corre 5 000 m por semana em uma quadra de esportes,
que tem uma pista curta e outra longa. Em uma certa semana, ele treinou seis dias,
sendo que a cada dia correu uma vez na pista longa e duas na pista curta. Na semana
seguinte, ele treinou sete dias, sendo que a cada dia correu uma vez em cada pista.
Podemos, ento, armar que:
(a) a pista longa 500 m mais longa do que a curta;
(b) a pista longa quatro vezes maior do que a curta;
(c) a pista longa cinco vezes maior do que a curta;
OBMEP 2010

69

Nvel 3
(d) a pista longa 600 m mais longa do que a curta;
(e) a pista longa trs vezes maior do que a curta.
6. Brincos e brincos Numa certa povoao africana vivem 800 mulheres, 3% das
quais usam apenas um brinco. Das demais, a metade usa dois brincos e a outra
metade, nenhum. Qual o nmero total de brincos usados por todas as mulheres
dessa povoao?
(a) 776

(b) 788

(c) 800

(d) 812

(e) 824

7. Perguntas e respostas Ana, Bento e Lucas participam de um concurso que consta


de 20 perguntas, com as regras seguintes.
Cada resposta certa vale 5 pontos.

Cada resposta errada acarreta a perda de 3 pontos.

Cada resposta em branco acarreta a perda de 2 pontos.


Ana
Bento
Lucas

certas
12
13
12

erradas
3
7
4

em branco
5
0
4

Usando os resultados do concurso da tabela e escrevendo os nomes dos trs em ordem


decrescente de classicao no concurso, obtemos:
(a) Ana, Bento, Lucas;

(c) Ana, Lucas, Bento;

(b) Lucas, Bento, Ana;

(e) Bento, Ana, Lucas.

(d) Lucas, Ana, Bento;

8. Qual a carga? O limite de peso que um caminho pode transportar corresponde


a 50 sacos de areia ou a 400 tijolos. Se esse caminho j carrega 32 sacos de areia,
quantos tijolos, no mximo, ele ainda pode carregar?
(a) 132

(b) 144

(c) 146

(d) 148

(e) 152

9. Quanto mede a cerca? Uma cerca reta de arame tem 12 postes igualmente
espaados. A distncia entre o terceiro e o sexto poste de 3,3 m. Qual o comprimento
da cerca, em metros?
(a) 8,4

(b) 12,1

(c) 9,9

(d) 13,2

(e) 9,075

1
10. Dzima peridica Sabendo que 0,333. . . = , qual a frao irredutvel equivalente
3
a 0, 1333 . . .?
(a)

70

1
13

(b)

1
15

(c)

1
30

(d)

2
15

OBMEP 2010

(e)

1 333
10 000

Nvel 3
11. Valor absoluto O valor absoluto |a| de um nmero a qualquer denido por

|a| = 0

se a > 0,
se a = 0,
se a < 0.

Por exemplo, |6| = 6, | 4| = 4 e |0| = 0. Quanto vale N = |5| + |3 8| | 4|?


(a) 4

(b) 4

(c) 14

(d) 14

(e) 6

12. O peso das frutas Marcos quer pesar, numa balana de dois pratos, uma banana,
uma ma e um mamo. Em cada uma das guras dadas, a balana est em equilbrio,
isto , os contedos que esto no prato da direita tm o mesmo peso que os que esto
no prato da esquerda. Em duas das trs pesagens foi utilizado um peso de 200 gramas.
Podemos armar que as trs frutas tm um peso total, em gramas, de

(a) 250;

(b) 300;

(c) 350;

(d) 400;

(e) 450.

13. Maratona Andr treina para a maratona dando voltas em torno de uma pista
circular com 100 m de raio. Para percorrer 42 km, o nmero de voltas que Andr
precisa dar est entre:
(a) 1 e 10;

(b) 10 e 50;

(c) 50 e 100;

(d) 100 e 500;

(e) 500 e 1 000.

14. Dobrando papel Uma folha quadrada foi dobrada duas vezes ao longo de suas
diagonais, obtendo-se um tringulo. Em seguida, foi feito um corte reto na folha
dobrada, paralelo ao lado maior desse tringulo, passando pelos pontos mdios dos
outros lados, conforme a ilustrao dada.

Desdobrando a folha, obteve-se um buraco quadrado no meio da folha. A rea do


buraco corresponde a qual frao da rea de toda a folha quadrada original?
(a)

1
2

(b)

1
6

(c)

3
8

(d)

3
4

(e)

1
4

15. Encontre o nmero Qual o menor nmero inteiro positivo N tal que N/3, N/4,
N/5, N/6 e N/7 sejam todos nmeros inteiros?
(a) 420

(b) 350

(c) 210

(d) 300

OBMEP 2010

(e) 280
71

Nvel 3
16. Equao quadrtica Se 3 e 1/3 so as razes da equao ax2 6x + c = 0, qual
o valor de a + c?
9
18
(a) 1
(b) 0
(c)
(d)
(e) 5
5
5
17. Cubo Os vrtices de um cubo so numerados de 1 a 8, de tal modo que uma das
faces tem os vrtices {1, 2, 6, 7} e as outras cinco tm os vrtices {1, 4, 6, 8}, {1, 2, 5, 8},
{2, 3, 5, 7}, {3, 4, 6, 7} e {3, 4, 5, 8}. Qual o nmero do vrtice que est mais distante
do vrtice de nmero 6?
(a) 1

(b) 3

(c) 4

(d) 5

(e) 7

18. Time de basquete O grco dado mostra o


nmero de pontos que os oito jogadores de basquete do time da escola marcaram no ltimo jogo.
Qual o nmero total de pontos marcados pelo
time?
(a) 54 (b) 8 (c) 12 (d) 58 (e) 46
19. O caminho da formiguinha Uma formiguinha vai caminhar de A at C, podendo
passar apenas uma vez pelo ponto B e usando somente os caminhos indicados na gura.

B
A

Qual o nmero de maneiras diferentes que ela pode escolher para caminhar de A at
C?
(a) 3
20. Operao
vale 1 0?
(a) 1

(b) 5

(c) 7

(d) 8

(e) 9

Dados dois nmeros reais a e b, considere a


(b) 0

(c) 2

(d) 2

b = a2 ab + b2 . Quanto

(e) 1

21. Indo para a escola O diagrama de barras mostra


a distribuio dos alunos de uma escola de acordo com
o tempo que gastam no trajeto de casa para a escola.
As fraes de minuto foram desconsideradas; por exemplo, se um aluno gasta 40 minutos e 15 segundos neste
trajeto, considera-se que o tempo gasto de 40 minutos.

nmeros de alunos
100
menos de 20 min
de 20 a 40 min
50

de 41 a 60 min
mais de 60 min

10

Responda s perguntas seguintes justicando sua resposta.


(a) Quantos alunos gastam menos do que 20 minutos para chegar escola?
(b) Quantos alunos tem esta escola?
72

OBMEP 2010

Nvel 3
(c) Quantos alunos gastam mais do que 40 minutos para chegar escola?
(d) verdade que a maioria dos alunos gasta mais do que 20 minutos para chegar
escola?
22. Campeonato de futebol No ltimo campeonato de futebol do bairro em que moro
participaram seis equipes, denominadas A, B, C, D, E e F. Cada equipe disputou, com
cada uma das outras, exatamente uma partida. A tabela de classicao do campeonato fornecida a seguir, sendo V o nmero de vitrias, E o nmero de empates, D
o nmero de derrotas, GP o nmero de gols marcados e GC o nmero de gols sofridos
para cada equipe.

(b) A tabela est incompleta. Determine a


quantidade de vitrias da equipe F, a
quantidade de derrotas da equipe D e a
quantidade de gols marcados pela equipe
F, representados na tabela por x, y e z.

Equipe
A
B
C
D
E
F

V
4
2
0
1
0
x

E
1
1
3
1
1
1

23. Poste eltrico Uma companhia de eletricidade instalou um poste num terreno plano. Para xar bem o
poste, foram presos cabos no poste, a uma altura de 1,4
metros do solo e a 2 metros de distncia do poste, sendo
que um dos cabos mede 2,5 metros, conforme a gura.

D
0
2
2
y
4
0

GP
6
6
2
3
1
z

1,4 m

(a) Quantas partidas foram disputadas?

GC
2
6
6
6
5
3

2,5 m
2m

Um professor de Matemtica, aps analisar estas medidas, armou que o poste no est
perpendicular ao solo. Voc acha que o professor est certo? Justique sua resposta.
24. Equaes recprocas Briot (matemtico ingls, que viveu de 1817 a 1882) e Runi
(matemtico italiano, que viveu de 1765 a 1822) desenvolveram mtodos para encontrar
solues para as equaes chamadas recprocas. Nesta questo, voc vai desenvolver,
passo a passo, a essncia desses mtodos. O item (a) uma preparao para os demais
itens.
1
1
1
(a) Se y = x + , calcule as expresses x2 + 2 e x3 + 3 em termos de y.
x
x
x
1
5
(b) Determine todas as razes reais da equao x2 5x + 8 + 2 = 0.
x x
(c) Determine todas as razes reais de x4 5x3 + 8x2 5x + 1 = 0.
(d) Determine todas as razes reais de x6 2x5 5x4 + 12x3 5x2 2x + 1 = 0.
25. Atirando echas Manoel testa sua pontaria lanando cinco echas que atingiram o
alvo nos pontos A, B, C, D e E, de coordenadas A = (1, 1), B = (2,5; 1), C = (1, 4),
D = (4, 4) e E = (6, 5).
OBMEP 2010

73

Nvel 3
A tabela mostra quantos pontos se ganha quando
a echa acerta um ponto dentro de cada uma das
trs regies, conforme mostra a gura.
(a) Marque os pontos A, B, C, D e E.
(b) Quantas echas ele acertou no interior do
menor crculo?
(c) Ao todo, quantos pontos Manoel fez?

26. Festa de aniversrio A festa de aniversrio de Andr tem menos do que 120
convidados. Para o jantar, ele pode dividir os convidados em mesas completas de seis
pessoas ou em mesas completas de sete pessoas. Em ambos os casos, so necessrias
mais do que 10 mesas e todos os convidados cam em alguma mesa. Quantos so os
convidados?
27. Medida do cateto Na gura dada, ABCD um retngulo e ABE e CDF so tringulos retngulos. A rea
do tringulo ABE 150 cm2 e os segmentos AE e DF medem, respectivamente, 15 e 24 cm. Qual o comprimento do
segmento CF ?

15

F
E

24
C

28. Sequncia de Peri Usando apenas os dgitos 1, 2, 3, 4 e 5, Peri construiu a


sequncia
1, 2, 2, 3, 3, 3, 4, 4, 4, 4, 5, 5, 5, 5, 5, 1, 1, 1, 1, 1, 1, 2, 2, 2, 2, 2, 2, 2, . . . ,
comeando com um 1, seguido de dois 2, trs 3, quatro 4, cinco 5, seis 1, sete 2, e assim
por diante. Qual o centsimo termo dessa sequncia?
29. rea em azulejo A gura dada foi montada com 12 azulejos quadrados de lados iguais a 10 cm. Qual a rea da regio
destacada?
30. Os cartes de Capitu Capitu tem cem cartes numerados de 1 a 100. Todos
cartes tm uma face amarela e a outra vermelha e o nmero de cada carto est
escrito em ambas as faces. Os cartes foram colocados sobre uma mesa, todos com a
face vermelha voltada para cima. Capitu virou todos os cartes de nmero par e depois
todos os cartes de nmero mltiplo de 3, colocando-os com a face amarela voltada
para cima. Quantos cartes caram com a face vermelha para cima?
31. Enchendo o tanque Para encher de gua um
tanque em forma de um bloco retangular de 3 m de
comprimento, 50 cm de largura e 0,36 m de altura,
um homem utiliza um balde cilndrico, de 30 cm de

36 cm
3m
50 cm

74

OBMEP 2010

Nvel 3

dimetro em sua base e 48 cm de altura, para pegar gua numa


fonte. Cada vez que ele vai fonte, ele enche 4/5 do balde e
no caminho derrama 10% do seu contedo. Estando o tanque
inicialmente vazio, quantas viagens fonte o homem ter de
fazer para que a gua no tanque chegue a 3/4 de sua altura?

48 cm

30 cm

32. Fator primo Qual o maior fator primo de 2 006?


33. Altura de salrio Entre 1986 e 1989, a moeda do nosso pas era o cruzado (Cz$).
De l para c, tivemos o cruzado novo, o cruzeiro, o cruzeiro novo e, hoje, temos o real.
Para comparar valores do tempo do cruzado e de hoje, os economistas calcularam que
1 real equivale a 2 750 000 000 cruzados. Imagine que a moeda no tivesse mudado e
que Joo, que ganha hoje 640 reais por ms, tivesse que receber seu salrio em notas
de 1 cruzado, somente. Se uma pilha de cem notas de 1 cruzado mede 1,5 cm de altura,
qual seria a altura (em quilmetros) do salrio do Joo?
(a) 26,4

(b) 264

(c) 26 400

(d) 264 000

(e) 2 640 000

34. S bala H 1 002 balas de banana e 1 002 balas de ma numa caixa. Lara tira, sem
olhar o sabor, duas balas da caixa. Se q a probabilidade de as duas balas serem de
sabores diferentes e p a probabilidade de as duas balas serem do mesmo sabor, qual
o valor de q p?
(a) 0

(b)

1
2 004

(c)

1
2 003

(d)

2
2 003

(e)

1
1 001

35. Distncia ao centro Um ponto P est no centro de um quadrado de 10 cm de


lado. Quantos pontos da borda do quadrado esto a uma distncia de 6 cm de P ?
(a) 1

(b) 2

(c) 4

(d) 6

(e) 8

36. Potncias e potncias Se 2 22x = 4x + 64, qual o valor de x?


(a) 2

(b) 1

(c) 1

(d) 2

(e) 3

37. Um raio de luz Dois espelhos formam um ngulo de 30 no ponto V. Um raio


de luz parte de um ponto S paralelamente a um dos espelhos e reetido pelo outro
espelho no ponto A, como mostra a gura.
S
Depois de uma certa quantidade de reexes, o raio retorna
a S. Se AS e AV medem, ambos, 1 metro, qual o comprimento (em metros) do trajeto percorrido pelo raio de luz?

(a) 2
(b) 2 + 3
(c) 1 + 2 + 3
(d) 2(1 + 3)

OBMEP 2010

A
30

(e) 5 3
75

Nvel 3
38. Diferena de quadrados Determine o valor de (666 666 666)2 (333 333 333)2 .
39. Escada de nmero Na gura, o nmero 8 foi obtido somandose os dois nmeros diretamente abaixo de sua casa. Fazendo-se
o mesmo para preencher as casas em branco, obtm-se o 42 na
casa indicada. Qual o valor de x?
(a) 7

(b) 3

(c) 5

(d) 4

(e) 6

42

8
3

40. Diferena de potncias Seja n = 9 867. Se voc calculasse n3 n2 , qual seria o


algarismo das unidades encontrado?
(a) 0

(b) 2

(c) 4

(d) 6

(e) 8

41. Parbola girada O grco da parbola y = x2 5x + 9 rodado de 180 em torno


da origem. Qual a equao da nova parbola?
(a) y = x2 + 5x + 9

(c) y = x2 + 5x 9

(b) y = x2 5x 9

(d) y = x2 5x + 9

(e) y = x2 5x 9

42. Logotipo A gura mostra a marca de uma empresa, formada por


dois crculos concntricos e outros quatro crculos de mesmo raio, cada
um deles tangente a dois dos outros e aos dois crculos concntricos.
O raio do crculo menor mede 1 cm. Qual , em centmetros, o raio
do crculo maior?
43. Padeiro cansado Um padeiro quer gastar toda sua farinha para fazer pes. Trabalhando sozinho, ele conseguiria acabar com a farinha em 6 horas. Com um ajudante, o
mesmo poderia ser feito em 2 horas. O padeiro comeou a trabalhar sozinho e, depois
de algum tempo, cansado, ele chamou seu ajudante e assim, aps 150 minutos a farinha
acabou. Durante quantos minutos o padeiro trabalhou sozinho?
(a) 30

(b) 35

(c) 40

(d) 45

(e) 50

44. Muitas diagonais Calcule o nmero de diagonais de um prisma hexagonal reto,


como o da gura esquerda. Calcule o nmero de diagonais do poliedro obtido a
partir de um cubo pelo corte de seus oito vrtices, como o da gura direita. (Esse
poliedro muito utilizado na fabricao de dados, pois o corte prximo a cada um de
seus vrtices arredonda o dado e facilita a sua rolagem.)
*

*
*

*
*

*
*

76

OBMEP 2010

Nvel 3
45. Promoo de sabonete Uma loja de sabonetes realiza uma promoo com o anncio Compre um e leve outro pela metade do preo. Qual seria uma outra promoo
que a loja poderia fazer oferecendo o mesmo desconto percentual?
(a) Leve dois e pague um

(d) Leve trs e pague um

(b) Leve trs e pague dois

(e) Leve quatro e pague trs

(c) Leve cinco e pague quatro


46. Qual o ngulo? Na gura, os dois tringulos ABC e DEF so equilteros.
Qual o valor do ngulo x?
B

(a) 30

(b) 40

(c) 50
G

(d) 60

75

(e) 70

65
D

47. Caixa de papelo A gura mostra um pedao de papelo que ser dobrado e
colado ao longo das bordas para formar uma caixa retangular. Os ngulos nos cantos
do papelo so todos retos. Qual ser o volume, em cm3 , da caixa?
(a)

1 500

(b)

3 000

(c)

4 500

(d)

6 000

(e) 12 000
48. Soma de vizinhos Numa sequncia, cada termo, a partir do terceiro, a soma
dos dois termos imediatamente anteriores. Se o segundo termo 1 e o quinto termo
2005, qual o sexto termo?
(a) 3 002

(b) 3 008

(c) 3 010

(d) 4 002

(e) 5 004

49. Algarismos crescentes Quantos nmeros entre 10 e 13 000, quando lidos da esquerda para a direita, so formados por algarismos consecutivos e em ordem crescente?
Por exemplo, 456 um desses nmeros, mas 7 890 no .
(a) 10

(b) 13

(c) 18

(d) 22

(e) 25

50. Bloco girante Num bloco de 1 2 3 centmetros, marcamos trs faces com as
letras X, Y e Z, como na gura. O bloco colocado sobre um tabuleiro de 8 8
cm com a face X virada para baixo, em contato com o tabuleiro, conforme mostra a
gura. Giramos o bloco de 90 em torno de uma de suas arestas de modo que a face Y
que virada para baixo (isto , totalmente em contato com o tabuleiro). Em seguida,
giramos novamente o bloco de 90 em torno de uma de suas arestas, mas desta vez de
modo que a face Z que virada para baixo.
OBMEP 2010

77

Nvel 3
Giramos o bloco mais trs vezes de 90 em torno de
uma de suas arestas, fazendo com que as faces X, Y e Z
quem viradas para baixo, nessa ordem. Quantos quadradinhos diferentes do tabuleiro estiveram em contato
com o bloco?
(a) 18

(b) 19

(c) 20

(d) 21

(e) 22
x
f (x)

51. Iterando um ponto A funo f dada pela tabela

1
4

2
1

3
3

4
5

5
2

Por exemplo, f (2) = 1 e f (4) = 5. Quanto vale f (f (f (f (. . . f (4) . . .))))?


2004 vezes
(a) 1

(b) 2

(c) 3

(d) 4

(e) 5

52. Esmeralda e o 21 Esmeralda escreveu em ordem crescente todos os nmeros de 1


a 999, sem separ-los, formando o nmero
12345678910111213 . . . 997998999.
Quantas vezes aparece o agrupamento 21, nessa ordem?
1
4

(e)

1
120

53. Muitos fatores Qual o valor do produto 1


(a)

10
125

(b)

5
9

(c)

3
5

(d)

8
15

1
9

54. Falta um ngulo Quanto mede, em graus, o ngulo da


gura?
(a) 20

(b) 25

(c) 30

(d) 35

1
1
1
?
16
225

30

(e) 40
50

40

55. Soma de distncias Da gura, conclumos que |z x| + |w x| igual a


x y

(b) 12

1 0

(a) 11

z
3,7

9,3

(c) 13

(d) 14

(e) 15

56. Espiral do Artur Artur quer desenhar uma espiral de 4


metros de comprimento, formada de segmentos de reta. Ele
j traou sete segmentos, como mostra a gura. Quantos
segmentos ainda faltam traar?
(a) 28
78

(b) 30

(c) 24

(d) 32

(e) 36

OBMEP 2010

Nvel 3
57. Quais so os ngulos? A gura mostra um retngulo
e suas duas diagonais. Qual a armativa correta a respeito
dos ngulos x e y indicados na gura?
(a) x < y

(c) 2x = 3y

(b) x = y

(e) x = 3y

(d) x = 2y

................................................................................
.
... .
.
..
................................................................................
........ .
.
.. . .
. ..........
.
.....
.
.
..... .
..... .
.....
.
.
.
.
.....
.
.....
.
.....
.....
.
.
.
.
.....
.
.....
.
.....
.....
.
.
.
.
.....
.....
.
.....
.....
.
.
.
..
.
.
.
.....
.
.....
.....
.
.
.....
.
.
.
.....
...
.
.
.....
.
.
.....
.
.
.....
..... ..... .
.
.
..... ...... .
.
.
.
.
.
.....
.
.
...
..
.
.
.
.
.
.
.
.
..... ...... .
.
..... ......... .
.
.
..
.
.
.
.
.....
.
.
.....
.....
.....
.
.
.
..
.
.
.....
.
.....
.
.
.....
.
.....
.
.
.
.....
..
.
.....
.
.
.
.
.....
.
.....
.....
.
.....
.
.
..
.
.
.
.
.....
.....
.....
.
.....
.
.
.
..
.
.
..... .
..... .
. ........
. ........
.
.
.. .
.
.
................................................................................
.................................................................................
. ..
. ...
.... .
. ...

58. Raiz menor Qual a menor das razes da equao 2(x 3 5)(x 5 3) = 0?
59. Comparando reas Seja v a soma das reas das regies
pertencentes unicamente aos trs discos pequenos na gura
(em cinza claro) e seja w a rea da regio interior pertencente
unicamente ao maior disco (em cinza escuro). Os dimetros
dos crculos so 6, 4, 4 e 2. Qual das igualdades dadas
verdadeira?
(a) 3v = w
(c) v = w
(e) v = w
(b) 3v = 2w

(d) v = 3w

60. Menor raiz Qual a menor raiz da equao


(a)

1
3

(b)

1
2

(c)

1
3

(d)

1
4

|x 1|
= 6?
x2
3
(e)
2

61. Toalha redonda Uma mesa quadrada tem 1 metro de lado. Qual o menor dimetro
(em metros) de uma toalha redonda que cubra completamente o tampo da mesa?

(a) 1
(b) 1,5
(c) 2
(d) 2
(e) 3
62. Solues reais Qual o conjunto formado por todos os valores reais positivos de
x tais que (x 1)(x 2)(x 3) < 0?
(a) (1, 2)

(b) (1, 3)

(c) (0, 1) (2, 3)

(d) (0, 3)

(e) (0, 2)

63. Cossenos crescentes Num tringulo retngulo, denimos o cosseno de um ngulo


cateto adjacente
.
agudo por cos =
hipotenusa
B

O tringulo retngulo da gura tem cateto OA = 1.


Escreva, em ordem crescente, os cossenos dos ngulos
de 25 , 41 e 58 .

N
17
0

16
25
1

M
A

64. Central telefnica Os ramais de uma central telefnica tm apenas 2 algarismos,


de 00 a 99. Nem todos os ramais esto em uso. Trocando a ordem de dois algarismos
de um ramal em uso, ou se obtm o mesmo nmero ou um nmero de um ramal que
no est em uso. Qual o maior nmero possvel de ramais em uso?
(a) Menos do que 45

(c) Entre 45 e 55

(b) 45

(d) Mais do que 55


OBMEP 2010

(e) 55

79

Nvel 3
65. Horrio de avio Um nibus, um trem e um avio partem no mesmo horrio
da cidade A para a cidade B. Se eu tomar o nibus, cuja velocidade mdia de 100
km/h, chegarei cidade B s 20 horas. Se eu tomar o trem, cuja velocidade mdia de
300 km/h, chegarei cidade B s 14 horas. Qual ser o horrio de chegada do avio
se sua velocidade mdia for de 900 km/h?
66. Discos de papelo Para fabricar nove discos de papelo circulares para o Carnaval
usam-se folhas quadradas de 10 cm de lado, como indicado na gura. Qual a rea
(em cm2 ) do papel no aproveitado?
(a) 25
(b) 22,5
(c) 21,5
(d) 21
(e) 22
67. Armaes absolutas Determine quais so as armaes verdadeiras.
(a) | 108| > 100

(b) |2 9| = 9 2

(c) | 6a| = 6|a|

(d) |5 13| = |5| |13|

(e) |a2 + 5| = a2 + 5

x
x+y
?
68. Frao radical Se = 5, quanto
y
2y
(a)

5
2

(b) 3 2

(c) 13y

(d)

25y
2

(e) 13

69. rea de tringulo A gura mostra um retngulo


KGST e um tringulo KGR. Os ngulos K RT e RGS
so iguais. Se T R = 6 e RS = 2, qual a rea do tringulo
KGR?

(a) 12 (b) 16 (c) 8 2 (d) 8 3 (e) 14

70. Pares de inteiros Quantos so os pares diferentes de inteiros positivos (a, b) tais
1
a+
b = 13 ?
que a + b 100 e
1
+b
a
(a) 1

(b) 5

(c) 7

(d) 9

(e) 13

71. Qual a soma? Se x + |x| + y = 5 e x + |y| y = 6, qual o valor da soma x + y ?


(a) 1
80

(b) 11

(c)

9
5

(d) 1
OBMEP 2010

(e) 11

Nvel 3
72. Crculo intermedirio Na gura, os trs crculos so
concntricos, e a rea do menor crculo coincide com a rea
do maior anel, destacado em cinza. O raio do menor crculo
5 cm e do maior 13 cm. Qual o raio (em cm) do crculo
intermedirio?

(a) 12
(c) 10 65
(e) 12 2

(b) 11
(d) 5 3
73. Fraes incompletas Encontre os algarismos que esto faltando em cada um dos
espaos marcados com traos.
(a)

126
21
=
8

(b)

8
33

4
5

74. Tringulos impossveis Quais dessas guras esto erradas?

75. Razo de reas Se um arco de 60 num crculo I tem o mesmo comprimento que
um arco de 45 num crculo II, encontre a razo entre a rea do crculo I e a rea do
crculo II.
(a)

16
9

(b)

9
16

(c)

4
3

3
4

(d)

(e)

6
9

76. Inequao errada Sendo x > 0, y > 0, x > y e z = 0, encontre a nica desigualdade
falsa.
(a) x + z > y + z
(b) x z > y z

(c) xz > yz
y
x
(d) 2 > 2
z
z

(e) xz 2 > yz 2

77. Equaes geomtricas Resolva as equaes dadas geometricamente, ou seja, interpretando o valor absoluto |a b| como a distncia entre a e b.
(a) |x 5| = 2

(b) |x + 3| = 1

(c) |3x 7| = 9

(d) |x + 2| = |x 5|

78. Pista circular A pista de um autdromo tem 20 km de comprimento e forma


circular, conforme gura.
OBMEP 2010

81

Nvel 3
Os pontos marcados na pista so A, que o ponto de partida; B, que dista 5 km de A no sentido do percurso; C,
que dista 3 km de B no sentido do percurso; D, que dista
4 km de C no sentido do percurso; e E, que dista 5 km
de D no sentido do percurso. Um carro que parte de A e
para aps percorrer 367 km estar mais prximo de qual
dos cinco pontos?
(a) A

(b) B

(c) C

(d) D

(e) E

79. Maior comprimento No diagrama dado, todos os quadradinhos tm 1 cm de lado. Qual dos segmentos dados o de maior
comprimento?
(a) AE
(b) CD + CF
(c) AC + CF
(d) F D
(e) AC + CE
80. Desigualdade entre inteiros Quantos dentre os nmeros 5, 4, 3, 2, 1, 0,
1, 2, 3 satisfazem a desigualdade 3x2 < 14?
(a) 1

(b) 2

(c) 3

(d) 4

(e) 5

81. Equao cbica Sobre a equao 2 007x3 + 2 006x2 + 2 005x = 0, o certo armar
que:
(a) no possui razes;

(d) tem apenas uma raiz real;

(b) tem trs razes reais distintas;

(e) tem trs razes positivas.

(c) tem duas razes iguais;


82. O perfume de Rosa Rosa ganhou um vidro de perfume com o formato de um
cilindro com 7 cm de raio da base e 10 cm de altura. Depois de duas semanas usando
o perfume, restaram 0,45 litros no vidro. Qual a frao que representa o volume que
Rosa j usou?
83. Igualdade com inteiros Quais nmeros naturais m e n satisfazem a equao
2 n + 1 = m2 ?
84. O caminho da pulga Para percorrer um caminho reto de 10 metros de comprimento, uma pulga usa a seguinte estratgia: a cada dia, ela percorre a metade do
caminho que falta. Assim, ela percorre 5 metros no primeiro dia, 2,5 metros no segundo, e assim por diante (o tamanho da pulga pode ser desconsiderado).
(a) Quantos metros ela ter percorrido ao nal do stimo dia? E do dcimo?
(b) A partir de qual dia a pulga estar a menos de 0,001 m do nal do caminho?
82

OBMEP 2010

Nvel 3
85. Uma soma alternada Se Sn = 1 2 + 3 4 + 5 6 + + (1)n+1 n para cada
inteiro positivo n, ento S1992 + S1993 igual a
(a) 2;

(b) 1;

(c) 0;

(d) 1;

(e) 2.

86. O raio da circunferncia Um arco de circunferncia mede 300 e o seu comprimento de 2 km. Qual o nmero inteiro mais prximo da medida do raio do crculo,
em metros?
(a) 157

(b) 284

(c) 382

(d) 628

(e) 764

87. Quatro passageiros Em um txi, um passageiro pode se sentar na frente e trs


passageiros atrs. De quantas maneiras podem se sentar quatro passageiros de um taxi
se um desses passageiros quiser car na janela?
88. Os cinco crculos Cinco discos de mesmo raio esto dispostos como mostra a gura. Quatro centros so os vrtices de um
quadrado e trs esto alinhados. Trace uma reta que divida a gura
formada pelos cinco discos em duas partes de mesma rea.

......
.......
............ ..............
.... ...... ..... ......
...
.. ..
..
..
..
..
.
..
.
.
.
..
.
.
.
.
.
.
.
.
.
.
.
.
.
.
.
..
.
..
...
.
..
.
...
.. ...
..
....
.. .....
..
............
............
...........
...........
..........
..........
. .....
. .....
.
..
.. ..... .... .....
..
.. .....
...
.. ....
.. ....
..
...
.. ..
..
..
..
..
..
.
..
..
..
.
.
.
.
.
.
.
.
.
.
.
.
.
.
.
.
.
.
.
.
.
.
.
.
..
..
.
..
..
.....
.....
..
.. ...
.. ...
..
...
. .
. .
.
...
............. ..............
.............
............. ..............
............

B...........................................................................................................C

89. O tringulo e o quadrado Na gura dada, ABCD um


quadrado cujo lado mede 1 cm, E o ponto mdio da diagonal
BD e F o ponto mdio do segmento BE. Qual a rea do tringulo CBF ?

. ...
.
. ...
.
......
. ... ............
.
.
.
.
.
...........
....
.
.
.
.
.
...
.
.
..
.
.
.
.
..
...
.
.
.
...
.
.
.
...
.
.
.
..
.
.
.
..
.
...
.
.
.
...
.
.
.
..
.
.
.
..
.
...
.
.
.
...
.
.
.
...
.
.
.
..
.
.
..
.
...
.
.
... .
.
.
.. .
.
.. .
.
... .
.
.
.
... .
.
.
.
.
.
..
...................................
.
..................................
.
...

E
r

90. Uma refeio Um sanduche e um prato de refeio custam R$ 5,00 e


R$ 7,00, respectivamente. De quantas maneiras pode-se comprar s sanduiches, s
pratos de refeio ou alguma combinao de sanduiches e pratos de refeio com
R$ 90,00, sem deixar troco?
y

91. Plano cartesiano O ponto P = (a, b) est marcado


na gura ao lado. Marque os pontos:

P
r

(a) A = (a + 1, b/2);

(b) B = (a/2, b 1);

(c) C = (a, b);

(d) D = (1 a, b 2).

2
3

92. Soma dos terminados em 9 A soma Sn = 9 + 19 + 29 + 39 + + an denota a


soma dos primeiros n nmeros naturais terminados em 9. Qual o menor valor de n
para que Sn seja maior do que 10 5 ?
93. Trs cilindros Trs cilindros de volumes V1 , V2 e V3 tm alturas e raios das bases
iguais a 10 e 10 cm, 10 e 5 cm e 20 e 5 cm.
OBMEP 2010

83

Nvel 3

.............................
...............................................
.
.
.
. ............
.
.
.
.
.
.
.
.
.
.
.
.
.
.
.
.
.
.
.
.
.
.
.
.
.
.
.
.
.
. ..................................... .
. .
. ..
.
.............
. ...................................
..

...................
.........................
...
.
.
.
.
.
.
.
.
.
.
.
.
.
.
.
.
.
.
.
.
.
.
.
.
.
.
.
.
.
.
.
.
.
. ................. .
..
.
....... .....
.. ........... ...

.........................
......................
.
.
.
.
.
.
.
.
.
.
.
.
.
.
.
.
.
.
.
.
.
.
.
.
.
.
.
.
.
.
.
.
.
.
.
.
.
.
.
.
.
.
.
.
.
.
.
.
.
.
.
.
.
.
.
.
.
.
.
.
.
.
.
.
.
.
.
.
.
.
.
.
.
.
.
.
.
.
.
.
.
.
.
.
.
. .. .
.
. ....... ........ .
....
.
....
...
. ..................

(a) Escreva em ordem crescente os volumes V1 , V2 e V3 dos trs cilindros.


(b) D as dimenses de um cilindro cujo volume V4 esteja entre V2 e V3 .
(c) D as dimenses de um cilindro cujo volume V5 esteja entre V1 e V3 .
94. Porcentagem de mortalidade Se 15% dos membros de uma populao foram
afetados por uma doena e 8% dos afetados morreram, a porcentagem da mortalidade
em relao populao inteira foi de:
(a) 1,2% ;

(b) 1,8% ;

(c) 8% ;

(d) 12% ;

(e) 23% .

95. Agenda de aulas Eliane quer escolher o seu horrio para a natao. Ela quer ir
a duas aulas por semana, uma de manh e outra de tarde, no sendo no mesmo dia,
nem em dias seguidos. De manh, h aulas de natao de segunda-feira a sbado, s
9h, s 10h e s 11h e de tarde, de segunda a sexta-feira, s 17h e s 18h. De quantas
maneiras distintas pode Eliane escolher o seu horrio?
96. Jogo de cartas Um grupo de amigos disputa um jogo no qual 16 cartas (sendo
quatro ases, quatro reis, quatro damas e quatro valetes) esto inicialmente dispostas
em quatro pilhas de quatro cartas. O jogo consiste em mover sucessivamente a carta
superior de uma pilha e coloc-la sobre uma outra pilha, at obter quatro novas pilhas,
em que na primeira pilha s tenha ases, na segunda, s valetes, na terceira s damas
e na quarta pilha s reis. Ganha o jogo quem zer o menor nmero de movimentos.
Com quantos movimentos sempre possvel terminar o jogo? Na gura dada, aparece
a disposio inicial das cartas nas pilhas.
pilha 1

pilha 2

rei de

dama de

dama de

valete de
s de

s de

s de

valete de

pilha 3
rei de
valete de
dama de
dama de

pilha 4
valete de
rei de
s de

rei de

97. Fraes inteiras Quantos nmeros inteiros positivos n existem tais que o quociente
2n2 + 4n + 18
seja um inteiro?
3n + 3
98. Quatro prefeitos e um crculo Quatro prefeitos decidem construir uma rodovia
circular que passe dentro dos limites de suas cidades. Como as quatro cidades no esto
sobre um mesmo crculo, os prefeitos contratam uma empresa para elaborar um projeto
84

OBMEP 2010

Nvel 3
para a construo de uma rodovia circular equidistante das quatro cidades. Qual o
maior nmero de projetos geogracamente distintos que a empresa pode elaborar?
99. Fatoriais Se n um nmero inteiro positivo, denotamos por n! o produto de
todos os inteiros de 1 a n. Por exemplo, 5! = 1 2 3 4 5 = 120 e 13! =
1 2 3 4 5 12 13. Por conveno, escrevemos 0! = 1! = 1. Encontre trs
nmeros inteiros a, b e c entre 0 e 9, que sejam distintos e tais que o nmero de trs
algarismos a b c seja igual a a! + b! + c!.
100. O Riquinho Riquinho distribuiu 1 000,00 reais entre os seus amigos Antnio,
Bernardo e Carlos da seguinte maneira: deu, sucessivamente, 1 real ao Antnio, 2
reais ao Bernardo, 3 reais ao Carlos, 4 reais ao Antnio, 5 reais ao Bernardo etc. Qual
foi a quantia recebida por Bernardo?
101. Retngulo com dimenses inteiras As diagonais de um retngulo medem

1 993 cm. Quais so as dimenses do retngulo, sabendo que elas so nmeros inteiros?
102. Mltiplos de 3 e quadrados perfeitos Escreve-se em ordem crescente os mltiplos
de 3 que, somados com 1, sejam quadrados perfeitos, ou seja, 3, 15, 24, 48, . . . . Qual
o mltiplo de 3 na 2 006a posio?
103. Cinco cartas Cinco cartas esto sobre uma mesa, e cada uma tem um nmero numa
face e uma letra na outra. Simone deve decidir se a seguinte frase verdadeira: Se
uma carta tem uma vogal numa face, ento ela tem um nmero par na outra. Qual
o menor nmero de cartas que ela precisa virar para tomar uma deciso correta?
...............
. .
. .
. .
.2.
. .
. .
. .
...............
. .

...............
. .
. .
. .
.3.
. .
. .
. .
...............
. .

...............
. .
. .
. .
.M.
. .
. .
. .
...............
. .

...............
. .
. .
. .
. A.
. .
. .
. .
...............
. .

...............
. .
. .
. .
.E.
. .
. .
. .
...............
. .

104. O lucro de uma companhia Uma companhia tem um lucro de 6% nos primeiros
R$ 1 000,00 reais de venda diria e de 5% em todas as vendas que excedam
R$ 1 000,00 reais, nesse mesmo dia. Qual o lucro dessa companhia, em reais, num
dia em que as vendas alcanam R$ 6 000,00 reais?
(a) 250

(b) 300

(c) 310

(d) 320

(e) 360

105. Sequncia triangular Encontre o 21o termo da sequncia que comea assim:
1; 2 + 3; 4 + 5 + 6; 7 + 8 + 9 + 10; 11 + 12 + 13 + 14 + 15; . . .
OBMEP 2010

85

Nvel 3
106. O jardim octogonal A gura mostra a planta de um
jardim de uma cidade, feita num papel quadriculado. O
jardim tem a forma de um polgono de oito lados com
uma roseira quadrada no centro, cercada de grama. A
rea total do jardim de 700 m2 . Para colocar uma cerca
em volta do jardim e da roseira, o prefeito dispe de, no
mximo, R$ 650,00.
Qual o maior preo que o prefeito poder pagar pelo
metro dessa cerca?

..........................
.....
.....
.....
....
.....
....
..
.
...
.
.. .
.....
.
...
......
.
.
.. ...
..
..
..
.
.
..
..
..
..
..
.
.
..
..
..
.
..
..
..
.
.
.. roseira..
.
..
..
..
..
..
..
.
.
..
..
.
..
..
..
..
..
.
.
..
..
.. ...
.. ...
..
.
..
.. ..
.. ..
.. .
..
....
.
.
..
.....
..
.
.....
....
.....
....
............................

107. Nmero de caracteres Numa folha de papel cabem 100 caracteres na largura e
100 na altura. Nessa folha so escritos sucessivamente os nmeros 1, 2, 3, e assim por
diante, com um espao entre cada um e o seguinte. Se no nal de uma linha no houver
espao para escrever o nmero seguinte, ele escrito no comeo da linha seguinte. Qual
o ltimo nmero escrito na folha?
108. A rvore de Emlia A rvore de Emlia cresce de acordo com
a seguinte regra: aps duas semanas do aparecimento de um galho,
esse galho produz um novo galho a cada semana e o galho original
continua crescendo. Depois de cinco semanas, a rvore tem cinco
galhos, como mostra a gura. Quantos galhos, incluindo o galho
principal, a rvore ter no nal de oito semanas?

q q q

.
.
.
.
.
.
.
.
.
.
.
.
.
.
.
.
.
.
.
.
.
.
.
.
..
.
.
.
.
.
.
.
.
. .
.
.
. ....
.
.
. .. .
. .
.
. ..
. ..
.
. ....
. ...
.
......
.
.
.
.
.
.
....
.
.
......
.
.
.
.....
.
.
.
.
.
.
.
.
.
.
.
.
.
.
.
.
.
.
.
.
.
.
.
.
.
.
.
.
.
.
.
..
.
.
.
..
.
.
..
..
...
.
..
.
.
........... .
.
.......... .
.
.
.
.
.
.
.
.
.
.
.
.
.
.
.
.
.
.
..
.
..
.
..
.
. ..
.
. ...........
. .........
.
.
.
.
.
.
.
.
.
.
.
.
.
.
.
.
.
.
.
.
.
.
.
.
.
.

q
q

109. Um teste vocacional Foi aplicado um teste vocacional em 1 000 alunos de uma
escola. A tabela a seguir apresenta os resultados, por rea de estudo e sexo.
Exatas

Humanas

Biolgicas

Masculino

232

116

207

Feminino

112

153

180

Se um aluno for escolhido ao acaso, determine a probabilidade desse aluno ser:


(a) da rea de exatas;
(b) da rea de humanas, sendo do sexo masculino;
(c) do sexo feminino, sendo da rea de biolgicas.

110. Dois setores circulares A rea do crculo da gura mede


20 cm2 . Se AOB = 60 e C OD = 30 , quanto mede a rea da
regio do crculo que est destacada?

86

OBMEP 2010

As

B
s

.................
.........................
........
.....
....
. ...
.....
...
....
........
.... ..
.. ......
... ...
..
..
.
...
..
..
..
..
.
..
..
..
.
..
..
..
..
..
..
..
.
..
..
..
..
.
.
..
.
.
.
..
..
.
.
.
.
.
.
.
..
.
..
.
.. .
.
.
.
.
.. . .
.
.
.
.. .
.
.
.
.
.
.............................
...............................
.
..
....
.
.
.
....
.
....
.
.
.
....
.
....
.
.
.
....
.
....
.
.
.
....
.
.
....
.
.
....
.
.
....
.
.
.... .
..
.... ..
..
....
...
..
..
..
..
..
..
..
..
..
...
...
...
...
....
.
.
....
.....
....
....
.....
....
.......
......... ..............
...................
......

sC

sD

Nvel 3
111. Compra de televisores Maria encomendou um certo nmero de televisores para
o estoque de uma grande loja, pagando R$ 1 994,00 por televisor. Ela reparou que,
no total a pagar, no aparece o algarismo 0, nem o 7, nem o 8 e nem o 9. Qual foi o
menor nmero de televisores que ela pode ter encomendado?
112. Distncia entre nmeros Considere os nmeros reais a, b, c e d representados em
uma reta, conforme mostra a gura. Determine quais das armaes so verdadeiras
e quais so falsas.
a
4

d
0

(a) |a| < 4

(d) |a| > |b|

(g) |a b| < 4

(j)

(b) |b| < 2

(e) |c| < |d|

(h) |a b| 3

(k) |b c| > 3

(c) |c| < 2

(f)

(i)

(l)

|a| < |d|

|c d| < 1

|b c| < 2
|c a| > 1

113. Cartes premiados Uma loja distribui 9 999 cartes entre os seus clientes. Cada
um dos cartes possui um nmero de quatro algarismos, entre 0001 e 9999. Um carto
premiado se a soma dos primeiros dois algarismos for igual soma dos dois ltimos;
por exemplo, o carto 0743 premiado. Prove que a soma dos nmeros de todos os
cartes premiados divisvel por 101.
114. O preo da gasolina Encher o tanque de gasolina de um carro pequeno custava,
em valores atualizados, R$ 29,90 em 1972 e R$ 149,70 em 1992. Qual dos valores
abaixo melhor aproxima o percentual de aumento do preo da gasolina nesse perodo
de 20 anos?
(a) 20%

(b) 125%

(c) 300%

(d) 400%

(e) 500%

115. O tringulo de moedas Um menino tentou alinhar 480 moedas em forma de um


tringulo, com uma moeda na primeira linha, duas moedas na segunda linha, e assim
por diante. Ao nal da tentativa, sobraram 15 moedas. Quantas linhas tem esse
tringulo?
116. Circunferncia e tringulo retngulo Inscreve-se uma circunferncia num tringulo retngulo. O ponto de tangncia divide a hipotenusa em dois segmentos que
medem 6 e 7 cm. Calcule a rea desse tringulo.
1 1
1
117. Soma de razo 1 Se Sn = + 2 + + n , qual o menor nmero inteiro positivo
2
2 2
2
n tal que Sn > 0,99?
118. Soma de razes quadradas

r2 5
.
3, mostre que 6 =
2

(b) Se s = 215 + 300, mostre que s2 > 1 015.


(a) Se r =

2+

OBMEP 2010

87

Nvel 3
119. Duas rodas Na gura dada, a roda A gira a 1 200 voltas por
minuto e a roda B a 1 500 voltas por minuto. Calcule os raios dessas
duas rodas.

..
.....
....... .
..... ............
.......
...
.......................
.........
...
.
..........
.
............. ..
.. ..
.. ....... .........
.. .....
.. ..
.. .
.. .
. ..
. ..
..
..
. ..
...
..
..
..
..
.
..
..
.
..
..
.
.
.
..
.
.
.
.
..
..
.
.
.
.
.
.
.
...
..
.
..
..
. ..
.
.
..
.
.
....
.. ....
.. .
.. .
. ...
.. ......
.
.
.
.
.
............. .
............ .
. .....
...
. ........ .........
.
.
.............
.....
.
.
.
.
.
.
.
.
.
.
.
.

9 cm

120. Dois divisores O nmero 248 1 divisvel por dois nmeros compreendidos entre
60 e 70. Quais so esses nmeros?
(a) 61 e 63

(b) 61 e 65

(c) 63 e 65

(d) 63 e 67

(e) 67 e 69

121. Rede de estaes Um servio de vigilncia vai ser instalado num parque na forma
de uma rede de estaes. As estaes devem ser conectadas por linhas de telefone, de
modo que qualquer uma das estaes possa se comunicar com todas as outras, seja por
uma conexo direta, seja por meio de, no mximo, uma outra estao.
Cada estao pode ser conectada diretamente por um cabo a, no mximo, trs outras estaes. O diagrama mostra um exemplo de uma
rede desse tipo, conectando sete estaes. Qual o maior nmero de
estaes que podem ser conectadas dessa maneira?

....
........
... ...
.
.
.
.
.
.
.. ..
.. ...
.. .
.
..... .....
..... ....
..
..
.
.
.....
.
.....
.
....
.
.....
. .....
..... ........ . ........
. .....
.....
.........
.... ....
.. .. .... ..
.
.
.......... ........... .
. ..
.
.
.
.
.
.
.
.. ..
..
... .............. ............. ...
. .. .. .
.........
. .
....
.... ..............
. .............
.
.
.
. ....... . .
.
....... . .
. .
.
.
.
.
. .
.
.
.
. .
.......
.
......
. .
. ........ .
.... ..
. ....
.
.
......
... ...... ........
. ..
..
.........
... ...
.. . .
... ...
. ............ ............ ..
.
.
.
.
.
.
.
.
.
.
.
.
.
.
... .............. .............. ...
.........
.........
.........
....
....
....

122. Bolas brancas e pretas Uma caixa tem exatamente cem bolas pretas e cem bolas
brancas. Repetidamente, trs bolas so retiradas da caixa e substitudas por outras
bolas, que esto em um saco, da maneira seguinte.
BOLINHAS REMOVIDAS
3 pretas

SUBSTITUDAS POR
= 1 preta

2 pretas e 1 branca

= 1 preta e 1 branca

1 preta e 2 brancas

= 2 brancas

3 brancas

= 1 preta e 1 branca

Qual pode ser o contedo da caixa depois de seguidas aplicaes desse procedimento?
(a) 2 pretas

(b) 2 brancas

(c) 1 preta

(d) 1 preta e 1 branca

(e) 1 branca.

123. O cubo Alice tem uma folha de cartolina de 60 por 25 cm. Ela quer cortar a
folha para montar um cubo, com arestas medindo um nmero inteiro de centmetros.
Permitindo cortes mas no permitindo superposio, qual o cubo de maior volume
que ela pode construir?
124. Um quadrado e um tringulo Na gura, ABCD
um quadrado, cuja rea mede 7/32 da rea do tringulo
XY Z. Qual a razo entre XA e XY ?

X
...
....
..... ..
..... .
A..............................................................................B
...
....
...
...

..
.
. ..
.
.....
.....
.
.
.
.
. ..
.
.....
.....
. ...
.
.
.
.
.
.....
.....
. .
.
. ..
.
.
.
.....
.....
.
.
..
.
.
..
.....
.
.
.....
.
.
..
..
.
.
..
.
.
.....
.
.
.
.....
.
.
..
...
.
.
.
.
............................................................................
..
..
.
............................................................................

125. A urna Uma urna tem seis bolas numeradas de 1 a 6. Se duas bolas so extradas,
qual a probabilidade de a diferena entre os nmeros dessas duas bolas ser igual a 1?
88

OBMEP 2010

Nvel 3
126. Soma das razes de uma equao Determine a soma das razes distintas da
equao x2 + 3x + 2 = |x + 1|.
127. Produto de trs nmeros No diagrama dado, cada um dos 10 crculos representa
um algarismo. Preencha o diagrama com uma igualdade vlida, colocando, em cada
crculo, um dos algarismos de 0 a 9 e utilizando cada algarismo uma nica vez.

..........
..........
..
.
..
.
.
.
.
.
.
.
..
.
..
...........
........ .

....
.....
......... ...........
... .... .... ....
..

....
....
.....
......... ........... ...........
... .... .... .... .... ....
..

....
....
....
.....
......... ........... ........... ...........
... .... .... .... .... .... .... ....
..

...................................................................... ......................................................................................................... = ............................................................................................................................................

128. rea do tringulo Determine a rea do tringulo


ABC mostrado na gura.

y 6

A..... = (1, 2)
..
.
..
.

. .....
. ......
.
.....
.
.....
.
.
.....
.
.....
.
.
.....
.
.....
.
.
.....
.....
.
.
.....
.
.....
.
.
.....
.
.....
.
.
.....
.
....
.
.
.
.
..
.
.
.
....
....
.
.
.
....
....
.
.
.
.
.
.
....
....
.
.
.
.
....
....
.
.
.
..
.
.
....
.
....
.
.
.
....
....
.
.
.
....
....
.
.
. ......
. . ...
. .
. ......
. ...
. ..
...
.
.
.
..

B = (8, 0)
x

C = (1, 6)

129. Duas tabelas As linhas da primeira tabela dada so todas progresses aritmticas
de uma mesma razo e as colunas dessa tabela so todas progresses aritmticas de
uma mesma razo. Na segunda tabela dada foi utilizada a mesma lei de formao, mas
algum apagou alguns nmeros deixando apenas trs. Qual o nmero que estava na
posio indicada com ?
5

11

14

17

12

15

18

21

24

19

22

25

28

31

26

29

32

35

38

33

36

39

42

45

39
87
56

130. A sequncia abc A lei de formao da sequncia 10, a, 30, b, c, . . . , a partir de seu
terceiro termo, consiste em tomar o dobro da soma dos dois termos imediatamente
anteriores. Qual o valor de c?
131. Permetro e diagonal O permetro de um retngulo ABCD mede 20 m. O menor
comprimento que pode ter a diagonal AC, em metros, :

(a) 0;
(b) 50;
(c) 10;
(d) 200;
(e) 20 5.
132. As idades numa classe Numa classe na escola, todos os alunos tm a mesma
idade, exceto sete deles que tm 1 ano a menos e dois deles que tm 2 anos a mais.
A soma das idades de todos os alunos dessa classe 330. Quantos alunos tem nessa
classe?
OBMEP 2010

89

Nvel 3
133. A mesa redonda Uma mesa redonda tem 1,40 metros de dimetro.
Para uma festa, a mesa ampliada colocando-se trs
tbuas de 40 cm de largura cada uma, como mostra
a gura. Se cada pessoa mesa deve dispor de um
espao de 60 cm, quantos convidados podero se sentar
mesa?

............
.................
......
...
....
..
...
..
..
..
..
..
..
..
..
.
.
.
.
.
.
.
................................
.
................................
.
.
.
.
.
.
.
.
.
.
.
.
.
.
..
..
.
..
..
.
..
..
...
..
...
..
......
....... ..........
............
...

mesa fechada

............
.................
......
...
....
..
...
..
..
..
..
..
..
..
.
..
.
.
.
.
.
.
.
................................
.
.
................................
.
.
.
.
.
.
.
................................
.
.
.
.
.
.
................................
.
.
.
.
.
................................
.
................................
.
.
.
.
.
.
.
.
.
.
.
.
................................
.
................................
.
.
.
.
.
.
.
.
.
.
.
.
.
..
.
..
.
..
..
.
..
..
...
..
...
......
...
...... ........
...........
......

mesa ampliada

134. Brincadeira com sete nmeros Sete nmeros inteiros positivos esto escritos
consecutivamente em ordem crescente numa mesma linha. Determine se possvel
colocar entre esses nmeros cinco sinais de + e s um de = de tal modo que
resulte uma igualdade.

135. Um terreno compartilhado Trs amigas compraram um


terreno quadrado e querem reparti-lo em trs terrenos de mesma
rea, conforme indicado na gura, pois no canto do terreno indicado por A se encontra uma boa fonte de gua. A que distncia
do vrtice C do terreno devem car os pontos de divisa M e N
indicados na gura?

B ............................................................................................................... A
M
C

.
. ..
.
.
.
.... . .
.... .. .
.
.
.
.
.... .. .
.... .. .
.
.
.
.... .. .
.
.
.... ... .
.
.
.
.
....
.
.
....
.
.
.
.
..
....
....
.
.
..
.
..
.
.
.
.
..
....
.
.
..
....
.
.
.
..
.
.
.
.
..
....
.
.
..
....
.
.
.
..
.
.
.
. .......
..
.
. .......
..
.
.
.
.
.
...
..
.
.....
..
.
.
.
.
.
.
.
.
..
.
.
..
.
.
.
.
.
.
..
..
.
.
.
.
.
.
..
.
.
..
.
.
.
.
.
.
.
..
.
.
..
.
.
.
.
.
..........................................
...........................................
.
..
.

136. As duas partculas Duas partculas percorrem um caminho circular de 120 m


de comprimento. A velocidade de uma delas 2 m/s maior do que a da outra e ela
completa cada volta num tempo que 3 segundos inferior ao da outra. Qual a
velocidade de cada partcula?

137. Queda livre Um corpo em queda livre demora 11 segundos para tocar o solo. No
primeiro segundo ele percorre 4,5 m e, em cada segundo seguinte, a distncia percorrida
aumenta em 9,8 m. Qual a altura da queda e quantos metros ele percorreu no ltimo
segundo?

138. Um caminho triangular Janete passeia por um caminho de forma triangular


ABC, com o lado AB medindo 1 992 m. Ela gasta 24 minutos para percorrer esse
lado AB e, depois, com a mesma velocidade, ela percorre o outro lado BC seguido da
hipotenusa CA em 2 horas e 46 minutos. Qual o comprimento do lado BC?

139. O preo do feijo A tabela e o grco dados mostram a evoluo do preo mdio
de trs tipos A, B e C de feijo na bolsa de alimentos durante os primeiros quatro
meses de um certo ano. Desses trs tipos, os que apresentaram, respectivamente, o
maior e o menor aumento percentual do preo nesse perodo so:
(a) A e B;
90

(b) A e C;

(c) B e C;
OBMEP 2010

(d) C e A;

(e) C e B.

Nvel 3
R$

6
110

100

90

80

d
t

t
d

60

fev.

jan.

fev.

mar.

abr.

65,67

83,33

96,67

103,33

73,30

80,50

99,55

109,50

64,50

71,57

89,55

100,00

jan.

70

d
t

.
..
..
..
..
..
..
..
..
..
..
..
.. ..
.. ...
.
..
..
.. ....
.. .....
..
..
.. ....
.. ..... .
.. ..... ...
.. ..... ....
. .....
..
......
..
..
....
....
..
..
...
...
..
..
..
..
... ...
.. ....
.
.. ..
.
..
..
.
.
...
...
.
.
...
..
...
...
.... ..
..
...
.... ..
.
.
. .
.
....
. .. ..
.
. ..
....
...
.
.
....
..
....
.
.
... .
.
.... .
..
... .
... .
.
..
.... ..
... ..
.
.
..
..
.
.. ......
....
.
..
.. ....
. .....
. ......
.......
.. .
...

t: A

mar. abr.

d: B

: C

140. Interseo de tringulos Os trs tringulos da gura se cortam em 12 pontos


diferentes. Qual o nmero mximo de pontos de interseo de trs tringulos quaisquer?
.
...
...
.. .
.
.. ..
.. ..
..
..
..
..
..
.
..
.
..
..
..
..
..
..
.
..
.
..
..
..
.
..
.
....
.....
..
. .
..
..
.........................
.. ...........................
.......... ...
........... ...
.
..
.
..
........................ ...
.................... ...
... .......
.
..
.
..
..
..
. ...............
..
..............
....
......
..
..
..
..
.
...............
...........
........................
.
...........
..
.
......
... ..........
.
..
.
.
...................
..
....................
................
..
.
... ..................
...
....
..............
..
..
.
..
..
............
. ........
..
.............
.
..
.
...
.
..... ... .
.... ... .
..
...
..
..
..
.........
.. .....................
.........
..
.. .. .................
.
. ...........
...
..
.
.. ..............
..
......... .
...........
.
.
.
.
..
.
.........
.
..
.........
............. ......................
............ ......................
..
..
..
..
..
.........
..
.........
..
........ .................. ........................ ...
..
.
........................... ............................ ...
..
..
.
................ .
. ....
.
................
...... ............
........................
.. .
..
..
..
...........
.........
..
.........
............
..
..
...........
..
.........
.........
...........
..
..
..
.
..
............
.........
.........
............
..
..
...............
.........
...
.
.
.........
............
..
.........
.........
..
......... ....
......... ....
.........
........
..

141. Comparar tringulos Na gura esto indicados


os comprimentos de todos os segmentos. Demonstre
que AC divide ao meio o ngulo DAB.

s
s
s

B
r

C
r

................
....................
...........
..
...........
..
.........
.......
..
..
.. .
... ...
..
..
..... ......
..... .......
..
..
.
.....
....
.....
....
..
.....
.....
..
....
....
.
..
...
....
..
....
.....
..
.....
.
.
....
....
....
....
..
..
.....
.....
....
....
..
.....
..
.....
....
.
....
..
..
..
....
.....
..
.....
....
.
.
..
....
.. .........
....
.. .........
.
....
....
........
.........
..
..
..
...
.......................................................................................................
.......................................................................................................
...
.

12

18

27

12

142. Queima de velas Dois tipos de vela tm o mesmo comprimento mas so feitas
de material diferente. Uma delas queima completamente em trs horas e a outra em
quatro horas, ambas queimando com velocidade uniforme. Quantos minutos depois
das 13 horas devem ser acesas simultaneamente as duas velas para que, s 16 horas, o
comprimento de uma seja o dobro do da outra?
(a) 24

(b) 28

(c) 36

(d) 40

(e) 48

143. Uma distrao Em vez de multiplicar certo nmero por 6, Jlia se distraiu e
dividiu o nmero por 6. O erro cometido por Jlia foi de aproximadamente:
(a) 100% ;

(b) 97% ;

(c) 83% ;
OBMEP 2010

(d) 17% ;

(e) 3% .
91

Nvel 3
144. Problema de nota Um professor prope 80 problemas a um aluno, informando que
lhe atribuir cinco pontos por problema resolvido corretamente e lhe descontar trs
pontos por problema no resolvido ou resolvido incorretamente. No nal, o aluno ca
com oito pontos. Quantos problemas ele resolveu corretamente?
145. Quadrados e tringulos Na gura dada, temos 16 pontos formando um reticulado
quadrado e duas retas, r e s, perpendiculares entre si.

(a) Quantos quadrados podemos construir, de tal maneira que seus vrtices pertenam
ao reticulado, porm nenhum de seus lados seja paralelo, nem reta r, nem reta
s?
(b) Quantos tringulos retngulos issceles podemos construir de tal maneira que seus
vrtices pertenam ao reticulado, porm nenhum de seus lados seja paralelo, nem
reta r, nem reta s?
146. Clculo de reas Em cada uma das guras a seguir tem-se um quadrado de lado
r. As regies hachuradas em cada uma destas guras so limitadas por lados desse
quadrado ou por arcos de crculos de raio r de centros nos vrtices do quadrado.
Calcule cada uma dessas reas em funo de r.

(b)

(a)

147. Sequncia de algarismos Todos os nmeros naturais de 1 em diante foram escritos


consecutivamente, formando uma sequncia de algarismos, como segue.
1234567891011121314151617181920212223 . . . .
Qual o algarismo que aparece na posio de nmero 206 788?
148. Soma constante Coloque os nmeros 663, 664, 665, 666, 667, 668, 669, 670 e
671, sem repetir, numa tabela 3 3, de tal maneira que a soma em cada linha, em
cada coluna e em cada diagonal seja 2 001. Caso isso no seja possvel, justique sua
resposta.
92

OBMEP 2010

Nvel 3
149. Contando os zeros Quantos zeros existem no nal do nmero 92007 + 1?
150. Crculos dentro do quadrado Dentro de um quadrado so colocados crculos, dois
a dois disjuntos ou, ento, tangentes externamente. Se o lado do quadrado mede 1 cm,
ser possvel colocar tantos desses crculos de tal modo que a soma de seus raios, em
centmetros, seja maior do que 2 008?
151. Construindo um nmero Encontre todos os nmeros de oito algarismos formados
somente com os algarismos 1, 2, 3 e 4, cada um deles duas vezes, tais que:
(a) exista um nico algarismo entre os dois algarismos 1;
(b) existam dois algarismos entre os dois algarismos 2;
(c) existam trs algarismos entre os dois algarismos 3 e
(d) existam quatro algarismos entre os dois algarismos 4.
152. Nmero na circunferncia Os algarismos 1, 2, 3, 4, 5, 6, 7, 8 e 9 foram escritos
(numa ordem desconhecida) ao redor de uma circunferncia. Lendo esses algarismos de
trs em trs no sentido horrio, formam-se nove nmeros de trs algarismos. Determine
a soma desses nove nmeros.
153. Cada pea em seu lugar Cinco peas de metal, confeccionadas, respectivamente,
de ouro, prata, bronze, platina e nquel, foram colocadas em cinco cofres, numerados
de 1 a 5. Cada cofre contm uma pea e o problema consiste em descobrir qual pea
est em qual cofre. Na porta de cada cofre est escrita uma informao. Das cinco
informaes, quatro so falsas e a nica que verdadeira a que aparece na porta do
cofre que contm a pea de ouro. As informaes nas portas dos cofres so as seguintes.
Cofre 1: O ouro est no cofre 2 ou 3.
Cofre 2: A prata est no cofre 1.
Cofre 3: O bronze no est aqui.
Cofre 4: O nquel est no cofre cujo nmero inferior, em uma unidade, ao que
contm o ouro.
Cofre 5: A platina est no cofre cujo nmero superior, em uma unidade, ao que
contm o bronze.
154. Soma de quadrados Encontre trs nmeros, numa progresso aritmtica de razo
2, tais que a soma de seus quadrados seja um nmero formado de quatro algarismos
iguais.
155. Adivinhe o nmero Certo nmero deixa resto 1 quando dividido por 3, deixa resto
2 quando dividido por 4, deixa resto 3 quando dividido por 5 e deixa resto 4 quando
dividido por 6. Qual o menor nmero inteiro positivo que satisfaz essas propriedades?
156. Um cdigo Na expresso abaixo, cada letra corresponde a um algarismo, sendo que
letras diferentes correspondem a algarismos diferentes. Determine esses algarismos.
6 AOBM EP = 7 M EP AOB
OBMEP 2010

93

Nvel 3
157. Calculando distncias O tringulo ABC
equiltero, com lados medindo 3 cm, e o tringulo
CBD retngulo, com lados medindo 3, 4 e 5
cm, conforme a gura dada. Calcule a distncia
entre os pontos A e D.

3
A

3
3

158. Calculando lados de um tringulo O tringulo


ABC equiltero e o ponto P tal que P A = 3 cm,
P B = 4 cm e P C = 5 cm. Calcule o comprimento dos
lados do tringulo ABC.

5 cm

3 cm

4 cm

159. Amigo oculto Um grupo de cinco amigos decide brincar de amigo oculto, cada um
compra um presente para seu amigo oculto. Pelas regras do jogo, cada um d exatamente um presente e recebe exatamente um presente. De quantas maneiras podem os
presentes ser distribudos, de modo que ningum d presente para si mesmo?
160. Contando solues Quantos so os pares de nmeros inteiros positivos (x, y) tais
que
xy
= 144 ?
x+y
161. Determinando uma sequncia Numa certa sequncia de 80 nmeros, qualquer
termo, salvo as duas extremidades, igual ao produto de seus termos vizinhos. O
produto dos 40 primeiros termos da sequncia 8 e o produto de todos os termos
tambm 8. Determine os termos da sequncia.
l..................................................................................................
.....
....
.
.
.
.
.
.
.
.
l
.
.
.
.
.
.
.
.
.
.
.
.
l
.
.
.
.
.
.
.
.
.
.
jardim
.
.
l
.
.
.
.
.
.
.
.
.
.
.
.
l
.
.
.
.
.
.
.
.
.
.
.
.
l.................................................................................................
.....
....

162. Construindo uma cerca Carina est desenhando a planta


de um jardim retangular que ter um de seus lados num muro
reto de pedras. Ela comprou 140 m de cerca, em pedaos de 1 m
cada um, para cercar os outros trs lados. Ela no pode cortar
esses pedaos e deve gastar todos eles.

(a) Se os dois lados vizinhos ao muro de pedra tm 40 m cada um, qual ser o
comprimento do terceiro lado?
(b) possvel que o maior dos lados a ser cercado tenha 85 m? E 65 m? Justique.
163. Um quadriltero especial Os ngulos ABC e C DA do
quadriltero ABCD da gura so retos e os quatro lados do
quadriltero medem nmeros inteiros que so todos distintos.
Se AD = 7 e BC = 11, quanto medem os lados AB e CD?

OBMEP 2010

11

7
y
D

94

Nvel 3
164. Trs quadrados Dois quadrados, ABCD com uma rea de 30 cm2 e F HIJ com
uma rea de 20 cm2 , tm seus lados AD e HI sobre uma reta, conforme a gura. Se
o ponto E do segmento AH for tal que BEF G um quadrado, calcule a rea desse
quadrado.
G
C

B
F

165. Bolinha de gude Trs amigos jogam uma partida de bolinha de gude, convencionando que o perdedor de cada rodada dobra as bolinhas dos outros jogadores, ou seja,
ele d aos outros dois um nmero tal de bolinhas que eles quem com o dobro do que
tinham no incio da rodada. O primeiro jogador perdeu a primeira rodada, o segundo
jogador a segunda, o terceiro a terceira e todos terminaram com 64 bolinhas cada um.
Com quantas bolinhas cada um dos trs amigos comeou essa partida?
166. Uma soma Calcule o valor da soma
1
1
1
1
1
S=
+
+
+ +
+
.
12 23 34
2 006 2 007 2 007 2 008
167. Dobrando papel Uma folha ABCD retangular com 1 000 cm2 de rea foi dobrada
ao meio e, em seguida, desdobrada segundo M N, conforme a gura. Em seguida, foi
dobrada e desdobrada novamente, segundo M C e, nalmente, dobrada e desdobrada
segundo a diagonal BD. Calcule a rea do pedao de papel indicado na gura, que
limitado pelos trs vincos.
A

B
F

168. Uma rea No tringulo ABC, M o ponto mdio do lado AC, D um ponto do
lado BC, tal que AD a bissetriz do ngulo B AC, e P o ponto de interseo de AD
e BM. Sabendo que AB = 10 cm, AC = 30 cm e a rea do tringulo ABC mede
100 cm2 , calcule a rea do tringulo ABP.
169. ltimos algarismos Quais so os dois ltimos algarismos do nmero
2 008

8 + 88 + 888 + + 88 88 ?
OBMEP 2010

95

Nvel 3
170. Idades mltiplas Quando Isabel nasceu, sua me estava fazendo aniversrio de 20
anos. Se Isabel e sua me viverem mais 100 anos, quantas vezes tero sido mltiplas
as idades das duas?
171. Blocos diferentes Ana tem um cubo de 10 cm de lado. Ela cortou o cubo em
cubinhos de 1 cm de lado e, com esses cubinhos, ela brinca de formar outros blocos
retangulares, mas sem que sobrem cubinhos. Por exemplo, ela formou um bloco de
10 20 5. No total, quantos blocos diferentes ela pode construir com esses cubinhos,
sem que sobre nenhum?
5 cm
10 cm
20 cm

172. Quadro negro Joana escreveu os nmeros de 1 a 10 000 no quadro negro e, depois,
apagou todos os mltiplos de 7 e 11. Qual foi o nmero que cou na posio 2 008?
173. Conjunto sem mltiplos Qual o maior nmero possvel de elementos de um
subconjunto de {1, 2, . . . , 100} tal que nenhum de seus elementos seja um mltiplo de
algum outro?
174. Brincando com a calculadora Digite numa calculadora um nmero qualquer de
trs algarismos. Em seguida, digite o mesmo nmero obtendo, assim, um nmero de
seis algarismos, da forma a b c a b c. Divida esse nmero por 7, divida o resultado por 11
e, nalmente, divida o nmero obtido por 13. O que aconteceu? Por que voc obteve
esse resultado?
175. No galinheiro Um galinheiro com 240 m2 de rea deve abrigar galinhas e pintinhos,
sendo desejvel que haja um espao livre de 4 m2 para cada galinha e 2 m2 para cada
pintinho. Alm disso, cada pintinho come 40 g de rao por dia e cada galinha come
160 g por dia, sendo permitido um gasto dirio mximo de 8 kg de rao.
(a) Represente algebricamente as condies do problema.
(b) Represente gracamente, no plano cartesiano xOy, as condies do problema.
(c) Esse galinheiro comporta 20 galinhas e 80 pintinhos? E 30 galinhas e 100 pintinhos?
(d) Qual o nmero mximo de galinhas que podem ser colocadas no galinheiro,
respeitando os espaos desejveis e o gasto mximo de rao? E de pintinhos?
176. Um nmero perfeito Um nmero natural n dito perfeito se a soma de todos os
seus divisores prprios, isto , divisores diferentes de n, igual a n. Por exemplo, 6 e
28 so perfeitos, pois 6 = 1 + 2 + 3 e 28 = 1 + 2 + 4 + 7 + 14. Sabendo que 231 1
um nmero primo, mostre que 230 (231 1) um nmero perfeito.
96

OBMEP 2010

Nvel 3
177. Quinze minutos a mais Dois carros partem, ao mesmo tempo, de uma cidade A
em direo a uma cidade B. Um deles viaja velocidade constante de 60 km/h e o
outro velocidade constante de 70 km/h. Se o carro mais rpido faz a viagem de A
a B em 15 minutos a menos do que o outro carro, qual a distncia entre essas duas
cidades?
178. Outros caminhos Partindo de sua casa para chegar na escola, Jlia deve caminhar
oito quarteires para a direita e cinco quarteires para cima, conforme indicado na
gura dada.

Ela sabe que existem muitas maneiras diferentes de fazer o percurso casa-escola, sempre
seguindo o caminho mais curto. Como ela uma menina muito curiosa, ela gostaria de
sempre fazer caminhos diferentes. Quantos desses caminhos existem da casa de Jlia
at a escola?
179. Escrevendo no tabuleiro Um tabuleiro quadrado de trs linhas por trs colunas
contm nove casas. De quantos modos diferentes podemos escrever as trs letras A, B
e C em trs casas diferentes, de tal modo que, em cada linha, esteja escrita exatamente
uma dessas trs letras?
180. Frao e percentagem Se na frao x/y diminuirmos o numerador x de 40% e o
denominador y de 60%, ento a frao x/y
(a)diminui 20%;

(b)aumenta 20%;

(c)diminui 50%;

(d)aumenta 50%.

181. Tringulos sobrepostos Dois tringulos retngulos congruentes possuem catetos


que medem 4 cm e 7 cm. Na gura dada, esquerda, os tringulos foram desenhados
de modo a coincidirem os catetos de 7 cm. Assim, AB = 7 cm e AD = BC = 4 cm.
J na gura direita, eles foram desenhados de modo a coincidirem as hipotenusas,
donde AD = BC = 4 cm e AC = BD = 7 cm.
D

Calcule as reas sombreadas nas duas guras.


OBMEP 2010

97

Nvel 3
182. Dois motoristas Dois motoristas viajam da cidade A at a cidade B e, imediatamente, regressam cidade A. O primeiro motorista viaja a uma velocidade constante
de 80 km/h, tanto na ida quanto na volta. O segundo motorista viaja at a cidade B
a uma velocidade constante de 90 km/h e retorna velocidade constante de 70 km/h.
Qual desses motoristas gasta menos tempo no percurso total de ida e volta?
183. Soma e inverte Usando somente as duas operaes +1 = somar 1 e
i = menos o inverso, podemos formar vrias sequncias a partir de um nmero
inicial. Por exemplo, iniciando com o nmero 3, podemos formar a sequncia
1 +1 4 i
5 +1
1 +1 3 i
4
+1
+1
i
3 4 5 .
5
5
4
4
4
3
Iniciando com 0, com qual sequncia obteremos novamente o 0, usando apenas essas
duas operaes +1 e i?
184. Carro ex Um carro denominado ex se ele pode ser abastecido com gasolina
ou com lcool. O consumo de um carro costuma ser dado (no Brasil) em quilmetros
por litro, que indicamos por km/l. J o custo desse consumo dado pelo preo do
quilmetro rodado, em reais por km. Suponha que os preos do litro de lcool e de
gasolina sejam, respectivamente, R$ 1,59 e R$ 2,49.
(a) Digamos que um certo carro ex rode 12,3 km por litro de gasolina. Qual deve
ser o consumo de lcool desse carro para que a utilizao do lcool seja nanceiramente mais vantajosa que a de gasolina?
(b) Com os preos dados, em que condies mais vantajoso, nanceiramente, o
uso do lcool em vez do de gasolina? D um exemplo numrico que satisfaa as
condies.
Daqui em diante, suponha que o consumo de um certo carro ex seja de x km/l com
x
gasolina e de
+ 1 km/l com lcool.
2
(c) Escreva a expresso da funo g(x) que fornece o custo de rodar 100 quilmetros
com esse carro utilizando gasolina e a expresso da funo a(x) que fornece o
custo de rodar 100 quilmetros utilizando lcool.
(c) Para que o custo seja o mesmo, tanto com lcool como com gasolina, qual deve
ser o consumo em km/l para a gasolina e para o lcool?
(d) Com o consumo dado, em que condies mais vantajoso, nanceiramente, o
uso do lcool em vez do de gasolina? D um exemplo numrico que satisfaa as
condies.
185. Contando tringulos Na gura dada esto marcados onze pontos sobre dois segmentos. Quantos tringulos podem ser formados com esses onze pontos?

98

OBMEP 2010

Nvel 3
186. Quadrado perfeito Existe um nmero de oito algarismos da forma
9999
que seja um quadrado perfeito?
187. Diferena quase nula Qual o menor nmero inteiro positivo n tal que

n n 1 < 0,01?
188. Conjunto de Cantor Desenhe um segmento de reta com uma unidade de comprimento, denotando-o por C1 . Remova a tera parte central (sem remover as extremidades) e denote por C2 o que sobrou. Agora, remova a tera parte central (sem as
extremidades) de cada um dos dois segmento de reta que constituem C2 , denotando
por C3 o que sobrou. Esse processo pode ser continuado, sempre removendo, em cada
estgio, a tera parte central de cada segmento em Cn para formar Cn+1 . O conjunto de
Cantor formado pelos elementos de C1 que nunca so removidos, em etapa alguma.
C1
C2
C3
(a) Na gura dada, indique os nmeros nas extremidades dos segmentos C1 , C2 e C3 .
4
1 4 3
e
pertencem ao conjunto de Cantor?
(b) Quais dentre os pontos , ,
3 9 81 81
(c) Quais so os comprimentos de C3 , C4 e C5 ? Voc consegue encontrar alguma
expresso para o comprimento de Cn ?
189. Enchendo uma piscina Uma piscina vazia foi abastecida de gua por duas
torneiras A e B, ambas de vazo constante. Durante quatro horas, as duas torneiras
caram abertas e encheram 50% da piscina. Em seguida, a torneira B foi fechada e,
durante duas horas, a torneira A encheu 15% do volume da piscina. Aps este perodo,
a torneira A foi fechada e a torneira B aberta. Durante quanto tempo essa torneira
teve de car aberta para que ela, sozinha, terminasse de encher a piscina?
190. Probabilidade de ser um nmero par Uma urna tem nove bolas, numeradas de
1 a 9. Jos e Maria retiram, cada um, simultaneamente, uma bola da urna. Com as
bolas retiradas eles formam um nmero de dois algarismos, sendo que o nmero que
est escrito na bola de Jos o algarismo das dezenas e o nmero que est escrito na
bola de Maria o algarismo das unidades. Qual a probabilidade desse nmero ser
par?
191. Mltiplo de 7 Mostre que se o produto N = (n + 6m)(2n + 5m)(3n + 4m), com m
e n nmeros inteiros positivos, for um mltiplo de 7 ento esse produto N tambm
mltiplo de 73 = 343.
OBMEP 2010

99

Nvel 3
192. Os ngulos 15 e 75 Na gura dada, ABCD um quadrado com uma unidade
de lado e o tringulo BCE equiltero. O ponto M o ponto mdio do segmento
CE, o segmento DN perpendicular a BM e o segmento BM perpendicular a CE.
(a) Calcule os comprimentos dos lados do tringulo DBN.

(b) Use o item (a) para calcular o cosseno, o seno


e a tangente dos ngulos de 15 e 75 .

193. Crculos tangentes Na gura dada esto desenhados dois


crculos concntricos de raios r e R, sendo r < R, e doze
crculos compreendidos entre os dois primeiras, todos de raio
x. Alm disso, os quatorze crculos so disjuntos ou tangentes.
Rr
.
2

R
4+ 6 2

.
(b) Mostre que
=
r
4 6+ 2
(a) Mostre que x =

194. Mudando a base Um tringulo issceles tem uma base de 10 cm


e dois lados iguais medindo 13 cm. possvel cortar esse tringulo em
dois outros tringulos de tal modo que, juntando esses tringulos de outra
maneira obtenhamos um outro tringulo issceles (evidentemente com a
mesma rea)?

.
.
..
..
..
. ..
. .
. ..
. ..
. ..
.
.
.
.
.
.
.
.
.
.
.
.
.
.
.
.
.
.
.
.
.
.
.
.
.
.
.
.
.
.
.
.
.
.
.
.
.
.
.
.
.
.
.
.
.
.
.
.
.
.
.
.
.
.
.
.
.
.
.
.
.
.
.
.
.
................................
................................
.

13

13

10

195. Clube de Matemtica Eu fao parte de um Clube de Matemtica, onde tenho


o mesmo nmero de colegas homens do que de colegas mulheres. Quando um garoto
falta, trs quartos da equipe so de meninas. Eu sou homem ou mulher? Quantas
mulheres e quantos homens tem o clube?
196. Uma calculadora diferente Davi tem uma calculadora muito original, que efetua
apenas duas operaes, a adio usual (+) e uma outra operao, denotada por , que
satisfaz
(i) a a = a,

(ii) a 0 = 2a e

(iii) (a b) + (c d) = (a + c) (b + d),
para quaisquer nmeros inteiros a e b. Quais so os resultados das operaes
(2 3) + (0 3) e 1 024 48?
197. Cercando o globo terrestre O raio do globo terrestre mede, aproximadamente, 6 378 km no Equador. Suponhamos que um o esteja
ajustado exatamente sobre o Equador.

100

OBMEP 2010

Nvel 3
Em seguida, suponhamos que o comprimento do o seja aumentado em 1 m, de modo
que o o e o Equador quem como crculos concntricos ao redor da Terra. Um homem
em p, uma formiga, ou um elefante so capazes de passar por baixo desse o?
198. Comprimento de uma corda Numa circunferncia de 10 cm de raio, o segmento
AB um dimetro e o segmento AC uma corda de 12 cm. Determine a distncia
entre os pontos B e C.
199. Dois irmos A diferena de idade entre dois irmos de trs anos. Um ano atrs,
a idade do pai desses irmos era o dbro da soma das idades dos irmos e, dentro de
vinte anos, a idade do pai ser a soma das idades desses dois lhos. Qual a idade de
cada um dos irmos?
200. Canelonis de ricota Todo domingo, Pedro prepara
canelonis para o almoo. Primeiro, ele corta retngulos de
massa de 16 por 12 cm e, depois, cola os dois lados mais longos, superpondo uma faixa de 2 cm.

16 cm
10 cm
2 cm

Dessa forma, ele obtm cilindros que recheia com ricota, gastando 500 g de ricota por
cilindro. Num belo domingo, com o mesmo nmero de retngulos de massa de 16 por
12 cm, ele decide produzir os cilindros colando os lados menores, sempre superpondo
uma faixa de 2 cm. Nessa situao, ele vai gastar mais ou menos ricota que antes?
Quanto?
201. Clculo de segmentos As medidas do retngulo ABCD A
so de 1 200 por 150 m. Alm disso, P est no prolongamento
do lado BC e dista 350 m de C. Determine as medidas de
D
AP, P Q, P D, CQ e DP.

1200
Q

B
150
C
350

202. Pr chegar junto! Ada e Luisa treinam todos os dias, cada uma delas sempre
com a mesma velocidade, para a grande corrida que vai acontecer no nal do ano na
escola. O treino comea num ponto A e termina no ponto B, distantes 3 000 m. Elas
partem no mesmo instante, mas quando Luisa termina a corrida, ainda faltam 120 m
para Ada chegar ao ponto B. Ontem, Luisa deu uma chance para Ada: Partimos ao
mesmo tempo, mas eu parto alguns metros antes do ponto A para chegarmos juntas.
Quantos metros antes do ponto A deve partir Luisa para chegar junto com Ada?
203. Um professor enfurecido Para castigar os alunos de sua turma por indisciplina,
o professor Zerus decidiu descontar da nota mensal de cada aluno uma percentagem
igual nota da prova, isto , quem tirou 60, ter um desconto de 60% na nota, quem
tirou 20, um desconto de 20% na nota e assim por diante. A nota mensal mxima
100.
(a) Quem vai car com a maior nota?
(b) E a menor?
OBMEP 2010

101

Nvel 3
(c) Alunos que tiraram notas boas reclamaram que vo car com a mesma nota de
alunos que tiraram notas ms. Eles tem razo?

204. O percurso de um atleta Um atleta resolveu fazer uma corrida de 15 km. Comeou
correndo 5 km na direo Sul, depois virou para o Leste, correndo mais 5 km e, novamente, virou para a direo Norte, correndo os 5 km restantes. Aps esse percurso,
constatou, para seu espanto, que estava no ponto de onde havia partido. Descubra
dois possveis pontos sobre o globo terrestre de onde esse atleta possa ter iniciado sua
corrida.
A

205. reas iguais Na gura dada, o tringulo ABC retngulo


e os semicrculos dados tm dimetros AB, BC ou AC. Mostre
que a rea sombreada igual rea do tringulo ABC.

206. Funo denida por rea A funo f,


denida para cada y satisfazendo 0 y < 2,
dada por f (y) = rea do quadriltero sombreado,
conforme indicado na gura dada.

r
6

(a) Escreva as equaes das retas r e s.

(b) Determine f (0).


(c) Escreva a expresso de f (y), para 0 y < 2.

(d) Esboce o grco de f (y).

y
-2 -1

1 2 3

207. PA e PG Determine quatro nmeros distintos a1 , a2 , a3 e a4 que sejam termos


consecutivos de uma progresso aritmtica e tais que os nmeros a1 , a3 e a4 formem
uma progresso geomtrica.
208. Plano cartesiano Dizemos que um ponto do plano cartesiano um ponto inteiro se
suas coordenadas so inteiras. Dado um inteiro positivo n, denote por f (n) o nmero de
pontos inteiros que esto sobre o segmento que liga a origem ao ponto inteiro (n, n+3),
sem contar as extremidades. Mostre que

2, se n mltiplo de 3,
f (n) =

0, se n no mltiplo de 3 .
209. Trabalhando com um quadriltero No
quadriltero ABCD da gura dada, tem-se AB = 5,
BC = 17, CD = 5 e DA = 9. Determine DB, sabendo
que sua medida um nmero inteiro.
102

OBMEP 2010

.
.............
..............
..
..
..
..
.
.
..
..
..
..
..
..
.
.
..
..
..
..
..
..
.
..
..
..
..
..
.
.
..
..
..
..
..
.
.
..
..
..
..
..
..
.
.
..
..
.
..
..
.
.
.
..
................................................................................
.................................................................................
.
..
.

A.......................................................

Nvel 3
210. O tringulo de Reuleaux O tringulo de Reuleaux a gura
formada a partir de um tringulo equiltero, substituindo os lados
por arcos de circunferncia centrados nos vrtices do tringulo e de
raios iguais ao lado do tringulo. Qual a rea de um tringulo de
Reuleaux, se os lados do tringulo equiltero inicial medem 1 cm?
211. Interseo de crculos Na gura dada foram desenhados
trs crculos de raio r centrados nos vrtices do tringulo equiltero ABC de lado a. Se 1 a < r < a, esses trs crculos
2
so, dois a dois, concorrentes em trs pontos X, Y e Z exteriores ao tringulo ABC. Mostre que o tringulo XY Z
equiltero e calcule o comprimento do seu lado, em termos de
a e r.

C
Y

A
Z

212. Valor mximo A expresso


k?

k2
atinge seu maior valor com qual nmero natural
1,001k

213. Moedas falsas Aladim tem dez sacos de moedas, sendo que cada
saco tem somente moedas verdadeiras ou somente moedas falsas.
Cada moeda verdadeira pesa 10 g e cada moeda falsa pesa 1 g.

(a) Suponhamos que em cada saco existam exatamente dez moedas e que somente um
dos sacos seja de moedas falsas. Utilizando uma balana e efetuando apenas uma
pesagem, como deve Aladim proceder para descobrir qual o saco das moedas
falsas?
(b) Suponhamos que os sacos estejam cheios de moedas e que Aladim no saiba
quantos desses sacos sejam de moedas falsas. Como pode ele identicar, com
apenas uma pesagem, os sacos que tm moedas falsas?

214. Menor inteiro Sejam p e q inteiros positivos tais que


valor de p para que p + q = 2 005?

p
7
5
< < . Qual o menor
8
q
8

215. Mais reas... Um tringulo tem vrtices A = (3, 0), B = (0, 3) e C, onde C est
na reta de equao x + y = 7. Qual a rea desse tringulo?
216. Crculos tangentes Trs crculos, com raios medindo 1, 2
e 3 cm, so dois a dois tangentes exteriormente, como na gura
dada. Determine o raio do crculo tangente exteriormente aos
trs crculos.

OBMEP 2010

103

Nvel 3
217. Soma nita Cada um dos nmeros x1 , x2 , . . . , x2 004 pode ser igual a

2 + 1. Quantos valores inteiros distintos pode valer a soma

2 1 ou a

1 002

k=1

x2k1 x2k = x1 x2 + x3 x4 + x5 x6 + + x2 003 x2 004 ?

218. Mltiplos Seja a um nmero inteiro positivo que mltiplo de 5 e tal que a + 1
mltiplo de 7, a + 2 mltiplo de 9 e a + 3 mltiplo de 11. Determine o menor valor
possvel de a.
219. Equao de duas variveis Determine todos os pares de inteiros (x, y) tais que
9xy x2 8y 2 = 2 005.
220. Trapzio retngulo Seja ABCD um trapzio retngulo de bases AB e CD, com
ngulos retos em A e D. Dado que a diagonal menor BD perpendicular ao lado BC,
determine o menor valor possvel para a razo CD/AD.
221. Jogos de futebol Os doze alunos de uma turma de olimpada saam para jogar
futebol todos os dias aps a aula de matemtica, formando dois times de seis jogadores
cada e jogando entre si. A cada dia eles formavam dois times diferentes dos times
formados em dias anteriores. Ao nal do ano, eles vericaram que cada cinco alunos
haviam jogado juntos num mesmo time exatamente uma vez. Quantos times diferentes
foram formados ao longo do ano?
222. A soma dos algarismos de um nmero Denotemos por s(n) a soma dos algarismos do nmero n. Por exemplo s(2 345) = 2 + 3 + 4 + 5 = 14. Observemos que:
40 s(40) = 36 = 9 4;

500 s(500) = 495 = 9 55;

2345 s(2345) = 2331 = 9 259 .


(a) O que podemos armar sobre o nmero n s(n)?

(b) Usando o item anterior, calcule s(s(s(22 009 ))).

Sugesto: Mostre que o nmero procurado menor do que 9.

104

OBMEP 2010

Desaos

Desaos
1. Cadeia do menor nmero (N2/N3) Partindo do nmero 265 863 e utilizando
uma nica vez cada uma das operaes +, , e e tambm uma nica vez os
nmeros 51, 221, 6 817, 13 259, podemos obter vrios nmeros. Por exemplo, 54 911,
como segue.
221

51

13 259

+6 817

265 863 1 203 61 353 48 094 54 911


Encontre a cadeia que permite obter o menor nmero inteiro positivo.
2. Qual a metade? (N2/N3) Considere a gura ao lado,
em que AB = AE = ED = CD = CA e o arco CB um arco
de crculo centrado no ponto E. Voc sabe repartir essa gura
em duas partes idnticas, que possam ser superpostas?
3. Cada um em seu estado (N1/N2/N3) Amlia, Bruno, Constncia e Denise so
quatro amigos que se encontram sentados numa mesa quadrada, cada um ocupando
um lado da mesa. Um dos quatro mora no Amazonas, outro em So Paulo, outro no
Cear e o quarto na Bahia. Sabendo que valem as condies a seguir, quem mora na
Bahia?
direita de Amlia est quem mora no Amazonas.

Em frente Constncia est a pessoa que mora em So Paulo.


Bruno e Denise esto um ao lado do outro.

Uma mulher est esquerda da pessoa que mora no Cear.


4. Diviso (N1/N2) Numa diviso, aumentando o dividendo de 1 989 e o divisor de
13, o quociente e o resto no se alteram. Qual o quociente?
5. Extraterrestre (N1/N2) No planeta Staurus, os anos tm 228 dias, divididos em
12 meses de 19 dias. Cada semana tem 8 dias: Zerum, Uni, Duodi, Trio, Quati, Quio,
Seise e Sadi. Sybock nasceu num duodi, que foi o primeiro dia do quarto ms. Que
dia da semana ele festejar seu primeiro aniversrio?
6. Que famlia! (N1/N2) Numa famlia, cada menino tem o mesmo nmero de
irmos que de irms, e cada menina tem o dobro de irmos que de irms. Qual a
composio dessa famlia?
7. Siga a pista (N1) Na pista de corrida dada, os sete pontos
de referncia so marcados a cada 50 m. Os atletas devem
fazer 2 km no sentido indicado pela echa, partindo do ponto
P. Marque o ponto C de chegada.
OBMEP 2010

105

Desaos
8. Cara ou Coroa (N2) Jernimo joga no tabuleiro dado com uma pea e um dado
da maneira descrita a seguir.
Colocando a pea na casa P (de partida), ele
lana uma moeda. Se der cara, avana duas
casas e se der coroa recua uma casa. Jernimo
lanou a moeda 20 vezes e conseguiu chegar na
casa C (de chegada). Quantas vezes a moeda
deu cara?
9. Contas do papagaio (N1) Rosa tem um papagaio que faz contas de um modo
estranho. Cada vez que Rosa diz dois nmeros ele faz a mesma conta. Por exemplo:
se Rosa diz 4 e 2 o papagaio responde 12;
se Rosa diz 5 e 3 o papagaio responde 12;
se Rosa diz 3 e 5 o papagaio responde 14;
se Rosa diz 9 e 7 o papagaio responde 24;
se Rosa diz 0 e 0 o papagaio responde 1.
Se Rosa diz 1 e 8 o que responde o papagaio?
10. As frias de Toms (N1/N2) Durante as frias de Toms, houve 11 dias chuvosos.
Durante esses 11 dias, se chovia pela manh havia sol sem chuva tarde, e se chovia
tarde, havia sol sem chuva pela manh. No total, Toms teve 9 manhs e 12 tardes
sem chuva. Quantos dias duraram as frias de Toms?
11. Maratona de Matemtica (N3) Numa maratona de Matemtica, o nmero de
questes muito grande. O valor de cada questo igual sua posio na prova:
um ponto para a questo 1, dois pontos para a questo 2, trs pontos para a questo
3, quatro pontos para a questo 4, e assim por diante. Joana totalizou 1 991 pontos
na prova, errando apenas uma questo e acertando todas as outras. Qual questo ela
errou? Quantas questes tinha a prova?
1
1 1 1 1 1
e
, cuja
12. Fraes ignoradas (N1) Escolhi quatro fraes dentre , , , ,
2 4 6 8 10 12
soma 1. Quais foram as fraes que eu no escolhi?
13. Caminho de maior total (N2) As regras do jogo so as seguintes.
(a) Partindo da casa com o nmero 3 destacado, deve-se chegar casa TOTAL
deslocando-se somente por linhas ou colunas e calculando-se os pontos.
(b) Quando nos deslocamos por uma linha, s podemos adicionar, por exemplo passando da 3 para a 6 ao lado, obtemos 3 + (6) = 3 pontos.

(c) Quando nos deslocamos por uma coluna, s podemos subtrair, por exemplo passando da 3 para a 5 abaixo, obtemos 3 5 = 2 pontos.

(d) S permitido passar uma vez por cada casa.


Qual o caminho que d o maior total?
106

OBMEP 2010

Desaos
3
5
8
4
0

6
7
3
1
2

9
2
5
6
7

9
1
4
8
TOTAL

14. Produtos em linha (N1/N2/N3) Em cada uma das oito casas brancas do quadro
dado, escrevemos um algarismo dentre os algarismos 1, 2, 3, 4, 5, 6, 7, 8 e 9 de modo que
os produtos efetuados em linha reta, seguindo as echas, forneam os valores indicados
dentro dos casas em cinza. Em qual casa se encontra o nmero 2?

15. Cdigo Postal (N2/N3) Para fazer a separao em regies da correspondncia


que deve ser entregue, um servio postal indica sobre os envelopes um cdigo postal
com uma srie de cinco blocos de pontos e bastes, que podem ser lidos por um leitor
tico. Os algarismos so codicados como segue.

A leitura se faz da direita para a esquerda. Por exemplo: o cdigo postal 91720 se
escreve como
, ou seja,

.
Note que a codicao de 94, que
, tem um eixo vertical de simetria.
Encontre os cdigos entre 47000 e 47999 que apresentam um eixo vertical de simetria.

OBMEP 2010

107

Desaos
16. Anis olmpicos (N1/N2/N3) Os nmeros de 1 a 9 foram colocados dentro de
cinco anis olmpicos, de tal modo que dentro de cada anel a soma 11.

Disponha os nove nmeros de outra maneira, para que a soma dentro de cada anel seja
sempre a mesma e a maior possvel.
17. Partidas de Denise (N2/N3) Denise e Antnio jogam uma srie de oito jogos,
em que o vencedor da primeira partida ganha um ponto, o da segunda dois pontos, o
da terceira quatro pontos, o da quarta oito pontos, e assim por diante, multiplicando
por dois o nmero de pontos de uma partida para a outra. No nal, Denise ganhou
31 pontos a mais que Antnio e no houve empate em nenhuma das partidas. Quais
partidas Denise ganhou?
18. Sete quadrados (N1/N2) Voc sabe repartir um quadrado em sete quadrados
menores?
19. Ilha misteriosa (N1/N2/N3) Numa misteriosa ilha havia 13 camalees cinza,
15 camalees marrons e 17 camalees vermelhos. Quando dois camalees de cores
diferentes se encontram, os dois tomam a terceira cor. Por exemplo, se um cinza se
encontra com um vermelho, ento os dois cam marrons. Por causa de uma tempestade, ocorreram dois encontros cinza-vermelho, trs encontros marrom-vermelho e um
encontro cinza-vermelho. Quantos camalees de cada cor caram na ilha?
20. Universo hostil (N3) Num deserto h cobras, ratos e escorpies. Cada manh,
cada cobra mata um rato. Cada meio-dia, cada escorpio mata uma cobra. Cada
noite, cada rato mata um escorpio. Ao nal de uma semana, noite, s restava um
rato. Quantos ratos havia na manh do incio da semana?
21. O jogo das chas Para iniciar um jogo com seus amigos, Manoel coloca oito chas
em cada uma das nove casas do tabuleiro mostrado na gura. Para ganhar o jogo, ele
precisa mover as chas pelo tabuleiro de tal modo que, ao nal, todas as chas estejam
colocadas e tenha sido alcanada uma outra congurao de chas na qual, em cada
linha, cada coluna e cada diagonal, reste o mesmo nmero de chas (mas no a inicial, com oito chas em cada casa). Na primeira jogada, ele coloca mais trs chas na
casa 3 e tira todas da casa 2, cando com 5 na mo, para prosseguir. Quantas chas ele

108

OBMEP 2010

Desaos
deve colocar em cada uma das outras sete casas para ganhar o jogo, mantendo as chas
das casa 2 e 3 inalteradas depois dessa primeira jogada?



2
3

0
chas

11
chas




22. Um sistema Nas igualdades AB + BC = CD e AB BC = BA, cada letra


representa um algarismo. Quanto vale A + B + C + D?
23. Constelaes oridas Rosa, Margarida e Dlia so trs constelaes em forma de
buqus de ores. Sabemos que:
(a) o nmero de estrelas de Dlia, que a menor das trs constelaes, o produto
de dois quadrados;
(b) o nmero de estrelas de Rosa tambm um produto de dois quadrados;
(c) Dlia e Rosa, juntas, tm o mesmo nmero de estrelas de Margarida;
(d) Margarida tem 28 561 estrelas.
Quantas estrelas possuem, cada uma, Dlia e Rosa?
24. Dois meses iguais A seguir mostramos o calendrio de abril de 2 005.
D

3
10
17
24

4
11
18
25

5
12
19
26

6
13
20
27

7
14
21
28

S
1
8
15
22
29

S
2
9
16
23
30

Qual o primeiro ms depois desse, de 2005 ou de 2006, que teve uma pgina igual?
25. A faixa e o quadrado Uma faixa retangular de cartolina mede 5 por 1 cm. Corte
a faixa com quatro cortes retilneos de modo a poder montar um quadrado com as
peas obtidas, mas sem superposio das peas.
26. Um nmero e seu sxtuplo Um nmero de trs algarismos e seu sxtuplo so
formados pelos mesmos algarismos. A soma dos algarismos desse nmero 17 e a de
seu sxtuplo 21. Qual esse nmero? Existe mais do que um nmero com essas
propriedades?
OBMEP 2010

109

Desaos
27. Oito dentro de um retngulo Coloque os nmeros de 1 a 8
dentro dos crculos do retngulo dado, de tal modo que a diferena
entre dois nmeros ligados por um segmento seja sempre maior do
que 1.

.................
.................
.................
.................
..
.. ....
..
.. .....
.. .
. .....
. ......
...
...
... .
.
.
...
...
... .
.
.
.
.
.
.
...
... .
.
...
...
. ...
.
.
.
.
................
................
.
.
.
.
.
.
.
.
.
.
.
.
. ..
. ....
.
.
.
.
. ...
. .....
.
.
..
.
....
..
. ....
..
..
....
.. .
.
.
.
.. .
.. ...
. ...
.. .
..
.
.
.
.................
.................
.................
.................

28. Uma estratgia com um nmero muito grande Carlos escreveu consecutivamente todos os nmeros de 1 a 60, ou seja,
1234567891011121314 57585960.
Depois ele riscou 100 algarismos de tal modo que o nmero formado com os algarismos
que no foram riscados fsse o maior possvel, sem mudar a ordem inicial em que os
algarismos foram escritos. Qual esse nmero?
29. Um nmero surpreendente Um certo nmero surpreendente divisvel por 9,
tem nove algarismos diferentes, nenhum dos quais igual a 0 e tal que:
(a) o nmero formado pelos 2 primeiros algarismos divisvel por 2;
(b) o nmero formado pelos 3 primeiros algarismos divisvel por 3;
(c) o nmero formado pelos 4 primeiros algarismos divisvel por 4;
(d) o nmero formado pelos 5 primeiros algarismos divisvel por 5;
(e) o nmero formado pelos 6 primeiros algarismos divisvel por 6;
(f) o nmero formado pelos 7 primeiros algarismos divisvel por 7;
(g) o nmero formado pelos 8 primeiros algarismos divisvel por 8.
Qual esse nmero?
30. Qual o erro? Uma das armaes dadas falsa:
(a) Andr mais velho do que Bruno;
(b) Cludia mais nova do que Bruno;
(c) A soma das idades de Bruno e Cludia o dobro da idade de Andr;
(d) Cladia mais velha do que Andr.
Quem o mais velho? E o mais novo?
31. Soma Neste exerccio, as letras representam algarismos. Determine cada uma das parcelas da soma dada.

abcdef
abcdef
+
ghij
.................................................
................................................

def hjf
32. Bolinhas Rogrio colocou seis bolinhas sobre a mesa, de modo a
formar dois quadrados, como na gura. Ele percebe que tem mais
uma bolinha. Complete a gura formada pelas bolinhas com essa
bolinha a mais, de tal modo que forme trs quadrados.
110

OBMEP 2010

................................................
.
.
.
.
...............................................
.
.
.
.
.
.
.
.
.
.
.
.
.
.
.
.
.
.
.
.
.
.
.
.
.
.
.
.
.
.
.
.
.
.
.
.
.
.
.
.
.
.
.
.
.
.
.
.
.
.
.
.
.
.
.
.
.
.
.
.
.
.
.
.
.
.
.
.
.
.
.
.
.
.
.
.
.
.
.
.
...............................................
.
.
................................................
.

Desaos
33. Um nmero que no divisvel por 5 Determine quais nmeros naturais n
entre 2 001 e 2 007 tornam o nmero 1n + 2n + 3n + 4n no divisvel por 5.
34. Quatro fraes e um inteiro Quantos nmeros naturais a, b, c e d, todos distintos,
1 1 1 1
existem tais que
+ + +
seja um inteiro?
a b c d
35. O Rei Arthur e o Drago das Trs Cabeas e Trs Caudas O Rei Arthur
teve que lutar com o Drago das Trs Cabeas e Trs Caudas. Sua tarefa foi facilitada
quando conseguiu arranjar uma espada mgica que podia, a cada golpe, fazer uma e
somente uma das seguintes coisas:
(a) cortar uma cabea;
(b) cortar duas cabeas;
(c) cortar uma cauda;
(d) cortar duas caudas.
Alm disso, a Fada Morgana lhe revelou o segredo do drago:
(a) se uma cabea cortada, cresce uma nova;
(b) se duas cabeas so cortadas, nada mais acontece;
(c) no lugar de uma cauda cortada nascem duas caudas novas;
(d) se duas caudas so cortadas, cresce uma nova cabea;
(e) o drago morre se perder as trs cabeas e as trs caudas.
Para matar o drago, de quantos golpes o Rei Artur vai precisar, no mnimo?
36. O passeio do cavalo Num tabuleiro de 5 5 casas, um cavaleiro do jogo de xadrez
est na casa marcada com A. Depois ele segue movendo, marcando as casas por onde
passa, como na gura.
A B C D E F G H
A

G
H
B

F
D

Partindo da casa H, o cavaleiro se move pelo tabuleiro at ter passado por todas as 25
casas. Descreva o trajeto que ele fez.
37. As faces do cubo Oito dados so agrupados formando um cubo. Quantas faces
dos dados permanecem visveis?
OBMEP 2010

111

Desaos
38. Data fatdica Em 1950, um profeta anunciou que o m do mundo ocorreria em
11 de agosto de 1999, que denotamos por 11081999. Como nada aconteceu nesse dia,
ele refez seus clculos e fez a seguinte previso: O m do mundo ocorrer na prxima
data que se escrever com oito algarismos diferentes. Voc consegue descobrir essa
data?
39. Todos com o 2 Qual operao que devemos fazer com todos os cinco nmeros
418, 244, 816, 426 e 24 para obter cinco nmeros que tenham todos o algarismo 2?
(a) Dividir por 2.
(b) Somar 4.
(c) Dividir por 6.
(d) Subtrair 5.
(e) Multiplicar por 3.
40. Tortas da vov Soa foi levar uns docinhos para sua av: foram sete docinhos de
amora, seis de cco e trs de chocolate. Durante o caminho, a gulosa Soa comeu dois
docinhos. Qual das situaes abaixo possvel?
(a) Sua av no recebeu docinhos de chocolate.
(b) Sua av recebeu menos docinhos de cco do que de chocolate.
(c) Sua av recebeu o mesmo nmero de docinhos de cada uma das trs variedades.
(d) Existem duas variedades de docinhos das quais sua vov recebeu o mesmo nmero.
(e) O nmero de docinhos de amora que sua vov recebeu maior que o das outras
duas variedades somadas.
41. Famlia Stimo O Sr. e Sra. Stimo tm sete lhos, todos nascidos em 1o de abril;
na verdade, em seis 1o de abril consecutivos. Neste ano, para seus aniversrios, a Sra.
Stimo fez um bolo com velinhas para cada um de seus lhos, sendo o nmero de velas
em cada bolo igual ao nmero de anos do aniversariante. Joo Stimo, o lho que mais
gosta de Matemtica, reparou que, nesse ano, o nmero total de velinhas o dobro do
que havia dois anos atrs e que h dois bolos a mais. Quantas velinhas sero acesas
desta vez?
42. O salta-cha Temos dez chas numeradas colocadas em linha reta, como na gura
dada.
......
......
......
......
......
......
......
......
......
......
......... .......... .......... .......... .......... .......... .......... .......... .......... ..........
.. ... ... ... ... ... ... ... ... ... ... ... ... ... ... ... ... ... ... ...
..
..
..
..
..
..
..
..
..
..
.
.
..
..
..
..
..
..
..
..
..
.
.
.
.
.
.
.
.
.
.
.
.
.
..
. .
. .
. .
. .
. .
. .
. .
. .
. .
.
.. ... ... ... ... ... ... ... ... ... ... ... ... ... ... ... ... ... ... ...
........ .......... .......... .......... .......... .......... .......... .......... .......... ..........
.......
......
......
......
......
......
......
......
......
......

9 10

Queremos arrum-las em cinco pilhas, com duas chas em cada pilha. A regra para
isso que s podemos movimentar uma cha fazendo-a saltar sobre uma ou mais chas,
ou sobre uma nica pilha j formada. Um exemplo de trs movimentos o seguinte.
A cha 7 pode saltar sobre a cha 8 e formar uma pilha com a 9;

a cha 5 pode saltar sobre as chas 6 e 7 e formar uma pilha com a 8 e


a cha 6 pode saltar sobre as chas 5, 4 e 3 formar uma pilha com a 2.

112

OBMEP 2010

Desaos
......
.
......
....
...
....
...
......
......
...
.....
...
.....
......
......
...
....
....
...
......
......
....
...
....
...
......
......
...
.....
...
.....
.
......
......
.
..
.....
......
.....
......
...
...
...... ........
...
...... .......
...
..
...
.
...
...
.... ......
...
..... ......
.......... ........... ........... ........... ........... ........... ........... ........... ........... ...........
.......... ........... ........... ........... ........... ........... ........... ........... ........... ...........
.
.
.
..
..
..
..
..
..
..
..
..
.
..
.
..
..
..
..
..
..
..
..
.
.
..
..
..
..
..
..
..
..
..
.
.
..
..
..
..
..
..
..
..
..
.
..
... ... ... ... ... ... ... ... ... ... ... ... ... ... ... ... ... ... ... ...
......... .......... .......... .......... .......... .......... .......... .......... .......... ..........
......
......
......
......
......
......
......
......
......
.....

? ?

9 10

.....
........
... ....
.
..
.
.
.
.
.
.
..
.. ...
.........
.......
.......... ........... ........... ...........
.......... ........... ........... ...........
.
.
.
..
..
..
.
..
.
..
..
.
.
..
..
..
.
.
..
..
..
.
..
... ... ... ... ... ... ... ...
......... .......... .......... ..........
......
......
......
.....

.....
.....
........ ..........
... .... ... ....
. .
.
..
.
.
.
.
..
.
.
.
..
..
.. ... ... ...
......... ...........
.......
......
.......... ........... ...........
.......... ........... ...........
.
..
..
..
.
..
.
..
.
.
..
..
.
.
..
..
.
..
... ... ... ... ... ...
......... .......... ..........
......
......
.....

9 10

Como formar cinco pilhas de duas chas com apenas cinco movimentos?
43. O menor Qual o menor nmero, 52 002 ou 32 002 + 42 002 ?
44. O maior resultado Qual o maior resultado que podemos encontrar quando
dividimos um nmero de dois algarismos pela soma de seus algarismos?
45. Dois mil Digamos que o peso de um nmero seja a soma de seus algarismos. Qual
o menor nmero que pesa 2 000?
46. No cabeleireiro Trs clientes esto no cabeleireiro, pagando cada um a sua conta
no caixa.
O primeiro cliente paga uma quantia igual ao montante que h no caixa e recebe
10 reais de troco.
O segundo cliente efetua a mesma operao que o primeiro.
O terceiro cliente efetua a mesma operao que o primeiro.

Encontre o montante que estava inicialmente no caixa, sabendo que, ao m das trs
operaes, o caixa cou zerado.
47. O macaco e a raposa O macaco diz para a raposa: Voc v as trs pessoas que
esto correndo l longe? Eu sei que o produto de suas idades 2 450 e que a soma de
suas idades o dobro da sua idade. Voc pode me dizer suas idades? No, responde
a raposa. E se eu te disser que o mais jovem dos trs o nico louro, voc pode agora
descobrir as idades? Ento a raposa d as idades das trs pessoas.
Porque a raposa no pode responder inicialmente? E porque pode responder depois?
48. Nova sequncia Encontre a lei que forma a sequncia 425, 470, 535, 594, 716,
802, . . . e d seus prximos dois termos.
49. Retngulo quase quadrado Um certo terreno retangular quase quadrado, pois
sua largura e seu comprimento medem nmeros inteiros que diferem exatamente por
uma unidade de medida. A rea desse terreno, em unidades quadradas, um nmero
de quatro algarismos, sendo iguais o das unidades de milhar e o das centenas, bem
como o das dezenas e o das unidades. Quais so as possveis dimenses desse terreno?
50. Onde est o erro? Seja x soluo de x2 + x + 1 = 0. Ento x = 0 e, por isso,
podemos dividir ambos os membros da equao por x, obtendo x + 1 + 1/x = 0. Da
equao original temos que x + 1 = x2 , portanto, x2 + 1/x = 0, isto , x2 = 1/x
ou, ainda, x3 = 1, de modo que x = 1. No entanto, substituindo x = 1 na equao
x2 + x + 1 = 0 original, encontramos 3 = 0, o que no est exatamente correto. Onde
erramos?

OBMEP 2010

113

Solues do Nvel 1

Solues do Nvel 1
1. Qual o nmero? A opo correta (d).
Como 96 8 = 12, temos 8 12 = 96. Observe que a soluo equivalente a resolver
96
a equao 8x = 96, cuja raiz x =
= 12.
8
2. Muro em 15 dias A opo correta (c).
Se o pedreiro assenta 8 metros por dia, em 15 dias assentar 15 8 = 120 metros.
3. Medindo pilhas de papel A opo correta (e).
Como a espessura de cada folha 0,1 mm, a altura de um pacote com 500 folhas
500 0,1 mm = 50 mm. Logo, a altura de cada pilha ser de 60 50 mm = 3 000 mm
= 3 m, aproximadamente, a altura de uma sala de aula.
4. Quanto pesa? A opo correta (b).
Soluo 1: Retirando-se dois saquinhos e quatro bolas de cada prato, a balana
continua equilibrada e restam trs saquinhos no prato esquerda e seis bolas no prato
da direita. Logo, o peso de trs saquinhos igual ao peso de seis bolas. Da conclumos
que o peso de um saquinho igual ao peso de duas bolas.
Soluo 2: Denotando o peso de um saquinho por x e o peso de uma bola por y,
o equilbrio da balana fornece a equao 5x + 4y = 2x + 10y, da qual decorre que
3x = 5x 2x = 10y 4y = 6y, ou seja, x = 2y.
5. Calcule a diferena A opo correta (e).
Para que a diferena seja a maior possvel devemos escolher o maior nmero de trs
algarismos pares diferentes e o menor nmero de trs algarismos mpares diferentes.
O maior nmero de trs algarismos pares diferentes 864 e o menor nmero de trs
algarismos mpares diferentes 135. A diferena entre eles 864 135 = 729.
6. Qual o volume? A opo correta (b).
As guras mostram que os volumes ocupados pelos lquidos correspondem, aproximadamente, a mais do que da metade no frasco A, o que elimina as opes (a) e (e),
metade no frasco B e a menos da metade no frasco C, o que elimina (c) e (d). O nico
grupo de fraes que corresponde a essas estimativas 2/3 (mais do que a metade),
1/2 (metade) e 1/4 (menos do que a metade).
7. Descontos e descontos A opo correta (b).
Soluo 1: A pessoa ir pagar 120 reais menos o desconto, que de 30% sobre 120,
ou seja, de 0,3 120 = 36 reais. Assim, a pessoa paga 120 36 = 84 reais.
Soluo 2: Como o desconto de 30%, a pessoa pagar 70% de 120, ou seja,
0,7 120 = 84 reais.
114

OBMEP 2010

Solues do Nvel 1
8. O carro de Maria A opo correta (a).
Soluo 1: Se num percurso de 25 km ela gasta 3 litros, ento para percorrer 100
km Maria gastar 4 3 = 12 litros. Logo, para percorrer 600 km, o carro gastar
6 12 = 72 litros. Como cada litro custa 0,75 reais, 72 litros custaro 0,75 72 = 54
reais.
Soluo 2: Podemos usar a regra de trs para calcular quantos litros sero gastos em
600 km. Temos:
3 litros 25 km
x litros 600 km.
Como essa regra de trs direta, resulta 25x = 3 600 e, portanto,
x=3

600
= 72 litros.
25

Como cada litro custa 0,75 reais, 72 litros custaro 0,75 72 = 54 reais.
9. Calculando distncias A opo correta (a).
Soluo 1: Pelo enunciado, temos AC = 50, BD = 45
e AD = 80. Da gura segue que
CD = AD AC = 80 50 = 30.
Logo,
BC = BD CD = 45 30 = 15 km.
Soluo 2: Pela gura, temos que 45 BC = 80 50. Logo, BC = 15 km.
10. Pesando caixas A opo correta (e).
Na gura podemos ver uma coluna com trs caixas, quatro colunas com duas caixas e
trs colunas com uma caixa. Logo, o total de caixas
1 3 + 4 2 + 3 1 = 14.
Como cada caixa pesa 25 kg, o peso do monte de caixas 14 25 = 350 kg.
11. Consumo de gua A opo correta (c).
Lembre que a mdia aritmtica de n nmeros um ensimo da soma desses nmeros.
Por exemplo, a mdia aritmtica dos nmeros 3, 6, 8 e 26
3 + 6 + 8 + 26
43
=
= 10,75.
4
4
Analogamente, o consumo mensal mdio a soma dos consumos mensais dividida pelo
nmero de meses. Logo, o consumo mensal mdio igual a
12,5 + 13,8 + 13,7 + 11,4 + 12,1
= 12,7 m3 .
5
OBMEP 2010

115

Solues do Nvel 1
12. Folheando um livro A opo correta (c).
Entre 1 e 100, o algarismo 5 aparece nos nmeros 5, 15, 25, 35, 45, 50, 51, 52, 53,
54, 55, 56, 57, 58, 59, 65, 75, 85 e 95. A primeira folha contm as pginas 1 e 2, a
segunda folha as pginas 3 e 4, a terceira folha as pginas 5 e 6, e assim sucessivamente.
Logo, as duas pginas que compem cada folha tm a seguinte numerao: um nmero
mpar e o nmero par seguinte. Assim, esto numa mesma folha as duplas de nmeros
49, 50; 51, 52; 53, 54; 55, 56; 57, 58; 59, 60 e nesse grupo temos seis folhas. Por outro lado,
de 1 a 48, temos cinco folhas com as pginas 5, 15, 25, 35 e 45 e, de 61 a 100, temos
quatro folhas com as pginas 65, 75, 85 e 95. Conclumos que o total de folhas com o
algarismo 5 em sua numerao 6 + 5 + 4 = 15.
13. Calculando a soma A opo correta (c).
A partir de qualquer crculo, obtemos inicialmente a sequncia 0, 1, 2, 3, 4, 5, 6, 7, 8, 9.
Subtraindo uma unidade dos mpares e somando uma unidade aos pares, a sequncia
torna-se 1, 0, 3, 2, 5, 4, 7, 6, 9, 8. Agora fcil vericar que a maior soma possvel
com trs nmeros consecutivos 8 + 9 + 6 = 23.

14. Desenhando o cubo A opo correta (b).


Vemos que o cubo (a) igual ao cubo (e) e o cubo (c) igual ao cubo (d). Como no
podemos trocar a estrela com o crculo cheio mantendo o crculo oco no topo, vemos
que a alternativa correta (b).
15. Crculos concntricos A opo correta (c).
r

Observe que a gura simtrica em relao reta r que passa


pelo centro comum das circunferncias. Para cada regio cinza
de um lado de r existe uma regio branca equivalente do outro
lado de r, e vice-versa. Logo, a rea da regio cinza igual
rea branca. Portanto, a rea da regio cinza igual metade
da rea do crculo maior.
16. Brincando com engrenagens A opo correta (a).
A engrenagem desta questo formada por dois
discos dentados. Quando um deles gira no sentido
horrio, o outro gira no sentido anti-horrio.
116

OBMEP 2010

Solues do Nvel 1
As cinco opes da resposta mostram a bandeira do disco esquerda, numa posio que
corresponde a uma rotao desse disco no sentido anti-horrio por um certo ngulo.
Nesse caso, a engrenagem direita girou por esse mesmo ngulo no sentido horrio,
levando a bandeirinha para a posio indicada na primeira alternativa.
17. Troca de garrafas A opo correta (d).
Como 43 = 10 4 + 3, numa primeira vez as 43 garrafas vazias podem ser trocadas
por 10 garrafas cheias, sobrando ainda 3 vazias. Agora, consumindo o leite dessas
10 garrafas, camos com 13 vazias, que desta vez podem ser trocadas por 3 cheias,
sobrando 1 vazia. Finalmente, consumindo o leite das 3 garrafas cheias, sobram 4
vazias, que podem ser trocadas por 1 cheia. Portanto, o total de garrafas cheias de
leite que podem ser obtidas nesse processo 10 + 3 + 1 = 14.
18. Retngulo e quadrados A opo correta (c).
Soluo 1: Como os quadrados menores tm 1 m2
de rea, cada um deles tem lado igual a 1 m. Pela
gura, conclumos que BC = 2 m. Como ABCD
um quadrado, segue que BC = CD = AD = 2 m.
Sendo CDEF tambm um quadrado, temos

H
1

DE = 2 m. Pela gura, temos


AH = AB + BH = 2 + 3 + 5,
EJ = AD+DE+AJ e AJ = AH. Segue que EJ = 2+2+5 = 9. Como EG = AH = 5,
as dimenses do terreno so 9 m de comprimento por 5 m de largura. Portanto, sua
rea de 9 5 = 45 m2 .
Soluo 2: Quadriculando o retngulo maior
com quadrados de 1 m2 de rea, obtemos
o retngulo BF GH formado por 12 quadrados
de 1 m2 de rea, os dois quadrados ABCD
e DCF E formados por quatro quadrados de
1 m2 de rea e o quadrado AHIJ formado por 25
quadrados de 1 m2 de rea. Portanto, a rea pedida
de 12 + 4 + 4 + 25 = 45 m2 .
19. Quantas fatias de bolo? A opo correta (e).
Temos um total de 8 3 = 24 fatias de bolo que foram comidas. Como todos comeram
bolo, inicialmente cada um dos nove amigos comeu uma fatia, sobrando, ainda, 249 =
15 fatias para serem comidas por nove pessoas. Se todos os nove amigos tivessem
comido menos do que duas dessas 15 fatias, poderamos escrever o nmero 15 como
uma soma de nove parcelas, cada uma delas sendo 0 (os que no comeram alguma das
15 fatias) ou 1 (os que comeram uma das 15 fatias), o que claramente no possvel.
Logo, obrigatoriamente algum comeu pelo menos duas dessas 15 fatias. Como todos
j haviam comido, inicialmente, uma fatia, conclumos que algum comeu 3 fatias, no
mnimo.
OBMEP 2010

117

Solues do Nvel 1
20. Mosaicos quadrados A opo correta (a).
No primeiro mosaico, temos 3 + 3 + 1 + 1 = 8 azulejos pretos, no segundo, temos
4+4+2+2 = 12, no terceiro, temos 5+5+3+3 = 16 e no difcil perceber (e vericar)
que os prximos mosaicos tm 20 e 24 azulejos pretos, pois a cada novo mosaico so
usados mais quatro azulejos pretos, um em cada lado. Como 8 + 12 + 16 + 20 + 24 = 80,
possvel construir exatamente cinco mosaicos. Finalmente, o nmero total de azulejos
brancos nesta sequncia de cinco mosaicos
12 + 22 + 32 + 42 + 52 = 1 + 4 + 9 + 16 + 25 = 55.

21. Quanto custa? Comprando trs cadernos por 6 reais cada um, ainda sobram 4
reais para Ester, de modo que a quantia que ela possui 3 6 + 4 = 22 reais.
(a) Se o irmo lhe empresta 4 reais, ela ca ento com 22 + 4 = 26 reais e pode
comprar 2 cadernos a 6 reais cada um, sobrando 26 2 6 = 26 12 = 14 reais
para 7 canetas. Conclumos que o preo de cada caneta 14 7 = 2 reais.

(b) Como Ester possui 22 reais, se ela comprar 2 cadernos, sobram-lhe


22 2 6 = 22 12 = 10 reais. Como cada caneta custa 2 reais, ela poder
comprar 10 2 = 5 canetas.
22. Encontre o nmero O nmero 24 deve ser escrito como uma soma de trs algarismos. Inicialmente, note que os algarismos 0, 1, 2, 3, 4 e 5 no podem ser usados.
Realmente, se um deles fosse usado, por exemplo o algarismo 5, ento teramos que
encontrar dois algarismos cuja soma fosse 19, pois 24 5 = 19; mas, sabemos que isso
no possvel. O mesmo ocorre com os algarismos 0, 1, 2, 3 e 4. Logo, o nmero da
casa de Jlia s pode ser composto pelos algarismos 6, 7, 8 e 9.
(a) Se os trs algarismos so iguais, ento o nmero da casa de Jlia 888.
(b) Se apenas dois desses algarismos so iguais, esses dois algarismos devem ser iguais
a 9 e o terceiro deve ser 6, obtendo os nmeros 699, 969 e 996. De fato, com dois
algarismos 8, recamos no caso anterior e, com dois 6 ou dois 7, a soma d, no
mximo 14, restando, no mnimo, 10 para o terceiro, o que no possvel.

(c) Se os trs algarismos so distintos, ento esses algarismos so 7, 8 e 9. De fato, se


ocorrer um 6, a soma dos outros dois deve ser 24 6 = 18, portanto precisamos de
dois 9 e recamos no caso anterior. Assim, restam apenas as alternativas seguintes:
789, 798, 879, 897, 978 e 987.
23. Campeonato de futebol
(a) A equipe A disputa com as cinco equipes B, C, D, E e F ; a equipe B, alm da
partida contra A, j computada, ainda disputa quatro partidas com as equipes
118

OBMEP 2010

Solues do Nvel 1
C, D, E e F ; a equipe C, ainda disputa com as equipes D, E e F ; a equipe D
ainda disputa com as equipes E e F e a equipe E ainda disputa com a equipe F.
No total, temos 5 + 4 + 3 + 2 + 1 = 15 partidas disputadas.
Outra maneira de contar : Podemos formar grupos de duas letras e contar, lembrando que AB e BA, por exemplo, so a mesma partida: AB, AC, AD, AE,
AF, BC, BD, BE, BF, CD, CE, CF, DE, DF e EF d 15 partidas.
Outra maneira de contar : Cada uma das seis equipes disputou, com cada uma
das outras cinco, exatamente uma partida. Portanto, foram disputadas um total
1
de (6 5) = 15 partidas.
2
(b) Cada equipe disputou cinco partidas, portanto, a soma do nmero de vitrias,
empates e derrotas de cada equipe, igual a cinco. Assim, temos x + 1 + 0 = 5,
portanto, x = 4, e temos 1 + 1 + y = 5, portanto, y = 3.
Outra maneira de calcular x e y: Sabemos o nmero total de empates (que sempre
envolvem duas equipes), dado por 1 + 1 + 3 + 1 + 1 + 1 = 8. Portanto, o nmero de
vitrias (ou derrotas), igual nmero de partidas, 15, menos a metade do nmero
de empates, 4, ou seja, o nmero total de vitrias (ou de derrotas) 15 4 = 11.
Assim, 4 + 2 + 1 + x = 11, portanto, x = 4, e 2 + 2 + y + 4 = 11, portanto, y = 3.
Finalmente, o nmero total de gols marcados no campeonato igual ao nmero
total de gols sofridos, que 2 + 6 + 6 + 6 + 5 + 3 = 28. Assim,
28 = 6 + 6 + 2 + 3 + 1 + z,
ou seja, z = 10.
Resumindo, o nmero x de vitrias da equipe F 4, o nmero y de derrotas da
equipe D 3 e o nmero z de gols marcados pela equipe F 10.
24. Dividindo o paraleleppedo
(a) Em centmetros, as dimenses do bloco maior so 320 60 75 e as dos blocos
menores so 80 30 15. Logo, o comprimento foi dividido por 4 = 320 80,
a largura foi dividida por 2 = 60 30 e a altura foi dividida por 5 = 75 15.
Portanto, teremos um total de 4 2 5 peas, conforme a gura.

15
80

15
15

80

15
30

15
30

80
80

(b) O volume de um bloco dado por comprimentolarguraaltura. Logo, o volume


de cada um dos blocos menores 80 30 15 = 36 000 cm3 . Como o peso
dado em metros cbicos, devemos reduzir o volume de cada bloco para metros
cbicos. Para isso, deslocamos a vrgula seis casas para a esquerda, obtendo
36 000 cm3 = 0,036 m3 . Como 1 m3 pesa 900 kg, ento cada bloco menor de 0,036
m3 pesar 0,036 900 = 32,4 kg.
OBMEP 2010

119

Solues do Nvel 1
25. Uma calculadora A partir do nmero 1 no visor devemos aplicar sucessivamente
as operaes das teclas A e B para obter o nmero desejado. Observe que, para obter o
nmero 2 a partir do nmero 1, podemos apertar tanto a tecla A quanto a B, portanto,
A
B
em cada uma das respostas dadas, podemos trocar cada 1 2 por 1 2.
A

(a) 1 2 4 5 10.

(b) 1 2 3 6 7 14 15.
A

(c) 1 2 3 6 12 24 25 50 100.

26. Ano bissexto


(a) Uma semana tem sete dias. Na diviso de 365 por 7 encontramos quociente 52 e
resto 1. Logo, o ano comum tem 52 semanas e 1 dia. Portanto, a frase correta
O ano comum tem sete semanas e um dia. Como o ano bissexto tem 366 dias,
ele possui 52 semanas e 2 dias. Portanto, o correto dizer O ano bissexto tem
sete semanas e dois dias.
(b) Se um ano comum inicia numa tera-feira, ento a sua 52a semana inicia numa
tera e termina numa segunda, ou seja, a 52a semana dada por tera quarta
quinta sexta sbado domingo segunda. Como esse ano tem 52 semanas e
mais 1 dia, o ltimo dia deste ano ser uma tera. Logo, o ano seguinte iniciar
numa quarta.
(c) No caso do ano bissexto, devemos considerar um dia a mais do que no item
anterior. Logo, o seu ltimo dia ser uma quarta e, portanto, o ano seguinte
iniciar numa quinta-feira.
27. Nmeros triangulares Notamos que o segundo nmero triangular obtido a partir
do primeiro acrescentando-se 2, o terceiro obtido do segundo acrescentando-se 3 e
assim por diante. Essa observao nos mostra como calcular os prximos nmeros
triangulares sem fazer muitas contas. Por exemplo, j sabemos que o quarto nmero
triangular 10, donde o quinto ser 10 + 5 = 15 e o sexto sendo, ento, 15 + 6 = 21.
Assim, podemos escrever os nmeros triangulares at passar de 100.
1 3 6 10 15 21 28 36 45
+2

+3

+4

+5

+6

+7

+8

+9

45 55 66 78 91 105
+10

+11

+12

+13

+14

Logo, os nmeros triangulares menores do que 100, so 1, 3, 6, 10, 15, 21, 28, 36, 45,
55, 66, 78 e 91. Assim, temos 13 nmeros triangulares menores do que 100.
28. Livros separados Denotando por n o nmero de livros que a bibliotecria vai colocar em cada estante, temos 130 n = nmero de estantes para os livros de Matemtica
e 195 n = nmero de estantes para os livros de Portugus. Isso mostra que n deve
ser um divisor comum de 130 e de 195, pois o nmero de estantes utilizadas inteiro.
Sabemos que, quando aumentamos o denominador de uma frao, esta frao diminui;
por exemplo, 27/10 menor do que 27/8. Logo, quanto maior for o denominador n,
menores sero as fraes 130/n e 195/n, o que signica que menor ser o nmero de
120

OBMEP 2010

Solues do Nvel 1
estantes utilizadas. Vemos, assim, que n deve ser o maior divisor comum (MDC) de 130
e 195. Como as decomposies desses nmeros em fatores primos so 130 = 2 5 13
e 195 = 3 5 13, segue que o MDC de 130 e 195 5 13 = 65.
Logo, a bibliotecria vai colocar 65 livros em cada estante, o nmero de estantes para
os livros de Matemtica 130 65 = 2 e o nmero de estantes para os de Portugus
195 65 = 3, o que d um total de 2 + 3 = 5 estantes.

29. Alunos com culos Nosso problema aqui encontrar o nmero de alunos da classe.
1 1
1
Como 1/6 dos alunos usam culos e, desses, 1/3 so meninas, temos que =
3 6
18
dos alunos so meninas que usam culos. Como
alunos que usam culos meninas que usam culos corresponde a meninos que usam culos
1
6

1
18

1
1
3
1
2
1
1
e
=

=
= , conclumos que da classe consiste de meninos que
6 18
18 18
18
9
9
usam culos, que so em nmero de 4. Portanto,
1
da classe corresponde a 4 alunos
9
9
da classe corresponde a 4 9 = 36 alunos
9
Assim, o nmero de alunos na classe 36.
30. Quadrado mgico Para facilitar nossas contas, conveniente reduzir todas as
fraes que aparecem na tabela a um mesmo denominador. Como 0,4 = 4/10 e
0,5 = 5/10, podemos reescrever a tabela como segue, em que indicamos com as letras a, b, c, d e e os nmeros que devem ser calculados.
a
b
4/10

c
5/10
5/10

6/10
d
e

Olhando para a diagonal ascendente, vemos que a soma dos elementos dessa diagonal
4/10 + 5/10 + 6/10 = 15/10. Como a soma dos elementos da terceira linha deve
ser igual a essa soma dos elementos da diagonal, obtemos 4/10 + 5/10 + e = 15/10,
donde e = 6/10. Tambm obtemos, na segunda coluna, 5/10 + 5/10 + c = 15/10, donde
c = 5/10. Colocando esses valores de c e e na tabela, obtemos
a
b
4/10

5/10
5/10
5/10

6/10
d
6/10

Agora, a primeira linha fornece a + 5/10 + 6/10 = 15/10, donde a = 4/10. Da terceira
coluna, obtemos 6/10 + d + 6/10 = 15/10, donde d = 3/10; do mesmo modo, obtemos
b = 7/10 e a tabela est completa.
OBMEP 2010

121

Solues do Nvel 1
4/10
7/10
4/10

5/10
5/10
5/10

6/10
3/10
6/10

31. Trs algarismos A partir da igualdade (AB)2 = CAB e denotando o nmero


de dois algarismos AB por x, temos x2 = (AB)2 = CAB = C 100 + x, ou seja,
x2 x = C 100. Portanto, o produto x(x 1) = x2 x divisvel por 100. Levando
em conta a fatorao 100 = 22 52 , dividimos a resoluo em trs casos, conforme a
maior potncia de 5 que divide x.
1o Caso: 52 divide x.
Como x um nmero de dois algarismos, os possveis valores de x so 25, 50 e 75.
Construmos uma tabela.
x
25
50
75

x1
24
49
74

x (x 1)
600
2 450
5 550

Portanto, o nmero x(x1) um mltiplo de 100 somente se x = 25. Como 252 = 625,
nesse caso temos C = 6.
2o Caso: 5 divide x, mas 52 no divide x.
Ento, necessariamente, 5 divide x 1, pois 5 divide x (x 1). Mas isso uma
impossibilidade, porque 5 no pode dividir os dois nmeros consecutivos x 1 e x
(lembre que os nmeros divisveis por 5 terminam em 0 ou 5). Logo, esse caso est
excludo.
3o Caso: 5 no divide x.
Ento 52 divide x1. Como no Caso 1, temos x1 = 25,50 ou 75 e os possveis valores
de x so 26, 51 e 76. Construmos uma tabela.
x
26
51
76

x1
25
50
75

x (x 1)
650
2 550
5 700

O produto x (x 1) um mltiplo de 100 somente se x = 76, mas esse caso tambm


est excludo, pois 762 = 5 776 tem mais do que trs algarismos.
Assim, a nica possibilidade A = 2, B = 5 e C = 6, com soma A + B + C = 13.
32. Pintando quadradinhos Para pintar a faixa conforme o modelo, o
retngulo padro (aquele que se repete por toda a faixa) o retngulo de 5 linhas e 4 colunas mostrado na gura. Nele, temos 7
quadradinhos pintados e 13 no pintados. Precisamos saber quantos
retngulos padro cabem na faixa. A faixa tem 250 colunas e cada
retngulo padro tem 4 colunas. Da diviso de 250 por 4 temos que
250 = 462+2, e conclumos que na faixa cabem 62 retngulos padro,
sobrando ainda duas colunas.
122

OBMEP 2010

Solues do Nvel 1

62 retngulo
padro

1 retngulo
padro

2 colunas
a mais

Nos 62 retngulos padro temos 62 13 = 806 quadradinhos no pintados. Agora


falta vericar quais so os quadradinhos no pintados nas duas colunas nais. A gura
mostra como so as duas colunas, de acordo com o modelo. Nessas colunas temos 6
quadradinhos no pintados. Assim, o nmero de quadradinhos no pintados em toda
a faixa 806 + 6 = 812.
33. A cisterna do Joo O dia 1o de janeiro comea com 156 litros de gua na cisterna
e, a partir da, a cisterna recebe gua da chuva e perde gua para regar as ores. Como
no dia 8 no houve alterao na quantidade de gua na cisterna, o nmero de litros de
gua na cisterna no dia 8
156 + gua de chuva do dia 1 ao dia 7 gua para regar do dia 1 ao dia 7.

O enunciado diz que a segunda parcela da expresso acima a soma dos nmeros da
terceira coluna, que 2,5 + 0 + 5 + 0 + 3 + 0 + 4,5 = 15 e a terceira parcela a soma
dos nmeros da segunda coluna da tabela, que 6 + 9 + 0 + 4 + 9 + 0 + 11 = 39. Assim,
o nmero de litros na cisterna, meia noite do dia 8, 156 + 15 39 = 132.
34. O mltiplo de 13 A opo correta (a).
Como 119 268 916 divisvel por 13, j que 9 174 532 13 = 119 268 916, podemos
concluir que os nmeros divisveis por 13 so aqueles obtidos somando ou subtraindo
mltiplos de 13 ao nmero 119 268 916. Dentre os nmeros apresentados, o nmero
119 268 916 13 = 119 268 903
o nico divisvel por 13.
35. Um bilho A opo correta (e).
Arnaldo disse que 1 bilho = 1 000 000 1 000 000 = 1 000 000 000 000 = 1012 . O
Professor Piraldo corrigiu-o, dizendo que 1 bilho = 1 0001 000 000 = 1 000 000 000 =
109 . A diferena
1 000 000 000 000 1 000 000 000 = 999 000 000 000.
36. Energia de abelha A opo correta (b).
A energia gasta por uma abelha para voar 7 000 quilmetros a mesma que 7 000
abelhas gastam para voar 1 quilmetro cada. Como o nmero de litros de mel foi
multiplicado por 10, temos energia suciente para que 10 vezes esse nmero de abelhas
voem 1 quilmetro cada, ou seja, 70 000 abelhas.
OBMEP 2010

123

Solues do Nvel 1
37. Perda de safra A opo correta (a).
1
1
Como um quinto de 100 000 5 100 000 = 20 000 e um quarto de 100 000 4 100 000 =
25 000, conclumos que a perda da safra est avaliada entre 20 000 e 25 000 reais. Logo,
um possvel valor para a perda de R$ 21 987,53.

38. Placa decorativa A opo correta (c).


Traando paralelas aos lados, podemos dividir a placa
em quadrados de 1 metro de lado, conforme indicado na
gura. Ento, a rea pintada igual a 12 metades desses
quadrados, ou, equivalentemente, 6 desses quadrados.
Como a placa total tem 16 desses quadrados, conclumos que a frao da rea pintada em relao rea da
6
3
placa
= .
16
8

1m
1m
1m
1m
1m
1m

39. O suco do Diamantino A opo correta (a).


O refresco composto por 20% de um litro, ou seja, 0,2 litros de suco e por 80% de
um litro, ou seja, 0,8 litros de gua. Logo, a mistura nal tem 0,2 litros de suco e
3 + 0,8 = 3,8 litros de gua. A porcentagem de suco em relao ao volume da mistura
, ento,
0,2
2
5
volume de suco
=
=
=
= 5%.
volume total
4
40
100
40. Uma eleio Joo recebeu 2/7 do total de votos, Rosa recebeu 2/5 do total de
votos e Marcos recebeu 1 2/7 + 2/5 = 1 24/35 = 11/35 do total de votos. O
vencedor foi aquele que obteve a maior frao dos votos. Para comparar essas fraes,
igualamos seus denominadores, obtendo 2/7 = 10/35 e 2/5 = 14/35. Assim, temos
2
11
2
<
<
,
7
35
5
Joo Marcos Rosa
e, portanto, Rosa venceu a eleio. ( interessante notar que a resposta no depende
do nmero de alunos da turma.)
41. Soma de potncias A opo correta (a).
Temos 26 + 26 + 26 + 26 44 = 4 26 44 . H vrias maneiras de calcular isso.
Soluo 1: 4 26 44 = 4 (22 )3 44 = 4 43 44 = 44 44 = 0.
Soluo 2: 4 26 44 = 4 26 43 = 4 26 (22 )3 = 4 26 26 = 0.
Soluo 3: 4 26 44 = 22 26 (22 )4 = 28 28 = 0.
42. Seis retngulos A opo correta (e).
124

OBMEP 2010

Solues do Nvel 1
A partir da gura, vemos que o comprimento a dos
retngulos menores o dobro da sua largura b, isto
, a = 2b. Temos, ento,

a + b = 2b + b = 3b = 21,
ou seja, b = 7 cm e a = 14 cm. Portanto, o comprimento do retngulo maior 4b = 28 e sua rea
21 28 = 588 cm2 .

b
a

43. Duas populaes A opo correta (e).


Seja p a populao de Tucupira h trs anos. Como essa populao cresceu 50%,
atualmente Tucupira tem p + 50% de p habitantes, ou seja,
p+

50
p = p + 0,5 p = 1,5 p habitantes.
100

Como a populao de Pirajussara no cresceu nesses 3 anos e h 3 anos era igual


de Tucupira, podemos concluir que a populao atual de Pirajussara p. Como a
soma das populaes das duas cidades, hoje, de 9 000, obtemos p + 1,5 p = 9 000,
donde p = 9 000/2,5 = 3 600. Assim, a soma das duas populaes, h trs anos, era de
3 600 2 = 7 200 habitantes.
44. Trs balanas A opo correta (d).
Na primeira balana temos 3 + 1 = 6 . Na segunda, temos 2 + 4 = 8 , o
que equivalente a 1 + 2 = 4 . Logo, (3 + 1) + (1 + 2) = 6 + 4 , ou seja,
4 +3 = 10 . Assim, ser necessrio colocar 10 quadrados no prato direito da balana
(3) para que ela que equilibrada.
45. Poucos domingos A opo correta (c).
Um ano normal tem 365 dias e o ano bissexto 366. Da diviso de 365 por 7, obtemos
365 = 52 7 + 1 e da diviso de 366 por 7 obtemos 366 = 52 7 + 2. Logo,
ano normal = 52 semanas + 1 dia
ano bissexto = 52 semanas + 2 dias
Portanto, um ano normal ou bissexto tem, no mnimo, 52 e, no mximo, 53 domingos
(um domingo para cada uma das 52 semanas e, talvez, um outro domingo para o dia
ou os dois dias que completam o ano).
Cada um dos 12 meses do ano tem, no mnimo, 28 dias e, no mximo, 31 dias, portanto,
tem, no mnimo, 4 domingos e, no mximo, 5 domingos. Levando em conta que
12 4 = 48, conclumos que
i) Num ano de 52 domingos sobram ainda 52 48 = 4 domingos. Cada um desses
car num ms diferente, porque nenhum ms pode ter seis domingos; logo, temos
quatro meses com 5 domingos.
ii) Analogamente, num ano com 53 domingos restam 5 domingos, que caro um em
cada ms diferente. Portanto, nesse caso, teremos cinco meses com 5 domingos.
OBMEP 2010

125

Solues do Nvel 1
46. Metade de potncia A opo correta (e).
Antes de dividir a expresso por 2, colocamos 210 em evidncia, obtendo
212 + 3 210 = 210 22 + 3 1 = 210 7. Assim,
212 + 3 210
210 7
=
= 29 7.
2
2
47. Minutos demais A opo correta (d).
Dividindo 2 880 717 por 60, obtemos 2 880 717 = 48 011 60 + 57. Isso signica que
2 880 717 min = 48 011 h + 57 min. Podemos, ento, escrever:
2 880 717 min = 48 000 h + 11 h + 57 min.
2 000 dias

Os 2 000 dias no interferem no horrio que estamos procurando, e como 18 horas e 27


minutos so exatamente 17 horas e 87 minutos, a resposta 18h87min 11h57min =
6h30min.
48. Dois nibus A opo correta (b).
1
O nmero total de alunos nos dois nibus 57 + 31 = 88 e 2 88 = 44. Para que cada
nibus tenha o mesmo nmero de alunos, 5744 = 13 alunos devem passar do primeiro
para o segundo nibus.

49. Cubo de papelo A opo correta (e).


Com as peas ilustradas ao lado podemos construir um cubo.
50. Algarismo das unidades A opo correta (c).
O ltimo algarismo de um mltiplo de 5 0 ou 5; os que terminam em 0 so pares e
os que terminam em 5 so mpares. Como 1 3 5 97 99 mpar, por ser
um produto de nmeros mpares, e um mltiplo de 5, segue que seu algarismo das
unidades 5.
51. Regio sombreada A opo correta (b).
A parte sombreada consiste em 10 metades de quadrados mais 3 quadrados inteiros, o
1
que equivale a 2 10 + 3 = 5 + 3 = 8 quadrados inteiros. Logo, a frao que representa
a parte sombreada
rea de 8 quadrados
8
4
rea sombreada
=
=
= .
rea total
rea de 18 quadrados
18
9
52. Colorindo um mapa A opo correta (b).
O estado A pode ser pintado de trs formas: verde, azul ou amarelo. Para qualquer
estado vizinho, por exemplo, o estado B, temos duas possibilidades, e os demais estados tm suas cores determinadas. Logo, podemos colorir o mapa de 32 = 6 formas.

126

OBMEP 2010

Solues do Nvel 1
Abaixo ilustramos duas dessas maneiras de pintar o mapa; em ambas, o estado A tem
a mesma cor.

53. Pintando um tabuleiro A opo correta (c).


Para satisfazer as condies do problema, as cinco casas das diagonais, marcadas com *,
devem ter cores diferentes. Por isso, precisaremos de, no mnimo, cinco cores distintas.
Denotemos essas cinco cores distintas por 1, 2, 3, 4 e 5 e vamos determinar como
podemos escolher as cores para as quatro casas restantes de modo a satisfazer as
condies pedidas. Uma maneira dada direita, a seguir.
*

*
*

3
*

1
4
2

2
3
4

4
1
5

Logo, possvel pintar as quatro casas restantes sem utilizar mais cores. Assim, bastam
cinco cores. A seguir, mostramos outras trs maneiras de colorir as casas.
2

3
1

2
4
5

4
1
2

3
2
4

1
4
3
5

2
4

54. Nmero X,Y Temos X,Y = X +

1
3
4

3
2
1

1
2
3

3
4
2

2
1
5

5
4
2

Y
10X + Y
=
e sabemos que
10
10

3
10X + Y
= X, Y = (X + Y ).
10
10
Logo, 10X + Y = 3X + 3Y, ou seja, 7X = 2Y. Conclumos que 2Y mltiplo de 7 e,
como Y um nmero inteiro entre 1 e 9, s temos a possibilidade Y = 7, donde X = 2.
Assim, o nmero 2,7.
55. Construo de casas Como as casas so vizinhas, podemos pensar nelas como
uma la de casas com seis posies. Vamos dividir a contagem em casos, de acordo
com o nmero de casas de madeira que podem ser construdas.
(a) Nenhuma casa de madeira: aqui h apenas uma maneira de construir as casas,
ou seja, todas de alvenaria.
OBMEP 2010

127

Solues do Nvel 1
(b) Uma casa de madeira: aqui temos seis maneiras de construir as casas, pois a casa
de madeira pode ser qualquer uma delas, sendo as outras de alvenaria.
(c) Duas casas de madeira: as casas de madeira podem ocupar as seguintes posies:
1 e 3, 1 e 4, 1 e 5, 1 e 6, 2 e 4, 2 e 5, 2 e 6, 3 e 5, 3 e 6 ou 4 e 6. Aqui temos 10
maneiras.
(d) Trs casas de madeira: as casas de madeira podem ocupar as seguintes posies:
1, 3 e 5; 1, 3 e 6; 1, 4 e 6; 2, 4 e 6. Aqui temos quatro maneiras.
(e) Quatro ou mais casas de madeira: impossvel, pois fcil ver que, nesse caso,
sempre teremos duas casas de madeira contguas.
Dessa forma, h 1 + 6 + 10 + 4 = 21 maneiras de planejar a construo.
56. Comparao de grandezas A opo correta (c).
Temos 1 000 + 0,01 = 1 000,01 e 1 000 0,01 = 1 000

1
= 10, bem como
100

1 000
1 000
= 1 = 1 000 100 = 100 000
0,01
100
e 0,01/1 000 = 0,00001. Finalmente, 1 000 0,01 menor do que 1 000 (no sendo preciso efetuar o clculo para obter esta concluso), de modo que o maior desses nmeros
1 000/0,01.
57. Maior nmero de seis algarismos A opo correta (c).
Soluo 1: Para que seja o maior possvel, o nmero deve comear com o maior
algarismo. Para termos seis algarismos sem mudar a ordem, o maior 8 e, depois, 7.
Agora faltam quatro algarismos para completar o nmero, portanto, escolhemos 9 103.
Logo, o nmero 879 103.
Soluo 2: As opes D e E no servem, pois a ordem foi alterada. Como nas opes
A, B e C no foi alterada, basta escolher o maior nmero dentre essas opes, que C.
58. Qual o numerador? A opo correta (a).
Como

1
4
1
6
=
e = , ento n s pode ser igual a 5.
6
24 4
24

59. Correndo menos A opo correta (a).


km
1
km
6 min = 10
6
h = 1 km.
h
h
60
Percorrendo essa mesma distncia de 1 km em 8 minutos, a velocidade ser

Soluo 1: A distncia percorrida d = 10

v=

1 km
60
1 km
15
=
=
km/h =
km/h = 7,5 km/h .
1
8 min
8
2
8 60 h

Soluo 2: Podemos usar diretamente a regra de trs, como segue.


128

OBMEP 2010

Solues do Nvel 1
Velocidade em km/h
10

Tempo em horas
6
60
8
60

Velocidade e tempo so grandezas inversamente proporcionais (aumentando a veloci8


6
dade, diminui o tempo), logo x/10 = 60 / 60 = 6/8, portanto, x = 60/8, ou seja, a
velocidade ser
60
15
x=
km/h =
km/h = 7,5 km/h .
8
2
60. Cinco vizinhas Helosa chega a seu andar depois de Elza, mas antes de Cludia
signica que Helosa mora acima de Elza e abaixo de Cludia e Quando Sueli chega
ao seu andar, Helosa ainda tem 2 andares para subir, e o mesmo ocorre a Patrcia
quando Elza chega ao seu andar signica que Helosa mora dois andares acima de Sueli
e Patrcia dois andares acima de Elza. Entretanto, como Sueli no mora no primeiro
andar e Helosa mora dois andares acima de Sueli, ou Sueli mora no segundo e Helosa
no quarto ou Sueli mora no terceiro e Helosa no quinto. Mas
5o andar Cludia
Cludia mora acima de Helosa, portanto Helosa no pode
4o andar Helosa
morar no ltimo andar, o quinto. Assim, Sueli mora no se3o andar Patrcia
gundo andar, Helosa no quarto e Cludia no quinto. Final2o andar
Sueli
mente, Patrcia mora dois andares acima de Elza, logo Elza
o
1 andar
Elza
mora no primeiro andar e Patrcia no quarto andar.
61. Potncias de 9 A opo correta (d).
920 + 920 + 920 = 3 920 = 3 32

20

= 3 340 = 341 .

62. Dois nmeros A opo correta (c).


Como a diferena 989 e o menor nmero tem dois algarismos (sendo, portanto, maior
do que 9), o nmero de trs algarismos deve ser maior do que 989 + 9 = 998, de modo
que a nica opo 999. Assim, o nmero de dois algarismos 10 e a soma dos dois
999 + 10 = 1 009.
63. Menor natural A opo correta (d).
Observe que 10n 1 um nmero que tem todos os seus algarismos iguais a 9. Note,
tambm, que um mltiplo de 37, da forma 37 n, s termina em 9 se n terminar em
7. Ento, os menores mltiplos de 37 terminados em 9 so 37 7 = 259, 37 17 = 629
e 37 27 = 999. Como 999 = 103 1, segue que n = 3.
64. Imunes gripes A opo correta (a).
Contraram a gripe 0,15% de 14 000 000, ou seja,
0,15
140 000 000 = 0,0015 14 000 000 = 21 000
100
pessoas. Portanto, no contraram a gripe 14 000 000 21 000 = 13 979 000 pessoas.
OBMEP 2010

129

Solues do Nvel 1
65. O cdigo secreto A opo correta (b).
O cdigo s pode ser formado com os algarismos 1, 2, 3, 4, 5, 6, 7, 8, e 9.
Da primeira informao temos que 1, 2 e 3 no fazem parte do cdigo
(nmeros que no fazem parte esto sublinhados nas tabelas). Da terceira informao, conclumos que 6 faz parte do cdigo, e sua posio
___6___ ou ___ ___6.

1
4
6
5
8

2
5
1
4
4

3
6
2
7
3

Da segunda informao segue que 4 e 5 no fazem parte do cdigo e a


posio do 6 no cdigo ___ ___6. Da ltima informao s temos
que o cdigo da forma 8 ___6. Com a quarta informao completamos
o cdigo: 876.

1
4
6
5
8

2
5
1
4
4

3
6
2
7
3

66. Parnteses, colchetes e chaves A opo correta (e).


As ordens de prioridade para resolver uma expresso so
parnteses colchete chaves
2o

1o

3o

e
multiplicaes e divises somas e subtraes .
2o

1o

Assim,
2 2 2 2 2 2(4 2)
2

=22 22 24
2

=22 22 222
4

= 2 2 2 2 (2)

= 2 2 2 (4) = 2 2 2 + 4
6

= 2 2 6 = 2 12 = 10.
12

67. Ordenando fraes A opo correta (a).


Soluo 1: O mnimo mltiplo comum (MMC) dos denominadores 30. Reduzindo
todas as fraes a esse denominador comum, temos
40
4
= ,
3
30
Ordenando,

Conclumos que

130

4
24
= ,
5
30

4
20
= ,
6
30

3
18
= ,
5
30

6
36
=
5
30

12
18
20
24
36
40
<
<
<
<
<
.
30
30
30
30
30
30
3
4
4
6
4
2
< < < < < .
5
5
6
5
5
3
OBMEP 2010

2
12
=
.
5
30

Solues do Nvel 1
Soluo 2: Escrevendo as fraes na forma decimal, temos
4
= 1,33 . . . ,
3

4
= 0,8,
5

Logo,

4
= 0,66 . . . ,
6

3
= 0,6,
5

6
= 1,2 e
5

2
3
4
4
6
4
<
<
<
<
<
5
5
6
5
5
3
0,4

0,6

0,66...

0,8

1,2

2
= 0,4.
5

1,33...

68. Nmeros de trs algarismos A opo correta (e).


Por serem maiores do que 200, seus algarismos das centenas s podem ser 3 ou 5.
Comeando com 3, temos 315 e 351 (que no repetem algarismos) e 311, 313, 331, 335,
353, 333 e 355 (repetindo algarismos), ou seja, nove nmeros.
Comeando com 5, basta trocar o 3 com o 5 nos nmeros acima. Logo, temos 9 desses
nmeros. Assim, temos um total de 18 nmeros que podem ser escritos usando apenas
os algarismos 1, 3 e 5.
69. Velocidade de maratona A opo correta (d).
O tempo que o vencedor gastou foi de
13h45min 11h330min = 2h15min =2 +

9
1
h = h.
4
4

Logo, a velocidade mdia, em km/h,


espao percorrido em km
42
168
= 9 =
= 18,6 km/h.
tempo gasto em horas
9
4
70. Bilhetinhos com nmeros A opo correta (c).
Se todas as alunas escrevessem o nmero 1, o produto seria 1, que no est entre
as opes. Logo, 2 ou 4 so fatores do produto e, por isso, o produto deve ser uma
potncia de 2. O maior produto possvel seria obtido no caso em que todas as 5 alunas
escrevessem o nmero 4, e o produto seria
4 4 4 4 4 = 45 = 210 = 1 024.
Logo, podemos eliminar 2 048. Agora temos que:
100 e 120 so divisveis por 5, logo no so potncias de 2;

768 divisvel por 3 (7 + 6 + 8 = 21), logo no potncia de 2.


A nica resposta possvel 256 = 28 . Seria, por exemplo, o caso em que duas alunas
escrevessem o nmero 2 e trs escrevessem o nmero 4, com 256 = 2 2 4 4 4.
71. Produto de fraes A opo correta (d).
1

1
2

1
3

1
4

1
1
1 2 3 4
= =
5
2 3 4 5
5

OBMEP 2010

131

Solues do Nvel 1
72. Produto mximo A opo correta (a).
Basta examinar os produtos dos nmeros naturais cuja soma 11.
e
e
e

11 = 1 + 10
11 = 3 + 8
11 = 5 + 6

1 10 = 10
3 8 = 24
5 6 = 30

11 = 2 + 9
11 = 4 + 7

e
e

2 9 = 18
4 7 = 28

73. Quem o cubo? A opo correta (c).


Temos 3m = 81 = 34 , donde m = 4. Logo, m3 = 43 = 4 4 4 = 64.
74. Qual o maior? A opo correta (c).
Somando 3 a todos os membros, obtemos a 1 + 3 = b + 2 + 3 = c 3 + 3 = d + 4 + 3,
de modo que a + 2 = b + 5 = c = d + 7, mostrando que c o maior dos nmeros.
75. Quatro formiguinhas A opo correta (b).
O trajeto de Biloca 3 diagonais + 4 larguras + 2 comprimentos. O trajeto de Pipoca
de 25 dm compreende 5 diagonais, logo o comprimento de uma diagonal 25 5 =
5 dm. O trajeto de Tonica de 37 dm compreende 5 diagonais mais 4 larguras da lajota,
ou seja, 25 + 4 larguras = 37, donde 4 larguras = 37 25 = 12 dm e a largura de
uma lajota 3 dm. O trajeto de Cotinha de 32 dm compreende 5 comprimentos + 4
larguras, ou seja, 5 comprimentos + 12 = 32, donde 5 comprimentos = 32 12 = 20
e o comprimento de uma lajota 4 dm. Assim, Biloca percorre
3 diagonais + 4 larguras + 2 comprimentos = 15 + 12 + 8 = 35 dm.
35

43

24

76. Trocando gurinhas A moeda de troca de Guilherme so as gurinhas de aranha,


portanto calculamos o valor-aranha das gurinhas que Clia quer trocar, usando as
informaes dadas.
(a)

(d)

(e)

4 borboleta = 12 tubaro = 24 periquito = 72 aranha


43

122

(d)

(e)

5 tubaro = 10 periquito = 30 aranha


103

52

(d)

(b)

3 cobra = 9 periquito = 27 aranha


33

93

243

(c)

6 macaco = 24 aranha
64

(d)

6 periquito = 18 aranha
63

Logo, Clia receber 72 + 30 + 24 + 27 + 18 = 171 gurinhas de aranha.


77. Soma de fraes A opo correta (d).
10
100
10
10 + 20 + 30 + 40
+
=
+
= 10 + 0,1 = 10,1
10
10 + 20 + 30 + 40
10
100
132

OBMEP 2010

Solues do Nvel 1
78. Geometria com palitos A opo correta (c).
Para o tringulo foram usados 6 3 = 18 palitos, sobrando, ento, 60 18 = 42
palitos para formar os trs lados do retngulo. Da gura, temos que a largura do
retngulo formada por seis palitos, logo o comprimento formado por 1 42 6) = 18
2
palitos. Como cada palito tem 5 cm de comprimento, a rea do retngulo dada por
6 5 18 5 = 30 90 = 2 700 cm3 .
largura

comprimento

79. Um incndio e o bombeiro A opo correta (c).


O sobe-desce do bombeiro a partir do degrau do meio at chegar ao ltimo degrau
dado por
sobe
sobe sobe
+5 7 +8 +7 ,
desce

de modo que o bombeiro sobe 8 + 5 = 13 degraus acima do degrau do meio, chegando


ao ltimo degrau da escada. Portanto, a escada tem 13 degraus acima do degrau do
meio, e igualmente 13 degraus abaixo do degrau do meio. Portanto, a escada tem
13 + 1 + 13 = 27 degraus. Veja um esquema da movimentao do bombeiro.
(3)sobe 8
(1)sobe 5

ltimo
degrau

Degrau
do meio
(2)sobe 7

(4)sobe 7

80. rvore genealgica A opo correta (c).


Na gura vemos que o pai de Evaristo Jos. O irmo de Jos Jean. O pai de Jean
Lus. O irmo de Lus Andr.
irmo do pai de Evaristo = irmo de Jos = Jean

Jos
pai do irmo do pai de Evaristo = pai de Jean = Lus
Jos

Jean
irmo do pai do irmo do pai de Evaristo = irmo de Lus = Andr
Jos

Jean
Lus
OBMEP 2010

133

Solues do Nvel 1
81. Colcha quadrada A opo correta (b).
A colcha formada de 5 5 = 25 quadradinhos, todos iguais. J os tringulos so
de dois tipos, o tipo I, que corresponde a meio quadrado e o tipo II, que corresponde
a 1/4 de um quadradinho. A parte em cinza composta de 8 tringulos do tipo I, 8
tringulos do tipo II e 4 quadrados, ou seja,
8 tringulos tipo I + 8 tringulos tipo II +4 quadrados = 10 quadrados.
4 quadrados

2 quadrados

Logo, a frao correspondente parte cinza

40
10
=
= 40%.
25
100

82. Falsas igualdades A opo correta (e).


Nenhuma igualdade est correta.
(i) Errada: 3 106 + 5 102 = 3 000 000 + 500 = 3 000 500 = 8 108 .
1
1
(ii) Errada: 23 + 23 = 23 + 3 = 8 + = 1 = 20 .
2
8
(iii) Errada, a multiplicao precede a soma: 5 8 + 7 = 40 + 7 = 47 = 75.
(iv) Errada, a diviso precede a soma: 5 + 5 5 = 5 + 1 = 6 = 2.

83. Menor valor da soma A opo correta (c).


Seja N o nmero dado por N = 3a = 4b = 7c. Ento, o nmero N um mltiplo de
3, 4 e 7. Portanto, quando fatoramos o nmero N em fatores primos, aparecem, pelo
menos, os fatores 2, 3 e 7, o primeiro dos quais com expoente, no mnimo, igual a 2.
Segue que N um mltiplo de 22 37 = 84. Por outro lado, os nmeros a = 47 = 28,
b = 3 7 = 21 e c = 4 3 = 12 satisfazem as igualdades 3a = 4b = 7c. Logo, a = 28,
b = 21 e c = 12 so os menores valores possveis para a, b e c e 28 + 21 + 12 = 61 o
menor valor possvel para a + b + c.
84. Procurando um quadrado perfeito A opo correta (d).
Fatorando 120 obtemos 120 = 23 3 5. Para obter um quadrado perfeito, todos os
expoentes dessa decomposio devem ser pares, logo basta multiplicar 120 por
2 3 5 = 30.
De fato, temos, 120 30 = 23 3 5 2 3 5 = 24 32 52 = (22 3 5)2 = 602 .
85. Visitas num museu A opo correta (c).
Observe que os nicos algarismos que no aparecem no nmero 1 879 564 so 0, 2 e
3. O prximo nmero com todos os algarismos distintos ocorrer quando mudar o
algarismo das centenas e tivermos 1 879 6 . Logo, o menor nmero ser 1 879 602 e
ainda faltam 1 879 602 1 879 564 = 38 visitantes.
86. Ligando nmeros por echas A opo correta (e).
, formado por seis echas,
O caminho-padro o que se repete, a saber,
sempre comeando nos mltiplos de 6, ou seja, em 0, 6, 12, etc. Vamos averiguar qual
134

OBMEP 2010

Solues do Nvel 1
a posio de 1 997 em relao ao mltiplo de 6 mais prximo. Dividindo 1 997 por 6,
obtemos 1 997 = 6 332 + 5, correspondendo a 336 caminhos-padro mais o resto de
5 echas. Portanto, 1 998 mltiplo de 6 mais prximo de 1 997, ocupando a primeira
posio no caminho-padro. Assim,
1998

1999

1997

2000

o caminho que ocorre entre 1 997 e 2 000.


87. Mltiplos de 9 Um nmero s um mltiplo de 9 se a soma dos seus algarismos
for um mltiplo de 9.
(a) O nmero deve ter 9 algarismos iguais a 1, ou seja, 111 111 111.
(b) Devemos usar o maior nmero possvel de algarismos iguais a 2, todos cando nas
casas mais direita. Assim, o menor nmero 12 222.
88. A orista Se a orista vender as ores sem desidrat-las, ela vai apurar um total
de 49 1,25 = 61,25 reais. O peso das ores depois da desidratao
1

2
5
49 = 49 = 14 kg.
7
7

Logo, vendendo as ores desidratadas, ela apura um total de 14 3,25 = 45,50 reais.
Assim, a orista ganha mais no processo sem a desidratao.
89. Divisores Como 2, 3, 5 e 7 so primos, os divisores do nmero N = 2a 3b 5c 7d
so os nmeros da forma 2m 3n 5p 7q , com 0 m a, 0 n b, 0 p c e
0 q d. Portanto, N tem (a + 1) (b + 1) (c + 1) (d + 1) divisores. Decompondo
378 em fatores primos, encontramos 378 = 2 33 7, portanto queremos a, b, c e d tais
que
(a + 1) (b + 1) (c + 1) (d + 1) = 2 33 7.
Por outro lado, para N ser mnimo, os expoentes devem ser ordenados do maior para
o menor, isto , a b c d.

Armamos que d > 0, pois se d = 0 ento a + 1, b + 1 ou c + 1 tem dois fatores maiores


do que 1. Se a + 1 = mn, com m n > 1, temos que
2a = 2mn1 = 2m1 2mnm = 2m1 (2m )n1 2m1 8n1 > 2m1 7n1 ,
onde na penltima desigualdade usamos o fato que m 3. Assim, temos que
2a 3b 5c 7d > 2m1 3b 5c 7n1
e, portanto, encontramos um nmero com a mesma quantidade de divisores, mas
menor. O argumento igual no caso em que b + 1 ou c + 1 tem dois fatores. Assim, d 1 e restam somente as possibilidades dadas na tabela seguinte.
OBMEP 2010

135

Solues do Nvel 1
a
20
13
8
6

b
2
2
6
5

c
2
2
2
2

d
1
2
1
2

(a + 1)(b + 1)(c + 1)(d + 1) = 378


21 3 3 2
14 3 3 3
9732
7633

Por ltimo, como

27
220 32 52 71
=
> 1,
213 32 52 72
7

213 32 52 72
25 7
= 4 >1
28 36 52 71
3

28 36 52 71
22 3
=
> 1,
26 35 52 72
7

temos que o valor de N 26 35 52 72 . Portanto, a = 6, b = 5, c = 2 e d = 2.


90. O produto dos algarismos
(a) Como 12 = 2 6 = 4 3 = 2 2 3, devemos utilizar os algarismos 1, 2, 3, 4 e
6 cujos produtos sejam 12. Assim, temos:
nmeros com 2 algarismos: 26,
nmeros com 3 algarismos:
com os algarismos 1, 2 e 6:
com os algarismos 1, 3 e 4:
com os algarismos 2, 2 e 3:

62, 34 e 43;
126, 162, 216, 261, 612 e 621;
134, 143, 314, 341, 413 e 431;
223, 232 e 322.

(b) Se P (n) = 0, ento o produto de seus algarismos igual a zero e, portanto, pelo
menos um dos algarismos do nmero n zero. De 1 a 199 temos 18 nmeros com
zero s nas unidades, 9 nmeros com zero s nas dezenas e ainda o nmero 100,
totalizando 28 nmeros:
10, 20, . . . , 90, 110, . . . , 190, 101, 102, . . . , 109 .
0 s nas unidades

0 s nas dezenas

(c) Queremos encontrar os inteiros positivos menores do que 200, cujo produto dos
algarismos seja maior do que 37 e menor do que 45. Por exemplo, 58 um desses
nmeros, porque 58 = 40. Em primeiro lugar, note que no existem nmeros cujo
produto dos algarismos seja 38, 39, 41, 43 e 44, porque esses nmeros possuem um
fator primo maior do que 10 e, portanto, no podem ser escritos como produto de
dois ou trs algarismos. Logo, restam apenas 40 e 42. Assim, os nmeros menores
do que 200 cujo produto dos algarismos
40 so 58, 85, 158 e 185;
42 so 67, 76, 167 e 176.

(d) O valor de P (n) o maior possvel quando n = 99 ou n = 199, quando


P (99) = P (199) = 81.
136

OBMEP 2010

Solues do Nvel 1
91. Suco de laranja Se Davi comprar seis garrafas individualmente, ele gastar
6 2,80 = 16,80 reais, que um valor maior do que o preo de uma caixa com
seis. Portanto, ele deve comprar a maior quantidade possvel de caixas. Nesse caso,
como 22 = 3 6 + 4, ele deve comprar trs caixas e quatro garrafas individualmente,
caso em que gastar 3 15 + 4 2,80 = 56,20 reais, que o mnimo possvel.
92. A casa da Rosa Como o quarto quadrado, com uma rea de 16 m2 , suas
dimenses so 4 4 m. Da mesma forma, as dimenses do quintal quadrado so
2 2 m. A sala tem uma rea de 24 m2 e uma dimen- ............................................................................................................................................................................................
.
.
.
.
.
.
.
.
.
.
.
.
.
.
.
4
6
.
.
.
.
.
so igual do quarto; portanto, as dimenses da sala so ...........
.
.
.
.
.
.
.
.
.
.
.
.
.
.
.
.
.
.
.
.
.
.
.
.
.
.
.
.
.
.
.
.
.
.
.
.
6 4 m. Assim, as dimenses totais da casa so 10 6 ....... Quarto .......4 Sala
.
.
.
.
.
.
.
.
.
.
.
.
.
.
.
.
.
.
2
.
.
.
.
.
.
.
.
m e a rea total da casa de 60 m . Logo, a cozinha tem ..........
.
.
.
.
.
.
.
.
.
.
.
..........................................................................................
.
.
.
.........................................................................................
.
.
.
.
.
.
.
.
.
.
.
.
.
.
.
.
.
. Quintal
.
uma rea de
.
.
.
.
.
.
.
.
.
.
.
.
.
.
.
.
2

Cozinha

.
.
.
.
.
.
.
.
.
.
.
.
.
.
.
.
.
.
.
.
.
.
.
.
.
.
.
.........................................................................................
.
.
..........................................................................................
.

60 16 24 4 = 16 m .

93. O passeio do Matias Observe que h 12 ruas, ou seja, lados de 100 metros, entre
os quatro quarteires. Tambm h quatro esquinas, marcadas com na gura, em
que se encontram trs ruas. Sempre que Matias passar por uma dessas quatro esquinas,
...... ......
..... .....
usar duas dessas trs ruas. Assim, pela regra que ele mesmo ..............................................................................................-.....................................................................................................................
.
......
.....
.
.
.
. 6
.
. 6
.
.
.
.
.
.
.
.
.
.
.
.
.
.
.
.
.
.
.
.
.
.
.
.
.
.
.
.
.
.
.
.
.
.
.
.
.
.
.
se imps, quando voltar a passar numa dessas quatro esquinas, .......... ...........
.
.
.
.
.
.
.
.
.
.
.
.
.
.
.
.
.
.
.
.
.
.
.
.
.
.
.
.
.
.
.
.
.
.
.
.
.
.
.
.
.
.
.
.
.
.
.
.
termina o passeio. Portanto, em todo caminho que percorrer, ......... ............................................................. ............................................................. ...........
.
.
.
.
.
.
.
.
.
.
.
.
h, pelo menos, duas dentre essas quatro esquinas em que ........... ....................................................... .......................................................
.
.
.
.
.
.
.
.
.
.
.
.
.
.
.
.
.
.
.
.
.
.
.
.
.
.
.
.
.
.
.
.
.
.
.
.
.
.
.
.
.
.
.
.
.
.
.
.
.
.
.
.
.
.
no usou todas as ruas que chegam a essas esquinas. Assim, o ...... .....
.
.
.
.
.
.
.
.
.
.
.
.
.
.
.
.
.
.
.
.
.
.
.
.
.
.
.
.
.
.
.
.
.
.
.
.
.
.
.
.
.
.
.
.
.
.
.
.
.
.
.
.
.
.
.
.
.
.
.
....
.
.
caminho de comprimento mximo usa no mximo 10 ruas, ou ........ ......................................... ?.......................................- ...........
...
.
.
.
..
..
.
.
.
.
.
.
r
.......
......
seja, tem um total de 1 000 m. Na gura desenhamos um dos ............................................................................................................................
P
trajetos mximos.
94. O adesivo ocial Como o quadrado pintado da cor azul pode estar em qualquer
lugar, temos seis possveis formas de escolher a posio desse quadrado. Entre os cinco
quadrados restantes, precisamos pintar dois de amarelo, o que podemos fazer de 10
maneiras. Os trs quadrados restantes so pintados de verde. Portanto, o prefeito tem
6 10 = 60 formas diferentes de escolher o adesivo.
95. Adio de nmeros Efetuando a adio
1 1 1

a 000
a 998
+ a 999
................................
................................
997
encontramos
997 = 22 997, onde
portanto, a = 7.

= a + a + a + 1. Logo, 22 = a + a + a + 1 e,

96. Cubo perfeito e divisibilidade Um cubo perfeito um nmero da forma a3 , onde


a um natural. Como 94 = (32 )4 = 38 , os cubos perfeitos que dividem 38 so 1, 33 e
(32 )3 = 36 .
OBMEP 2010

137

Solues do Nvel 1
97. Localizao de um ponto O ponto indicado est quatro marcas direita de 19.
Entre 19 e 20 aparecem subdivises em 10 partes iguais, portanto, cada marca equivale
a 0,1 nessa escala. Assim, o ponto indicado 19,4.

18

19

20

98. Clculo de porcentagem Temos 58 acertos em 84 questes, portanto, a razo de


58
acertos
. Dividindo 58 por 84, encontramos, aproximadamente, 0,69047 em 1, ou
84
69,047 em 100. Logo, o percentual , aproximadamente, 69,047%.
99. Comparao de algarismos Os nmeros que estamos procurando so maiores do
que 400 e menores do que 600, portanto, o algarismo das centenas s pode ser 4 ou 5.
Como so nmeros ascendentes, o algarismo das dezenas menor do que o algarismo
das unidades. Vejamos como escolher os algarismos das dezenas e das centenas.

56

57
4
58

59

67
68
4

69

67
68
5

69

78
79

78
79

89

89

Logo, temos 10 nmeros ascendentes com algarismo das centenas igual a 4 e seis nmeros ascendentes com algarismo das centenas igual a 5. Assim, temos 16 nmeros
ascendentes entre 400 e 600.
100. Muro colorido Observamos que no momento em que escolhermos a cor de dois
tijolos vizinhos, a cor de todos os demais tijolos estar decidida.
............................................................................
.
.
.
.
............................................................................
.
.
.
.
.
.
.
.
.
.
.
.
.
.
.
.
.
.
.
.
.
.
.
.
.
.
.
.
.
.
.
.
.
.
.
.
.
........................................................................................
........................................................................................
..............
.
.
.
.
.
.
............
.
.
.
.
.
.
.
.
.
.
.
.
.
.
.
.
.
.
.
.
.
.
.
.
.
.
.
.
.
.
.
.
.
.
.
.
.
.
.
.
.
.
...................................................
............................................................................
..........................
.
.
.
...................................................
.
.
.
.
.
.
.
.
.
.
.
.
.
.
.
.
.
.
.
.
.
.
.
.
.
.
.
.
.
.
.
.
.
.
.
.
.
.
.
.
.
.
................................................................
.........................................................................................
.......................................
.
.
.
.
.
.
.
.
.............
.
.
.
.
.
.
.
.
.
.
.
.
.
.
.
.
.
.
.
.
.
.
.
.
.
.
.
.
.
.
.
.
.....................................................................................................
.
.
.
.
.
.
.
.
.
.....................................................................................................
.
.

Assim, denotando os tijolos de acordo com uma de suas trs cores A, B ou C, e seguindo
a exigncia de no ter tijolos de mesma cor se tocando, obtemos uma distribuio como
a da gura. Como a maior quantidade de tijolos est marcada com A, num total de
seis, e os tijolos amarelos so os mais baratos, devemos escolher tais tijolos amarelos.
Por outro lado, temos a mesma quantidade de tijolos B e C, quatro de cada tipo,
portanto, podemos escolher quatro tijolos azuis e quatro vermelhos. Assim, o menor
valor a ser pago na compra dos tijolos desse muro 6 6 + 4 7 + 4 8 = 96 reais.
138

OBMEP 2010

Solues do Nvel 1
101. Divisores e fatorao Como o produto dos dois fatores 96, eles so divisores
de 96. Decompondo 96 em fatores primos, encontramos 96 = 25 3, portanto, seus
divisores so 1, 2, 3, 4, 6, 8, 12, 16, 24, 32, 48 e 96.
Os divisores 96, 48, 32, 24 e 16 no servem, pois seus quadrados j so maiores do que
208, sobrando 1, 2, 3, 4, 6, 8 e 12, cujos quadrados so 1, 4, 9, 16, 36, 64 e 144.
Agora fcil ver que a nica possibilidade 64 + 144 = 208. Como 8 12 = 96, os
nmeros so 8 e 12.
102. O retngulo do Lus Faremos a diviso com retngulos. Observamos que 24 = 64
e 12 = 6 2, portanto, Lus pode fazer um primeiro corte a 4 cm no lado de 10 cm
e outro corte a 2 cm do corte anterior. Depois desses cortes, resta um retngulo de
tamanho 6 4. Por ltimo, como 16 = 4 4, basta fazer mais um corte a 4 cm no
lado que mede 6 cm. Os cortes esto ilustrados na gura seguinte, com indicao das
dimenses dos lados e das reas.
4

24

12

16

103. Comparao de nmeros Fatorando os nmeros e extraindo as razes, obtemos

121
= 112
= 11,

3
3
3
= 9
= 9e
729
4
4
4 74 = 2 7 = 14 .
38 416 =
2
Logo, em ordem crescente, temos

729,

121 e

38 416.

104. As moedas Atribuindo o valor 1 s coroas e 1 s caras e somando os resultados


depois de cada jogada, inicialmente a brincadeira comea com soma 7 e queremos
chegar a cara e coroa alternadas, de modo que a brincadeira termina em 1 ou em
1. Observamos que, em cada passo da brincadeira, temos as seguintes possibilidades:
trocamos duas coroas por duas caras e o valor da soma diminui em 4; trocamos uma
cara e uma coroa por uma coroa e uma cara e o valor da soma ca inalterado; ou
trocamos duas caras por duas coroas e o valor da soma aumenta em 4. Portanto,
impossvel partir de 7 como soma inicial e chegar a 1, mas vejamos que, efetivamente,
possvel chegar a 1, isto , a quatro caras e trs coroas. Como queremos obter quatro
caras no consecutivas, precisamos de, pelo menos, quatro jogadas.

OBMEP 2010

139

Solues do Nvel 1
As quatro jogadas, que fazem a soma passar de 7 para 3, de 3 para 1 e ento permanecer em 1, esto ilustradas na gura.
.....
..............
.... ......
..
.
..
.
.
.
.
. coroa .
.
.
.
.
.
..
.
..
.
...
..
.... .....
..........
.....

.....
..............
.... ......
..
.
..
.
.
.
.
. coroa .
.
.
.
.
.
..
.
..
.
...
..
.... .....
..........
.....

.....
..............
.... ......
..
.
..
.
.
.
.
. coroa .
.
.
.
.
.
..
.
..
.
...
..
.... .....
..........
.....

.....
..............
.... ......
..
.
..
.
.
.
.
. coroa .
.
.
.
.
.
..
.
..
.
...
..
.... .....
..........
.....

.....
..............
.... ......
..
.
..
.
.
.
.
. coroa .
.
.
.
.
.
..
.
..
.
...
..
.... .....
..........
.....

.....
..............
.... ......
..
.
..
.
.
.
.
. coroa .
.
.
.
.
.
..
.
..
.
...
..
.... .....
..........
.....

.....
..............
.... ......
..
.
..
.
.
.
.
. coroa .
.
.
.
.
.
..
.
..
.
...
..
.... .....
..........
.....

..
.............
..... .......
..
..
..
.
.
.
.
.
. cara .
.
.
.
.
.
..
..
..
..
...
.... ....
...........
.....

..
.............
..... .......
..
..
..
.
.
.
.
.
. coroa .
.
.
.
.
.
..
..
..
..
...
.... ....
...........
.....

..
.............
..... .......
..
..
..
.
.
.
.
.
. coroa .
.
.
.
.
.
..
..
..
..
...
.... ....
...........
.....

..
.............
..... .......
..
..
..
.
.
.
.
.
. coroa .
.
.
.
.
.
..
..
..
..
...
.... ....
...........
.....

..
.............
..... .......
..
..
..
.
.
.
.
.
. coroa .
.
.
.
.
.
..
..
..
..
...
.... ....
...........
.....

..
.............
..... .......
..
..
..
.
.
.
.
.
. coroa .
.
.
.
.
.
..
..
..
..
...
.... ....
...........
.....

.
.............
...... .......
..
..
.
..
.
.
.
.
.
.
. cara .
.
.
.
..
..
.
..
...
.... .....
...
........
.....

.
.............
...... .......
..
..
.
..
.
.
.
.
.
.
. cara .
.
.
.
..
..
.
..
...
.... .....
...
........
.....

.
.............
...... .......
..
..
.
..
.
.
.
.
.
.
. coroa .
.
.
.
..
..
.
..
...
.... .....
...
........
.....

..
............
..... .......
..
..
.
..
.
.
.
.
.
.
. coroa .
.
.
.
..
..
.
..
...
.... .....
...
........
.....

..
............
..... .......
..
..
.
..
.
.
.
.
.
.
. coroa .
.
.
.
..
..
.
..
...
.... .....
...
........
.....

..
............
..... .......
..
..
.
..
.
.
.
.
.
.
. cara .
.
.
.
..
..
.
..
...
.... .....
...
........
.....

.....
.............
.... ......
..
.
..
.
.
.
.
.
.
. cara .
.
.
.
..
..
.
..
...
.... .....
..
.........
.....

.....
.............
.... ......
..
.
..
.
.
.
.
.
.
. coroa .
.
.
.
..
..
.
..
...
.... .....
..
.........
.....

.....
.............
.... ......
..
.
..
.
.
.
.
.
.
. cara .
.
.
.
..
..
.
..
...
.... .....
..
.........
.....

.....
.............
.... ......
..
.
..
.
.
.
.
.
.
. coroa .
.
.
.
..
..
.
..
...
.... .....
..
.........
.....

.....
.............
.... ......
..
.
..
.
.
.
.
.
.
. coroa .
.
.
.
..
..
.
..
...
.... .....
..
.........
.....

.....
.............
.... ......
..
.
..
.
.
.
.
.
.
. cara .
.
.
.
..
..
.
..
...
.... .....
..
.........
.....

.....
.............
.... ......
..
.
..
.
.
.
.
.
.
. cara .
.
.
.
..
..
.
..
...
.... .....
..
.........
.....

.....
..............
.... ......
..
.
..
.
.
.
.
. cara .
.
.
.
.
.
.
.
..
..
..
..
.... ...
...... ......
......
.

.....
..............
.... ......
..
.
..
.
.
.
.
.coroa .
.
.
.
.
.
.
.
..
..
..
..
.... ...
...... ......
......
.

.....
..............
.... ......
..
.
..
.
.
.
.
. cara .
.
.
.
.
.
.
.
..
..
..
..
.... ...
...... ......
......
.

.....
..............
.... ......
..
.
..
.
.
.
.
.coroa .
.
.
.
.
.
.
.
..
..
..
..
.... ...
...... ......
......
.

.....
..............
.... ......
..
.
..
.
.
.
.
. cara .
.
.
.
.
.
.
.
..
..
..
..
.... ...
...... ......
......
.

.....
..............
.... ......
..
.
..
.
.
.
.
.coroa .
.
.
.
.
.
.
.
..
..
..
..
.... ...
...... ......
......
.

.....
..............
.... ......
..
.
..
.
.
.
.
. cara .
.
.
.
.
.
.
.
..
..
..
..
.... ...
...... ......
......
.

..
.............
..... .......
..
..
..
.
.
.
.
.
. cara .
.
.
.
.
.
..
..
..
..
...
.... ....
...........
.....

..
............
..... .......
..
..
.
..
.
.
.
.
.
.
. cara .
.
.
.
..
..
.
..
...
.... .....
...
........
.....

105. O preo do frango A opo correta (b).


Como 81 = 34 , o valor do frango triplicou quatro vezes. O nmero de meses transcorridos foi 4 6 = 24 meses, isto , dois anos, ou seja, em janeiro de 2002 o frango atingir
o preo de R$ 81,00.
106. Excurses a Foz do Iguau Temos um nibus com 27 19 = 8 lugares livres e
ainda precisamos acomodar os 53 8 = 45 participantes em nibus de 27 lugares.
claro que um nibus s no suciente, portanto, precisamos de dois nibus e teremos
2 27 45 = 9 lugares livres no ltimo nibus. Ficaram 27 9 = 18 pessoas no nibus
incompleto.
107. As fraes de Laura Como a frao igual a um nmero inteiro, o seu numerador
deve ser um mltiplo do seu denominador. Vamos testar todas essas possibilidades e
escolher as que satisfazem as condies do problema.
3 + 6 + 11
3+5+6
= 7,
= 10 e
2
2

5 + 6 + 11
= 11 no satisfazem;
2

2 + 5 + 11
= 6 satisfaz;
3

3 + 6 + 11
= 4 no satisfaz;
5

2 + 5 + 11
= 3 satisfaz e
6

2+3+6
= 1 no satisfaz.
11

Assim, temos somente as duas respostas seguintes.


m
m
m
2 + 5 + 11
m
= 6
m
3

.......................................................
.......................................................

108. Clculo da unidade A opo correta (e).


140

OBMEP 2010

m
m
m
2 + 5 + 11
m
= 3
m
6

.......................................................
.......................................................

Solues do Nvel 1
Como o algarismo da unidade de qualquer potncia de 5 5, segue que o algarismo da
unidade de cada fator do produto 5 + 1 = 6. Mas, 6 6 = 36, ou seja, o produto
de dois nmeros terminados em 6 tambm um nmero terminado em 6. Logo, o
algarismo da unidade desse produto 6.
109. Nmeros cruzados
7 5
8
5 7
6 3
4
7 5

2
8 5
1
2
8 7
9 2

8 8
0
7 5
4
6 4
5

110. Ovos e mas A opo correta (b).


Como o enunciado e a resposta so percentuais podemos, nesse caso, estipular qualquer
preo e qualquer unidade monetria, que a resposta ser, sempre, a mesma. O mais
simples, portanto, supor que, inicialmente, uma dzia de ovos custava 100 e, portanto,
que dez mas tambm custavam 100. Como o preo dos ovos subiu 10%, o novo valor
dos ovos 110. O preo das mas diminuiu 2%, portanto, o novo preo de dez mas
98. Assim, enquanto antes gastava-se 200 na compra de uma dzia de ovos e dez
mas, agora gasta-se 110 + 98 = 208. Da, temos que o aumento foi de 8 em 200, o
que corresponde ao percentual de
8
4
=
= 4%.
200
100
111. Diviso de nmeros decimais A opo correta (a).
Efetuando a diviso, temos
254,88
254 880
144 177 10
=
=
= 1 440 .
0,177
177
177
112. Almoo dos amigos Os preos de um prato mais uma vitamina so
13 , 14 , 16 , 17 , 18 , 20 , 20 , 21 , 23 .
7+6

7+7

7+9

11+6

11+7

11+9

14+6

14+7

14+9

Dentre esses, os que diferem por 6 so 14 e 20, ou 17 e 23. Logo, temos duas solues:
ou Denise gasta 7 + 7 = 14 e Jlio 14 + 6 = 11 + 9 = 20, ou Denise gasta 11 + 6 = 17
e Jlio 14 + 9 = 23.
113. Somas de trs em trs Inicialmente, observe que se a maior soma de trs desses
nmeros for 9, ento todos os nmeros devem ser menores do que 7, ou seja, 1, 2, 3,
4, 5 ou 6. Por outro lado, se a menor soma de trs desses nmeros distintos for 6,
ento eles no podem incluir 5 ou 6, restando 1, 2, 3 e 4. Vericamos que esses so os
nmeros, pois
1 + 2 + 3 = 6, 1 + 2 + 4 = 7, 1 + 3 + 4 = 8
OBMEP 2010

e 2 + 3 + 4 = 9.
141

Solues do Nvel 1
114. O passeio do Jorge Lembrando que a distncia entre as rvores ao longo do
caminho de 5 m, ilustramos o sentido do percurso de Jorge nas guras.
P...-...........s.....-...........s.....-..........s.....-...........s
s
.... ...... .. ...... .. ...... .. ...... .
... .....
. ..... . ...... . ...... .

P.........................s...........................s...............s................s
s
....
...
..
.
.. ...... .. ...... .
.
...... . ...... .

P.........................s...........................s..........-.....s.........-.......s
s
....
...
..
.
.. ...... .. ...... .
. ...... . ...... .

.
.
.
.
.
.
.
.
.
.
.
.
.
.
.
.
.
.
.
.
.
.
.
.
.
.
.
.
.
.
.
.
.
.
.
.
.
.
.
.
.
.
.
.
.
.
.
.
.
.
.
.
.
.
.
.
.
.
.
.
.
.
.
.
.
.
.
.
.
.
..............................................................
...............................................................
.
.

.
.
.
.
.
.
.
.
.
.
.
.
.
.
.
.
.
.
.
.
.
.
.
.
.
.
.
.
.
.
.
.
.
.
.
.
.
.
.
.
.
.
.
.
.
.
.
.
.
.
.
.
.
.
.
.
.
.
.
.
.
.
.
.
.
.
.
.
.
.
..............................................................
...............................................................
.
.

s?

s s?


6
s
6
s
s
-

.
.
.
.
.
.
.
.
.
.
.
.
.
.
.
.
.
.
.
.
.
.
.
.
.
.
.
.
.
.
.
.
.
.
.
.
.
.
.
.
.
.
.
.
.
.
.
.
.
.
.
.
.
.
.
.
.
.
.
.
.
.
.
.
.
.
.
.
.
.
..............................................................
...............................................................
.
.

s?

s s
s?
 

(a) Caminhando inicialmente 32 m, ele toca em sete rvores, parando 2 m depois da


ltima rvore que tocou.
(b) Voltando 18 m, ele toca em quatro rvores, parando 1 m depois da ltima que
tocou.
(c) Ao retornar 22 m, ele toca em cinco rvores, parando 1 m depois da ltima rvore
que tocou.
Assim, Jorge tocou em 7 + 4 + 5 = 16 rvores.
115. A descoberta do algarismo
algarismos, temos,
com 1
com 2
com 3

Separando os nmeros cujos quadrados tm 1, 2 e 3


algarismo: 1, 2, 3
algarismos: 4, 5, 6, 7, 8, 9
algarismos: 10, 11, 12,. . . , 31.

At 312 , o nmero j tem 3 + 12 + 66 = 81 algarismos. Abreviando algarismo por


algs, temos
12 , 22 , 32 , 42 , . . . , 92 , 102 , . . . , 312 .
13 algs

26=12 algs

322=66 algs

Assim, faltam 100 81 = 19 algarismos para o 100o . Como s 1002 tem 5 algarismos,
e como 19 = 4 4 + 3, teremos mais 4 nmeros de 4 algarismos cada um, que so 322 ,
332 , 342 e 352 , e mais os 3 algarismos (milhar, centena, dezena) do nmero 362 = 1 296,
como segue.
12 , 22 , 32 , 42 , . . . , 92 , 102 , . . . , 312 , 322 , 332 , 342 , 352 , 12 9 6
13 algs

26=12 algs

322=66 algs

44=16 algs

100o alg

Assim, vemos que o algarismo 9 ocupa a 100a posio.


116. OBMEP Como peso de B+ peso de E = 6 e peso de M + peso de P = 6, segue
que os pesos de M, P, B e E so todos menores do que 6. Como no h dois discos
de mesmo peso, M, P, B e E no podem pesar 3 e, portanto, os pesos desses quatro
discos s podem ser 1, 2, 4 e 5. Agora, peso de X+ peso de O = 13 e peso de
Z+ peso de O = 9, portanto, peso de X = peso de Z + 4. Assim, a nica opo para
os pesos de Z e de X 3 e 7. Por excluso, o peso de O 6. Assim, obtemos
peso de O + peso de B + peso de M + peso de E + peso de P = 6 + 6 + 6 = 18.
117. Prdio misterioso Primeiro observamos que os elevadores denotados por A, C, D,
E, F e H conduzem a recintos fechados em algum dos dois andares e, portanto, no
levam sada. Desconsiderando esses elevadores, nosso desenho de elevadores teis
o seguinte.
142

OBMEP 2010

Solues do Nvel 1
qqqqqqqqqqqqqqqqqqqqqqqqqqqqqqqqqqqqqqqqqqqqqqqqqqqqqqqqqqqqqqqqqqqq
qq
q B
qq
q
qqqqqqqqqqqqqqqqqqqqqqqqqqqqqqqqqqqqq
qqqqqqqqqqqqqqqqq
qqq
qq
qq
qq
qq
qq
qq
qq
q
qqqqqqqqqqqqqqqqqqqqqqqqqqqqqqqqqqqqqqqqqqqqqq
qq
qq
qq
qq
qq
qq
q
qq G q
qq
qq
qq
qqqqqqqqqqqqqqqqqq
qq
q
qq
qq
qqq
qq J
q
sada
qqqqqqqqqqqqqqqqqqqqqqqqqqqqqqqqqqqqqqqqqqqqqqqqqqqqqqqqqqqqqqqq

qqqqqqqqqqqqqqqqqqqqqqqqqqqqqqqqqqqqqqqqqqqqqqqqqqqqqqqqqqqqqqq
q
qq
qq
B qqqq
qqqqqqqqqqqqqqqqqq
qqqqqqqqqqqqqqqqqqqqqqqqqqqqqqqqqqqqqqqqqqqqqqqqqqq
qq
qq
qq
qq
qq
qq
qq
qq
qq
qq
qq
qq
qq
qq
qq
qq
qq
qq
qq
qq
qq
qqqqqqqqqqqqqqqqqqqqqqqqqqqqqqq
qq
qq
qq
qq
qq
qq
qq
qq G qq
qq
qqqqqqqqqqqqqqqqqqqqqqqqqqqqqqqqqqqqqqq
qqqqqqqqqqqqqqqqqq
qq
qq
qq
qq
qq
qq J
qq
q
qqqqqqqqqqqqqqqqqqqqqqqqqqqqqqqqqqqqqqqqqqqqqqqqqqqqqqqqqqqqqqqqq

entrada

Assim, o caminho mais curto entre a entrada de um andar at a sada do outro consiste
em primeiro pegar o elevador B, depois o J e, por ltimo, o G.
118. Soma de fraes
Soluo 1: Transformando as fraes em nmeros decimais, obtemos
1
1
1
1
909

= 0,1 0,01 + 0,001 0,00001 = 0,0909 =


.
10 100 1 000 10 000
10 000
Soluo 2: Efetuando a soma das fraes, obtemos
1
1
1
1 000 100 + 10 1
909
1

=
=
.
10 100 1000 10 000
10 000
10 000
27
119. Biblioteca Ao comprar 140 livros, a biblioteca cou com
do nmero de livros,
25
2
2
portanto, 140 corresponde a
dos livros da biblioteca. Se
corresponde a 140
25
25
1
25
livros,
corresponde a 140 2 = 70 livros e
a 70 25 = 1 750 livros. A opo
25
25
correta (a).
120. Comparao de fraes Para que uma frao seja menor do que 1, o numerador
deve ser menor do que o denominador. Eliminando as repeties, obtemos a lista
seguinte.
1
2
2
1
e
(b) 2 fraes com denominador 3:
3
3
1
2
(c) 2 fraes com denominador 4: ,
4
4
(a) 1 frao com denominador 2:

3
4

1/2

4
1 2 3
, ,
e
5 5 5
5
2
1
3
4
,
,
(e) 2 fraes com denominador 6: ,
6
6
6
6

(d) 4 fraes com denominador 5:

1/3

(f) 6 fraes com denominador 7:

1/2

5
6

2/3

1 2 3 4 5
6
, , , ,
e
7 7 7 7 7
7

OBMEP 2010

143

Solues do Nvel 1
(g) 4 fraes com denominador 8:

1
,
8

2
,
8

3
,
8

1/4

(h) 6 fraes com denominador 9:

1 2
, ,
9 9

4
,
8
1/2

3
,
9

4 5
, ,
9 9

1/3

6
8

5
,
8

7
8

3/4

6
,
9

8
7
e
.
9
9

2/3

Assim, temos 27 dessas fraes.


121. Diviso com resto Se a diviso de 2 007 por algum nmero deixar resto 5, ento
esse nmero divide 2 007 5 = 2 002. Assim, calculamos todos os divisores de 2 002 =
2 7 11 13, listados na coluna da direita da tabela seguinte.
.
.
.
.
. 1
.
.
.
.
.
.
.
.
.
.
.
.
.
.
.
.
.
.
.
.
2 002 ........ 2 ........... 2
.
.
.
.
.
.
.
.
1 001 ............ 7 ........... 7, 14
.
.
.
.
.
.
.
143 ............ 11 ............ 11, 22, 77, 154
.
.
.
.
.
.
.
13 ...........13 ........... 13, 26, 91, 182, 143, 286, 1 001, 2 002
.
.
.
.
.
.
.
.
1 ......... .........
Como o resto 5 deve ser menor do que o divisor, dividindo 2 007 por qualquer um dos
14 nmeros seguintes deixa resto 5:
7, 11, 13, 14, 22, 26, 77, 91, 143, 154, 182, 286, 1 001 e 2 002.
122. Panelas Convertendo 1 kg em 1 000 g, temos que as duas panelas juntas, mais a
carne, pesam 645 + 237 + 1 000 = 1 882 g. Logo, cada panela, mais o seu contedo de
carne, deve pesar 1 882 2 = 941 g. Assim, Jos colocou:
941 645 = 296 g

941 237 = 704 g

nessas duas panelas.


123. Domins Como 2 3 = 6, podemos comear supondo que os dois domins ............p..............p.....p.......
......p.......
. . p.
........p..p...
e ........p....p.............p....p....p..... estejam na posio certa. Se isso for verdade, e como 1 3 = 3, resulta que
o algarismo na dezena do resultado deve ser 3, portanto precisamos trocar o domin
..p....p.......
. . p.
......p.......
. .
........p.....
.............
. p .p .
. . p.
.
. . .
.............
pelo domin ........p........p.........p....p....p..... , de tal forma que o 3 que na dezena. Como temos um 2 na
..p...........
. . .
centena do resultado, a centena do primeiro nmero precisa ser um 4. Com essa troca,
a posio dos domins ca correta, como pode ser visto na gura.
.............
. . .
.............

...........................................
.
.
.
.
.
. s .
.
.
.
.
.
. s s.
.
.
.
.
.
.
...........................................
.
.
.....s................
..
.
.
. s .
.
.
.
.
...................s..
.
..
........................................... . s s.
.
.
. s .
.
. s s. . s s.
.
. .
.
.
.
. .
.
.
. .
.
.
.s
. . s s.
.
s.
.
. .
........................................... ......................
.
.
.
.

.......................................s...
.
. s s. s . .
.
.
. s .
.
.
.
.
.
.
.
. s s. s s.
.
.
...........................................
.
.
...........................................
.
.
.
.
.
. s . s s s.
.
.
.
.
.
.
.
.
. s s s.
.
.
...........................................
.
.
144

OBMEP 2010

5412
3
16236

...........................
...........................

Solues do Nvel 1
124. Cdigo secreto A nica maneira de obter 360 = 23 32 5 como produto de trs
nmeros de um algarismo cada um 360 = 9 8 5 . Como A o menor dos trs,
A = 5. Logo B = 8 e C = 9, ou B = 9 e C = 8, ambas opes com AA + BB + CC =
55 + 88 + 99 = 242. Logo, temos duas possibilidades para o cdigo ABC, a saber, 589
ou 598.
125. Os doze pontos No total, temos 11 possveis quadrados, mostrados nas guras
seguintes.
5 quadrados
s
s
s
s

s
s
s
s

4 quadrados

s
s

s
s
s
s

s
s
s
s

.
.
..
...
....
....
.. .. ... ...
.. .... .... ....
.. ...... ....
.
..
..
..
..
.
..
...
..
....
..
..
..
..
.. ..
.. ...
.. .. .
.
..
..
..
..
..
.. ..... ..... ...
.
...
.. ...... ....
. .. .
.. .. .. ..
.. ... .. .. . .. .
.. .
.. .. .... .. ...
...
..
..
.. . .. .. ...
..... ...... .......
...... ....... ........
.
.
.... ...
.. .
.
.. ...
..
.. .... ....
.
.. ....
..
..
.
.. ..
..
.
.. ...
..
..
..
.. .. .
..
..
.. ...
...
..
..
..
..
..
.
..
..
.. ....... ....
.. ........ ....
.. ... ... . .
....
...
...
...
.
.

.
...........
.
...........
.
.
.
.
.
.
.
.
.
.
.
.
.
.
.
.
.
.
.
.
.
.
.
.
................................
.
.
.
.
................................
.
.
.
.
.
.
.
.
.
.
.
.
.
.
.
.
.
.
.
.
.
.
.
.
.
.
.
.
.
.
.
.
.
.
.
.
.
.
................................
.
.
.
................................
.
.
.
.
.
.
.
.
.
.
.
.
.
.
.
.
.
.
.
.
.
.
.
.
.
.
.
.
.
...........
.
.
.
...........

s
s

s
s

s
s

2 quadrados
s
s
s
s

s
s
s
s

..
..
...............
.. ......... ..
.. ..
........ .........
....... .........
.....
. ...
.....
.....
........
.......
. .
..... .
.
.......
.. ..
.. .
....
...
..
..
..
.
.. .
...
.
. ..
. .
.. ...
... ..
.... .
. .. .
. . ..
.....
.....
..
......
.....
.
....... .........
........ ........
.
.. . .
.. ......... ..
.
............. .
.. .. ..

s
s

s
s

126. Relgio Vamos tentar uma data e um horrio no mesmo ano de 1994. J que com
os nmeros dados no podemos alterar o dia nem para 29 nem para 30, sem alterar
o ano, ento a data procurada no est no ms 05. O seguinte ms possvel o 08.
Como precisamos da data mais prxima possvel, observemos que podemos formar o
dia 01, sobrando os algarismos 0, 2, 4 e 5 para formar a hora. A menor hora possvel
que podemos formar com esses algarismos 02h45m, de modo que a data procurada
1o de agosto de 1994, s 02 horas e 45 minutos.
......
...................
.......... ..............
........
.....
.....
....
.....
....
....
...
....
...
...
...
...
..
...
..
..
..
.
..
..
..
..
.
..
..
..
..
..
.
..
..
.
.
.
.
.
.
.
.
.
.
.
.
.
.
.
.
.
.
.
.
.
.
.
.
.
.
.
.
.
.
.
.
.
.
.
.
.
.
.
.
.
.
.
.
.
..............
.
...............
.
..............
.
.
.
................
.
.
.
.
.
.
.
.
.
.
.
.
.
.
.
.
.
.
.
.
.
.
.
.
.
.
.
.
.
.
.
.
.
.
.
.
.
.
.
.
.
..
.
.
.
.
..
.
.
.
.
.
.
.
.
.
.
.
.
..
.. .
.
.
.
.
.
.
.
.. ..............
.
............... ...
.
................ ...
.
.
.. ..............
.. .
.
..
..
..
...
..
...
..
..
...
..
....
...
....
....
.....
....
......
.....
.......
............................
......................

......
...................
.......... ..............
.....
........
.....
....
.....
....
...
....
...
....
..
...
..
..
..
..
..
.
..
..
..
..
..
.
..
..
..
..
..
.
..
..
.
.
.
.
.
.
.
.
.
.
.
.
.
.
.
.
.
.
.
.
.
.
.
.
.
.
.
.
.
.
.
.
.
.
.
.
.
.
.
.
.
.
.
.
.
..............
.
...............
.
..............
.
.
.
................
.
.
.
.
.
.
.
.
.
.
.
.
.
.
.
.
.
.
.
.
.
.
.
.
.
.
.
.
.
.
.
.
.
.
.
.
.
.
.
.
.
..
.
.
.
.
..
.
.
.
.
.
.
.
.
.
.
.
.
..
.. .
.
.
.
.
.
.
.
.. ..............
.
............... ...
.
................ ...
.
.
.. ..............
.. .
.
..
..
..
.
...
..
...
...
....
...
....
....
.....
....
.....
.....
.......
.....
.......
.......................
......................

01 08 94

28 05 94

14 h 00

02 h 45

127. Lpis A opo correta (b).


Vamos ver em quantas caixas podemos colocar o nmero mximo de lpis, que 6 por
caixa. Nas 13 caixas no possvel, pois 13 6 = 78 maior do que o nmero 74
do total de lpis. Em 12 caixas podemos ter 12 6 = 72, sobrando uma caixa, com
74 72 = 2 lpis.
128. Contagem A cada 10 pginas, o algarismo 1 aparece uma vez nas unidades e, a
cada 100 pginas, aparece 10 vezes nas dezenas. Contando o nmero de pginas que
contm o algarismo 1 em cada faixa abaixo, temos
(a) 20 vezes entre 1 e 99:
1, 11, 21, 31, 41, 51, 61, 71, 81, 91, num total de 10 vezes na unidade;
10, 11, 12, 13, 14, 15, 16, 17, 18, 19, num total de 10 vezes na dezena.
OBMEP 2010

145

Solues do Nvel 1
(b) 120 vezes entre 100 e 199:
101, 111, 121, 131, 141, 151, 161, 171, 181, 191: 10 vezes na unidade;
110, 111, 112, 113, 114, 115, 116, 117, 118, 119: 10 vezes na dezena;
100, 101, 102, . . . , 199, num total de 100 vezes na centena.
(c) 20 vezes entre 200 e 299:
201, 211, 221, 231, 241, 251, 261, 271, 281, 291: 10 vezes na unidade;
210, 211, 212, 213, 214, 215, 216, 217, 218, 219: 10 vezes na dezena.
At a pgina 299, o nmero 1 aparece 20 + 120 + 20 vezes, faltando, portanto, apenas
171 160 = 11 vezes. Os dois primeiros que aparecem depois de 299 so dois na
unidade, em 301 e 311, e os nove primeiros das dezenas, em 310, 311, 312, 313, 314,
315, 316, 317 e 318. Assim, o livro tem 318 pginas.
129. Viagem a Recife A opo correta (b).
No momento em que a informao foi dada, o tempo de vo que faltava era de 1h20min,
4
ou 4/3 de hora. Logo, nesse momento, a distncia at Recife era de 864 3 = 1 152
km. Como estvamos a 1 222 km da cidade de partida, a distncia entre essa cidade e
Recife deve ser 1 152 + 1 222 = 2 374 km. Dentre as opes dadas, a mais prxima
2 400 km.
130. Praa Como a 5a casa da Maria a 12a casa do Joo, a diferena entre as contagens
de 7 casas e, portanto, a 1a casa da Maria a 8a casa do Joo. Como a 30a casa da
Maria a 5a casa do Joo, a 32a da Maria a 7a do Joo. A casa seguinte j a 8a do
Joo, ou seja, a 1a da Maria. Assim, a praa tem 32 casas.
131. Sequncia de guras As guras se repetem num grupo de seis, sempre terminando
com 2, tanto o 1o quanto o 166o grupo. Como 996 = 6 166, a ltima gura do 166o
grupo, ou seja, a 966a gura, 2.
996o

1 000o

, , , , , 2, . . . , , , , , , 2 , , , , , , 2, . . .
1o grupo de 6

166o grupo de 6

(a) A 1000a gura, portanto, .

(b) O primeiro est na 3a posio, o segundo na 1 6 + 3 = 9a posio, o terceiro


na 2 6 + 3 = 15a , o quarto na 3 6 + 3 = 21a posio, e assim por diante, at
o milsimo , que aparece na 999 6 + 3 = 5 997a posio.
132. A brincadeira com o quadrado
Soluo 1: Convertendo metros em milmetros, temos 1 m = 1 000 mm. Assim, o
quadrado cou dividido em 1 000 1 000 = 106 quadradinhos, cada um com 1 mm
de lado. Colocando lado a lado todos os 106 quadradinhos, teremos um retngulo de
comprimento
1 + 1 + + 1 = 106 1 = 106 mm = 1 km.
106 parcelas

146

OBMEP 2010

Solues do Nvel 1
Soluo 2: O quadrado tem rea igual a 1 m2 = 106 mm2 . A rea do retngulo
a mesma do quadrado. Como a largura do retngulo mede = 1 mm, resulta que o
comprimento c do retngulo, em milmetros, mede

106
c=
=
= 106 mm.

1
133. O cdigo da Arca do Tesouro Nas duas tabelas seguintes mostramos unicamente
os grupos de trs nmeros em casas sucessivas, horizontais ou verticais, cuja soma seja
14.

2
7

7
4
6

9
3

4
4

7
1
6

9
3
2

7
2
5

5
6
3

9
3
2
7
1
6

4
5
5
2
7

1
8
5

Assim, quando eliminamos esses nmeros da tabela inicial, os nmeros que sobrevivem
so somente os indicados na tabela seguinte.
5
6

4
8
2
Portanto, a soma dos nmeros que restam 5 + 4 + 6 + 4 + 8 + 2 = 29, que o cdigo
da Arca do Tesouro.
134. Operaes com decimais Temos

(0,2)3 + 1
0,008 + 1
1,008
=
=
= 0,84 .
0,2 + 1
1,2
1,2

135. Fatores inteiros Como o produto dos dois fatores 96, eles so divisores de 96 =
25 3, ou seja, os possveis fatores positivos so 1, 2, 3, 4, 6, 8, 12, 16, 24, 32, 48 e 96.
Os nicos com quadrado menor do que 208 so 1, 2, 3, 4, 6, 8 e 12, cujos quadrados
so 1, 4, 9, 16, 36, 64 e 144.

A nica maneira de obter 208 como soma de dois dos nmeros listados acima,
64 + 144 = 208. Assim, os nicos fatores positivos so 8 e 12. Logo os nicos fatores inteiros cuja soma dos quadrados 208 so 8 e 12 ou, ento, 8 e 12.
136. Divisibilidade A opo correta (b).
O nmero divisvel por 45 = 5 9, portanto divisvel por 5 e 9. Todo nmero
divisvel por 5 termina em 0 ou 5. Assim, b = 0 ou b = 5. Todo nmero divisvel
por 9 tem como a soma dos seus algarismos um nmero que mltiplo de 9. Logo,
6 + a + 7 + 8 + b = 21 + a + b mltiplo de 9. Como a 9, e b = 0 ou b = 5, temos
21 21 + a + b 21 + 9 + 5 = 35. Mas, o nico mltiplo de 9 entre 21 e 35 27. Logo,
21 + a + b = 27. Conclumos que a + b = 6 e o nmero procurado 61 785 ou 66 780.
OBMEP 2010

147

Solues do Nvel 1
137. Nmero simples Com 1 algarismo, temos os nmeros simples 1 e 2; com 2 algarismos, temos os 22 = 4 nmeros simples 11, 12, 21 e 22; com 3 algarismos, temos os 23 = 8
nmeros simples 111, 112, 121, 122, 211, 212, 221 e 222. Com 4 algarismos, temos
24 = 16 nmeros simples, com 5 algarismos, temos 25 = 32 nmeros simples e com 6
algarismos, temos 26 = 64 nmeros simples. Como um nmero inferior a 1 milho tem,
no mximo, 6 algarismos, resulta que existem exatamente 2 + 4 + 8 + 16 + 32 + 64 = 126
nmeros simples menores do que 1 milho.
138. Venda de TV Sejam a o algarismo da dezena de milhar e b o da unidade. Como
o nmero divisvel por 72 = 8 9, temos que 79b um nmero par divisvel por
8. Testando os valores de b = 0, 2, 4, 6 e 8, vemos que, necessariamente, b = 2. Um
nmero divisvel por 9 se a soma dos seus algarismos for um mltiplo de 9. Ento,
a + 6 + 7 + 9 + 2 = a + 24 um mltiplo de 9 e, portanto, a = 3. Assim, na fatura
constava R$ 36 792,00 e, portanto, cada TV custou 36 792 72 = 511 reais.
139. Chocolate Como 8 1,35 = 10,8 maior do que 10, Henrique comprou 7 barras de
chocolate e recebeu 10 71,35 = 0,55 reais, ou 55 centavos, de troco.
140. O quadradinho Simplicando, obtemos
1,6
Assim,

6 400 000
= 16 000 = 1,6 10 000 .
400

= 10 000.

141. Dois nmeros Como 12 o MDC dos dois nmeros e cada um tem dois algarismos,
os nicos candidatos so os mltiplos de 12 menores do que 100, ou seja,
12, 24, 36, 48, 60, 72, 84 e 96.
Como 1 728 = 12 12 12 = 26 33 , os mltiplos 60 (com fator 5) e 84 (com fator 7)
no so divisores de 1 728. Tambm 1 728 12 = 144 e 1 728 96 = 18, de modo que
a lista reduz a 24, 36, 48 e 72, com 24 72 = 36 48 = 1 728. Como o MDC de 24 e
72 24, temos uma nica soluo, a saber, 36 e 48, cujo produto 1 728 e o MDC
12.
142. As idades dos irmos Dividindo 2 000 por 7, obtemos 2 000 = 7 285 + 5.
Logo, 2 000 dias equivalem a 285 semanas, mais 5 dias. Como o dia 13 de maro de
2 007 caiu em uma tera-feira, contando os 5 dias restantes, temos que o aniversrio
do irmo de Carlos cair em um domingo. Agora, dividindo 2 000 por 365, obtemos
2 000 = 365 5 + 175. Assim, 2 000 dias equivalem a, aproximadamente, cinco anos e
meio, portanto Carlos estar com 12 anos de idade.
143. A mistura de concreto A opo correta (e).
De acordo com os dados do problema, misturamos 1 kg de cimento com 3 kg de areia
e 5 kg de terra. Isso equivale a misturar 5 kg de cimento com 15 kg de areia e 25 kg
de terra, e essa mistura pesa 5 + 15 + 25 = 45 kg.
148

OBMEP 2010

Solues do Nvel 1
144. Ponto na escala A distncia entre os pontos inicial e nal de 12,62 12,44 =
0,18 unidades. Como esto marcados 18 intervalos, o comprimento de cada um deles
de 0,18 18 = 0,01 unidades. O ponto P est na 6a posio direita de 12,44,
portanto corresponde a 12,44 + 0,01 6 = 12,50.
145. O pomar do Francisco A opo correta (c).
De acordo com os dados do problema, podemos observar que temos dois pares de
rvores vizinhas: as laranjeiras so vizinhas dos limoeiros e as macieiras so vizinhas
das pereiras. Como so cinco leiras e as macieiras e pereiras no esto do lado das
laranjeiras e limoeiros, resulta que as tangerineiras esto na terceira la, a do meio.
146. Quatro quadrados Se a rea de cada quadrado 3 cm2 e cada um deles est
3
cm2 . Como h
dividido em 16 quadradinhos, ento a rea de cada quadradinho
16
um total de 6 quadradinhos superpostos nos 4 quadrados, temos que a rea da gura

9
87
3
= 12 =
= 10,875 cm2 .
436
16
8
8
147. O o de arame A opo correta (d).
A gura composta de 3 semicrculos, o que exclui as opes (b), (c) e (e), e 4 segmentos
de reta, o que exclui a opo (a), que s tem 3 segmentos.
148. Sequncia de fsforos A opo correta (c).
Observe que o nmero de fsforos da sequncia formado da seguinte maneira:
primeiro termo =
segundo termo =
terceiro termo =

3 + 3 = 2 3 = (1 + 1) 3;
3 + 3 + 3 = 3 3 = (2 + 1) 3;
3 + 3 + 3 + 3 = 4 3 = (3 + 1) 3.

Logo, o oitavo termo da sequncia (8 + 1) 3 = 9 3 = 27.


149. O trajeto das formiguinhas
(a) O trajeto de M a N compreende 14 comprimentos e 12 larguras das lajotas. Logo,
seu comprimento 14 6 + 12 4 = 84 + 48 = 132 cm.
Como as duas formiguinhas percorrem a mesma distncia, cada uma deve andar
132 2 = 66 cm.
(b) Vamos acompanhar, desde o incio, o percurso feito por Maricota at completar
os 66 cm:
2 comprimentos

26=12

2 comprimentos
12+42=54

1 largura

3 comprimentos

4+12=16

1 largura

18+16=34

1 comprimento

4+54=58

6+58=64

2 larguras +
8+34=42

1/2 largura .
2+64=66

O caminho de Maricota at o ponto de encontro est indicado na gura.


OBMEP 2010

149

Solues do Nvel 1
r
M .............................................................................12
- .....
.

.
.
.
.
34
.
.
.............................................................
.............................................................
.
.
.
.
.
.
.
.
.
.
16
.
.
.
.
.
.
.
.
.
.
.
.
.
.
.........................................
.
........................................54
.
.
.
.
.
.
42
.
.
.
.
.
64 .....................58
....................
.
.
.
.
.
.
.
66 .
.
.
.
.
.
.
.
.
.
.
.
.
.
.
.
.
.
.............................................................
.............................................................
.
.
.
.
.
.
.
.
.
.
.
.
.
.
.
.
.
.
.
.
.
.
.
.
.
.
.
.
.
.
.
.
.
.
.
.
.
.
.
.
.
.
.
.
.
.........................................
.
.
.
.
........................................
.
.
.
.
.
.
.
.
.
.
.
.
.
.
.
.
.
.
.
....................
.
.
.
.....................
.


?
r

ponto de encontro

150. A soma 100


(a) Inicialmente observe que, como a soma dos trs nmeros 100 e o maior deles
igual soma dos outros dois, ento duas vezes o maior nmero 100, ou seja, o
maior nmero 50.
(b) Como 50 no primo, os outros dois nmeros so primos e tm soma igual a 50.
Por exemplo, 3 e 47 so primos e 3 + 47 = 50. Portanto, os nmeros 3, 47 e 50
formam uma soluo do problema.
(c) Existem outras solues para o problema. Para encontr-las, escrevemos a lista de
todos os primos entre 1 e 50, ou seja, 2, 3, 5, 7, 11, 13, 17, 19, 23, 29, 31, 37, 41, 43 e 47
e, para cada um desses nmeros, vericamos se a diferena para 50 tambm
primo. Encontramos um total de quatro solues
Soluo
Soluo
Soluo
Soluo

1
2
3
4

3
7
13
19

47
43
37
31

50
50
50
50

151. Cdigo de barras Lembre que a primeira e a ltima barra no fazem parte do
cdigo.
(a) Primeiramente, escrevemos o CEP dado com os algarismos 0 e 1:
00101 10100 00110 00001 11000 00011 00101 11000 .
3

Em seguida, escrevemos o cdigo de barras desse CEP:

||||||||||||||||||||||||||||||||||||||||||

(b) Primeiramente, escrevemos o cdigo de barras dado com os algarismos 0 e 1 em


grupos de 5 algarismos:

| ||||| ||||| ||||| ||||| ||||| ||||| ||||| ||||| |


01010 11000 01010 00110 11000 11000 01010 11000

Em seguida, escrevemos o CEP, que 20240020.


150

OBMEP 2010

Solues do Nvel 1
152. Atletas da escola O nmero total de alunos na escola dado
pela frao 12/12, que podemos representar gracamente por um
retngulo dividido em 12 partes iguais.
Denotemos por V, F e NE o nmero de alunos que jogam somente vlei, somente futebol
e nenhum desses dois esportes, respectivamente. A informao dada, em termos das
partes desse retngulo, a seguinte:
o 1/4 dos alunos que jogam somente vlei corresponde a trs partes;

o 1/3 dos alunos que jogam somente futebol corresponde a quatro partes;

o 1/12 dos alunos que no jogam nem vlei nem futebol corresponde a uma parte.
V
F

V
F

V
F

F
NE

(a) Sobram 4 partes do retngulo para os alunos que jogam vlei e futebol, ou seja,
esses 300 alunos correspondem a 4/12 = 1/3 do total dos alunos da escola. Logo,
o total de alunos na escola 300 3 = 900.
1
(b) O total de alunos que jogam somente futebol 900 = 300.
3
(c) Os alunos que jogam futebol so os que jogam s futebol mais os que jogam
futebol e vlei, ou seja, 300 + 300 = 600.
11
(d) O total de alunos que praticam um dos esportes
900 = 825, pois 1/12 dos
12
alunos no jogam nem futebol, nem vlei.
153. Dzima peridica
1
(a) Dividindo 1 por 22, obtemos 22 = 0,0454545 . . . . Observe que o algarismo 4 est
nas posies pares, ou seja, segunda, quarta, sexta, e assim por diante, enquanto
que o algarismo 5 est nas posies mpares, ou seja, a terceira, a quinta, a stima,
e assim por diante. Como 1 997 um nmero mpar, temos que o algarismo da
1 997a casa decimal 5.
1
(b) Dividindo 1 por 27, obtemos 27 = 0,037037037 . . . . Observe que os algarismos 0,
3 e 7 se repetem, sucessivamente, a cada trs casas decimais, sendo que

o algarismo 0 est nas posies 1a , 4a , 7a , . . . , ou seja, aquelas que, divididas


por 3, deixam resto 1;
o algarismo 3 est nas posies 2a , 5a , 8a , . . . , ou seja, aquelas que, divididas
por 3, deixam resto 2 e
o algarismo 7 est nas posies 3a , 6a , 9a , . . . , ou seja, aquelas que so mltiplas de 3.
.

Como a diviso 1 997 3 deixa resto 2, o algarismo da 1 997a


casa decimal 3.

OBMEP 2010

.
.
1 997 ................3...................
...
..
2 665

151

Solues do Nvel 1
154. Ana na corrida Transformando minutos em horas, temos que 20 minutos correspondem a 20/60 = 1/3 de hora. Assim, a velocidade de Ana deve ser maior do que
1
v = 5/ = 15 km/h.
3
155. Quadradinhos e o buraco Contando os quadradinhos retirados de cada linha,
temos que o nmero desses quadradinhos 1 + 3 + 5 + 15 + 10 + 2 = 36. Como cada
quadradinho tem 1 cm2 de rea, a rea do buraco 36 cm2 .
Para obter o permetro do buraco, podemos simplesmente contar quantos lados de
quadradinhos tm o buraco, obtendo 42 lados, de modo que o permetro mede 42 cm.
Entretanto, uma maneira alternativa de descobrir o permetro do buraco observar
que ele se estende por 6 linhas e 15 colunas, sendo que cada linha e cada coluna
ocupada pelo buraco contm exatamente dois lados de quadradinho que fazem parte
do permetro. Logo, o permetro do buraco mede 2 (6 + 15) = 42 cm.
156. Quadrados perfeitos no retngulo Para resolver esse problema, convm listar os
quadrados perfeitos de dois algarismos, que so
42 = 16, 52 = 25, 62 = 36, 72 = 49, 82 = 64 e 92 = 81,
bem como os quadrados perfeitos de trs algarismos, que so
102 = 100, 112 = 121, 122 = 144, 132 = 169, 142 = 196, 152 = 225,
162 = 256, 172 = 289, 182 = 324, 192 = 361, 202 = 400, 212 = 441,
222 = 484, 232 = 529, 242 = 576, 252 = 625, 262 = 676, 272 = 729,
282 = 784, 292 = 841, 302 = 900 e 312 = 961.
Em particular, vemos que todo quadrado perfeito de trs algarismos um nmero
terminado em 0, 1, 4, 5, 6 ou 9.
Assim, estabelecemos que, dos quadrados perfeitos de dois algarismos,
25, 36 e 81 no podem aparecer na terceira coluna, assinalada com X.
Para essa coluna, restam apenas os quadrados perfeitos 16, 49 e 64,
portanto, temos trs opes, como segue.
(I)

1
6

4
9

(II)

(III)

X
X

6
4

(a) Vamos examinar cada uma das trs opes.


Opo (I): Os quadrados perfeitos de trs algarismos terminados em 6 so 196,
256, 576 e 676. Como nenhum quadrado perfeito de dois algarismos
1
termina em 2 ou 7, os quadrados perfeitos 256, 576 e 676 no podem
1 9 6
aparecer na segunda linha, restando, apenas, 196.
Agora, os nicos quadrados perfeitos de dois algarismos terminados em 1 e 9 so,
respectivamente, 81 e 49. Obtemos a soluo seguinte, que a nica dentro da
Opo (I).
8
1
152

4
9

OBMEP 2010

1
6

Solues do Nvel 1

Opo (II): Os quadrados perfeitos de trs algarismos terminados em 9 so 169,


289, 529 e 729, de modo que a segunda linha pode ser preenchida apenas com o
quadrado perfeito 169. Na primeira coluna s pode aparecer o nmero 81, por ser
o nico quadrado perfeito de dois algarismos terminado em 1.
1

4
9

8
1

4
9

Temos, agora, duas opes para preencher a ltima casa em branco: 1 ou 3.


No entanto, nem 814 nem 834 so quadrados perfeitos. Assim, a opo (II)
impossvel.
Opo (III): Os quadrados perfeitos de trs algarismos terminados em 4 so 144,
324, 484 e 784, de modo que a segunda linha pode ser preenchida apenas com
o quadrado perfeito 144 e, na primeira coluna s pode aparecer o nmero 81.
Agora, a nica escolha para a casa em branco o nmero 6.
8
1

6
4

8
1

6
4

6
4

No entanto, 866 no quadrado perfeito. Logo a opo (III) tambm impossvel.


(b) Pelo que vimos acima, existe apenas uma soluo, encontrada no item precedente,
8 4 1
.
a saber,
1 9 6
157. Aula de diviso
(a) Temos 38 4 = 34 = 2 17 = 1 34, portanto, = 17 e = 2, ou = 34 e
= 1.
(b) Temos 75 = 6 12 + 3, portanto, = 3 e = 6.

(c) Temos 3 7 = 21. Os possveis restos da diviso por 3 so 0, 1 e 2, portanto,


= 21 e = 0, ou = 22 e = 1 ou, ainda, = 23 e = 2.

(d) Temos 42 = 5 8 + 2, portanto, podemos trocar o divisor pelo quociente para


obter = 8 e = 2.
158. Linhas de nibus
(a) Fatorando, temos 15 = 3 5 e 25 = 52 , portanto o menor mltiplo comum de 15 e
25 75 = 3 52 . Assim, os dois nibus passaro juntos novamente no ponto a cada
75 minutos, ou seja, a cada 1h15min. Logo, os nibus passaro juntos novamente
no ponto perto da casa de Quinzinho, s 7h30min + 1h15min = 8h45min.
(b) Para obter os horrios em que os nibus passaro juntos no ponto de nibus
perto da casa de Quinzinho, devemos ir somando 1h15min, obtendo 8h45min,
10h, 11h15min, 12h30min, 13h45min, 15h, 16h15min, 17h30min, 18h45min, 20h,
21h15min, 22h30min e 23h45min. O prximo nibus s passa depois da meia
noite.
OBMEP 2010

153

Solues do Nvel 1
159. Quadrados dentro de um retngulo
(a) Como o menor quadrado tem 1 cm de lado, o lado do quadrado
A mede 1 4 = 4 cm e o lado do quadrado B mede 4 + 1 = 5
cm. O quadrado C tem um lado em comum com o quadrado
B, portanto, o quadrado C tambm tem 5 cm de lado. Assim,
o lado do quadrado maior mede 5 + 5 + 4 = 14 cm.

.
....
......................14...................
.
.
.
.
.
.
.
.
.
.
.
.
.
.
.
.
.
.
.
.
.
.
.
.
.............................................
. 5 . 5 . 4 .
.
.
. C . B . A.
. .
.
.
. .
.
.
.............
.
..............................................
.
.
.

(b) Os lados do retngulo medem 14 cm e 14 + 5 = 19 cm, portanto, o permetro do


retngulo 14 2 + 19 2 = 66 cm.
160. Festa na escola
(a) A professora + 16 alunos + 1 monitor + 5 pais = 23 pessoas comero os pes
de queijo. Para que cada pessoa possa comer pelo menos 5 pes de queijo, ser
necessrio comprar, no mnimo, 5 23 = 115 pes de queijo. Cada po de queijo
100
pesa, em mdia,
= 10 gramas, de modo que ser necessrio comprar
10
10 115 = 1 150 gramas de pes de queijo.
Como a preciso da balana de 100 g, arredondamos 1 150 g para 1 200 g e
obtemos a quantidade de po de queijo que a professora deve comprar, em gramas.
1 200
(b) Como
= 12, temos que a professora gastar 12 3,20 = 38,40 reais.
100
1 200
(c) A quantidade de pes de queijo comprada foi de
= 120 pes. Logo, sobraro
10
120 115 = 5 pes de queijo.
161. Ai que fome
(a) Maria possui 50,50+70,25+40,10+50,05 = 2,50+1,75+0,40+0,25 = 4,90
reais.
(b) Tirando a passagem, restam R$ 4,00 para Maria fazer seu lanche. Observe que
Maria no pode escolher uma empada e, se escolher um sanduche, no pode mais
comprar um refrigerante. Assim, Maria s tem as nove seguintes opes de lanche.
Opo 1
Sanduche: R$ 2,20
Suco: R$ 1,20
Cocada: R$ 0,40
Total: R$ 3,80
Opo 5
Pastel: R$ 2,00
Suco: R$ 1,20
Cocada: R$ 0,40
Total: R$ 3,60

154

Opo 2
Sanduche: R$ 2,20
Suco: R$ 1,20
Bombom: R$ 0,50
Total: R$ 3,90

Opo 6
Pastel: R$ 2,00
Suco: R$ 1,20
Bombom: R$ 0,50
Total: R$ 3,70

Opo 3
Sanduche: R$ 2,20
Refresco: R$ 1,00
Cocada: R$ 0,40
Total: R$ 3,60

Opo 7
Pastel: R$ 2,00
Refresco: R$ 1,00
Sorvete: R$ 1,00
Total : R$ 4,00

OBMEP 2010

Opo 4
Sanduche: R$ 2,20
Refresco: R$ 1,00
Bombom: R$ 0,50
Total: R$ 3,70

Opo 8
Pastel: R$ 2,00
Refresco: R$ 1,00
Bombom: R$ 0,50
Total: R$ 3,50

Opo 9
Pastel: R$ 2,00
Refresco: R$ 1,00
Cocada: R$ 0,40
Total: R$ 3,40

Solues do Nvel 1
162. Advinhe Somando 50 + 50 = 100 obtemos trs dgitos e 41 32 = 9 tem um,
portanto, os nmeros procurados no podem ser maiores do que 49, nem menores do
que 42. Como 43 primo (bem como 47), meus nmeros so quaisquer dentre 42, 43,
44, 45, 46, 47, 48 e 49, que no tm divisor comum diferente de 1.
163. Produto de consecutivos Em primeiro lugar, note que se trs nmeros so consecutivos, ento um deles divisvel por 3. Portanto, qualquer nmero que seja o produto
de trs ou mais nmeros consecutivos, deve ser divisvel por 3. Mas, dentre os nmeros
dados, apenas 1 680 divisvel por 3 e, alm disso,
1 680 = 24 3 5 7 = 5 6 7 8.
Logo, 1 680 o nico dentre os trs nmeros dados que pode ser escrito como um
produto de quatro nmeros consecutivos.
164. Palndromos
(a) O prximo 2 112.
(b) O prximo palndromo mpar 3 003.
(c) Para ser primo, o palndromo no pode ter quatro algarismos, pois todo nmero
palndromo de quatro algarismos do tipo abba, que divisvel por 11, j que
abba = a00a + bb0 = a 1001 + b 110 = a 11 91 + b 11 10
= (91a + 10b) 11.
O primeiro nmero palndromo de cinco algarismos 10 001 = 73 137, que no
primo. Os prximos possveis candidatos so
10 101 = 3 367 3 e 10 201 = 101 101.
Assim, o primeiro nmero palndromo primo depois de 929 10 301.
165. O maior MDC Designemos por d o mximo divisor comum dos seis nmeros.
Ento, esses seis nmeros de dois algarismos so mltiplos distintos de d e podemos reformular a pergunta: queremos saber qual o maior nmero d que possui seis mltiplos
distintos menores do que 100.
Note que d, 2d, 3d, 4d, 5d e 6d so todos mltiplos de d. Logo, a melhor situao possvel
quando esses seis nmeros so os mltiplos considerados. Para isso, preciso que
6d 99. Dividindo 99 por 6, obtemos o quociente 16 e o resto 3, ou seja, 99 = 616+3.
Logo, d = 16. Portanto, os seis nmeros de dois algarismos cujo MDC o maior possvel
so 16, 32, 48, 64, 80 e 96. O MDC desses seis nmeros 16.
166. Quantidade de gua na Terra Denotemos V = 1 360 000 000. Lembre que 1% =
1/100, portanto, 1% de V igual a 1 360 000 000/100 = 13 600 000. Segue que:
97% =

97
= 0,97 e 97% de V vale 97 13 600 000 = 1 319 200 000;
100
OBMEP 2010

155

Solues do Nvel 1

40 000 000
1
= 0,0294 = 2,94
= 2,94%;
1 360 000 000
100

1,8% =

1,8
= 0,018 e 1,8% de V vale
100
1,8 13 600 000 = 24 480 000;

0,0096 = 0,96

1
= 0,96% e 0,96% de V vale
100
0,96 13 600 000 = 13 056 000;

1
250 000
= 0,00018 = 0,018
= 0,018%;
1 360 000 000
100
1
= 0,001% e 0,001% de V vale
0,00001 = 0,001
100

0,001 13 600 000 = 13 600.


Assim, a tabela completa a seguinte.
Especicaes
gua salgada
gua doce
Gelo
gua subterrnea
Lagos e rios
Vapor de gua

Volume de gua em km3


1 319 200 000
40 000 000
24 480 000
13 056 000
250 000
13 600

Percentual
97%
2,94%
1,8%
0,96%
0,018%
0,001%

Forma decimal do percentual


0,97
0,0294
0,018
0,0096
0,00018
0,00001

167. Balas Primeiramente, precisamos saber de quantas maneiras podemos obter 14


como soma de trs parcelas inteiras, cada uma delas maior do que ou igual a 3, isto ,
14 = . . . + . . . + . . . .
3

14 =

14 =

14 =
As parcelas possveis so

14 =

14 =

3
3
3
4
4

+
+
+
+
+

3
4
5
4
5

+
+
+
+
+

8
7
6
6
5

Agora, para cada uma dessas possibilidades, podemos fazer diferentes distribuies
entre as trs crianas, conforme a tabela seguinte. Observe que, quando as trs parcelas
so diferentes, temos seis possibilidades e, quando duas so iguais, temos apenas trs
possibilidades.
156

OBMEP 2010

Solues do Nvel 1
14 = 3 + 3 + 8

14 = 3 + 4 + 7

14 = 3 + 5 + 6

14 = 4 + 4 + 6

14 = 4 + 5 + 5

1a criana
3
3
8
3
3
4
4
7
7
3
3
5
5
6
6
4
4
6
4
5
5

2a criana
3
8
3
4
7
3
7
3
4
5
6
3
6
3
5
4
6
4
5
4
5

3a criana
8
3
3
7
4
7
3
4
3
6
5
6
3
5
3
6
4
4
5
5
4

Assim, temos 3 + 6 + 6 + 3 + 3 = 21 maneiras diferentes de distribuir as balas entre as


trs crianas.
168. Minutos Observemos primeiramente que
5
5
h = 60 min = 50 min,
6
6
de modo que a prova durou 4h50min. Somando as horas e os minutos, obtemos
12h35min + 4h50min = 16h85min.
Mas, 85 min = 1h25min. Logo, a prova termina s 16h85min = 17h25min.
169. Menor nmero Um nmero s divisvel por 4 se o nmero formado pelos seus
dois ltimos algarismos for divisvel por 4. Assim, usando apenas os algarismos 1, 2,
3, 4 e 9, as nicas possibilidades so 12, 24, 32 ou 92. Como 9 o maior algarismo,
devemos coloc-lo o mais direita possvel, de modo que 9 deve ser o algarismo da
casa das dezenas, ou seja, nosso nmero termina com 92. Os outros algarismos 1, 3 e
4, devem aparecer em ordem decrescente esquerda de 92, ou seja, os trs primeiros
algarismos do nmero devem ser 134. Portanto, o nmero procurado 13 492.
170. Contas do papagaio
5

+14

(a) Temos 8 40 54 9 8. Logo, o papagaio grita 8.

(b) Devemos fazer a operao inversa daquela que o papagaio fez, comeando da
ltima operao, ou seja, somar 1 ao nmero, multiplicar o nmero por 6, depois
subtrair 14 e dividir por 5 o resultado:
+1

14

3 4 24 10 2.

Logo, Antnio soprou 2 no ouvido do papagaio.


OBMEP 2010

157

Solues do Nvel 1
+1

14

(c) Observe que 7 8 48 34 6,8. Como 6,8 no um nmero inteiro,

Antnio no vai sopr-lo ao ouvido do papagaio e, mesmo que soprasse, o papagaio


no saberia realizar a primeira operao,
que seria multiplicar
6,8 5.

(d) Quando Antnio sopra um nmero n, o papagaio faz as operaes


5

+14

n 5n 5n + 14

5n + 14 1 5n + 14

1.
6
6

O papagaio s saber calcular a resposta se 5n + 14 for divisvel por 6, ou


seja, se for da forma 6k, com k inteiro no-negativo. Se 5n + 14 = 6k, ento
5n + 2 = 6(k 2) e, multiplicando ambos os lados por 5, resulta
25n + 10 = 6(5k 10), donde n + 24n = 25n = 6(5k 10) 12 + 2, ou seja,
n = 6(5k 12) + 2 24n = 6(5k 12 4n) + 2. Assim, se Antnio sopra um
nmero n da forma 6m + 2, o papagaio faz as operaes
5

+14

6m + 2 30m + 10 30m + 24 5m + 4 5m + 3

e grita o nmero 5m + 3. Se n no for dessa forma, o papagaio permanece mudo.


Logo, Antnio s pode soprar os nmeros
2, 8, 14, 20, 26, 32, 38, . . .
e o papagaio s pode responder, respectivamente,
3, 8, 13, 18, 23, 28, 33, . . . .
171. Soma maior que 34 O maior nmero de quatro algarismos 9 999, cuja soma dos
algarismos 49 = 36. Os nmeros de quatro algarismos cuja soma dos algarismos 35
so 8 999, 9 899, 9 989 e 9 998. Logo, temos cinco nmeros de quatro algarismos
com soma dos seus algarismos maior do que 34, que so os nmeros 8 999, 9 899, 9 989,
9 998 e 9 999.
172. Nenhum 1 Fatorando 111 111, obtemos 111 111 = 3 7 11 13 37. Segue
da que possvel, sim, escrever o nmero 111 111 como um produto de dois fatores,
nenhum deles terminando em 1. Por exemplo, 111 111 = 3 37 037. Mas existem
outras possibilidades, como, por exemplo, 111 111 = 7 15 873.
Na verdade, possvel listar todas as possibilidades. So elas
3 37 037,

7 15 873,

13 8 547,

39 2 849,

77 1 443,

143 777,

33 3 367,

37 3 003,

259 429 e 273 407.

Logo, Roberto tem 10 opes para escrever 111 111 na forma desejada.

173. Nmeros equilibrados Note que se um nmero equilibrado tem os trs algarismos
distintos, diferentes de zero, ento, com os mesmos algarismos, obtemos seis nmeros
equilibrados. Para isso, basta trocar os algarismos de posio. Por exemplo, 123, 132,
213, 231, 312 e 321.
158

OBMEP 2010

Solues do Nvel 1
Se um dos trs algarismos do nmero equilibrado for 0, ento com esses algarismos
obtemos apenas quatro nmeros equilibrados, pois o 0 no pode estar na casa da
centena. Por exemplo, 102, 120, 201 e 210.
Assim, vamos variar apenas os algarismos da centena e da dezena. Como o algarismo da
unidade a mdia desses dois algarismos, esses dois algarismos devem ser ambos pares
ou ambos mpares. Listamos os possveis nmeros equilibrados a partir do algarismo
das centenas.
total de nmeros equilibrados

1 : ; 111 ;
2 : ; 201 ;
3:;
4:;
5:;
6:;
7:;
8:;
9:;

132
222
333
402

;
;
;
;

153
243
354
444
555
603

;
;
;
;
;
;

174
264
375
465
576
666
777
804

;
;
;
;
;
;
;
;

195
285
396
486
597
687
798
888
999

1 + 4 6 = 25
4 + 1 + 3 6 = 23
1 + 3 6 = 19
4 + 1 + 2 6 = 17
1 + 2 6 = 13
4 + 1 + 6 = 11
1+6=7
4+1=5
1

;
;
;
;
;
;
;
;
;

Somando, temos 121 nmeros equilibrados de trs algarismos.


174. Nmeros primos Os nmeros primos entre 70 e 110 so
71,

73,

79,

83,

89,

97,

101,

103,

107 e 109.

Subtraindo 1 de todos esses nmeros, obtemos a lista


70,

72,

78,

82,

88,

96,

100,

102,

106 e 108.

Dessa lista, os mltiplos de 3 so 72, 78, 96, 102 e 108. Logo, os nmeros procurados
so
24 = 72 3, 26 = 78 3, 32 = 96 3, 34 = 102 3 e 36 = 108 3.

De fato, temos 24 3 + 1 = 73, 26 3 + 1 = 79, 32 3 + 1 = 97, 34 3 + 1 = 103 e


36 3 + 1 = 109.

175. Quadro moderno

(a)

(b)

A gura (a) mostra como foi pintado o quadrado nas duas cores, mas ainda no sabemos
qual dessas partes azul ou verde. Para isso, dividimos o quadrado em quatro faixas
verticais, como na gura (b), com o que o quadrado cou dividido em 16 quadradinhos
iguais. A parte no hachurada compreende
4 meios quadrados + 8 quadrados = 10 quadrados.
2 quadrados

OBMEP 2010

159

Solues do Nvel 1
Logo, a parte no hachurada corresponde a 10/16 do quadro, ou 5/8 e, portanto, a
parte hachurada corresponde a
16 10
6
3

=
= .
16 16
16
8
Assim, a parte hachurada da gura a que foi pintada de azul e corresponde a 3/8 do
quadro.
176. Encontro de amigos Eu chegarei quando meu relgio marcar 10h05min, uma vez
que penso que meu relgio est adiantado cinco minutos. Como ele est atrasado dez
minutos, chegarei, na verdade, s 10h15min. Meu amigo chegar quando seu relgio
marcar 10h, pois ele pensa que o relgio dele est correto, mas, na realidade, sero
09h55min. Logo, meu amigo chegar vinte minutos antes de mim.
177. Trabalho comunitrio A resposta correta (b).
O nmero total de alunos dessa classe 22 + 18 = 40, dos quais 60% foram prestar trabalho comunitrio, isto , 0,6 40 = 24. O nmero mnimo de alunas que participaram
desse trabalho obtido quando o nmero de alunos que participaram mximo, ou
seja, quando todos os 22 alunos se envolverem no trabalho, restando o mnimo de duas
vagas para as alunas.
178. rea de trapzios A resposta correta (e).
Unindo os quatro trapzios, formamos um quadrado de 50 cm
de lado e, portanto, de 2 500 cm2 de rea. Como o buraco
quadrado tem 30 cm de lado, sua rea de 30 30 = 900 cm2 .
Assim, a rea de cada um dos quatro trapzios, em cm2 , dada
por (2 500 900) 4 = 1 600 4 = 400 .
179. Adivinhao J de incio sabemos que o maior dos dois nmeros
par, por ser o dobro do menor, mas no termina em zero, porque o maior e o
menor nmero no possuem algarismos em comum;
seu algarismo das dezenas 2, no mnimo, porque sua metade um nmero com
dois algarismos e
a soma de seus algarismos 9, no mximo, porque essa soma um dos algarismos
do menor nmero.
Logo, o menor candidato a maior dos dois nmeros 22 e o maior 72. Depois de
22, o nmero par seguinte 24, que desconsideramos porque sua metade 12, que
repete o algarismo 2. J 26 candidato nesse critrio, mas 28 no , por ter soma de
algarismos igual a 10. Continuando at 72, obtemos todos os candidatos, indicados na
tabela seguinte.
maior
menor
160

22
11

26
13

32
16

34
17

36
18

OBMEP 2010

44
22

54
27

62
31

72
36

Solues do Nvel 1
Por vericao, temos que 34 e 17 a nica soluo, tendo sido os dois nmeros em
que pensei.

180. Dezoito nmeros consecutivos Uma sequncia de dezoito nmeros consecutivos


sempre possui dois termos que so mltiplos de 9. A soma dos algarismos de um
mltiplo de 9 sempre um mltiplo de 9. Logo, toda sequncia de dezoito nmeros
consecutivos sempre possui dois termos que so divisveis por 9 e cuja soma de seus
algarismos tambm divisvel por 9. Agora, cada um desses dois nmeros tm trs
algarismos, portanto, os nicos mltiplos de 9 que podem ser a soma dos algarismos
so 9, 18 e 27. No entanto, 999 o nico nmero de trs algarismos cuja soma dos
algarismos 27 e a nica sequncia de dezoito nmeros consecutivos de trs dgitos
que o inclua a sequncia de 982 a 999, que no inclui nmero de trs algarismos com
soma de algarismos igual a 9 e um nico com essa soma igual a 27. Assim, as nicas
possibilidades para as somas dos algarismos dos dois mltiplos de 9 da sequncia so
(i) 9 e 9;

(ii) 9 e 18;

(iii) 18 e 18;

(iv) 18 e 27.

Vejamos alguns exemplos de cada um desses quatro casos.

(i) 9 e 9: um dos nmeros 144 e o outro 135 = 144 9 ou 153 = 144 + 9. Duas
possveis sequncias so
130 ,
1o

,
2o

,
10o

,
11o

141 ,

, 135 ,
5o

,
13o

4o

15o

,
5o

13o

,
7o

, 144 ,

, 153 ,
12o

6o

14o

, 144 ,
3o

11o

10o

,
4o

12o

2o

1o

,
3o

,
14o

17o

,
7o

,
15o

18o

,
8o

,
16o

,
9o

, 147 ; e

,
16o

6o

,
8o

,
9o

, 158 .
17o

18o

(ii) 9 e 18: um dos nmeros 900 e o outro 891 ou 909. Duas possveis sequncias
so
887 ,
1o

,
2o

,
10o

,
11o

898 ,
1o

, 891 ,
4o

,
12o

3o

12o

,
6o

, 900 ,
14o

,
4o

, 909 ,
11o

5o

13o

, 900 ,
2o

,
10o

,
3o

,
13o

,
15o

5o

,
15o

17o

7o

18o

,
8o

,
16o

,
9o

, 904 ; e

,
8o

16o

6o

14o

,
7o

,
9o

, 915 .
17o

18o

(iii) 18 e 18: um dos nmeros 828 e o outro 819 ou 837. Duas possveis sequncias
OBMEP 2010

161

Solues do Nvel 1
so
811 ,

1o

2o

10o

11o

10o

5o

12o

14o

17o

6o

18o

7o

8o

15o

9o

, 828 ; e

16o

, 837 ,

13o

8o

15o

, 828 ,

4o

, 819 ,

7o

14o

6o

13o

3o

5o

12o

2o

1o

4o

11o

823 ,

3o

,
9o

, 840 .

16o

17o

18o

(iv) 18 e 27: um dos nmeros 999 e temos uma nica opo para a sequncia, a
saber,
982 ,
1o

,
2o

,
10o

,
3o

,
11o

,
4o

,
12o

,
5o

,
13o

,
6o

,
14o

,
7o

,
15o

, 990 ,
8o

,
16o

9o

, 999 .
17o

18o

Analisemos, agora, cada caso. Nos casos (i) e (ii), um dos nmeros divisvel por 9,
que a soma de seus algarismos. No caso (iv), um dos nmeros 999, que divisvel
por 27. Finalmente, no caso (iii), um dos dois mltiplos de 9 necessariamente par,
pois so dois mltiplos consecutivos de 9. Logo, esse nmero um mltiplo de 2 e de
9, portanto um mltiplo da soma de seus algarismos, que 18.
181. Completar uma tabela Observe que em cada quadrado formado por quatro quadradinhos, o nmero que est na parte inferior, direita, a soma dos outros trs
nmeros. Assim, preenchemos a tabela.
0
1
2
3
4

1
2
1+2+2=5
10
17

2
5
2 + 5 + 5 = 12
27
54

3
10
5 + 10 + 12 = 27
66
147

4
3 + 4 + 10 = 17
10 + 17 + 27 = 54
147
A

Logo:
A = 66 + 147 + 147 = 360.

182. Procurando mltiplos de 9 Sempre existe uma diferena que um mltiplo de 9.


De fato, quando dividimos um nmero por 9, podemos encontrar nove restos diferentes,
a saber, 0, 1, 2, 3, 4, 5, 6, 7 ou 8. Logo, entre os dez nmeros do conjunto, pelo menos
dois deles tm mesmo resto quando divididos por 9, j que temos, no mximo, nove
restos diferentes. Quando tomamos a diferena desses dois nmeros que tm o mesmo
resto, obtemos um nmero com resto zero, ou seja, divisvel por 9.
162

OBMEP 2010

Solues do Nvel 1
183. Correndo numa praa A distncia que o atleta
percorre a cada volta completa igual ao permetro da
praa, de 2 900 + 2 600 = 3 000 m.
Como 15,5 km = 15 500 m e 53 000 + 500 = 15 500
m, o atleta d cinco voltas completas (partindo de P e
retornando a P) e ainda corre mais 500 m. Portanto, ele
para no ponto Q, 150 m alm do vrtice B, indicado na
gura.
184. Ovos para um bolo Como os 43 bolos tm a mesma receita, o nmero de ovos
que a doceira precisa um mltiplo de 43. Por outro lado, esse nmero tambm
um mltiplo de 2, 3, 4, 5 e 6, acrescido de 1. O MMC de 2, 3, 4, 5 e 6 60, mas
60 + 1 = 61 no mltiplo de 43. Precisamos, ento, encontrar um nmero com essas
duas propriedades:
um mltiplo de 43;

acrescido de 1 mltiplo de 2, 3, 4, 5 e 6.
Lembre, tambm, que como a receita gasta menos do que nove ovos, o nmero que
estamos procurando menor do que 43 9 = 387. Temos:
60 2 + 1 = 121
60 3 + 1 = 181
60 4 + 1 = 241
60 5 + 1 = 301
60 6 + 1 = 361

no mltiplo de
no mltiplo de
no mltiplo de
mltiplo de 43;
no mltiplo de

43;
43;
43;
43.

Podemos parar por aqui, porque os prximos nmeros sero maiores do que 387. Logo,
a doceira comprou exatamente 301 ovos.
185. Cortando uma cartolina Os lados do retngulo nal obtido aps os cortes so,
cada um, a metade dos lados da cartolina original. Assim, o permetro do retngulo
original o dobro do permetro do retngulo nal. Logo, o permetro da cartolina
antes do corte media 2129 = 258 cm.
Observao: Ao fazer um corte paralelo a um dos lados do tringulo e pelo ponto
mdio desse lado, o outro corte que formar o retngulo s pode ocorrer no ponto
mdio do outro lado, em vista da semelhana desses tringulos. Assim, o enunciado
contm um dado a mais, desnecessrio para quem reconhece semelhana de tringulos
e suas propriedades.
186. A soma errada primeira inspeo, podemos admitir que os trs algarismos
direita dos nmeros estejam corretos, isto , esto corretos os algarismos 0, 1, 3, 4, 5,
6 e 8. Portanto, dentre os algarismos 2, 7 e 9, um deles est errado. O algarismo 9
est correto, pois se o mudarmos, a soma com 2 no estar certa. Assim, sobram 2
e 7. Se o 7 estivesse errado, ento o 2 estaria correto, mas isso no possvel, pois
1 + 4 + 2 = 7. Logo, o 2 que est errado e deve ser substitudo. Olhando novamente
para a soma 1 + 4 + 2, vemos que o resultado um nmero com o algarismo da unidade
igual a 1. Logo, o algarismo 2 deve ser substitudo quatro vezes pelo 6. Fazendo essa
substituio, vericamos que a soma ca correta.
OBMEP 2010

163

Solues do Nvel 1
187. Nmero de cinco algarismos Para que a b c seja divisvel por 4, seus dois ltimos
algarismos devem formar um nmero divisvel por 4. Como os algarismos so 1, 2, 3,
4 e 5, as nicas possibilidades so b c = 12, b c = 24, b c = 32 e b c = 52. Por outro
lado, os nmeros divisveis por 5 terminam em 0 ou 5. Como 0 no est includo, segue
que d = 5, pois b c d divisvel por 5. Isso exclui a possibilidade bc = 52, porque no
podemos repetir o 5. At agora temos trs possibilidades, a saber,
a245e e a325e.

a 1 2 5 e,

Examinemos esses trs casos para escolher os algarismos a e e, lembrando que no


pode haver repetio.
a325e

a245e

a125e

e=3

e=4

e=1

e=3

e=1

e=4

41 253

31 254

32 451

12 453

43 251

13 254

mltiplo de 3

mltiplo de 3

mltiplo de 3

mltiplo de 3

mltiplo de 3

mltiplo de 3

Logo, o nmero 12 453.


188. Tabela misteriosa Observemos que:
na ltima coluna estaro os mltiplos de 9, porque essa coluna est em branco e
nenhum dos nmeros que aparecem na tabela mltiplo de 9;
na 5a linha estaro os mltiplos de 12, pois nessa linha que aparece o nico
mltiplo de 12 da tabela (a saber, 24);
na 4a coluna estaro os mltiplos de 10, pois 40 o nico mltiplo de 10 na tabela;

na 5a coluna teremos mltiplos de 7, pois 42 e 49 so os nicos mltiplos de 7 na


tabela;

na 2a linha estaro os mltiplos de 7, porque 1 e 7 so os nicos divisores de 49


menores do que 12;
na 3a coluna aparecero os mltiplos de 2, pois 2 o nico divisor comum de 22
e 24 diferente de 1;
na 3a linha aparecero os mltiplos de 11, pois 22 = 2 11 e os mltiplos de 2 j
esto na 3a coluna;
na 6a linha aparecero os mltiplos de 6, pois os divisores de 42 = 2 3 7
menores do que 12 e diferentes de 1 so 2, 3, 6 e 7. Os mltiplos de 2 e 7 j esto
em seus respectivos lugares. Faltam os mltiplos de 3 e 6. Os nicos mltiplos de
6 na tabela so 24 e 42, e 24 j aparece na 5a linha;
na 2a coluna e na 4a linha aparecero os mltiplos de 3 ou 5, pois 15 = 3 5;

na 1a coluna e na 1a linha aparecero os mltiplos de 4 ou 8, pois os divisores


comuns de 32 e 40, menores do que 12 e diferentes de 1, so 2, 4 e 8, mas os
mltiplos de 2 j esto na 3a coluna.

164

OBMEP 2010

Solues do Nvel 1
At aqui, a situao a seguinte.
4 ou 8
32

10
40

14

70

49

63

11

22

10

77

99

12

24

120

84

108

12

60

42

54

4 ou 8

3 ou 5

3 ou 5

15

Examinemos agora as possibilidades que se apresentam.


II - Trs nmeros repetidos

I - Repetio de ambos 30 e 60

8
32

5
20

2
8

10
40

7
28

9
36

4
32

5
40

2
16

10
80

7
56

9
72

56

35

14

70

49

63

28

35

14

70

49

63

11

88

55

22

110

77

99

11

44

55

22

110

77

99

24

15

30

21

27

12

15

30

21

27

12

96

60

24

120

84

108

12

48

60

24

120

84

108

48

30

12

60

42

54

24

30

12

60

42

54

III - Repetio de ambos 12 e 40

IV - Apenas um nmero repetido

8
32

3
12

2
8

10
40

7
28

9
36

4
32

3
24

2
16

10
80

7
56

9
72

56

21

14

70

49

63

28

21

14

70

49

63

11

88

33

22

110

77

99

11

44

33

22

110

77

99

40

15

10

50

35

45

20

15

10

50

35

45

12

96

36

24

120

84

108

12

48

36

24

120

84

108

48

18

12

60

42

54

24

18

12

60

42

54

Logo, a nica soluo a da tabela IV.


189. Habitantes e esporte O total de habitantes desta cidade praticamente 30 000 e
divisvel por 9 e 15. Logo, deve terminar em 0 ou 5 e a soma de seus algarismos deve
ser um mltiplo de 9. Como 29 970 o maior nmero que menor do que 30 000 e tem
fatores 9 e 15, podemos supor que essa seja a populao total da cidade. Logo,
2
29 970 = 3 996 e
15

2
29 970 = 6 660
9

OBMEP 2010

165

Solues do Nvel 1
o nmero de mulheres e de homens, respectivamente, que praticam esporte somente
nos ns de semana. A tabela dada indica que 8 563 + 7 582 = 16 145 pessoas no
praticam esporte. Logo, a cidade tem 16 145 5 = 3 229 pessoas que praticam esporte
regularmente e, portanto, 3 229 1 252 = 1 977 pessoas do sexo feminino praticam
esporte regularmente. A tabela completa a seguinte.
No praticam esporte
fem.
8 563

masc.
7 582

Praticam esporte
somente nos ns
de semana
fem.
masc.
3 996
6 600

Praticam
esporte
regularmente
fem. masc.
1 977 1 252

Populao
total
29 970

190. Botes luminosos A resposta correta (c).


A tabela mostra a cor de cada boto em cada etapa.
incio
apertando boto 1
apertando boto 3
apertando boto 5

1
azul
verde
verde
verde

2
azul
verde
azul
azul

3
azul
azul
verde
verde

4
azul
azul
verde
azul

5
azul
azul
azul
verde

6
azul
azul
azul
verde

7
azul
azul
azul
azul

8
azul
verde
verde
verde

Logo, os botes que caram com luzes verdes acesas no nal so 1, 3, 5, 6 e 8, o que
nos d um total de cinco botes.
191. Qual o nmero? O problema determinar os algarismos b, c, d, e
e f tais que o nmero b c d e f 1 seja o triplo de 1 b c d e f.

1bcdef
3
...................................
..................................
bcdef 1

De incio vemos que f = 7 e, a partir da, podemos ir descobrindo cada um dos


algarismos, como segue.
1bcde7
3
...................................
..................................
bcde71

1bcd57
3
...................................
..................................
bcd571

1bc857
3
...................................
..................................
bc8571

1b2857
3
...................................
..................................
b28571

Portanto, b = 4 e o nmero de partida 142 857.


192. Jardim variado Os tringulos 1, 2, 5 e 6 so retngulos, de modo que, para calcular suas reas, vamos enxergar
cada um deles como metade de um retngulo. Para que a
nossa estratgia funcione, precisamos saber dividir o terreno
N
retangular em retngulos menores.
Subdividimos o terreno em dezesseis retngulos de 15 por
40 m, como mostra a gura, cada um com uma rea de 1540
= 600 m2 . Ento temos que
a rea do tringulo 1 = rea do tringulo 5 =
a rea do tringulo 2 =
166

1
3

1
4 600 = 1 200 m2 ;
2

1
6 600 = 1 800 m2 e
2
OBMEP 2010

P
6

Solues do Nvel 1
a rea do tringulo 6 =

1
2 600 = 600 m2 .
2

Observe que a rea do tringulo 4 igual rea do terreno todo, subtrada das reas
dos tringulo 5 e 6 e da rea da regio esquerda de M R. Contando retngulos, vemos
que essa rea mede 10 600 = 6 000 m2 . Logo, a rea do tringulo 4 dada por
120 80 1 200 + 600 + 6 000 = 9 600 7 800 = 1 800 m2 .
Finalmente, a rea do tringulo 3 a rea total do terreno subtrada da soma das reas
j calculadas dos outros cinco tringulos, ou seja,
120 80 (2 1 200 + 2 1 800 + 600) = 9 600 6 600 = 3 000 m2 .
Para que o gasto seja o menor possvel, as ores mais caras devem ser plantadas nas
regies menores. Como a menor regio a 6, nela deve ser plantada a or mais cara, a
rosa, gastando 3,50 600 = 2 100 reais. A maior regio a 3, onde deve ser plantada
a or mais barata, o bem-me-quer, gastando 0,80 3 000 = 2 400 reais.

Nas regies 1 e 5, com reas iguais a 1 200 m2 , devem ser plantadas bromlias e
cravos, contribuindo com (3,00 + 2,20) 1 200 = 6 240 reais. Nas regies 2 e 4, com
reas iguais a 1 800 m2 , devem ser plantadas margarida e violeta, contribuindo com
(1,20 + 1,70) 1 800 = 5 220 reais.
Temos, ento, quatro diferentes maneiras de formar o jardim, mantendo o mesmo gasto
mnimo de 2 100 + 2 400 + 6 240 + 5 220 = 15 960 reais. Apresentamos a seguir uma das
quatro possibilidades de escolhas das ores com esse oramento mnimo.
Regio
1
2
3
4
5
6

rea m2
1 200
1 800
3 000
1 800
1 200
600

Preo m2
3,00
1,20
0,80
1,70
2,20
3,50

Flor
bromlia
margarida
bem-me quer
violeta
cravo
rosa

Total por or
3,00 1 200 = 3 600
1,20 1 800 = 2 160
0,80 3 000 = 2 400
1,70 1 800 = 3 060
2,20 1 200 = 2 640
3,50 600 = 2 100
TOTAL: 15 960

193. O algarismo 3 Vejamos todas as vezes que Luis escreveu o algarismo 3:


3 ; 1;

13, 23, 30, 31, 32, 33, . . . , 39, 43, . . . , 93 ; 2 + 11 + 6 = 19.


2

11

At aqui, ele escreveu vinte vezes o algarismo 3. Da temos


103 , 113 , 123 , 130 , 131 .
21a

22a

23a

24a

25a

Logo, ao escrever o nmero 131, ele escreveu o algarismo 3 pela 25a vez.
OBMEP 2010

167

Solues do Nvel 1
194. Soma de potncias Existe um padro para o algarismo das unidades de uma
potncia de 3: ele tem perodo 4, pois se repete de quatro em quatro vezes. De fato,
temos
3
35 = 243
32 = 9
36 = . . . 9
33 = 27
37 = . . . 7
34 = 81
38 = . . . 1
Como 444 mltiplo de 4, o algarismo das unidades de 3444 1.
Analogamente, o algarismo das unidades de potncias de 4 tem perodo 2. De fato,
temos
41 = 4
43 = 64
42 = 16
44 = 256
Lembrete: Todo nmero
terminado em 0 ou 5 divisvel por 5.

Como 333 mpar, o algarismo das unidades de 4333


4. Portanto, o algarismo das unidades de 3444 +4333
1 + 4 = 5, de modo que ele divisvel por 5.

195. Telefonemas Como Joo telefona para seus pais a cada trs dias, podemos montar
uma tabela indicando os dias da semana em que ocorreram os quatorze primeiros
telefonemas de Joo.
Domingo Segunda Tera Quarta Quinta Sexta Sbado
1o

6o

4o

2o

7o

5o

3o

8o
13o
11o
9o
14o
12o
10o
Analisando a primeira linha dessa tabela, percebemos que so sete telefonemas, um em
cada dia da semana e que, a partir do stimo telefonema, os dias comeam a se repetir.
Isso implica que os nmeros que aparecem na segunda linha da tabela so obtidos dos
nmeros que aparecem na primeira linha somando 7. Por exemplo, Joo telefonar
para seus pais aos domingos nos telefonemas de nmeros
1
1+7=8
8 + 7 = 15
15 + 7 = 22
22 + 7 = 29
29 + 7 = 36
.
.
.
ou seja, nos nmeros que deixam resto 1 quando divididos por 7. Com esse raciocnio,
podemos determinar o dia da semana em que cai uma ligao, analisando o resto da
diviso do nmero do telefonema por 7.
Domingo
1
8
.
.
.

Segunda
6
13
.
.
.

resto 1

168

resto 6

Tera Quarta
4
2
11
9
.
.
.
.
.
.

resto 4

resto 2

OBMEP 2010

Quinta
7
14
.
.
.

Sexta
5
12
.
.
.

Sbado
3
10
.
.
.

resto 0

resto 5

resto 3

Solues do Nvel 1
Dividindo 100 por 7, obtemos 100 = 7 14 + 2. Logo, o resto da diviso de 100 por 7
2 e segue que o centsimo telefonema ocorre numa quarta-feira.
196. O maior produto Observe que obtemos o maior resultado possvel se um dos
nmeros comear com o algarismo 5 e o outro com 4. Alm disso, como s temos cinco
algarismos, um dos dois nmeros deve ter somente um ou dois algarismos. Vejamos as
possibilidades que do o maior produto.
um dos fatores tem um algarismo:

bom usar
uma
calculadora.

5 3214 = 21 284 ; 4 3215 = 21 605.


um dos fatores tem dois algarismos:

532 41 = 21 812 ; 531 42 = 22 302 ; 521 43 = 22 403 ;


432 51 = 22 032 ; 431 52 = 22 412 ; 421 53 = 22 313.
Logo, o melhor resultado 431 52 = 22 412.
197. O caminho da Joaninha Os nmeros primos que aparecem na tabela so 23,
73, 37, 17, 79, 19, 37, 53 e 251. Logo, s h dois caminhos que Dona Joaninha pode
percorrer. Um o apresentado na gura. O outro idntico, exceto que o azulejo 87
ca esquerda, passando entre 87 e 231 e, depois, seguindo horizontalmente.

C
198. O lugar dos amigos Observe que 3 o nico nmero dentro das trs guras e 1
o nico que no est dentro de um polgono, logo Celina ; 3 e Fbio ; 1. Agora, 4
o nico nmero dentro do tringulo e do crculo, logo Elisa ; 4. Nessa situao, 5
o nico dentro do tringulo, mas no do quadrado, assim Diana ; 5. Finalmente, 7
o nico nmero dentro de uma nica gura, logo Bento ; 7. Resta, ento, 2 dentro
do crculo, portanto, Guilherme ; 2 e Ana ; 6.
................
.................
.....
...
.....
...
...
..
...
..
..
..
..
.
..
..
..............................
.
..
..............................
.
.
.
.
.
.
.
.
.
.
.
.
.
.
.
.
.
.
.
.
.
. .
.
. ..
.
.
.
.
.
.
.
.
.
.
. .. ..
.
.
. ... ...
.
.
.
.
.
.
.
.
.
.
.
.. ..
.
.
.. ...
.
.
.
.
.
.
.
.
.
.
.
.
.. .
..
..
.. ..
.
...
..
.
.
.
.
.. .
.
.. .
.. ..
.. .
.
..
. .
.
..
. .
..
.
.
.. ... .
...
.
... ... .
....
.
.
.
.
... .
.
.
.
.
....
. ........ ...
.
... ..
..
.
.
.
.
..
.
.
................. .
..
.. .................
.
.
..............................
.
..............................
.
.
..
..
.. .
..
..
..
.
.
.
............................................
............................................
...
.
..
.

4
5

...............
......................
......
...
...
...
...
..
...
..
..
..
..
..
..............................
.
.
..
..............................
.
.
.
.
.
.
.
.
.
.
.
.
.
.
.
.
.
.
.
.
.
. .
.
. ..
.
.
.
.
.
.
.
.
.
.
. .. ..
.
.
. ... ...
.
.
.
.
.
.
.
.
.
.
.
.. ..
.
.
.. ...
.
.
.
.
.
.
.
.
.
.
.
.
.. .
..
..
.. ..
.
...
..
.
.
.
.
.. .
.
.. .
.. ..
.. .
.
..
. .
.
..
. .
..
.
.
.. ... .
...
.
... ... .
....
.
.
.
.
... .
.
.
.
.
....
. ........ ...
.
... ..
..
.
.
.
.
..
.
.
................. .
..
.. .................
.
.
..............................
.
..............................
.
.
..
..
.. .
..
..
..
.
.
.
............................................
............................................
...
.
..
.

E
D

OBMEP 2010

.................
..................
.....
...
.....
...
..
...
...
..
..
..
..
..
..
..............................
.
..
..............................
.
.
.
.
.
.
.
.
.
.
.
.
.
.
.
.
.
.
.
.
.
. .
.
. ..
.
.
.
.
.
.
.
.
.
.
. .. ..
.
.
. ... ...
.
.
.
.
.
.
.
.
.
.
.
.. ..
.
.
.. ...
.
.
.
.
.
.
.
.
.
.
.
.
.. .
..
..
.. ..
.
...
..
.
.
.
.
.. .
.
.. .
.. ..
.. .
.
..
. .
.
..
. .
..
.
.
.. ... .
...
.
... ... .
....
.
.
.
... .
.
.
..
.
.
....
. ........ ...
.
.. ..
..
.
.
.
.
..
.
.
................. .
..
.. .................
.
.
..............................
.
..............................
.
.
..
..
.. .
..
..
..
.
.
.
............................................
............................................
...
.
..
.

E A
D

169

Solues do Nvel 1
199. Quadrado perfeito? Lembre que um nmero um quadrado perfeito se na sua
decomposio em fatores primos os expoentes so todos pares. Por exemplo,
54 76 132 um quadrado perfeito, pois igual a (52 73 13)2 .
Como nenhum nmero elevado ao quadrado termina em 3, segue que N1 = 333 . . . 3
no um quadrado.
Temos que N2 = 666 . . . 6 = 2 333 . . . 3. Como 333 . . . 3 mpar, ento na decomposio de N2 em fatores primos aparece s um fator 2. Logo, N2 no um quadrado.
Vejamos a divisibilidade por 3. A soma dos algarismos desses nmeros
N3 ; 50 15 = 750

N4 ; 50 21 = 1 050

N5 ; 50 27 = 1 350

Como todas essas somas so divisveis por 3, essas trs somas tambm so divisveis
por 3. Logo, se algum deles fosse um quadrado perfeito, teria que ser divisvel por 9.
A soma dos algarismos de N3 e N4 no divisvel por 9, logo esses dois nmeros no
so divisveis por 9 e, consequentemente, no so quadrados perfeitos.
Como 1 350 divisvel por 9, ento N5 divisvel por 9. Temos
2727272727 . . . 27 9 = 303030 . . . 03
e
303030 . . . 03 3 = 101010 . . . 01,

portanto,

2727272727 . . . 27 = 32 303030 . . . 03 = 33 101010 . . . 01.


Note que 101010 . . . 01 tem 49 algarismos, dos quais 25 so iguais a 1 e os outros iguais
a 0. Logo, a soma de seus algarismos 25 e, portanto, no divisvel por 3. Assim,
2727272727 . . . 27 divisvel por 33 , mas no por 34 . Assim, conclumos que tampouco
N5 um quadrado perfeito.
200. Preenchendo quadradinhos A operao equivalente a
+

= 4

portanto, o lado esquerdo da igualdade um mltiplo de 4. Usando apenas os nmeros


1, 2, 3, 5 e 6, possvel vericar que as nicas possibilidades so
+

2 = 4 1

ou

1 = 4 2

Da, podemos concluir que


3 + 5 6

170

2 = 4 1

ou

OBMEP 2010

6 + 5 3

1 = 4 2

Solues do Nvel 1
so as nicas possibilidades de preenchimento.
1

201. Os trs nmeros Como 13 983 termina em 3, a soma dos


algarismos das unidades dos trs nmeros diferentes deve ser
13 ou 23. Como 23 no pode ser obtido na soma de 1, 2, 4 e
7, s temos uma opo, a saber, 2 + 4 + 7 = 13 .

2
4
7

....................................................................
....................................................................

Agora, a soma dos algarismos das dezenas deve ser 8 1 = 7 e, portanto, s pode ser
1 + 2 + 4 = 7. Completamos os algarismos das dezenas, tendo o cuidado de no repetir
o mesmo algarismo num mesmo nmero. Temos somente as trs opes seguintes.
1

1
2
4

4
1
2

2
4
7

....................................................................
....................................................................

1 3

4
2
1

2
4
7

....................................................................
....................................................................

8 3

2
4
7

....................................................................
....................................................................

8 3

9 8

Os algarismos das centenas devem somar 9, o que nos deixa duas possibilidades, 4+4+1
ou 1 + 1 + 7. Como nas trs opes o algarismo 4 ocorre em dois dos trs nmeros,
escolhemos a possibilidade 1 + 1 + 7 para a centena, para que no aparea repetido o
algarismo 4. Tambm precisamos cuidar para que no apaream repetidos o 1 e o 7,
o que elimina a terceira opo acima e nos leva a duas opes para as centenas, como
segue.
1

.........................................................................
........................................................................

.........................................................................
........................................................................

Finalmente, os algarismos das unidades de milhar devem somar 13 e fcil escolh-los.


Assim, Soa pode chegar a 13 983 de duas maneiras, como segue.
1

.........................................................................
........................................................................

OBMEP 2010

.........................................................................
........................................................................

171

Solues do Nvel 1
202. Preencher uma tabela Existem vrias maneiras de preencher a tabela, dependendo
da casa que escolhemos para ser preenchida, o que pode ser feito de vrias maneiras.
Vejamos um exemplo de como preencher a tabela. Inicialmente,
temos quatro casas que podem ser preenchidas, todas marcadas com
X. Escolhemos uma delas e preenchemos de acordo com a segunda
X X X
regra. Repetimos esse processo at a tabela estar completamente
1 2 X
preenchida.
;
1

3
1

;
2

3
1

4
2

3
1

4
2

Mas, para colocar em cada casa o maior nmero possvel, a idia , a cada vez, examinar
todas as casas que podem ser preenchidas e s preencher a casa em que podemos colocar
o maior nmero. Se em duas dessas casas o nmero a ser colocado for o mesmo,
preencheremos a que tem o menor nmero de casas vizinhas j preenchidos. Vamos l!

3
1

9
3
1

18
6
2

27
9
3
1

54
18
6
2

54
18
6
2

72
144

27
9
3
1

54
18
6
2

72
144

27
9
3
1

54
18
6
2

72
144

216
432
576

27
9
3
1

54
18
6
2

72
144
1178

216
432
576

27
9
3
1

54
18
6
2

72
144
1 178
3 516

27
9
3
1

54
18
6
2

72

27
9
3
1

54
18
6
2

72
144

216
432

27
9
3
1

54
18
6
2

72
144
1 178

216
432
576
1 754

216

27
9
3
1

6
2

6
2

9
3
1

3
1

216
432
576
1 754

Logo, o maior nmero que pode ser escrito na tabela 3 516.


203. Olimpada de Pequim Para iniciar, escolhemos um lugar para um dos atletas,
digamos, para Maria.
.................................
.
.................................
.
.
.
.
.
.
.
.
.
.
.
.
.
.
.
.
.
.
.
.
.
.
.
.
.
.
.
.
.
.
.
.
.
.
.
.
.
.
.
.
.
.
.
.
.
.
.
.
.
.
.
.
.
.
.
.
.
.
.
.
.
.
.
.
.
.
.
.
.
.
.
.
.
.
.
.
.
.................................
.
.
.
.................................
.

172

Maria

OBMEP 2010

Solues do Nvel 1
(a) Quem pratica natao est esquerda de Maria. Logo, s podemos ter a congurao abaixo.
.................................
.
.
.................................
.
.
.
.
.
.
.
.
.
.
.
.
.
.
.
.
.
.
.
.
.
.
.
.
.
.
.
.
.
.
.
.
.
.
.
.
.
.
.
.
.
.
.
.
.
.
.
.
.
.
.
.
.
.
.
.
.
.
.
.
.
.
.
.
.
.
.
.
.
.
.
.
.
.
.
.
.
.
.
.
.
.................................
.................................
.

Maria

Natao

(b) Quem pratica ginstica est frente de Juan. Existem duas nicas possibilidades:
Maria pratica ginstica ou Maria no pratica ginstica.
Maria no pratica ginstica

Maria pratica ginstica

Ginstica
Juan

.......................................
.
......................................
.
.
.
.
.
.
.
.
.
.
.
.
.
.
.
.
.
.
.
.
.
.
.
.
.
.
.
.
.
.
.
.
.
.
.
.
.
.
.
.
.
.
.
.
.
.
.
.
.
.
.
.
.
.
.
.
.
.
.
.
.
.
.
.
.
.
.
.
.
.
.
.
.
.
.
.
.
.
.
.
.
.
.
.
.
.
.
.
.
.
.
.......................................
.
......................................
.
.
.

.......................................
.
......................................
.
.
.
.
.
.
.
.
.
.
.
.
.
.
.
.
.
.
.
.
.
.
.
.
.
.
.
.
.
.
.
.
.
.
.
.
.
.
.
.
.
.
.
.
.
.
.
.
.
.
.
.
.
.
.
.
.
.
.
.
.
.
.
.
.
.
.
.
.
.
.
.
.
.
.
.
.
.
.
.
.
.
.
.
.
.
.
.
.
.
.
.......................................
.
......................................
.
.
.

Maria
Ginstica

Maria

Natao
Juan

Natao

(c) Como Tnia e David sentaram-se juntos, ento somente a segunda opo do item
anterior Maria no pratica ginstica pode satisfazer essa condio. Ela gera
as seguintes duas possibilidades.
Maria no pratica ginstica

Maria no pratica ginstica

Tnia
Ginstica

David
Ginstica
.......................................
.
......................................
.
.
.
.
.
.
.
.
.
.
.
.
.
.
.
.
.
.
.
.
.
.
.
.
.
.
.
.
.
.
.
.
.
.
.
.
.
.
.
.
.
.
.
Tnia .
.
.
.
.
.
.
.
.
.
.
.
.
.
.
.
.
.
.
.
.
.
.
.
.
.
.
.
.
.
.
.
.
.
.
.
.
.
.
.
.
.
.
.
.
.
.
.
.
.
......................................
.
.......................................
.

.......................................
.
......................................
.
.
.
.
.
.
.
.
.
.
.
.
.
.
.
.
.
.
.
.
.
.
.
.
.
.
.
.
.
.
.
.
.
.
.
.
.
.
.
.
.
.
.
David .
.
.
.
.
.
.
.
.
.
.
.
.
.
.
.
.
.
.
.
.
.
.
.
.
.
.
.
.
.
.
.
.
.
.
.
.
.
.
.
.
.
.
.
.
.
.
.
.
.
......................................
.
.......................................
.

Maria

Maria

Natao
Juan

Natao
Juan

(d) Como uma mulher sentou-se ao lado de quem pratica vlei, a segunda opo
acima que a correta, e temos duas possibilidades para o atleta que pratica
atletismo: David ou Maria.
Tnia
Ginstica

Tnia
Ginstica
.......................................
.
......................................
.
.
.
.
.
.
.
.
.
.
.
.
.
.
.
.
.
.
.
.
.
.
.
.
.
.
.
.
.
.
.
.
.
.
.
.
.
.
.
.
.
.
David .
.
.
.
.
.
.
.
.
.
.
.
.
.
.
.
.
.
.
Atletismo.
.
.
.
.
.
.
.
.
.
.
.
.
.
.
.
.
.
.
.
.
.
.
.
.
.
.
.
.
.
.......................................
.
.
.
......................................
.

.......................................
.
......................................
.
.
.
.
.
.
.
.
.
.
.
.
.
.
.
.
.
.
.
.
.
.
.
.
.
.
.
.
.
.
.
.
.
.
.
.
.
.
.
.
.
.
David .
.
.
.
.
.
.
.
.
.
.
.
.
.
.
.
.
.
.
Vlei .
.
.
.
.
.
.
.
.
.
.
.
.
.
.
.
.
.
.
.
.
.
.
.
.
.
.
.
.
.
.......................................
.
.
.
......................................
.

Maria
Vlei

Maria
Atletismo

Natao
Juan

Natao
Juan

204. Culturas diferentes


(a) (i) 03/12 signica 12 de maro para Ralph e 03 de dezembro para Jorge, portanto,
uma data ambgua.
OBMEP 2010

173

Solues do Nvel 1
(ii) 18/08 s pode ser mesmo 18 de agosto.
(iii) 05/05 s pode ser 05 de maio.
Logo, (i) uma data em que eles no podem se escrever.
(b) A data s ambgua quando o nmero do dia tambm puder representar o nmero
do ms, logo quando um nmero de 1 a 12. Por outro lado, nesses nmeros no
h ambiguidade quando o nmero do ms for igual ao nmero do dia. Por exemplo,
05/05 s pode ser 05 de maio. Por isso, em cada ms, eles devem evitar 11 dias.
Logo, os perodos mais longos em que eles no podem se escrever ocorrem em 11
dias consecutivos de janeiro de 02 a 12 de janeiro e em dezembro de 02 a 12
de dezembro. Observe que nos outros meses os perodos em que eles no podem
se escrever so menores. Por exemplo,
em abril eles no podem se escrever de 01/04 a 12/04, exceto em 04/04;
em setembro eles no podem se escrever de 01/09 a 12/09, exceto em 09/09.
205. Uma liquidao Na liquidao, exceto aos sbados, os produtos esto 50% mais
baratos. Nos sbados, com o desconto adicional de 20%, os produtos esto custando
80% dos preos fora dos sbados, ou seja
80% de 50% =

80
50
40

=
= 40% do preo original. .
100 100
100

Logo, Roberta deixou de economizar 60%, que corresponde aos R$ 50,40. Como
60% ; 50,40,
10% ; 50,40 6 = 8,4 e
100% ; 8,4 10 = 84,00,
o preo da cala antes da liquidao era de R$ 84,00.
206. Nmero com muitos zeros A resposta correta (d).
Vamos comparar os cinco nmeros sem efetuar clculos. Temos
3 + a = 3,000 . . . 0001 menor do que 4;
3a
3a
3
a
a
3

menor do que 3;
= 0,000 . . . 0003 menor do que 1;
3
3
= 3 102010 maior do que 10 e
=
=
1
0,000 . . . 0001
102010
0,000 . . . 0001
=
menor do que 0,000 . . . 0001 .
3

Assim, 3/a representa o maior nmero.


207. Corrida das tartarugas Vamos representar cada tartaruga numa reta, utilizando
sua letra inicial. Os dados nais da corrida esto representados na gura dada.
174

OBMEP 2010

Solues do Nvel 1
25

R
s

E
s

Ss

P
s

O
s

10

25

Logo, Sininha est 20 m frente de Elzinha e, portanto, Pulinha est 5 m frente de


Sininha. A ordem de chegada O, P, S, E e R.
208. Que memria... O nmero comea com 25 porque 52 a nica potncia de 5 com
dois algarismos.
2

Os candidatos aos dois ltimos algarismos so as potncias de 2 com dois algarismos,


a saber, 16, 32 e 64. Como 32 no serve, por apresentar o 2 repetido, temos as opes
2

ou

4 .

O algarismo do meio um mltiplo de 3, portanto, s pode ser 3, 6 ou 9, mas o 6


no pode ser repetido. Para escolher entre as duas opes acima, basta lembrar que
a soma dos cinco algarismos deve ser mpar e, como 2 + 5 mpar, a soma dos trs
ltimos deve ser par. Assim, a segunda opo acima ca descartada, pois no podemos
complet-la com um mltiplo de 3, restando, apenas os nmeros
2
O maior dos dois, 2

ou

6 .

6 , o cdigo bancrio de Esquecinaldo.

209. Uma frao irredutvel Para que a frao seja irredutvel, o numerador e o denominador no podem ter fator comum. Comeamos calculando os fatores primos de
N = 2 3 4 5 10, que so
2 3 4 5 6 7 8 9 10 .
22

23

23

25

32

Logo, a decomposio de N em fatores primos dada por


N = 28 34 52 7.
Podemos escolher diversas fraes que satisfazem o problema, como segue.
1
.
52 7
(ii) Se o numerador tem apenas um fator de N, temos as quatro fraes
(i) Se o numerador 1, temos a frao

28
;
34 52 7

28

34
;
28 52 7

34

52
28 34 7

OBMEP 2010

28

7
.
34 52
175

Solues do Nvel 1
(iii) Se o numerador tem dois fatores de N, temos as seis fraes
28 34 28 52 28 7 34 52 34 7
; 4
;
;
;
52 7
3 7 34 52 28 7 28 52

52 7
.
28 34

(iv) Se o numerador tem trs fatores de N, temos as quatro fraes


28 34 52
;
7

28 34 7
;
52

(v) Se o numerador N, temos a frao

28 52 7
34

34 52 7
.
28

28 34 52 7
.
1

Assim, ao todo, temos dezesseis dessas fraes irredutveis.


210. Transformar em decimal Temos:
5
14 20
34
1
2
+ 16
=
+
=
= 11 + = 11,3333 . . .
3
12
3
3
3
3
5
1
3
6
(b) 5 2
=5 2
= 5 = 4 = 3, 8
3
5
5
5
2
2
2
5
10
1
(c) 1 +
=1+
= 2 + = 2,25
3 = 1+
3 = 1+ 8 = 1+2
8
8
4
1 + 1+4
1+ 5
5
(a) 7

211. Uma sequncia especial Observe que:


os nmeros de 1 a 9 ocupam nove posies;

os nmeros de 10 a 99 ocupam 2 90 = 180 posies;

os nmeros de 100 a 199 ocupam 3 100 = 300 posies;


os de 200 a 299 ocupam 3 100 = 300 posies;

os de 300 a 399 ocupam 3 100 = 300 posies; etc.


100 , . . . 199 , 200 , . . . , 299 , 300 , . . . , 399 , 400 , . . . , 499 , 500 , . . . , 599 , 600 , . . . , 699
3100=300

3100=300

3100=300

3100=300

3100=300

3100=300

Assim, os algarismos usados para escrever de 1 a 699 ocupam 9 + 180 + 6 300 = 1 989
posies, logo faltam 2 009 1 989 = 20 posies. Como 20 = 3 6 + 2, precisamos
ainda escrever de 700 a 706, obtendo 21 posies, com o algarismo 6 ocupando a posio
21. Logo, o algarismo 0 que que ocupa a 2009a posio.
6

212. Cortar um retngulo Dividimos o retngulo


em 13 7 quadradinhos de 1 cm de lado cada
um. Agora, usamos que 13 = 1 + 3 + 4 + 5 =
6 + 7 = 0 + 13 para obter a diviso em 13 retngulos diferentes. Voc consegue encontrar outras
formas de fazer essa diviso?
176

OBMEP 2010

..............................................................................................................
.
.
.
.
.
.............................................................................................................
.
.
.
.
.
.
.
.
.
.
.
.
.
.
.
.
.
.
.
.
.............................................................................................................
.
.............................................................................................................. 1
.
.
.
.
.
.
.
.
.
.
.
.
.
.
.
.
.
.
.
.
.
.
.
.
.
.
.
.
.
.
.
.
.
.
.
.
.
.
.
.
.
.
.
.
.2
.
.
.
.
.
.
.
.
.
.
.
.
.
.
.
.
.
.............................................................................................................
.
.
.
..............................................................................................................
.
.
.
.
.
.
.
.1
.
.
.
.
.
.
.
.
.
.
.............................................................................................................
.
.
.
..............................................................................................................
.
.
.
.
.
.
.
.
.
.
.
.
.
.
.
.
.
.
.
.
.
.
.
.
.
.
.
.
.
.
.
.
.
.
.
.
.
.
.
.
.
.
.
.
.
.
.
.
.
.
.
.
.
.
.
.2
.
.
.
.
.
.
.
.
.
.
.
.
.
.
.
.
.
.
.
.
.
.
.
.
.
.
.
.
.
.
.
.
.
..............................................................................................................
.
.
.
.
.
.
.
.
.
.
.
.
.
.
.............................................................................................................
.
.
.
.
.
.
.
.
.
.
.
.
.
.
.
.
.
.
.
.
.
.
.
.
.
.
.
.
.
.
.
............................................................................................................. 1
.
.
..............................................................................................................
.
.
.
.
.
.
.
.

Solues do Nvel 1
213. Medida de ngulo A resposta correta (b).
Temos que AOC + C OE = 90 e C OE = DOY. Logo, AOC = 90 DOY. Como
DOY est entre 40 e 50 , segue que AOC est entre 90 50 = 40 e 90 40 = 50 .

214. Permetros e reas A rea do quadrado ( 3+3)2 = 3 +23 3+32 = 12+6 3


e a do retngulo

( 72 + 3 6) 2 = 144 + 3 12 = 12 + 6 3 .
Logo, eles tm a mesma rea. Vamos agora comparar os permetros. O do quadrado

4 ( 3 + 3) = 4 3 + 12

2)
6
e o do retngulo 2 ( 72 + 3 6 + 2) = 2 (6 2 + 3 6 + = 6 + 14 2 .

Como 4 3 < 6 6 e, tambm, 12 < 14 2, segue que 4 3 + 12 < 6 6 + 14 2. Assim,


o retngulo tem o maior permetro.
215. Clculo de ngulo Como AB = AC, o tringulo ABC issceles, logo ABC = ACB. Sendo
AD = BD, o tringulo ABD tambm issceles, logo
ABD = B AD. Temos, ento,

......
.
......
... . ...
... . ...
... . .......
... . .......
.
....
.... .
.
.... ..
....
....
.
.... . 39 .......
.
..
.
....
...
....
.
...
.
...
....
....
.
.
....
...
.
....
....
.
....
....
.
.
....
...
.
..
.
....
....
.
...
....
.
.
.
....
....
....
.
....
.
..
.
...
.
....
...
.
...
.
.
....
.
...
.
....
.
....
...
.
..
..
....................................................................................
........................................................................................
.... ..

ACB = ABC = ABD = B AD .

Na gura, esses trs ngulos iguais esto representados pela letra . Os ngulos internos
de ABC so + 39 , e . Logo, + 39 + + = 180 , ou seja, 3 = 180 39 =
141 . Assim, B AD = = 47 .
Lembrete 2: A
soma dos ngulos
internos de um
tringulo 180 :

Lembrete 1: Os
ngulos da base
de um tringulo
issceles so iguais:
B=C

A + B + C = 180 .

e AB = AC.

216. O caminho da formiga A resposta correta (c).

OBMEP 2010

177

Solues do Nvel 1
217. Menino mentiroso Claramente, Pedrinho encontrou Joozinho num dia em que
ele mente. O sbado est descartado pois, caso contrrio, ele estaria falando a verdade.
Assim, o encontro entre eles foi numa tera ou quinta-feira. No pode ter sido numa
tera-feira, porque ento o dia seguinte no poderia ser uma quarta. Logo, a nica
possibilidade para o dia do encontro dos dois quinta-feira.
218. Encontre os quatro nmeros Como os nmeros 1, 2, 3 e 6 satisfazem a propriedade, fcil vericar que, dado qualquer nmero inteiro n, os mltiplos n, 2n, 3n
e 6n de n tambm satisfazem a propriedade. Como estamos procurando nmeros de
trs algarismos e 999 6 = 166,5, basta considerar qualquer valor de n entre 100 e 166
para obter quatro nmeros de trs algarismos com a propriedade notvel.
219. Colando seis tringulos

1/8
1/16 V
IV
1/32

VI

III

1/4

1
I

II
1/2

O permetro da gura formada por treze segmentos, na sequncia de formao dos


tringulos, que podem ser descritos como segue.
1
cm no tringulo I,
2
1
1
1 segmento de cm e 1 segmento de cm no tringulo II,
2
4
1
1
1 segmento de cm e 1 segmento de cm no tringulo III,
4
8
1
1
1 segmento de cm e 1 segmento de
cm no tringulo IV,
8
16
1
1
cm e 1 segmento de
cm no tringulo V e
1 segmento de
16
32
1
2 segmentos de
cm no tringulo VI.
32
Soluo 1: Contando os comprimentos de segmentos, podemos ver que o permetro
mede
1
1
1
1
1
+3
21+2 +2 +2 +2
2
4
8
16
32
3
16 + 8 + 4 + 3
1 1 1
=3+
=2+1+ + + +
2 4 8 32
32
127
31
=
cm.
=3+
32
32
2 segmentos de 1 cm e 1 segmento de

178

OBMEP 2010

Solues do Nvel 1
Soluo 2: O contorno da gura, comeando no canto esquerdo e seguindo no sentido
anti-horrio, mede
1+

1
1
1
1
1
1 1 1
1 1 1
+ + +
+
+
+
+
+ + + +1
2 4 8 16 32 32 32 16 8 4 2

1
1
centmetros. A soma da PG de primeiro termo 1, razo e ltimo termo
dada
2
32
por
1
1
1
1
.
1 =2
64
2
32
Logo, o permetro da gura mede
2

1
1
1
1
127
+
=4
+ 2
=
cm.
32
32
32
32
32

Soluo 3: Observe que cada vez que agregamos um tringulo de lado a, trocamos
um segmento de comprimento a do permetro por dois segmentos de comprimento a,
de modo que o permetro aumenta em a.
Como o primeiro tringulo tem permetro de 3 cm, agregando um tringulo de lado
1
1
cm, a nova gura tem um permetro de 3 + cm; se agregamos mais um tringulo
2
2
1
1
1
cm, a nova gura tem permetro 3 + + cm. Seguindo esse processo,
de lado
4
2
4
depois do sexto tringulo, a gura tem permetro de
3+

1 1 1
1
1
1
127
+ + +
+
=3+1
=
cm,
2 4 8 16 32
32
32

1
1
1
onde usamos a soma da PG de primeiro termo , razo e ltimo termo , dada por
2
2
32
1
1

2 64

1
1
=1
.
2
32

220. Os livros da Elisa Seja N o nmero total de livros da Elisa. Como N + 1 um


mltiplo de 9 e 4, temos que N + 1 um mltiplo de 36. Logo, N + 1 36 ou 72, pois
Elisa tem menos do que 100 livros. Se N = 35, ento o nmero de livros de matemtica
36 9 1 = 3 e o nmero de livros de literatura 36 4 = 9. Mas, ento, Elisa
teria 24 + 3 + 9 = 36 livros, o que impossvel, porque 36 maior do que 35. Assim,
N = 71 e Elisa tem 72 9 1 = 7 livros de matemtica.
221. Substituindo pela soma Sabemos que qualquer nmero e a soma de seus algarismos
sempre deixam o mesmo resto quando divididos por 9. Assim, Mrcio substitui o
nmero inicial por outro, muito menor, com o mesmo resto na diviso por 9, e continua
assim, at chegar num nmero de um nico algarismo que, evidentemente, igual ao
resto da diviso de todos os nmeros obtidos anteriormente inclusive do primeiro
por 9. Assim, o que Mrcio faz , to somente, um processo de um passo apenas, que
consiste na substituio de nmeros naturais por seus restos na diviso por 9.
OBMEP 2010

179

Solues do Nvel 1
(a) Como 32 009 = 32 008 3 = (32 )1 004 3 = 91 004 3, o resto da diviso de 32 009 por
9 0. Logo, o nmero nal do processo de Mrcio 9.
(b) Observe que 172 = (18 1)2 = 182 2 9 + 1 = mltiplo de 9 + 1. Logo,
172 008 = (172 )1 004 = mltiplo de 9 + 1
e, portanto, 172 009 = mltiplo de 9 + 17 = mltiplo de 9 + 8. Logo, o nmero
nal do processo de Mrcio 8.
(c) Aplicando o processo aos nmeros da lista dos nmeros naturais 1, 2, 3, 4, 5, 6,
7, 8, 9, 10, 11, 12, . . . , a lista nal sempre 1, 2, 3, 4, 5, 6, 7, 8, 9, 1, 2, 3, . . . .
Como o resto da diviso do nmero 20 092 009 por 9 4, ento o ltimo nmero
da lista nal 4 e os seis ltimos algarismos da lista nal so . . . , 8, 9, 1, 2, 3, 4.
Portanto, essa lista tem os quatro algarismos 1, 2, 3 e 4 uma vez a mais do que os
algarismos 5, 6, 7, 8 e 9. Em particular, h mais 4 do que 5 na lista. O nmero de
vezes que aparece o 9 na lista o nmero de mltiplos de 9 que so menores do
que ou iguais a 20 092 009. Como 20 092 005 o maior mltiplo de 9 que menor
do que 20 092 009, temos que o algarismo 9 aparece 20 092 005 9 = 2 232 445
vezes na lista.
222. Uma brincadeira na sala de aula
(a) O nmero 1 s pode ser obtido por diviso a partir do 2, com 1 = 2 2 e o 2 s
pode ser obtido por diviso a partir do 4, com 2 = 4 2, mas o 4 pode ser obtido
por soma a partir do 1, com 4 = 1 + 3 ou por diviso a partir do 4, com 4 = 8 2.
Logo, temos duas maneiras de obter o 1 depois de trs operaes, a partir de 1 e
1 ; 4 ; 2 ; 1
de 8:
.
8 ; 4 ; 2 ; 1
(b) Com uma operao a mais, vemos que o nmero 8 pode ser obtido a partir do
5 por soma, com 8 = 5 + 3, ou do 16 por diviso, com 8 = 16 2. Logo, temos
trs maneiras de obter o 1 depois de quatro operaes, a partir de 2, 5 e 16:

2 ; 1 ; 4 ; 2 ; 1

5 ; 8 ; 4 ; 2 ; 1 .

16 ; 8 ; 4 ; 2 ; 1
(c) maneira anloga, vemos que podemos obter o 1 depois de cinco operaes, com
De
4 ; 2 ; 1 ; 4 ; 2 ; 1

10 ; 5 ; 8 ; 4 ; 2 ; 1
, bastando comear com os nmeros
13 ; 16 ; 8 ; 4 ; 2 ; 1

32 ; 16 ; 8 ; 4 ; 2 ; 1
4, 10, 13 e 32.

223. Calcule a idade No prximo ano, Laura e sua av estaro dois anos mais velhas do
que no ano passado. Logo, suas idades no ano passado so mltiplos de 8 que, somados
com 2, do mltiplos de 7. Procuremos esses nmeros.
mltiplos de 7 : 7 14 21 28 35 42 49 56 63 . . . 98 . . .
(mltiplos de 7) 2 : 5 12 19 26 33 40 47 54 61 . . . 96 . . .
180

OBMEP 2010

Solues do Nvel 1
Note que 40 e 96 so os nicos mltiplos de 8 menores do que 100 que aparecem na
segunda linha. Como Vov Ana tem menos do que 100 anos, podemos concluir que
ano passado ela tinha 96 anos e Laura 40. Logo, a idade atual de Laura 41 anos.
224. Divises e restos
Soluo 1: O dobro do nmero procurado um mltiplo de 5 acrescido de 1. Como
os mltiplos de 5 terminam em 0 ou 5, o dobro termina em 1 ou 6. Mas o dobro
um nmero par, logo termina em 6. Assim, o nmero termina em 3 ou 8 e, portanto,
dividido por 5, deixa resto 3.
Soluo 2: Sabemos que o nmero inteiro n procurado satisfaz 2n = 5m + 1, para
algum inteiro m. Ento o produto 5m = 2n 1 de 5 por m mpar, o que implica que
m mpar. Assim, m = 2k + 1, para algum inteiro k e, portanto,
2n = 5m + 1 = 5(2k + 1) + 1 = 10k + 6 = 2(5k + 3),
ou seja, n = 5k + 3 deixa resto 3 na diviso por 5.
225. Preenchendo o crculo Sabemos que
= 423 47 = 9. Por outro lado, temos
que
1 448 = 282 +

mltiplo de 282

nmero de 2 algarismos

Como 282 tem trs algarismos, conclumos que s pode ser o resto da diviso de
1 448 por 282. Efetuando essa diviso, obtemos 1 448 = 282 5 + 38. Logo, = 3 e
= 8. Obtemos, tambm, que = 5. Finalmente, obtemos
423

= 282, ou seja, 141 = 282, portanto, = 2 .


3

A sequncia completa a seguinte.


 
9

2/3

47 423

 

 
5

+ 38



282 1410

1448

 

OBMEP 2010



181

Solues do Nvel 2

Solues do Nvel 2
1. Populao A opo correta (a).
Como 1 milho = 1 000 000, temos 30,3 milhes = 30,3 1 000 000 = 30 300 000.
2. Rguas em 15 minutos A opo correta (e).
Se a mquina produz oito rguas em um minuto, em 15 minutos ela produzir 8 15 =
120 rguas.
3. Alturas iguais A opo correta (e).
Usaremos a notao a < b, que signica que a menor do que b ou, equivalentemente,
que b maior do que a. Assim, a < b < c signica que a menor do que b e b menor
do que c. Para simplicar, vamos denotar a altura de cada um dos irmos pela letra
inicial de seu nome.
Do enunciado temos:
(i) L maior do que A (L > A ou, equivalentemente, A < L);
(ii) M menor do que L (M < L);
(iii) A maior do que J ( A > J ou, equivalentemente, J < A);
(iv) J menor do que M (J < M).
De (i) e (iii) segue que J < A < L. Portanto, os irmos de mesma altura no esto entre
Jlio, Antnio e Luza. De (ii) e (iv) segue que J < M < L. Portanto, os irmos de
mesma altura no esto entre Jlio, Maria e Luza. Logo, a nica opo que Antnio
e Maria tenham a mesma altura.
4. Unidade A opo correta (c).
O produto dado tem um de seus fatores igual a 5, portanto, um mltiplo de 5, que
sempre tem o algarismo da unidade igual a 0 ou 5. Alm disso, como todos os fatores
so nmeros mpares, o produto um nmero mpar. Assim, seu algarismo da unidade
5.
5. Em que o? A opo correta (d).
Observe que a aranha utiliza oito os de apoio, numerados a partir do o A, iniciando
em 0. Logo,
sobre o o A aparecem os mltiplos de 8;

sobre o o B aparecem os (mltiplos de 8)+1;

sobre o o C aparecem os (mltiplos de 8)+2;

sobre o o D aparecem os (mltiplos de 8)+3;


sobre o o E aparecem os (mltiplos de 8)+4;
sobre o o F aparecem os (mltiplos de 8)+5;

sobre o o G aparecem os (mltiplos de 8)+6;


182

OBMEP 2010

Solues do Nvel 2
sobre o o H aparecem os (mltiplos de 8)+7.
Na diviso de 118 por 8 encontramos resto 6, o que signica que 118 dado por
(mltiplos de 8) + 6. Assim, 118 est sobre o o G.
6. Pontos ganhos A opo correta (c).
Segundo as regras da Copa do Mundo, uma vitria vale trs pontos e um empate vale
s um ponto. Como a seleo do Senegal tem uma vitria e dois empates, ela obteve
1 3 + 2 1 = 5 pontos.
7. Gols sofridos A opo correta (d).
Numa tabela de jogos, o nmero total de gols marcados sempre igual ao nmero total
de gols sofridos. Denotando por x o nmero de gols que sofreu a seleo do Uruguai,
vemos que 5 + 5 + 4 + 0 = 2 + 4 + x + 3, portanto, 14 = 9 + x, e temos que x = 5, ou
seja, a seleo do Uruguai sofreu 5 gols.
8. Qual o ngulo? A opo correta (c).
Nesta questo usaremos um importante teorema da Geometria Plana, como segue.
Teorema: A soma dos ngulos internos de um tringulo
B
sempre 180 .
50

Pelo teorema, temos A + B + C = 180 e, como B = 50 ,


segue que A + 50 + C = 180 , ou seja, A + C = 130 .
Como AD e CD so as bissetrizes dos ngulos A e C,
respectivamente, o teorema aplicado ao tringulo ADC
d a relao

1
1
A + C + ADC = 180 .
2
2

A
2

C
2

1
1
Mas 1 A + 2 C = 1 A + C = 2 130 = 65 , portanto, da igualdade acima decorre que
2
2
ADC = 180 65 = 115 .

9. Basquete A opo correta (a).


Analisando o grco, vericamos que os jogadores marcaram as seguintes quantidades
de pontos: Daniel 7, Ramon 8, Ian 2, Bernardo 11, Tiago 6, Pedro 12,
Ed 1 e Andr 7. O total 54 pontos.
10. Telefone A opo correta (a).
Vejamos a despesa em janeiro. Como 10 horas so gratuitas e Geni utilizou o telefone
por 15 horas e 17 minutos, ela deve pagar a tarifa xa mensal de 18 reais mais o
custo de apenas 5 horas e 17 minutos. Como o preo dado em minutos, passamos
o tempo a pagar para minutos. Sabemos que 1 hora = 60 minutos, portanto, 5 horas
= 5 60 = 300 minutos. Logo, 5h17min = 300 + 17 = 317. Assim, a conta telefnica
de Geni em janeiro foi de 18 + 317 0,03 = 18 + 9,51 = 27,51 reais.
OBMEP 2010

183

Solues do Nvel 2
Em fevereiro, Geni usou seu telefone por menos do que 10 horas, portanto nesse ms
ela s precisa pagar a tarifa xa mensal de 18 reais. Logo, a despesa de Geni com
telefone nesses dois meses foi de 27,51+18 = 45,51 reais.
11. rea A opo correta (e).
Soluo 1: A rea de um quadrado de lado l l2 e a rea da regio cinza a diferena
entre as reas dos quadrados maior e menor. O lado do quadrado maior a+b, portanto
sua rea (a + b)2 = a2 + 2ab + b2 . J o lado do quadrado menor a, portanto sua
rea a2 . Assim, a rea da regio cinza (a + b)2 a2 = a2 + 2ab + b2 a2 = 2ab + b2 .
Soluo 2: A rea de um retngulo o produto da largura pelo comprimento. Pelos
dados do problema, a largura da regio cinza (a + b) a = b.
Dividindo a regio cinza em dois retngulos, um
de largura b e comprimento a e o outro de largura
b e comprimento a + b (ver gura), vemos que a
rea da regio cinza a soma das reas desses dois
retngulos, ou seja,

rea

rea

a b + b (a + b) = ab + ab + b2
= 2ab + b2 .

Portanto, a rea da regio cinza 2ab + b2 .


Soluo 3: A regio cinza formada por dois retngulos de dimenses a b e um
quadrado de lado b. Logo, sua rea 2ab + b2 .
12. Comprando sorvete A opo correta (d).
Se comprar no supermercado A, Joana gastar 2 24 = 48 reais. Se comprar no
supermercado B, ela gastar 3 14 = 42 reais. Portanto, no supermercado B ela
economizar 6 reais em relao ao A.
13. Cartolina e barbante A opo correta (e).
Observando a frente da cartolina, vericamos que o barbante entra e sai pelos furos
da primeira linha. A opo (e) no possvel, pois no verso esses dois furos aparecem
como consecutivos ao percorrer o barbante, o que impede o barbante de continuar pelos
demais furos.
14. Amigos e fraes A opo correta (b).
Como cada amigo deu a Daniel a mesma quantia, digamos que Daniel tenha recebido
x reais de cada um de seus trs amigos. Inicialmente, ento, Adriano tinha 5x reais,
Bruno tinha 4x reais e Csar tinha 3x reais. Segue que o total de dinheiro inicial dos
trs amigos era de 5x + 4x + 3x = 12x reais. Como cada um de seus trs amigos
lhe deu x reais, Daniel tem agora 3x reais, o que representa a quarta parte do total
de 12x. Logo, ele agora possui 1/4 da quantia que seus trs amigos juntos possuam
inicialmente.
184

OBMEP 2010

Solues do Nvel 2
15. Escolhendo sorvetes A opo correta (d).
Vamos denotar cada sabor de sorvete pela sua letra inicial, ou seja, a aa,
b baunilha, c caj. Para enumerar todas as possibilidades de compra do sorvete
com quatro bolas, devemos considerar os seguintes casos:
quatro bolas do mesmo sabor (1a coluna ao
lado);
trs bolas do mesmo sabor e uma de sabor diferente (2a coluna ao lado);
duas bolas de um mesmo sabor e duas de outro
sabor (3a coluna ao lado);

aaaa aaab aabb


bbbb aaac aacc
cccc
bbcc
bbba
bbbc

duas bolas de um mesmo sabor e as outras duas


dos outros dois sabores (4a coluna ao lado).

aabc
bbac
ccab

ccca
cccb

Assim, obtemos 15 modos de fazer essa compra de sorvete.


16. Peas de um quadrado A opo correta (b).
Para que seja possvel montar o quadrado, o nmero total de quadradinhos deve ser
um quadrado perfeito. Um nmero inteiro um quadrado perfeito se ele igual ao
quadrado de algum nmero inteiro. Por exemplo, 1, 4, 9, 16 e 25 so quadrados
perfeitos, pois 1 = 12 , 4 = 22 , 9 = 32 , 16 = 43 e 25 = 52 . Observe que esses cinco
inteiros so os nicos quadrados perfeitos menores do que 30.
Contando o total de quadradinhos apresentados nas cinco opes de resposta obtemos
4 + 5 + 6 + 7 + 8 = 30. Portanto, devemos eliminar uma pea com 5 quadradinhos, para
restar 25, um quadrado perfeito, ou eliminar uma pea com 14 quadradinhos, para
restar 16, outro quadrado perfeito, ou eliminar uma com 21, para restar 9, ou eliminar
uma com 26, para restar 4, ou eliminar uma com 29 quadradinhos, para restar um
nico. Ocorre que no h peas com 14, 21, 26 ou 29 quadradinhos, restando a nica
opo de eliminar a pea (b), com 5 quadradinhos.
O nico quadrado que Pedro poderia ter montado com quatro
peas no usando a pea (b). Isto no signica que seja possvel montar um quadrado com as quatro peas restantes. Mas,
sabendo que devemos montar um quadrado de lado 5 com as
cinco peas (a), (c), (d) e (e), o problema j ca bem mais fcil.
A gura mostra como isso pode ser feito.

(c )

(d)

(a)

(e)

17. Paradas de nibus A opo correta (b).


Como a distncia entre a terceira e a sexta paradas 3 300 m, a distncia entre duas
paradas consecutivas 3 300 3 = 1 100 m. Portanto, a distncia entre a primeira e a
ltima paradas de 1 100 11 = 12 100 metros, ou seja, 12,1 quilmetros.

OBMEP 2010

185

Solues do Nvel 2
18. Desenho A opo correta (e).

(a)

(b)

(c)

(d)

Nas ilustraes (a), (b), (c) e (d) dadas, vemos que, iniciando o desenho no ponto P
e seguindo as setas de acordo com a ordem numrica, possvel completar cada um
desses desenhos sem tirar o lpis do papel.
Observe que, excetuando-se o vrtice de incio do traado e o vrtice de nalizao, os
demais vrtices do desenho devem possuir obrigatoriamente um nmero par de linhas
chegando at eles, pois a cada vez que se chega a um desses vrtices por uma linha,
deixa-se esse mesmo vrtice por outra linha.
Assim, impossvel fazer o traado da opo (e) do enunciado, que no pode ser
construdo sem tirar o lpis do papel, j que seus quatro vrtices externos possuem
trs linhas chegando a cada um deles.
19. Qual o cubo? A opo correta (e).
Ao cortar um canto do cubo, eliminamos um de seus vrtices. Como cada vrtice se
liga a trs arestas do cubo, uma representao do cubo cortado deve mostrar trs cortes
ao redor de um mesmo vrtice.

20. Quadrado mgico A soma dos nmeros de uma diagonal 4 + 0 + (4) = 0,


portanto, o valor da soma dos nmeros de cada linha, de cada coluna e da outra
diagonal tambm deve ser 0. Assim, obtemos de imediato os nmeros que faltam nas
casas cinza no primeiro tabuleiro, a saber, 16, 8 e 12, porque (12) + 16 + (4) = 0 na
primeira linha, (12) + 8 + 4 = 0 na primeira coluna e (12) + 0 + 12 = 0 na diagonal.

Agora, o nmero que falta na segunda linha do segundo tabuleiro 8, porque


8 + 0 + (8) = 0. Para a terceira linha, obtemos 16, pois 4 + (16) 12 = 0.
21. Torneio Denotemos as sete equipes pela sua letra inicial.
186

OBMEP 2010

Solues do Nvel 2
(a) Na primeira rodada do Grupo 1 foram disputadas trs partidas, AB, BC e
CA.
(b) Na primeira rodada do Grupo 2 foram disputadas seis partidas, DE, DF,
DG, EF, EG e FG.
(c) Na segunda rodada, cada equipe do Grupo 1 jogou quatro partidas, uma com
cada uma das equipes do Grupo 2. Como o Grupo 1 tem trs equipes, na segunda
rodada foram disputadas 3 4 = 12 partidas.
22. Truque numrico
(a) Vamos fazer o experimento com os nmeros 0, 5 e 4.
0
5
-4

6
6
6

-21

-21

30

-21

-24

-21

-45

-7

3
-15

-(02)=0
-(52)= -10

-(-42)= +8

-7

-7
-7

O resultado nal sempre 7.

(b) razovel, ento, conjecturar que, para qualquer nmero inicial escolhido, o resultado nal desse procedimento ser sempre 7. Seja x o nmero inicial. Temos,
ento, as operaes seguintes.
x

6x

-21

6x-21

6x-21
8
3

-2x

2x-7-2x = -7

Portanto, o resultado dessa mgica sempre ser igual a 7, qualquer que seja
o nmero inicialmente escolhido.
23. Jogando sinuca A bola muda a direo de sua trajetria cada vez que bate numa das beiradas da mesa.
Como a trajetria faz sempre um ngulo de 45 com
a beirada, a trajetria dessa bola, tacada a partir de
um canto, seguir sempre as diagonais dos quadrados
que ela cruzar. Traando essa trajetria, conclumos
que (b) a bola bater cinco vezes nas beiradas da mesa
antes de (a) cair na caapa superior esquerda.
Contando quadrados atravessados, vemos que (c) ela atravessar 23 quadrados pela
diagonal.
24. Tringulo issceles Por denio, um tringulo issceles se tiver dois lados
iguais. O terceiro lado chamado base do tringulo issceles, e os ngulos formados
entre a base e os dois lados iguais so os ngulos da base.
OBMEP 2010

187

Solues do Nvel 2
A

C
C

A gura mostra um tringulo issceles ABC, cujos lados iguais so AB e AC e a


base BC. Denotamos os ngulos ABC e ACB da base por B e C, respectivamente.
Demonstra-se que num tringulo issceles os ngulos da base so sempre iguais. No
tringulo da gura temos, portanto, B = C.
Passando resoluo desta questo, observe que A + B + C = 180 , j que a soma dos
ngulos internos de qualquer tringulo 180 . Pelos dados do problema, A = 20o e o
tringulo issceles, de modo que B = C. Logo, 180 = 20 + B + C = 20 + 2B e,
portanto, B = C = 80 .
O tringulo CBD tambm issceles, pois dado que CB = DB. Como a base
desse tringulo CD, seus ngulos de base so C DB = C, portanto, C DB = 80 . Considerando a soma dos ngulos internos desse tringulo CBD, obtemos
C BD + C DB + C = 180 . Substituindo os valores j obtidos, vemos que C BD + 80 +
80 = 180 , de modo que C BD = 20 . Assim, DBE = B 20 = 80 20 = 60 .

O tringulo DBE tambm issceles, porque tambm DB = BE. A base desse


tringulo DE e os ngulos iguais da base BE so E DB = DEB. Como
180 = B DE + DEB + DBE = 2 B DE + 60 ,
conclumos que B DE = 60 .
25. Pesando moedas Sejam A, B, C e D as quatro moedas aparentemente iguais.
Comparamos as moedas A e B na balana, colocando uma em cada prato. Dois
casos podem ocorrer: a balana ca em equilbrio ou a balana no ca em equilbrio.
Vamos analisar separadamente cada caso. Observe que, em ambos casos, s utilizamos
a balana duas vezes.

1o Caso: A balana ca equilibrada. Podemos concluir que A e B tm o mesmo peso,


portanto, so verdadeiras. Vamos ento comparar A com C. Para isso, mantemos A
na balana e colocamos C no lugar de B. Se houver equilbrio novamente, porque A
e C tm o mesmo peso e so, portanto, verdadeiras. Assim, A, B e C so verdadeiras
e a nica opo que D seja a moeda falsa. Se no houver equilbrio, C a moeda
falsa.
2o Caso: A balana no ca equilibrada. Logo uma das duas moedas, A ou B a
falsa. Substitumos A por C na balana. Se houver equilbrio, A a moeda falsa. Se
no houver equilbrio, a moeda falsa B.

188

OBMEP 2010

Solues do Nvel 2
26. Nmeros binomiais A opo correta (e).
Preenchendo o tabuleiro de acordo com as regras do problema, segue que
60 = ( + 17) + (2 +13) = 3 +30, donde = 10.
5

+6

11

+17

+7
2+13

60

27. Costuras da bola A opo correta (c).


Se somarmos os nmeros de lados de todos os polgonos (20 hexgonos e 12 pentgonos)
que compem a superfcie da bola, obteremos um valor que duas vezes o nmero de
costuras, pois cada costura lado comum de exatamente dois polgonos. Assim, temos
que 2 nmero de costuras = 12 5 + 20 6 = 180, donde o nmero de costuras 90.
28. Razo de reas A opo correta (a).
A grade um quadrado de lado igual a 5 cm, logo sua rea igual a 25 cm2 . A parte
sombreada da grade formada por quatro tringulos, sendo que dois deles tm base
1 cm e altura 2 cm e os outros dois tm base 1 cm e altura 3 cm. Logo a rea sombreada
1
1
igual a 2 2 (1 2) + 2 2 (1 3) = 5 cm2 e a rea no sombreada igual a 25 5 =
20 cm2 . Assim, a razo pedida 5/20 = 1/4.
29. S sorvete A opo correta (c).
Vamos primeiro analisar a informao contida na diagonal da tabela indicada pelos
nmeros dentro dos quadradinhos.

Esses nmeros indicam quantas foram as crianas que tomaram sorvetes com o mesmo
sabor pela manh e pela tarde: um tomou sorvetes de abacaxi, dois de banana, nenhum
de chocolate e um de doce de leite. Todos os outros estudantes comeram sorvetes de
sabores diferentes pela manh e tarde, num total de 64 (1 + 2 + 0 + 1) = 60.

OBMEP 2010

189

Solues do Nvel 2
30. Brincando com tabuleiro A opo correta (b).
Notamos primeiro que se uma casa tem o algarismo
0, ento nenhuma das casas vizinhas pode estar
pintada. Logo, as casas marcadas com um na
gura direita no foram pintadas.
Consideremos, agora, a casa do canto superior direito, na qual aparece o nmero 1. Ela tem trs
vizinhas, e j sabemos que duas delas no foram
pintadas.
Logo, a vizinha que sobra (a casa imediatamente
abaixo) foi pintada. Podemos aplicar o mesmo argumento s casas do canto inferior esquerdo e do
canto inferior direito.
Olhamos agora para o 2 na ltima linha. Como
esta casa j tem duas vizinhas pintadas, todas suas
outras vizinhas no foram pintadas.
Argumento idntico se aplica casa da segunda
linha e terceira coluna, pois nela aparece um 1 e
j temos uma de suas vizinhas pintadas. Logo, as
suas outras trs vizinhas no foram pintadas.
Finalmente, usamos o 3 que aparece na casa da
terceira linha e terceira coluna. Esta casa j tem
duas vizinhas pintadas, logo deve haver mais uma
de suas vizinhas pintada.
Esta vizinha s pode ser a casa em branco na gura acima, e podemos completar a
tabela. Conclumos que o nmero de casas pintadas 4.
31. Cartes numerados A opo correta (b).
A formao de um nmero de 6 algarismos ilustrada a seguir.
centena
de milhar

dezena
de milhar

unidade
de milhar

centena

dezena

unidade

Para se obter o menor nmero possvel, os menores algarismos devem estar o mais
esquerda possvel (na casa do milhar) e para se obter o maior nmero possvel os
maiores algarismos devem tambm estar o mais esquerda possvel (na casa do milhar).
Jorge joga primeiro: Para obter o menor nmero possvel, ele coloca o menor algarismo
que ele possui, que o 2, na casa da centena de milhar. Se ele no zesse isso, Larissa
colocaria seu 5 nesta casa na prxima jogada e obteria, assim, um nmero maior.
2

dezena
de milhar

unidade
de milhar

centena

dezena

unidade

Agora a vez de Larissa: Para obter o maior nmero possvel, ela coloca o maior algarismo que ela possui, que o 5, na casa das dezenas de milhar, pois a casa das centenas
de milhar j est ocupada.
190

OBMEP 2010

Solues do Nvel 2
2

unidade
de milhar

centena

dezena

unidade

Agora, Jorge tem os algarismos 4 e 6, e Larissa 1 e 3. Logo, os algarismos de Larissa


so menores do que os de Jorge, o que determina a estratgia de Jorge: ele deve tentar
colocar seus algarismos o mais direita possvel, com o 6 direita do 4. Por sua
vez, Larissa deve tentar colocar seus algarismos o mais esquerda possvel, com o 3
esquerda do 1.
Jorge joga: Ele coloca o algarismo 6 na casa das unidades.
2

unidade
de milhar

centena

dezena

Larissa joga: Ela coloca seu 1 na casa das dezenas.


2

unidade
de milhar

centena

Agora, Jorge tem apenas o algarismo 4 e Larissa o 3. Ele ento coloca o 4 na casa das
centenas e Larissa coloca o 3 na casa das unidades de milhar, acabando o jogo.
2

Assim, o nmero nal, obtido se os dois jogadores forem espertos, 253 416.
32. Faltam balas A opo correta (a).
Dividindo 237 por 37, obtemos 237 = 7 31 + 20. Logo, 237 no divisvel por 31.
Isso quer dizer que a professora realmente vai ter que comprar mais balas para que
todos os alunos recebam o mesmo nmero de balas. Devemos adicionar expresso
7 31 + 20 o menor inteiro positivo x tal que 7 31 + 20 + x seja mltiplo de 31. Como
20 + 11 = 31, basta que a professora compre 11 balas adicionais.
33. Artesos de braceletes A opo correta (d).
O arteso produz 6 braceletes a cada 20 minutos. Como 1 hora = 60 minutos =
3 20 minutos, o arteso produz 6 3 = 18 braceletes em uma hora. Como ele
trabalhou 12 horas 8 horas = 4 horas, o nmero de braceletes feitos pelo arteso
18 4 = 72. O auxiliar produz 8 braceletes a cada meia hora, portanto em 1 hora ele
produz 16 braceletes. Para produzir 72 braceletes ele precisar de 72/16 = 4,5 horas
= 4 horas e 30 minutos. Como ele inicia seu trabalho s 9 horas, ele terminar seu
trabalho s 9 + 4,5 = 13h30min.
34. Girando um pentgono A opo correta (b).
1
360 = 72 .
5
Soluo 1: Dividindo 252 por 72, obtemos 252 = 3 72 + 36. Como 36 = 72 2,
conclumos que uma rotao do pentgono de 252 em torno do seu centro corresponde
a uma rotao de um ngulo igual a trs vezes e meia o ngulo central.
O pentgono tem 5 lados. Logo, seu ngulo central mede

OBMEP 2010

191

Solues do Nvel 2
Soluo 2: Como 252 = 72 + 180 , podemos pensar na rotao de 252 como uma
rotao de 72 seguida de outra de 180 , conforme ilustrado na gura dada, em que O
o centro do polgono.
A
A

O
rotao de 72

rotao de 180

35. rea em funo da diagonal A opo correta (c).


A rea A de um retngulo o produto do comprimento
pela largura. Sejam a e b o comprimento e a largura do
retngulo. Assim, A = ab. O permetro desse retngulo
dado por 2a + 2b. Como o permetro 100, temos que
2a + 2b = 100, portanto, a + b = 50. Elevando ao
quadrado ambos os lados dessa ltima igualdade, obte-

.
......
..
..............................................................................................
................................................................................................
.
... .
.
.
.
.
.... .
..... .
.
.
.
.
.
.....
.
.....
.
.
.
.
.
.....
.
.....
.
.
.
.
.
.....
.
.....
.
.
.
..
.
.
.
.
.
.....
.
.....
.
.
.
..
.
.
.
.
.....
.
.
.....
.
..
.
.
.
.
.
.
.....
.....
.
.
.
..
.
.
.
.
.
.
.....
.....
.
.
.
..
.
.
.
.
.
.....
.
.....
.
.
.
..
.
.
.
.
.
.....
.
.....
.
.
.
..
.
.
.
.
.....
.
.
.....
.
.
.
..
.
.
.
.
.....
.
.
.....
.
..
.
.
.
.
.
.
.....
.
.....
.
.
..
.
.
.
.
.
.
.....
.....
.
.
.
..
.
.
.
.
.....
.
.....
.
.
.
..
......
. .
.
.....
. ........
. ........
. .
.
.
. .
. ...
..............................................................................................
.
................................................................................................
. .
..

mos a2 + b2 + 2ab = (a + b)2 = 502 = 2 500. Se x denota o comprimento da diagonal, o


Teorema de Pitgoras arma que x2 = a2 + b2 , portanto, x2 + 2A = x2 + 2ab = 2 500.
1
Conclumos que 2A = 2 500 x2 , ou seja, A = 1 250 x2 a expresso da rea do
2
retngulo em funo da diagonal x.
36. Valor de uma quadrtica A opo correta (d).
Elevando ao quadrado ambos os lados da igualdade x + y =
x2 + 2xy + y 2 = (x + y)2 = 82 = 64. Como xy = 15, conclumos que

8, obtemos

x2 + 6xy + y 2 = (x2 + 2xy + y 2 ) + 4xy = 64 + 4 15 = 124.


37. ngulos em funo de x A opo correta (c).
Completamos a gura marcando os ngulos e , lembrando que ngulos
opostos pelo vrtice so iguais. Lembremos que a soma dos ngulos internos
de um tringulo 180 . Olhando para o
a
3x
tringulo mais esquerda, vemos que

5x

b
2x

b
6x

3x + 4x + = 180 .

4x

Segue que = 180 7x. Considerando o tringulo do meio, temos


(180 7x) + 5x + = 180 .

Conclumos que = 2x. Finalmente, do tringulo da direita, temos que


+ 2x + 6x = 180 , ou seja, 2x + 2x + 6x = 180 . Assim, x = 18 .
192

OBMEP 2010

Solues do Nvel 2
38. Operao diferente A opo correta (c).
Pela denio, obtemos

2226
22 + 23 + 24 + 25 + 26
120
=
=
= 8.
46
4+5+6
15

39. Taxi caro A opo correta (c).


Como a bandeirada xa, temos 10,00 2,50 = 7,50 reais a serem gastos apenas
com os metros rodados. Cada trecho de 100 metros rodado custa R$ 0,10, ento com
R$ 7,50 posso fazer uma corrida de (7,50)/(0,10) = 750/10 = 75 trechos de 100 metros
cada um, ou seja, 75100 = 7 500 metros. Como 1 quilmetro tem 1 000 metros, segue
que, com R$ 10,00, posso pagar uma corrida de at 7 500 metros, ou 7,5 quilmetros.
40. Mltiplos de 3 ou 4 A opo correta (d).
Para encontrar o nmero de mltiplos de 3 compreendidos entre 1 e 601, basta usar o
algoritmo da diviso e observar que 601 = 200 3 + 1. Isso mostra que 3 1, 3 2,
. . . , 3 200 so os mltiplos de 3 entre 1 e 601, ou seja, temos 200 desses mltiplos.
Do mesmo modo, vemos que existem 150 mltiplos de 4 entre 1 e 601. Nesse total de
200 + 150 = 350, alguns nmeros aparecem contados duas vezes, pois so mltiplos de
3 e de 4 ao mesmo tempo; por exemplo, foram includos 12, 36 e 60 nos 200 mltiplos de
3 e tambm nos 150 mltiplos de 4. Lembre que os mltiplos de 3 e de 4 so, tambm,
mltiplos de 12. O mesmo argumento usado acima mostra que temos 50 mltiplos de
12 entre 1 e 601. Logo, o nmero de mltiplos de 3 ou 4 entre 1 e 601 350 50 = 300.
41. Lados de um paraleleppedo A opo correta (b).
240
Soluo 1: De xyz = 240, segue que xy =
. Substituindo em xy+z = 46, obtemos
z
240
+ z = 46, ou seja, z 2 46z + 240 = 0. As razes dessa equao so nmeros cuja
z
soma 46 e cujo produto 240, e fcil vericar que essas razes so e 6 e 40. Logo,
240
em
z = 6 ou z = 40. De maneira completamente anloga, a substituio de yz =
x
x + yz = 64 nos leva a x = 4 ou x = 60.
240
Agora, de xyz = 240, segue que y =
. Como y um nmero inteiro, ento xz
xz
um divisor de 240. De x = 4 ou x = 60 e z = 6 ou z = 40 segue que as possibilidades
para xz so
4 6 = 24, 4 40 = 160, 60 6 = 360, 60 40 = 2 400.
x

Vemos que s podemos ter x = 4 e z = 6, pois em qualquer outro caso o produto xz


240
240
no um divisor de 240. Segue que y =
=
= 10, donde
xz
46
x + y + z = 4 + 10 + 6 = 20.
Soluo 2: Somando xy + z = 46 e x + yz = 64, obtemos
(x + z)(y + 1) = (x + z)y + (x + z) = xy + z + x + yz = 46 + 64 = 110
OBMEP 2010

193

Solues do Nvel 2
e vemos que y + 1 um divisor de 110. Logo, temos as possibilidades
y + 1 = 1, 2, 5, 10, 11, 22, 55 e 110,
ou seja, y = 0, 1, 4, 9, 10, 21, 54 e 109. Por outro lado, y um divisor de 240, porque
xyz = 240 e, alm disso, y positivo, que nos deixa com as nicas possibilidades
y = 1, 4 e 10. Examinemos cada caso de y.
Se y = 1, ento 110 = (x + z)(y + 1) = (x + z) 2, portanto, x + z = 55. Como
tambm 46 = xy + z = x + z, esse caso y = 1 no possvel.
Se y = 4, ento 110 = (x + z)(y + 1) = (x + z) 5, portanto,
x + z = 22. Mas 240 = xyz = 4xz, portanto, xz = 60. Podemos vericar
(por exemplo, com uma lista de divisores de 60 ou, ento, resolvendo a equao
w2 22w + 60 = 0) que no h valores inteiros positivos de x e z que veriquem
essas duas condies x + z = 22 e xz = 60. Logo, esse caso y = 4 tambm no
possvel.
Se y = 10, ento 110 = (x + z)(y + 1) = (x + z) 10, portanto, x + z = 11. Mas
240 = xyz = 10xz, portanto, xz = 24. Podemos vericar (por exemplo, com uma
lista de divisores de 24 ou, ento, resolvendo a equao w2 11w + 24 = 0) que
os nicos valores inteiros positivos de x e z que veriquem essas duas condies
x + z = 11 e xz = 24 so x = 4 e z = 6.
Assim, a nica possibilidade x = 4, y = 10 e z = 6, com o que x + y + z = 20.
42. Pontos da reta A opo correta (b).
Notamos que a e b so nmeros maiores do que 1/2 e menores do que 1. Portanto, a +b
um nmero maior do que 1 e menor do que 2. Logo, a + b s pode ser representado
por m. Como a < b, segue que a b negativo e, portanto, s pode ser representado
por q. Quanto ao produto ab, notamos primeiro que, como a e b so positivos, seu
produto positivo. Por outro lado, temos b < 1 e a > 0, donde ab < a. Assim, o nico
nmero que pode representar ab p.
43. Velocidades A opo correta (d).
O menor tempo de percurso obtido quando se percorre o maior trecho com a maior
velocidade e o menor trecho com a menor velocidade. J o maior tempo obtido
quando se percorre o maior trecho com a menor velocidade e o menor trecho com a
maior velocidade. Assim, o tempo total gasto pelo piloto nos trs trechos de, no
240 300 400
240 300 400
mnimo,
+
+
= 15 horas e de, no mximo,
+
+
= 17 horas.
40
75
80
800
75
400
44. Comprimento de diagonal A opo correta (b).
Primeiro notamos que os tringulos AP S e CQR so congruentes, pois tm os trs ngulos iguais (um deles sendo reto)
e tambm um de seus lados (P S = QR). Do mesmo modo,
os tringulos BP Q e DRS tambm so congruentes. Sejam AP = x e BP = y. Ento a rea do tringulo AP S
1
1 x2 e a do tringulo BP Q 2 y 2 e a rea cortada foi de
2
1
1
2 2 x2 + 2 y 2 = x2 + y 2 .
OBMEP 2010

P y B
y
Q

S
D

194

Solues do Nvel 2
Assim, estabelecemos que x2 + y 2 = 200. Agora notamos que P R a hipotenusa do
tringulo retngulo P SR. Para calcular P R, basta saber o comprimento dos catetos
P S e RS. Mas P S a hipotenusa do tringulo retngulo AP S e do Teorema de
Pitgoras segue que (P S)2 = (AS)2 + (AP )2 = x2 + x2 = 2x2 . Do mesmo modo,
obtemos (RS)2 = 2y 2 . Logo,
(P R)2 = (P S)2 + (RS)2 = 2x2 + 2y 2 = 2(x2 + y 2 ) = 2 200 = 400,

ou seja, P R = 400 = 20 m.
45. Diviso de nmeros grandes claro que com nmeros to grandes, o objetivo
da questo no efetuar a diviso. Em vez disso, decompomos o nmero em partes
convenientes.
123 456 123 456 = 123 456 000 000 + 123 456 = 123 456 1 000 000 + 123 456
= 123 456 (1 000 000 + 1) = 123 456 1 000 001
Logo, 123 456 123 456 1 000 001 = 123 456.
46. Refrigerante no cinema A opo correta (c).
A economia teria sido equivalente a seis refrigerantes, permitindo a Joozinho mais um
cinema e mais um refrigerante. Logo, o ingresso do cinema cinco vezes o valor do
refrigerante.
47. Diviso de potncias A opo correta (c).
Soluo 1:

5050
(2 52 )50
250 5100
=
=
= 250 550 = (22 52 )25 = 10025 .
2525
(52 )25
550

Soluo 2:

(2 25)50
250 2550
5050
=
=
= 225 225 2525 = 10025 .
2525
2525
2525

48. Palitos de dois tamanhos A opo correta (a).


A quantidade de palitos mnima quando o nmero de palitos de 7 cm utilizado
o maior possvel. O segmento mede 200 cm. Dividindo 200 por 7, obtemos 200 =
28 7 + 4. Portanto, se tentssemos utilizar apenas palitos de 7 cm, deveramos utilizar 29 palitos, mas ainda sobrariam 3 cm. Para que no sobrem esses 3 cm, basta
substituir 3 dos 29 palitos de 7 cm por palitos de 6 cm. Temos 26 7 + 3 6 = 200.
Logo, o nmero mnimo de palitos 26 + 3 = 29. Devemos utilizar 26 palitos de
7 cm e 3 palitos de 6 cm.
Observao: Observe que a soluo equivale a encontrar nmeros inteiros x e y tais
que 200 =
7y
+
6x
e y seja o maior possvel, onde y denota o nmero de
mltiplo de 7

mltiplo de 6

palitos de 7 cm e x o de palitos de 6 cm.


49. Maior raiz A opo correta (d).
Soluo 1: Usando a fatorao a2 b2 = (a b)(a + b), temos
0 = (x 37)2 169 = (x 37)2 132 = (x 37 13)(x 37 + 13) = (x 50)(x 24).
OBMEP 2010

195

Solues do Nvel 2
Logo, as razes so 24 e 50.
Soluo 2: Extraindo a raiz quadrada em ambos os lados de (x 37)2 = 132 , temos
x 37 = 13, ou x 37 = 13. Assim, x = 50 ou x = 24.
50. Mquina com visor A opo correta (d).
O diagrama a seguir mostra os resultados de dois algarismos que podem ser obtidos a
partir do nmero 5, apertando cada uma das duas teclas.

A
B
A

59

86

32

5
A

95

11

95

68
65

23

47

29

14
B

41
A

83

51. Quadrado mgico parcial A opo correta (e).


De acordo com a regra de quadrado mgico, temos que a soma dos
nmeros da diagonal que contm y igual soma dos nmeros da
coluna que contm y, ou seja, 26 + 14 + y = y + x + 13. Segue que
26 + 14 = x + 13, donde x = 13.

1
26

14

y
x
13

52. rea do retngulo A opo correta (e).


Soluo 1: Observemos, primeiro, que a razo entre as reas de dois retngulos que
tm a mesma base igual razo entre suas alturas. De fato, na gura esquerda,
esto representados dois retngulos que tm a mesma base b e alturas h1 e h2 .

S1

h1

16

h1

S2

h2

12

27

h2

Suas reas S1 e S2 so dadas por S1 = b h1 e S2 = b h2 , portanto,


S1
bh1
h1
=
=
.
S2
bh2
h2
196

OBMEP 2010

Solues do Nvel 2
Aplicando essa observao aos dois pares de retngulos dados (ver gura acima,
direita) e denotando por S a rea do quarto retngulo, temos
S
h1
16
4
=
=
= ,
27
h2
12
3
1
de modo que S = 3 (27 4) = 36. Assim, a rea do retngulo ABCD

12 + 16 + 27 + 36 = 91.
Soluo 2: Sejam x e y lados dos retngulos de
reas 12 e 27, respectivamente, como indicado na
gura. Logo os outros lados desses retngulos so
12/x (retngulo de rea 12), 16/x (retngulo de
rea 16) e 27/y (retngulo de rea 27), como indicado na gura.

16
8
x

16

12
8
x

12

27

h1
h2

16
12
28
Assim, o comprimento do retngulo ABCD x + y e sua largura
+
=
.
x
x
x
12
27
y
27
9
Claramente,
=
, de modo que =
= . A rea de um retngulo o produto
x
y
x
12
4
28y
28
= 28+
=
do comprimento pela largura. Logo, a rea de ABCD A = (x+y)
x
x
y
9
28 + 28 . Assim, A = 28 + 28 = 28 + 7 9 = 91.
x
4
53. Lado do quadrado
Soluo 1: Sejam x e y o maior e o menor catetos, respectivamente, do tringulo
retngulo. Como o lado do quadrado ABCD mede 3 cm, temos x y = 3. Por outro
lado, como o lado de EF GH mede 9 cm, temos x + y = 9. Resolvendo o sistema,
encontramos x = 6 e y = 3. Logo, o lado do quadrado IJKL, que a hipotenusa do

tringulo retngulo, mede 62 + 32 = 45 = 3 5 cm, pelo Teorema de Pitgoras.


H

D
C

x x-y

x+y

B
L

P
E

Soluo 2: Os quadrados IJKL e M N OP tm como lados as hipotenusas dos tringulos retngulos dados, logo tm a mesma rea.
H

Superpondo-se as duas guras e fazendo esses dois quadrados coincidirem, encontramos oito tringulos e conclumos que
8 a rea do tringulo igual rea de EF GH menos a rea
de ABCD, ou seja, igual a 92 32 = 72. Logo, a rea de
cada tringulo 9 cm2 . Da gura, temos que a rea de IJKL
igual a 4 a rea do tringulo mais a rea de ABCD, ou
seja, igual a 4 9 + 9 = 45.
OBMEP 2010

M
I

G
D

C
A
P L
E

K
O

197

Solues do Nvel 2
Logo, o lado do quadrado IJKL mede

45 = 3 5 cm.

54. Maior nmero A opo correta (d).


Lembre que, se num produto, um dos fatores zero, ento o produto tambm zero.
Temos 2 0 2 006 = 0, 2 0 + 6 = 0 + 6 = 6, 2 + 0 2 006 = 2 + 0 = 2,
2 (0 + 6) = 2 6 = 12 e 2 006 0 + 0 6 = 0 + 0 = 0. Logo, o maior nmero
2 (0 + 6) = 12.
55. Operao A opo correta (e).

Temos que descobrir qual a regra dessa operao. Note que


2 4 = 10 = 2 4 + 2, 3 8 = 27 = 3 8 + 3, 4 27 = 112 = 4 27 + 4
e 5 1 = 10 = 5 1 + 5.

Uma hiptese plausvel que a regra


a b = a b + a. Segundo essa regra, temos

que

dene

operao

seja

4 (8 7) = 4 (8 7 + 8) = 4 64 = 4 64 + 4 = 260.
56. Terceiro lado A opo correta (e).
Lembre que, num tringulo, a soma de dois lados quaisquer deve ser maior que o
terceiro lado. Como 1 + 5 no maior do que 7, o terceiro lado no pode medir 1 cm.
57. Asterisco A opo correta (e).
1

3 2

25
25
3 2

=
=

=
.
=
6
24 8 3
24
8 3
24 24
24
Logo,

25
1
4
= =
, donde 25 = 4, ou seja, = 29.
24
6
24

58. Expresses algbricas Note que a gura um retngulo formado por um quadrado
de lado a e um retngulo de lados 1,5 e a. Logo, a2 a rea do quadrado e 1,5 a a
rea do retngulo. Assim, a2 + 1,5 a representa a soma dessas duas reas, ou seja, a
rea total da gura. J 4 a + 3 = 3 a + 1,5 + a + 1,5 o permetro da gura.
59. Faixa decorativa A opo correta (d).
Soluo 1: O comprimento da hipotenusa de cada um dos cinco
tringulos retngulos issceles da faixa mede 30 5 = 6 cm. O
quadrado formado por quatro desses tringulos tem lado igual a
6 cm, portanto, sua rea 36 cm2 . Logo, cada um dos tringulos
tem 36 4 = 9 cm2 de rea. Portanto, a rea da parte sombreada
mede 9 5 = 45 cm2 .

6 cm

Soluo 2: O comprimento da hipotenusa de cada um dos cinco tringulos retngulos


issceles da faixa mede 30 5 = 6 cm. Denotando os catetos desses tringulos por x,
198

OBMEP 2010

Solues do Nvel 2
o Teorema de Pitgoras fornece 36 = x2 + x2 = 2x2 , ou seja, x2 = 18, de modo que
a rea de cada um dos cinco tringulos da faixa mede 9 cm2 . Assim, a rea da parte
sombreada mede 5 9 = 45 cm2 .
60. Bicicleta e chocolate A opo correta (c).
2 barras d 3 h
, segue que
12 bombons d 2 h

Como

1 barra d 1,5 h = 1 h 30 min


.
3 bombons d 0,5 h = 30 min

Assim, Tio me emprestar a bicicleta por 1 h 30 min + 30 min = 2 horas.


61. Retas paralelas?
Soluo 1: No tringulo BCE, temos B EC = 180 (42 + 48 ) = 90 . No
tringulo AF D, temos AF D = 180 (28 + 62 ) = 90 . Logo, as retas EC e F D
so perpendiculares reta AB, de modo que so paralelas.
Soluo 2: No tringulo ABC, temos B CA = 180 (48 + 62 ) = 70 . Portanto,
E CA = 70 42 = 28 = F DA. Logo, as retas EC e F D so paralelas, pois cortam
a reta AD segundo o mesmo ngulo.
62. Menor nmero A opo correta (b).
5
5
5
Como x > 5, temos 0 < x 1 < x < x + 1. Portanto,
< <
. Tambm
x+1
x
x1
x
x+1
5
< 1 <
<
, pois 5 < x < x + 1. Assim, dentre os nmeros 5/x,
temos
x
5
5
5/(x + 1), 5/(x 1), x/5 e (x + 1)/5, o menor 5/(x + 1).
63. rea de quadrado A opo correta (a).
Denotemos por C e L o comprimento e a largura, respectivamente, de cada um dos
quatro retngulos. O permetro de cada retngulo dado por 2(C + L). Como esse
permetro mede 40 cm, obtemos C + L = 20 cm. Observe, na gura dada, que o lado
do quadrado ST U V dado por C + L. Assim, sua rea de (C + L)2 = 202 = 400 cm2 .
64. Operando fraes
1
1 1 1
1 1 1
1
= ,
= ,
=
,
2
2 2 3
6 3 4
12
1
1
1
1
1
+
+
+
+
= 1
2
6
12
20
30

(a) 1
(b)

1
1 2

1
1
2
3

1
1
4
3

1
1
4
5

1 1
1
1 1
1
=
,
=
.
4 5
20 5 6
30
1
1
1
1
1
1
1
1
1
+ + + + ;
2
2
3
3
4
4
5
5
6

1
1
6
5

cancelando as parcelas iguais de sinais opostos, resulta que


1
1
1
1
5
1 1
+ +
+
+
=1 = .
2 6 12 20 30
6
6
1
1
1
1
1
1 1
+
+
+
+ +
, comeamos
(c) Para calcular a soma + +
2 6 12 20 30 42
999 000
observando que todos os denominadores so produtos de nmeros consecutivos,
OBMEP 2010

199

Solues do Nvel 2
iniciando em 1; usando a decomposio de cada parcela dada no item (a), obtivemos, no item (b), que
1
1
1
1
1
1
+
+
+
+
=1 .
12 23 34 45 56
6

Mais geralmente, podemos provar que

1
1
1
1
1
1
1
+
+
+
+
+ +
=1
.
12 23 34 45 56
n (n + 1)
n+1

Assim,

1 1
1
1
1
1
1
999
+ +
+
+
+ +
=1
=
= 0,999.
2 6 12 20 30
999 000
1 000
1 000
65. ngulos e permetro O tringulo BCD issceles, porque tem dois lados iguais,
BD = BC, logo B DC = B CD.
Mas DBC = B CD, portanto os trs ngulos desse
tringulo so iguais, cada um valendo 180 3 =
60 , e o tringulo BCD equiltero. Assim,
BD = BC = CD = 115 m.

A
120 m
26
30
E

120 m
26

115 m

80 60
70

226 m

60

60 115 m
D

O tringulo ABE tambm issceles, porque tem dois ngulos iguais, logo os lados
AE e AB so iguais, portanto AB = AE = 120 m. Assim, o permetro da gura mede
120 2 + 115 2 + 226 = 696 m.
66. Desigualdade racional A opo correta (c).
1
1
1 4(x 2)
9 4x
Temos
< 4
4 < 0
< 0
< 0.
x2
x2
x2
x2
Para que uma frao seja negativa, o numerador e o denominador devem ter sinais
contrrios.
1o Caso: 9 4x > 0 e x 2 < 0. Devemos ter x < (9/4) e x < 2. Portanto, x < 2,
pois sendo menor do que 2, automaticamente x ser menor do que 9/4. Conclumos
que todo x < 2 satisfaz a desigualdade.
2o Caso: 9 4x < 0 e x 2 > 0. Devemos ter x > (9/4) e x > 2. Portanto, x > (9/4),
pois sendo maior do que 9/4, automaticamente x ser maior do que 2. Conclumos que
todo x > (9/4) satisfaz a desigualdade.
Juntando os dois casos, conclumos que x satisfaz a desigualdade se, e s se, x < 2 ou
x > (9/4).
67. Desigualdade dupla A opo correta (e).
Como os nmeros que aparecem so todos positivos, podemos elev-los ao quadrado
mantendo o sentido das desigualdades, obtendo
2 000 2 000 = 2 0002 < n(n + 1) < 2 0052 = 2 005 2 005.
Observe que n e n + 1 so inteiros consecutivos, portanto, as nicas opes so as
seguintes.
200

OBMEP 2010

Solues do Nvel 2
2 0002 < 2 000 2 001 < 2 0052

2 0002 < 2 001 2 002 < 2 0052

2 0002 < 2 002 2 003 < 2 0052

2 0002 < 2 003 2 004 < 2 0052

2 0002 < 2 004 2 005 < 2 0052


Logo, temos cinco possibilidades para n, a saber, 2 000, 2 001, 2 002, 2 003 e 2 004.
68. Dimetro do crculo Observe que OC um raio do crculo. Temos que OC =
AB = 5 cm, por serem as diagonais do retngulo OACB. Logo, o dimetro mede
10 cm.
69. Falta um ngulo A opo correta (d).
Lembre que a soma dos ngulos internos de um tringulo 180 . Do tringulo ST U,
temos que T SU = 180 (75 + 30 ) = 75 . Logo, esse tringulo issceles (por ter
dois ngulos iguais) e, portanto, T U = SU. Como T U = SV, segue que SU = SV.
Assim, o tringulo SU V tambm issceles e, portanto,
1
S V U = (180 50 ) = 65 .
2
70. Caf, bolo e gato Vamos listar os eventos ocorridos e contar o tempo gasto em
cada um. A primeira atividade foi colocar o gato fora da casa, logo nossa lista comea
com essa atividade e o tempo contado a partir dela.
Atividade
Gato fora de casa
Bolo no forno
Fazer o caf
Despertador toca
Gato entra em casa
Acabar de tomar o caf
Telefone toca
Desligar o telefone

Tempo depois que o gato


foi posto fora de casa
0 minutos
10 minutos
10 + 6 = 16 minutos
35 + 10 = 45 minutos
45 5 = 40 minutos
40 + 3 = 43 minutos
16 + (40 16) 2 = 28 minutos
28 + 5 = 33 minutos

Podemos, agora, dar as respostas.


(a) s 3h59min desliguei o telefone, o que ocorreu 33 minutos depois de colocar o
gato fora de casa. Como 59 33 = 26, coloquei o gato para fora s 3h26min.
(b) O despertador toca 45 minutos aps colocar o gato fora de casa.
(c) O gato j estava fora de casa por 28 minutos quando o telefone tocou.
Podemos saber exatamente a hora em que ocorreu cada atividade, conforme a tabela
seguinte.
OBMEP 2010

201

Solues do Nvel 2
Atividade
Gato fora de casa
Bolo no forno
Fazer o caf
Despertador toca
Gato entra em casa
Acabar de tomar o caf
Telefone toca
Desligar o telefone

Tempo depois que o gato


foi posto fora de casa
0 min
10 min
10 + 6 = 16 min
35 + 10 = 45 min
45 5 = 40 min
40 + 3 = 43 min
16 + (40 16) 2 = 28 min
28 + 5 = 33 min

Horrio
59 33 =
26 + 10 =
26 + 16 =
26 + 45 =
26 + 40 =
26 + 43 =
26 + 28 =
26 + 33 =

3h26min
3h36min
3h42min
4h11min
4h06min
4h09min
3h54min
3h59min

71. Muitos ngulos Na gura I, temos 63 + 18 + 95 = 176 , que menor do que


180 . Logo, esta gura est errada.
Na gura II, temos 112 + 72 = 184 , que maior do que 180 . Logo, esta gura est
errada.
Na gura III, temos 44 + 45 + 62 + 29 = 180 . Esta gura est correta.
72. Sinal de produto e de quociente
a
Como > 0 e 5 > 0, obtemos a > 0.
5
b
> 0, segue que b > 0, portanto, b < 0.
Como a > 0, temos 7a > 0. Como
7a
11
Como
> 0 e 11 > 0, obtemos abc > 0. Como b < 0 < a, segue que c < 0.
abc
18
> 0 e 18 < 0, obtemos abcd < 0. Como abc > 0, segue que d < 0.
Como
abcd

73. Sinais e radicais


Temos 3 11 = 9 11 = 99 . Como 100 > 99, obtemos

10 = 100 > 99 = 3 11, portanto, 10 3 11 > 0 e 3 11 10 < 0. Analogamente,

temos 10 26 = 100 26 = 2 600 . Como 2 601 > 2 600, obtemos

51 = 2 601 > 2 600 = 10 26 ,

portanto, 51 10 > 0 e 10 26 51 < 0.


26
Finalmente, 182 = 324 < 325 = 25 13
garante que 18 < 5 13 , de modo que 18 5 13 < 0.

Os nmeros negativos so (b) 3 11 10, (c) 10 26 51 e (e) 18 5 13 .


74. ngulos entre retas Temos 80 + y = 180 , portanto, y = 100 . Como as retas r
e s so paralelas, segue que 60 + x + 80 = 180 , donde x = 40 .
75. Variao de temperatura A variao de temperatura a diferena entre a mxima
e a mnima. Completamos a tabela dada com as variaes, como segue.
Dia
2a -feira
3a -feira
4a -feira
5a -feira
6a -feira
202

Temperatura
mxima,
em C
7
0
2
9
13

Temperatura
mnima,
em C
12
11
15
8
7
OBMEP 2010

Variao
da temperatura,
em C
7 (12) = 7 + 12 = 19
0 (11) = 0 + 11 = 11
2 (15) = 15 2 = 13
9 (8) = 9 + 8 = 17
13 (7) = 13 + 7 = 20

Solues do Nvel 2
Logo, a maior variao da temperatura ocorreu na sexta-feira.
76. Ordenando fraes A opo correta (d).
Lembre que a ordem entre fraes constitudas de inteiros positivos determinada pelo produto cruzado dos
a
c
inteiros, ou seja, < equivale armao ad < bc.
b
d
Desse modo, temos
1
2
4
3
4
5
1
< < < < <1< < ,
6
4
5
7
4
3
2
j que 4 < 6, 5 < 8, 14 < 20, 16 < 21, 3 < 4 (duas vezes) e 8 < 15, respectivamente.
Assim, 1/6 e 1/4 cam esquerda de 2/5, 4/3 e 5/2 cam direita de 3/4 e s 4/7
ca entre 2/5 e 3/4.
77. Frao de rea Observe que a regio em cinza na
gura dada tem a mesma rea que a do enunciado.
Como todos os retngulos tm a mesma largura, o retngulo maior est dividido em quatro partes iguais por
segmentos paralelos ao seu comprimento.
Assim, a regio cinza representa uma quarta parte do retngulo maior.
78. Uma a mais!
5
:
2

(a) duas fraes cuja diferena


5
5

4
4

5 5
10
5
+ =
= .
4 4
4
2

(b) duas fraes cujo produto

5
:
2

10 7
10
5
10 14

=
=
= .
7
8
7
4
4
2

(c) duas fraes cuja soma


1
17
+
6
3

5
:
2

17 2
15
5
17 1
=
=
= .
6
3
6
6
6
2

(d) duas fraes cujo quociente

5
:
2

5 2
5 3
5
= = .
3 3
3 2
2

OBMEP 2010

203

Solues do Nvel 2
3
Logo, a frao que est sobrando .
2
79. Qual o ngulo? A opo correta (b).

K
b T
L

180-b

Sejam T SM = x, S KT = y, K LS = , K T S = . O tringulo KLM issceles


porque tem dois lados iguais; consequentemente, seus ngulos da base so iguais, isto ,
K M S = K LS = . Analogamente, o tringulo KST tambm issceles e, portanto,
K ST = K T S = . Usaremos, agora, que a soma dos ngulos internos de um tringulo
180 . Acompanhe na gura:
No tringulo SM T temos x + + 180 = 180 , portanto, x = .

No tringulo KLM temos + + 30 + y = 180 , portanto, y = 150 2.


No tringulo KST temos + + 150 2 = 180 , portanto, = 15 .

Assim, x = 15 .
80. Operao circular Colocando x num dos crculos e aplicando a sucesso de opex+2
raes obtemos x =
+ 1, donde x = 4.
2
4
x+2+1
2

x
4

+2

x+2

+1

x+2
2

x+2
6

81. Pratos e copos Sejam c e p o nmero de copos e pratos que Iara pode comprar.
Observe que certamente c e p so nmeros inteiros e alm disso, como ela quer comprar,
no mnimo, quatro pratos e seis copos, temos p 4 e c 6. Como cada copo custa
R$ 2,50 e cada prato custa R$ 7,00, o custo de c copos e p pratos 2,5 c + 7 p. Mas
Iara s dispe de R$ 50,00, portanto, 2,5 c + 7 p 50. Assim, devemos encontrar dois
nmeros inteiros c e p que satisfaam p 4, c 6 e 2,5 c + 7 p 50.
Se Iara comprar quatro pratos, sobram 50 4 7 = 22 reais para os copos. Como
22 = 8 2,50 + 2, ela pode comprar mais oito copos (sobrando R$ 2,00).

Se Iara comprar cinco pratos, sobram 50 5 7 = 15 reais para os copos. Como


15 = 6 2,50, ela pode comprar mais seis copos.

204

OBMEP 2010

Solues do Nvel 2
Se Iara comprar seis pratos, sobram 50 6 7 = 8 reais para os copos, o que lhe
permite comprar apenas trs copos, que no o que ela quer.
Logo, Iara pode comprar quatro pratos e oito copos, ou cinco pratos e seis copos.
82. Desigualdades de inteiros A opo correta (c).
Somando 1 aos trs membros das duas desigualdades 5 < x 1 5 obtemos
5 + 1 < x 1 + 1 5 + 1, ou seja, 4 < x 6. Os valores inteiros de x que
satisfazem essas duas desigualdades so 3, 2, 1, 0, 1, 2, 3, 4, 5 e 6.
83. Nove quadrados A opo correta (d).

O lado de A mede 1 = 1 cm e o de B mede 81 = 9 cm. Agora temos:


Lado de G = lado de de B lado de A = 9 1 = 8 cm.
Lado de C = lado de B + lado de A = 1 + 9 = 10 cm.
Lado de F = lado de G lado de A = 8 1 = 7 cm.

Lado de H = lado de G + lado de F = 8 + 7 = 15 cm.

Lado de B + lado de C = lado de G + lado de F + lado de E, portanto,


9 + 10 = 8 + 7 + lado de E, ou seja, lado de E = 4 cm.
Lado de D = lado de C + lado de E = 10 + 4 = 14 cm.

Lado de I = lado de E + lado de D = 18 cm.


Finalmente, a rea de I mede 182 = 324 cm2 .

84. Muitas medalhas Denotemos por A, B, C e D o nmero de medalhas ganhas por


Andr, Bruno, Celina e Dalva, respectivamente. Ento A, B, C e D so inteiros no
negativos e A + B + C + D = 21. Temos que
Bruno ganhou o dobro de Celina, ou seja, B = 2 C.

Dalva ganhou trs a mais do que Bruno, ou seja, D = B + 3 = 2 C + 3.

Assim, atribuindo qualquer valor a C, automaticamente sabemos os valores de B e D.


Mas A + 2 C + C + 2 C + 3 = A + B + C + D = 21. Logo, A + 5 C = 18 e, portanto,
podemos expressar tambm A em termos de C, com A = 18 5 C. Observe que C 3,
pois se C = 4, ento A = 18 20 = 2, o que impossvel.
Como C um inteiro maior do que ou igual a 0 e menor do que 4, temos apenas as
possibilidades seguintes.
C
0
1
2
3

A
18
13
8
3

B
0
2
4
6

D
3
5
7
9

Como Andr foi o que recebeu mais medalhas, C = 3 no serve. O problema tem,
ento, trs possveis solues, listadas a seguir.
OBMEP 2010

205

Solues do Nvel 2
A
18
13
8

B
0
2
4

C
0
1
2

D
3
5
7

85. As somas so quadrados Com nmeros de 1 a 15, a soma de dois adjacentes , no


mnimo, 3 e, no mximo, 29. Os quadrados de nmeros inteiros de 3 a 29 so, apenas,
4, 9, 16 e 25. Veriquemos quais so os nmeros de 1 a 15 que podem ser adjacentes,
ou vizinhos.
Nmeros
Vizinhos
possveis

1
3
8
15

2
7
14

3
1
6
13

4
5
12

5
4
11

6
3
10

7
2
9

8
1

9
7

10
6
15

11
5
14

12
4
13

13
3
12

14
2
11

15
1
10

Os nmeros 8 e 9 s tm, cada um, apenas um possvel vizinho, logo eles devem ser
colocados no incio e no m da la, seguidos de seus nicos vizinhos.
8

Sobram os nmeros 2, 3, 4, 5, 6, 10,11 12, 13, 14 e 15. Na tabela de vizinhos, vemos


que, ao lado do 7, s podemos colocar o 2 e, ao lado do 2, s o 14. Temos, ento,
8

14

Consultando a tabela de vizinhos e os nmeros que sobram, chegamos resposta.


8

15

10

13

12

11

14

Veja, a seguir, a soluo passo a passo.


Formao da linha em cada etapa
?
?
?
?
?
?
?

14

8
8
8
8
8
8

1
1
1
1
1
1

?
?
?
?
?
15

?
?
?
?
?
10

?
?
?
?
?
6

?
?
?
?
3
3

?
?
?
13
13
13

?
?
12
12
12
12

?
4
4
4
4
4

5
5
5
5
5
5

11
11
11
11
11
11

14
14
14
14
14
14

2
2
2
2
2
2

7
7
7
7
7
7

9
9
9
9
9
9

Sobram
2, 3, 4, 5, 6, 10, 11, 12,
13, 14, 15
3, 4, 5, 6, 10, 11, 12, 13,
14, 15
3, 4, 5, 6, 10, 11, 12, 13,
15
3, 4, 6, 10, 12, 13, 15
3, 6, 10, 12, 13, 15
3, 6, 10, 13, 15
3, 6, 10, 15
6, 10, 15
Resposta

86. rea de uma regio Lembre que a rea de um tringulo


1
base altura,
2
onde a altura relativa base escolhida. No tringulo AEB, temos base = AB =
comprimento do retngulo e a altura relativa a essa base BC = largura do retngulo.
1
Logo, AB BC = 24 e AB BC = 48. Logo, a rea do retngulo 48 cm2 . Assim,
2
a rea pedida 48 (24 + 13) = 48 37 = 11 cm2 .
206

OBMEP 2010

Solues do Nvel 2
87. Potncias de 10 A opo correta (c).
105 (102 )2 103
105 104 103
0,00001 (0,01)2 1 000
=
=
0,001
103
103
105+(4)+3
106
=
= 3 = 106(3) = 103
103
10
88. Diferena de quadrados A opo correta (d).
Como (x + y)2 = x2 + 2xy + y 2 e (x y)2 = x2 2xy + y 2 , temos

20 = (x + y)2 (x y)2 = x2 + 2xy + y 2 x2 + 2xy y 2 = 4xy,

portanto xy = 5.
89. Um quadriltero Para que ABCD seja um paralelogramo, seus lados devem ser
dois a dois paralelos, isto , AB||CD e AD||BC. Como DAB + ABC = 180 , as retas
AD e BC so paralelas. Tambm as retas AB e DC so paralelas, pois temos dois
ngulos alternos internos de 45 entre essas retas. Assim, ABCD um paralelogramo.
90. Sexta-feira treze Como os dias da semana se repetem a cada 7 dias, a diferena
entre os dias da semana dada pelo resto ao dividir o nmero de dias transcorridos
por 7. Na tabela abaixo, temos
(a) na primeira linha, o nmero de dias entre o dia 13 de um ms e o dia 13 do ms
seguinte;
(b) na segunda linha, o resto obtido quando dividimos esse nmero por 7;
(c) na terceira linha, o resto obtido quando dividimos por 7 o nmero de dias entre o
13 de janeiro e o 13 do ms correspondente; assim, esse nmero obtido somando
os resultados obtidos na primeira linha, desde janeiro at o ms correspondente,
calculando, depois, o resto da diviso por 7.
J-F
31
3
3

F-M
28
0
3

M-A
31
3
6

A-M
30
2
1

M-J
31
3
4

J-J
30
2
6

J-A
31
3
2

A-S
31
3
5

S-O
30
2
0

O-N
31
3
3

N-D
30
2
5

Os valores iguais na ltima linha, signicam que, nesses meses, o dia 13 caiu no mesmo
dia da semana. Em particular, a ltima linha nos diz que 13 de fevereiro, 13 de maro
e 13 de novembro correspondem ao mesmo dia da semana. Assim, no mximo, temos
trs sextas-feiras treze.
No caso de trs sextas-feiras treze num mesmo ano, o 13 de janeiro ocorreu 3 dias antes
de sexta-feira, isto , numa tera-feira, e o dia 10 de janeiro aconteceu 3 dias antes,
isto , num sbado.
Observao: Note que uma sexta-feira 13 ocorre apenas quando o primeiro dia do
ms cair num domingo. Assim, uma outra maneira, talvez mais simples, de resolver o
problema determinar o nmero mximo de vezes em que o primeiro dia do ms caia
num domingo num ano que no seja bissexto.
OBMEP 2010

207

Solues do Nvel 2
91. Tringulos com lados inteiros A opo correta (b).
Para que trs nmeros a, b e c sejam os comprimentos dos lados de um tringulo, cada
um deles deve ser maior do que a diferena e menor do que a soma dos outros dois.
Sejam a b c os comprimentos dos lados do tringulo, de modo que c < a + b.
Agora, somando c a ambos os membros, temos 2c < a + b + c = 12, ou seja, 2c < 12,
de modo que c < 6. Alm disso, como 3c a + b + c = 12, temos que c 4, de modo
que 4 c < 6.
No caso c = 5, temos que a + b = 7 e os possveis valores de a e b so a = 2 e b = 5,
ou a = 3 e b = 4. No caso c = 4, temos que a + b = 8 e, portanto, a nica soluo
a = b = 4. Concluso: temos 3 possveis tringulos.

92. Festa de aniversrio Denotemos por m o nmero de mas e por p o nmero de


peras que Ana compra, de modo que o peso que ela leva na sacola 300m+200p gramas.
Como a sacola aguenta, no mximo, 7 000 gramas, temos 300m + 200p 7 000, que
equivale a 3m + 2p 70. Como as peras pesam menos, Ana deve levar uma quantidade
maior de peras e, portanto, uma menor de mas. Como Ana quer fazer tortas de
ambas frutas, precisa levar pelo menos 1 ma.
Se ela levar uma ma, temos 2p 70 3 = 67, portanto p 33,5, o que signica que
Ana pode levar mais 33 peras, num total de 34 frutas. Se ela levar duas mas, temos
2p 70 6 = 64, portanto p 32, o que signica que Ana pode levar mais 32 peras,
novamente num total de 34 frutas. Se ela levar trs mas, temos 2p 70 9 = 61,
portanto p 30,5, o que signica que Ana s pode levar mais 30 peras, num total de
33 frutas.
Nas contas feitas, vemos que, a cada ma que Ana levar a mais, ela precisa comprar
1,5 peras a menos. Assim, se levar mais do que duas mas, nunca poder levar mais
do que 30 peras, num total sempre inferior a 34 frutas.
Concluso: o nmero mximo de frutas que Ana pode levar 34 frutas (ou uma ma
e 33 peras, ou duas mas e 32 peras).
93. Os dois quadrados Se a a medida do lado do quadrado maior e b a medida
do lado do quadrado menor, ento sabemos do enunciado que a2 = b2 + 2 001 . Logo
2 001 = a2 b2 = (a + b)(a b) e, como a e b so nmeros inteiros, temos que a + b
e a b so divisores de 2 001. Mas, 2 001 = 3 23 29, portanto, temos 4 possveis
formas de fatorar 2 001 em dois fatores, a saber,
(a + b)(a b) = 2 001 1 = 667 3 = 87 23 = 69 29.
Como (a + b) + (a b) = 2a, resulta
2 001 + 1
= 1 001;
2
667 + 3
= 335;
(b) a + b = 667 e a b = 3, caso em que a =
2
87 + 23
(c) a + b = 87 e a b = 23, caso em que a =
= 55;
2
69 + 29
(d) a + b = 69 e a b = 29, caso em que a =
= 49.
2
(a) a + b = 2 001 e a b = 1, caso em que a =

208

OBMEP 2010

Solues do Nvel 2
Assim, as possibilidades para o lado maior so 1 001, 335, 55 e 49 cm.
94. A multiplicao O maior quadrado no retngulo de 85 135 aquele de 85 85.
Sobra, ento, um retngulo de 50 85, em que o maior quadrado mede 50 50.
Continuando assim, obtemos
85 135 = 852 + 502 + 352 + 152 + 152 + 52 + 52 + 52 .
.
.......................................................................................................................................................................................................
.
.
.
......................................................................................................................................................................................................
.
.
.
.
.
.
. 2. 2. 2.
.
.
.
.
.
.
.
.
.
.
.
.
.
.5 .
.
.
.
.
.................5...
.
.
.........5..........
.
.
.
.
.
.
. ... . ... .
.
.
.
.
.
.
.
.
.
.
.
.
.
.
.
.
.
.
.
.
.
.
.
.
.
.
.
.
.
.
.
.
.
.
.
.
.
.
.
.
.
.
.
.
.
.
.
.
.
.
.
.
.
.
.
2 .
.
.
.
.
.
.
.
.
.
.
.
.
.
.
.
.
.
.
.
.
.
.
.
.
.
.
.
.
.
.
.
.
.
.
.
.
.
.
.
.
2
.
.
.
.
......................
.
.
.
.
.
.
.
.
.
.
.
.
.......................
.
.
.
.
.
.
.
.
.
.
.
.
.
.
.
.
.
.
.
.
.
.
.
.
.
.
.
.
.
.
.
.
.
.
.
.
.
.
.
.
.
.
.
.
.
.
.
.
.
.
.
.
.
.
.
.
2 .
.
.
.
.
.
.
.
.
.
.
.
.
.
.
.
.
.
.
.
.
.
.
.
.
.
.
.
.
.
.
.
.
.
.
.
.
.
.
.
.
.
.
.
.
.
.
.........................................................................
.
.
.
..........................................................................
.
.
.
.
.
.
.
.
.
.
.
.
.
.
.
.
.
2
.
.
.
.
.
.
.
.
.
.
.
.
.
.
.
.
.
.
.
.
.
.
.
.
.
.
.
.
.
.
.
.
.
.
.
.
.
.
.
.
.
.
.
.
.
.
.
.
.
.
.
.
.
.
.
.
.
.
.
.
.
.
.
.
.
.
.
.
.
.
.
.
.
.
.
.
.
.
.
.
.
.
.
.
.
.
.
.
.
.
.
.
.
.
.
.
.
.
.
.
.
.
.
.
.
.
.
.
.
.
.
.
.
.
2
.
.
.
.
.
.
.
.
.
.
.
.
.
.
.
.
.
.
.
.
.
.
.
.
.
.
.
.
.
.
.
.
.
.
.
.
.
.
.
.
.
.
.
.
.
.
.
.
.
.
.
.
.
.
.
.
.
.
.
.
.
.
.
.
.
.
.
.
.
.
.
.
.
.
.
.
.
.
.
.
.
.
.
.
.
.
.
.
.
.
.
.
.
.
.
.
.
.
.
.
.
.
.
.
.
.
.
.
.
.
.
.
.
.
.
.
.
.
.
.
.
.
.
.
.
.
.
.
.
.
.
.
.
.
.
.
.
.
.
.
.
.
.
.
.
.
.
.
.
.
.
......................................................................................................................................................................................................
.......................................................................................................................................................................................................
.

35

15

15

85

50

95. Expresso fracionria A opo


xy
x
y
Soluo 1: Temos
=
x
x
x
portanto,
xy
=
x
Soluo 2: Se

correta (c).
y
x
y
1
= 1 e, como
= 2, resulta que
= e,
x
y
x
2
x y
1
1
=1 = .
x x
2
2

xy
2y y
y
1
x
= 2, ento x = 2y e, portanto,
=
=
= .
y
x
2y
2y
2

96. Diferena e soma de quadrados


(a) Como a2 b2 = (a + b)(a b), temos
1 6782 1 6772 = (1 678 + 1 677)(1 678 1 677) = 3 355.
(b) Como (a + b)2 = a2 + 2ab + b2 , temos
1 0012 + 1 0002 = (1 000 + 1)2 + 1 0002 = 1 0002 + 2 000 + 1 + 1 0002 =
= 2 1 0002 + 2 001 = 2 002 001.
(c) Como (a b)2 = a2 2ab + b2 , temos
19 9992 = (20 000 1)2 = (2 104 )2 4 104 + 1 =
= 4 108 4 104 + 1 = 399 960 001.
OBMEP 2010

209

Solues do Nvel 2
(d) Colocando em funo de 2 000, temos
2 0012 + 2 0022 + 2 0032 = (2 000 + 1)2 + (2 000 + 2)2 + (2 000 + 3)2 =
= 3 2 0002 + 12 2 000 + 14 = 12 024 014.
97. Um queijo triangular Para dividir o queijo em 5 partes iguais, suciente dividilo em 5k partes iguais e dar k partes a cada um. Observe que se dividirmos cada lado
para servir de base de dois tringulos equilteros menores, obtemos 4 = 22 tringulos
menores no total; dividindo cada lado para servir de base de trs ou quatro tringulos
equilteros menores, obtemos 9 = 32 ou 16 = 42 tringulos no total. Dessa maneira,
a menor diviso em 5k tringulos menores alcanada quando 5k um quadrado, ou
seja, quando k = 5. Essa partio mostrada na gura, em que o queijo foi partido
em 25 = 52 = 5k tringulos.

98. Notas de Matemtica Devemos encontrar os valores dos smbolos na soma

As duas notas so nmeros de dois algarismos e a soma deles tm trs algarismos, de


modo que a soma precisa ser maior do que 100 e menor do que 200. Assim, temos que
= 1. Mas, Cludia obteve 13 pontos a mais do que Joo, portanto,

+ 1 3
1
Agora, como a soma de e 3 termina em 1, temos que = 8 e, portanto,
Assim, as notas de Cludia e Joo so, respectivamente, 81 e 68.
99. Operao com raiz quadrada A opo correta (c).
210

OBMEP 2010

= 6.

Solues do Nvel 2
Observe que, denotando por A a expresso dada, temos

A2 = ( 6 + 2)( 3 2)
3+2

= ( 6 + 2)2 ( 3 2)2
3+2

= ( 6 + 2)2 ( 3 2)2 ( 3 + 2)

= ( 6 + 2)2 ( 3 2) ( 3 2)( 3 + 2)

= (6 + 2 12 + 2)( 3 2) ( 3)2 22

= (6 + 2 12 + 2)( 3 2)(1)

= (8 + 4 3)(2 3) = 4(2 + 3)(2 3)

= 4(22 ( 3)2 ) = 4 1 = 4.

Assim, A2 = 4 e, portanto, A pode ser 2 ou 2. Como 3 2 negativo e os outros


dois fatores de A so positivos, temos que A deve ser negativo, ou seja, A = 2.
100. Para a escola de bicicleta Seja t o tempo que Ctia gasta pedalando a 20 km/h.
Pedalando a 10 km/h, ela faz o percurso no dobro do tempo que pedalando a 20 km/h,
isto 2t. No entanto, como ela demora 45 minutos a mais, temos 2t t = 45, de modo
que t = 45 min. Logo, diariamente, ela sai da escola 45 minutos antes das 16h30m,
isto , s 15h45m, e o percurso at sua casa, que feito em 45 min a 20 km/h, tem
3
20 = 15 km. Para sair da escola s 15h45m e chegar em casa s 17h, ela deve
4
percorrer esses 15 km entre a escola e sua casa em 1h15m, o que corresponde a 5/4 de
hora. Portanto, Ctia deve manter uma velocidade de
60
15 km
=
km/h = 12 km/h.
5/4 h
5

101. Distncia na reta


Soluo 1: Como a maior distncia entre dois pontos 19 e a menor 2, desenhamos
uma reta numrica com os dois pontos 0 e 19 nas extremidades e o ponto 2 a duas
unidades de 0, obtendo os primeiros trs pontos na gura.
0.......................................2.....................................................................................................................................................................................................................................................................................................................................19
..
.
..
..
....
...
p3
p1
p2

Em seguida, colocamos dois outros pontos, para tentar fechar as distncias exigidas.
Como precisamos ter distncias 5 e 7, colocamos o ponto 7 na reta, o que nos d
distncias que no so incompatveis com os dados do problema, j que as distncias
entre esses 4 pontos so 2, 5, 7, 12, 17 e 19, conforme gura.
0.......................................2.................................................................................................7...................................................................................................................................................................................................................................................19
..
.
...
..
...
..
....
...
p2
p1
p3
p4

OBMEP 2010

211

Solues do Nvel 2
Desse modo, se nossa tentativa de colocar todos os pontos tiver xito, necessariamente
k = 12. Temos sorte pois, para obter as distncias 4, 8, 13 e 15, basta colocar o ponto
15 na reta, obtendo todas as distncias 2, 4, 5, 7, 8, 12, 13, 15, 17 e 19, conforme
gura.
0.......................................2.................................................................................................7...............................................................................................................................................................15.......................................................................19
..
.
...
..
...
..
.......
......
....
...
p5
p4
p3
p1
p2

Escolhendo 4 como terceiro ponto, obtemos uma outra distribuio de pontos com as
mesmas distncias entre eles, como na gura seguinte, em que, novamente, k = 12.
0...............................................................................4...............................................................................................................................................................12.............................14............................................................................................19
..
.
..
..
.......
......
.......
......
....
...
p5
p4
p3
p1
p2

Soluo 2: Como a maior distncia 19 podemos, supor que um ponto 0 e outro


19. Se um terceiro ponto for igual a a, teremos as distncias a 0 = a e 19 a
na lista de distncias dada. Como nessa lista de distncias aparecem os pares 2 e 17,
bem como 4 e 15, podemos escolher o nmero a = 2 ou a = 4 como terceiro ponto.
Escolhamos o ponto 2. Como 4 e 15 esto na lista das distncias, temos que 4 ou 15
outro ponto na reta; mas, 4 no pode ser um dos pontos porque a distncia 2, entre 2
e 4, no aparece duas vezes. Logo, 15 o quarto ponto na reta. Por ltimo, o quinto
ponto tem que estar a uma distncia 5 de um dos pontos e 7 de outro. Assim, o ponto
que falta o ponto 7 e a distncia desconhecida k = 19 7 = 12.
Tomando 4 como terceiro ponto, obteramos os pontos 12 e 14 como quarto e quinto
pontos e, novamente, a distncia desconhecida k = 12.

102. Nmero mpar A opo correta (c).


Lembremos que a soma ou diferena de nmeros de mesma paridade um nmero par:
par par = par

mpar mpar = par.

Observemos que n2 e n3 podem ser pares ou mpares, portanto n2 + 5 e n3 + 5 podem


ser mpares ou pares, dependendo de n ser par ou mpar. Restam as opes (a), (b) e
(c).
Soluo 1: Ambos n2 n e n2 +n so soma e diferena de dois nmeros que sempre tm
a mesma paridade, portanto, esses nmeros sempre sero pares, do mesmo modo que
n2 n+2 e n2 +n+2. Finalmente, a opo correta (c), porque n2 +n+5 = (n2 +n)+5,
que soma de um par e um mpar, sempre ser um nmero mpar, para todo valor
inteiro de n.
Soluo 2: Observemos que n2 n = n(n 1) e n2 + n = n(n + 1) so o produto de
dois nmeros consecutivos, portanto, so sempre pares, do mesmo modo que n2 n + 2
e n2 + n + 2. Finalmente, a opo correta (c), porque n2 + n + 5 = (n2 + n) + 5 a
soma de um par com um mpar, que sempre mpar, para todo valor inteiro de n.
103. Quatro nmeros inteiros A opo correta (e).
212

OBMEP 2010

Solues do Nvel 2
Como m, n, p e q so inteiros, tambm 7 m, 7 n, 7 p e 7 q so inteiros. Agora,
4=122 e
4 = (1) (2) 1 2

a nica decomposio de 4 em um produto de nmeros inteiros distintos. Segue que


(7 m) + (7 n) + (7 p) + (7 q) = (1) + (2) + 1 + 2 ,

portanto, m + n + p + q = 28.
104. As pginas do dicionrio Observemos que:
(a) a cada dez nmeros imprime-se uma vez o 1 nas unidades;
(b) a cada cem nmeros imprime-se dez vezes o 1 nas dezenas;
(c) a cada mil nmeros imprime-se cem vezes o 1 nas centenas.
Assim, de 1 at 999, imprime-se o algarismo 1 um total de 300 vezes, das quais 100
vezes nas unidades, 100 vezes nas dezenas e 100 vezes nas centenas.
De 1 000 at 1 999, imprime-se o algarismo 1 outras 300 vezes dentre unidades, dezenas
e centenas, mais 1 000 vezes na posio do milhar, portanto, entre 1 e 1 999, o nmero
de vezes que se imprime o 1 300 + 300 + 1 000 = 1 600.
Agora, entre 2 000 e 2 999, imprime-se o 1 mais 300 vezes, completando
1 600 + 300 = 1 900.
De 3 000 a 3 099, temos 20 algarismos 1, de 3 100 a 3 119, temos 32 algarismos 1 e, de
3 120 a 3 149, temos 32 algarismos 1, portanto, at 3 149, o nmero de vezes que se
imprime o 1 1 900 + 20 + 32 + 32 = 1 984. Como faltam 4 algarismos 1, o nmero de
pginas do dicionrio 3 152.
105. Soma de potncias de 2
n
Soluo 1: Observe que 28 + 211 + 2n = (24 )2 + 2 24 26 + (2 2 )2 . Logo, para n = 12,
temos 28 + 211 + 212 = (24 + 26 )2 . Assim, n = 12 uma soluo.
Soluo 2: Se 28 + 211 + 2n = k 2 , ento
28 + 23 28 + 2n
9 28 + 2n
2n
2n

= k2
= k2
= k 2 (3 24 )2
= (k 3 24 )(k + 3 24 ).

Logo, (k 3 24 ) e (k + 3 24 ) so potncias de 2, ou seja, k + 3 24 = 2a e


k 3 24 = 2b , com a + b = n e
2a 2b = (k + 3 24 ) (k 3 24 ) = 3 25 = 96.
Examinemos a lista das potncias de 2:
2, 4, 8, 16, 32, 64, 128, 256, . . . .
Constatamos que a diferena dessas potncias s 96 entre 128 = 27 e 32 = 25 . Logo,
a = 7 e b = 5. Assim, n = 7 + 5 = 12 a nica soluo.
OBMEP 2010

213

Solues do Nvel 2
106. Reverso de um nmero Lembremos que nmeros ab de dois algarismos, em que
a o algarismos das dezenas e b o das unidades, so dados por ab = a 10 + b. Por
exemplo, 47 = 4 10 + 7.
Seja ab um nmero de dois algarismos; seu reverso , ento, ba. Temos que
ab + ba = a 10 + b + b 10 + a = (a + b) 11.
Por outro lado, a, b 9, de modo que a + b 18. Como 11 um nmero primo e
a + b 18, 11 no divide a + b e, portanto, o produto (a + b) 11 s um quadrado
perfeito se a + b = 11. Assim, temos 8 nmeros satisfazendo a condio do problema:
29, 38, 47, 56, 65, 74, 83 e 92.
107. ngulos externos de um tringulo Observemos que os ngulos y, 150 e 160 so
ngulos externos de um tringulo, de modo que y +150 +160 = 360 . Assim, y = 50 .
Pela mesma razo, conclumos que z = 50 . Como x, y e z so ngulos internos de um
tringulo, temos x + y + z = 180 e, portanto, x = 80 .
108. Uma brincadeira Sejam a, b, c e d os nmeros procurados. So dados os nmeros
a+b+c
a+b+d
a+c+d
+ d,
+ c,
+b e
3
3
3

b+c+d
+ a,
3

mas no sabemos sua ordenao. No entanto,


90 = 17 + 21 + 23 + 29
a+b+c
a+b+d
a+c+d
b+c+d
+d+
+c+
+b+
+a
=
3
3
3
3
= 2(a + b + c + d)
e, portanto, 2(a + b + c + d) = 90, ou seja, a + b + c + d = 45. Seja, agora, d o maior
dentre os nmeros a, b, c e d. Ento
d = 29

a+b+c
45 d
= 29
,
3
3

e conclumos que d = 21.


109. Ovos e mas A opo correta (b).
Como o enunciado e a resposta so percentuais podemos, nesse caso, estipular qualquer
preo e qualquer unidade monetria, que a resposta ser, sempre, a mesma. O mais
simples, portanto, supor que, inicialmente, uma dzia de ovos custava 100 e, portanto,
que 10 mas tambm custavam 100. Como o preo dos ovos subiu 10%, o novo valor
dos ovos 110. O preo das mas diminuiu 2%, portanto, o novo preo de 10 mas
98. Assim, enquanto antes gastava-se 200 na compra de uma dzia de ovos e 10
mas, agora gasta-se 110 + 98 = 208. Da, temos que o aumento foi de 8 em 200, o
que corresponde ao percentual de
4
8
=
= 4%.
200
100

214

OBMEP 2010

Solues do Nvel 2
110. Dividir um cubo A opo correta (b).
Convertendo metros em milmetros, temos 1 m = 1 000 mm. Assim, o cubo cou dividido em 1 000 1 000 = 106 cubinhos, cada um com uma aresta de 1 mm. Empilhandose os 106 cubinhos sucessivamente um em cima do outro, obtemos uma coluna de
1 000 1 000 = 106 mm = 1 km de altura.
111. Uma expresso A opo correta (b).
a2
4a
22 a
a1(3)
a4
1 3 = a25 3 3 = a7
= a7
a5
(2 a)
2 a
2
2
7+4
3
a
a
1
=
=
= 3
2
2
2a
112. Uma igualdade Fatorando 96, temos 25 3 a2 = b3 . Para que 25 3 a2 seja um
cubo, o nmero a deve ter, pelo menos, a fatorao 2n 3n . Como queremos o menor
valor de a, tomamos a = 2n 3n e, assim,
25 3 a2 = 25 3 (2n 3m )2 = 25+2n 31+2m .
Logo, 5 + 2n e 1 + 2m so mltiplos de 3. Os menores valores de n e m so n = 2 e
m = 1. Portanto, a = 22 3 = 12.
113. Somas de trs em trs Somando de trs em trs quatro nmeros a, b, c e d, obtemos
os nmeros a + b + c, a + b + d, a + c + d e b + c + d, sendo
(a + b + c) + (a + b + d) + (a + c + d) + (b + c + d) = 3(a + b + c + d).
Como 30 = 6 + 7 + 8 + 9 = (a + b + c) + (a + b + d) + (a + c + d) + (b + c + d), resulta
30
30 = 3(a + b + c + d), donde a + b + c + d =
= 10. Como cada nmero igual
3
diferena entre a soma dos quatro nmeros e a soma dos outros trs, os nmeros
procurados so
10 6 = 4 , 10 7 = 3 ,

10 8 = 2 e 10 9 = 1.

114. O retngulo do Lus Faremos a diviso com retngulos. Observamos que 24 = 64


e 12 = 6 2, portanto, Lus pode fazer um primeiro corte a 4 cm no lado de 10 cm
e outro corte a 2 cm do corte anterior. Depois desses cortes, resta um retngulo de
tamanho 6 4. Por ltimo, como 16 = 4 4, basta fazer mais um corte a 4 cm no
lado que mede 6 cm. Duas opes de cortes esto ilustrados nas guras seguintes, com
indicao das dimenses dos lados e das reas.
4

24

12

16
6

6
8

OBMEP 2010

16

24
12

2
215

Solues do Nvel 2
115. Uma fbrica de blusas A opo correta (c).
Denotemos por x o nmero de unidades produzidas. Ento o custo de produo
500 + 2x reais. Na venda, o fabricante est recebendo 2,5x. Assim, ele ter lucro
quando 2,5x > 500 + 2x, isto , 0,5x > 500, ou x > 1 000.
116. Existncia de tringulos A opo correta (c).
A soma dos trs ngulos internos de um tringulo 180 . Logo, se um deles mede 90 , a
soma dos outros dois 90 e, por isso, no podem ser maiores do que 90 . Portanto, no
existem tringulos retngulos obtusngulos. Os seguintes exemplos de comprimentos
de lados mostram que cada um dos outros casos pode ocorrer:

(a) 2, 3, 3;
(b) 1, 1, 2;
(d) 3, 4, 5;
(e) 3, 4, 6.
117. Os doze pontos No total, temos 11 possveis quadrados, mostrados nas guras
seguintes.
5 quadrados
s
s
s
s

s
s
s
s

4 quadrados

.
...........
.
...........
.
.
.
.
.
.
.
.
.
.
.
.
.
.
.
.
.
.
......................
.
................................
.
.
.
.
...........
.
.
.
.
.
.
.
.
.
.
.
.
.
.
.
.
.
.
.
.
.
.
.
.
.
.
.
.
.
.
.
.
.
.
.
.
.
.
.
.
.
.
.
................................
.
.
................................
.
.
.
.
.
.
.
.
.
.
.
.
.
.
.
.
.
.
.
.
.
.
.
.
...........
.
.
.
...........

s
s

s
s
s
s

s
s
s
s

2 quadrados

s
s

s
s

s
s
s
s

s
s
s
s

... ...
.
. .............
.. ...... ...
.
. ..............
....... .........
.....
. ...
.....
...
....
.. ...
.. ...
.....
.....
.
.
...
.... .
.. ..
.. ..
.. ..
..
..
..
...
...
...
....
.
. .
.. .
......
.... ..
. ... .
. .. .
. . ..
.....
.....
....
....
....... .........
........ .........
..... ..
. . .
.. ......... ..
. .....
..... ...
..

..
..
..
...
...
...
.. ... .. ...
.. ........ ....
.. ....... ....
..
..
..
..
..
..
..
..
..
....
.. ...
..
..
.
..
.. ...
..
.. ....
.
.
.
..
.. ..... ..... ...
.
.. ..... ...... ...
.. .... . .. .. ...
.. .. . .. .. .. .. .
.. . . .. ..
....
.. . .....
..
.
.
.
.. . . .. . ... .
..... ... ... ... ...
...... ........ ........
.
.
....
..
....
..
.. ...
.
.
.. ....
..
..
.
..
.. ...
..
.. ...
.
.. . .
..
..
..
..
.. ..
..
..
..
...
..
..
...
.. ..... ....
.. ...... ...
.. . ..... ...
.. .. .....
...
....
...
...
.
.

s
s

s
s

s
s

118. O colar Sejam g o nmero de prolas grandes, p o de pequenas, a o peso (em


gramas) de uma prola grande e b o de uma pequena. Com essa notao, os dados so
os seguintes.
(a) O nmero total de prolas no colar g + p e temos g + p < 500.
(b) O peso das prolas grandes g a e o das pequenas p b.
(c) O peso total do colar g a + p b.
Antes de equacionar o problema, equacionamos as duas hipteses do problema.
(a) Ao substituirmos 70% das prolas grandes pelas pequenas, o nmero total de
prolas no colar ca composto por
30% g + p + 70% g = 0,3 g + (p + 0,7 g)
pequenas

grandes

prolas e seu peso ca sendo


0,3 g a

peso das grandes

(p + 0,7 g) b =

peso das pequenas

0, 4(g a + p b) .
40%

do peso inicial

(b) Analogamente, ao substituirmos 60% das prolas pequenas pelas grandes, o nmero
total de prolas no colar ca composto por
g + 60% p + 40% p = (g + 0,6 p) + 0,4 p
grandes

216

pequenas

OBMEP 2010

Solues do Nvel 2
prolas e seu peso ca sendo
(g + 0,6 p) a +

peso das grandes

0,4 p b

peso das pequenas

1,7(g a + p b)
170%

do peso inicial

Assim, as duas hipteses podem ser resumidas no sistema


(0,3) g a + (0,7) g b + p b = 0,4(g a + p b) ,
g a + (0,6) p a + (0,4) p b = 1,7(g a + p b).
Para resolv-lo, comeamos multiplicando ambas equaes por 10 e simplicamos, obtendo
(7 g + 6 p) b = g a ,
(7 g 6 p) a = 13 p b.
7g + 6p
a
13 p
= =
, resultando
g
b
7g 6p
49 g 2 36 p2 + 13 p g = 0 . Para fatorar essa expresso, desdobramos
Eliminando as incgnitas a e b, podemos escrever

13 p g = 49 p g 36 p g
e obtemos
0 = 49 g 2 + 13 p g 36 p2 = 49 g 2 + 49 p g 36 p2 36 p g
= g (49 g 36 p) + p (49 g 36 p)
= (g + p)(49 g 36 p) ,
de modo que (g + p)(49 g 36 p) = 0 , ou seja,
49 g = 36 p .
Como 36 e 49 so primos entre si e p e g so inteiros positivos, segue que g um
mltiplo de 36 e p um de 49, isto , g = 36 k e p = 49 k , para certos inteiros k e k
maiores do que 1. Decorre que 36 49 k = 36 p = 49 g = 49 36k, ou seja, k = k , de
modo que g = 36 k, p = 49 k e g + p = 85 k. Como g + p < 500, o colar s pode ter 85,
170, 255, 340 ou 425 prolas.
119. Mulheres votantes A opo correta (b).
52
40
A frao de mulheres na populao
e, delas, a frao que votante
. Logo,
100
100
a frao de mulheres votantes
40
52
52

100% =
100% = 0,208 100% = 20,8%.
100 100
250
120. Amigos do sculo XX Os dois amigos nasceram no mesmo ms e no mesmo ano,
com uma diferena de 7 dias, de modo que um nasceu no dia d/m/a e o outro no
dia (d + 7)/m/a. Com essas datas formamos os nmeros (d)(m)(a) e (d + 7)(m)(a).
Sabemos que
(d + 7)(m)(a) = (d)(m)(a) + 7 10k ,
OBMEP 2010

217

Solues do Nvel 2
onde k o nmero de algarismos de (m)(a). Observe que s podemos ter k = 3, se o
ms m tem um algarismo, ou k = 4, se o ms m tem dois algarismos. Como tambm
(d + 7)(m)(a) = 6 (d)(m)(a), resulta
7 10k = 5(d)(m)(a).

No caso k = 3, decorre que o amigo mais velho nasceu em


7 000
= 1 400,
5
70 000
= 14 000, que no uma
isto , 1o de abril de 1900. No caso k = 4, decorre
5
data vlida.
(d)(m)(a) =

121. Operao em uma frao Seja a/b uma frao qualquer e seja c um nmero
qualquer tal que
a+c
b
= .
b+c
a
Esta igualdade equivalente a (a + c)a = (b + c)b, ou seja, a2 + ac = b2 + bc ou, ainda
a c(a b) = b2 a2 = (b a)(a + b). Evidenciando a b, vemos que o que se quer
um nmero c tal que
(a b)(a + b + c) = 0.
Essa igualdade pode ser satisfeita de duas maneiras.
a
(a) Se a = b, temos simplesmente 1 =
e podemos somar qualquer nmero c aos
b
dois termos da frao para obter 1 novamente.

(b) Se a = b, precisamos ter a+b+c = 0 e, nesse caso, s podemos somar c = (a+b)


aos dois termos da frao a/b para obter b/a.
122. O nmero 119 Dados inteiros positivos d e r, dizemos que N dividido por d deixa
resto r se existir um inteiro n tal que N = nd + r. Se M dividido por d deixar o mesmo
resto r, ento existe um inteiro m tal que M = md + r. Nesse caso, se M > N, resulta
que m = n + p para algum inteiro n e, portanto,
M = md + r = (n + p)d + r = nd + r + pq = N + pd,
de modo que M N = pd um mltiplo de d. O mesmo ocorre se M < N.

Como 119 tem todas as propriedades arroladas, decorre que se N for algum nmero
com essas mesmas propriedades ento, necessariamente 119 N um mltiplo de 2,
3, 4, 5 e 6. Como o menor mltiplo comum de 2, 3, 4, 5 e 6 60, N tem as mesmas
propriedades de 119 se, e s se, 119 N for um mltiplo de 60. Assim, os nicos
nmeros N com as mesmas propriedades de 119 so da forma N = 119 + 60 p,
para algum inteiro p. Para obter N positivo, precisamos tomar p 1 e, para obter
N 2 007, precisamos tomar p 31, pois 119 + 60 32 = 2 039 > 2 007.
Assim, os nicos nmeros inteiros positivos e menores do que 2 007 com as mesmas
propriedades de diviso de 119 so
59, 119, 179, . . . , 1979 (= 119 + 60 31),
num total de 33 nmeros.
218

OBMEP 2010

Solues do Nvel 2
123. Fonte com trs torneiras Para simplicar, numeramos os 10 garrafes de acordo
com os respectivos tempos que levam para car cheios, de 1 a 10.
Soluo 1: Uma ideia utilizar o tempo que sobra de um garrafo para encher
outro garrafo, enchendo simultaneamente outros dois. As guras seguintes ilustram
a soluo. Na Figura I, as 3 torneiras gastam 10 minutos para encher os garrafes 10,
Figura I: 10 min

10

.
.
.
.
.
.
.
.
.
.
.
.
.
.
.
.
.
.
.
.
.
.
.
.
.
.
.
.
.
.
.
.
.
.
.
.
.
.
.
.
.
.
.
.
.
.
.
.
.
.
.
.
.
.
.................
.
.................

.
.
.
.
.
.
.
.
.
.
.
.
.
.
.
.
.
.
.
.
.
.
.
.
.
.
.
.
.
.
.
.
.
.
.
.
.
.
.
.
.
.
.
.
.
.
.
.
.
.
.
.
.
.
.................
.
.................

.
.
.
.
.
.
.
.
.
.
.
.
.
.
.
.
.
.
.
.
.
.
.
.
.
.
.
.
.
.
.
.
.
.
.
.
.
.
.
.
.
.
.
.
.
.
.
.
.
.
.
.
.................
.
.
.................

.
.
.
.
.
.
.
.
.
.
.
.
.
.
.
.
.
.
.
.
.
.
.
.
.
.
.
.
.
.
.
.
.
.
.
.
.
.
.
.
.
.
.
.
.
.
.
.
.
.
.
.
.
.
.................
.
.................

Figura II: 9 min

.
.
.
.
.
.
.
.
.
.
.
.
.
.
.
.
.
.
.
.
.
.
.
.
.
.
.
.
.
.
.
.
.
.
.
.
.
.
.
.
.
.
.
.
.
.
.
.
.
.
.
.
.................
.
.
.................

.
.
.
.
.
.
.
.
.
.
.
.
.
.
.
.
.
.
.
.
.
.
.
.
.
.
.
.
.
.
.
.
.
.
.
.
.
.
.
.
.
.
.
.
.
.
.
.
.
.
.
.
.
.
.................
.
.................

.
.
.
.
.
.
.
.
.
.
.
.
.
.
.
.
.
.
.
.
.
.
.
.
.
.
.
.
.
.
.
.
.
.
.
.
.
.
.
.
.
.
.
.
.
.
.
.
.
.
.
.
.
.
.................
.
.................

.
.
.
.
.
.
.
.
.
.
.
.
.
.
.
.
.
.
.
.
.
.
.
.
.
.
.
.
.
.
.
.
.
.
.
.
.
.
.
.
.
.
.
.
.
.
.
.
.
.
.
.
.................
.
.
.................

.
.
.
.
.
.
.
.
.
.
.
.
.
.
.
.
.
.
.
.
.
.
.
.
.
.
.
.
.
.
.
.
.
.
.
.
.
.
.
.
.
.
.
.
.
.
.
.
.
.
.
.
.
.
.................
.
.................

.
.
.
.
.
.
.
.
.
.
.
.
.
.
.
.
.
.
.
.
.
.
.
.
.
.
.
.
.
.
.
.
.
.
.
.
.
.
.
.
.
.
.
.
.
.
.
.
.
.
.
.
.................
.
.
.................

9, 8, 2 e 1 e, na Figura II, as 3 torneiras gastam mais 9 minutos para encher os garrafes


7, 6, 5, 4 e 3. Logo, o tempo total gasto de 19 minutos.
Soluo 2: Se tivssemos uma torneira s, o tempo gasto para encher os 10 garrafes
seria de 1 + 2 + + 9 + 10 = 55 minutos. Como temos trs torneiras e 55 = 3 18 + 1,
uma torneira, pelo menos, vai levar 19 minutos e as outras duas, 18 minutos cada. A
tabela seguinte mostra uma forma de fazer o trabalho em 19 minutos.
Torneira 1
Torneira 2
Torneira 3

10
8
7

9
5
6

3
4

2
1

124. A sequncia xyz Igualando os denominadores, vericamos que a sequncia dada


igual a
4 5 6 7
, , , , x, y, z.
8 8 8 8
Assim, o denominador sempre 8 e os numeradores so consecutivos. Logo,
8
9
10
5
x = = 1, y = e z =
= .
8
8
8
4
125. A mesa circular Se a prxima pessoa a se sentar vai ter que se
sentar ao lado de uma cadeira ocupada, isso signica que existem
no mximo 2 cadeiras consecutivas desocupadas. (Na gura, as
cadeiras ocupadas esto representadas por quadradinhos pretos e
as desocupadas por quadradinhos brancos.)

.....
....................
......... .............
....
......
....
.....
...
....
...
..
..
...
..
..
..
..
..
..
..
..
..
..
..
.
..
.
.
.
.
.
.
.
.
.
.
.
.
.
.
.
.
.
.
.
.
.
.
.
.
.
.
.
.
.
.
.
.
.
.
.
.
.
.
.
.
.
..
.
..
.
.
..
.
..
..
..
..
..
..
..
..
...
..
...
...
....
.....
....
....
......
.
..
........
.....................
...............

Podemos, ento, pensar nas cadeiras em grupos ordenados de 3 cadeiras, em que a


terceira j est ocupada. Logo, o menor valor de N 60 3 = 20.
126. Nmeros proporcionais A opo correta (d).
x
3
Como = , ento xz = 3y. Elevando ao quadrado ambos os membros dessa igualy
z
dade, obtemos x2 z 2 = 9y 2 .
OBMEP 2010

219

Solues do Nvel 2
127. Esportistas de uma escola A opo correta (c).
Denotemos por x o nmero de estudantes que praticam simultaneamente os dois esportes. Logo, o nmero de estudantes que pratica somente futebol 20 x e o que
pratica somente vlei 19 x. Portanto, os 15 estudantes que praticam exatamente
um esporte esto divididos em 15 = (20 x) + (19 x). Segue que x = 12 e resulta
que 20 + (19 x) = 27 estudantes praticam pelo menos um dos dois esportes. Assim,
13 = 47 27 estudantes no praticam nem futebol nem vlei.
128. Vamos ao teatro A opo correta (c).
3
Mrio pagou 3 e levou 5, portanto, pagou apenas do preo usual, tendo economizado
5
2
40
2
. Como =
, a economia foi de 40%.
5
5
100
129. Uma desigualdade A opo correta (c).
Note que o inverso de um nmero s maior do que 1 quando o nmero for positivo e
menor do que 1. Portanto,
1
> 1 0 < x 1 < 1 1 < x < 2 .
x1
130. A sala do Professor Newton A opo correta (c).
Como o nmero de alunos homens menor do que 15 e o das mulheres 15, temos que
15 < alunos homens + alunas mulheres < 15 + 15 = 30, ou seja, o nmero do total de
alunos est entre 15 e 30.
Soluo 1: Quando dividimos o nmero de alunos por 4 sobram 2 alunos, ento o
nmero de alunos par. Quando dividimos por 5 sobra um, ento o ltimo algarismo
do nmero de alunos 1 ou 6; sendo par, s pode ser 6. Assim, s temos dois possveis
valores, 16 ou 26. Descartamos 16 por ser divisvel por 4, de modo que o nmero de
alunos 26. Consequentemente, temos 26 15 = 9 alunos homens.

Soluo 2: Observemos que o nmero 6, dividido por 4, deixa resto 2 e, dividido por
5, deixa resto 1. Logo, se somamos a 6 um mltiplo comum de 4 e 5, o nmero obtido
tambm ter essa propriedade. O menor mltiplo comum de 4 e 5 20, portanto, os
possveis valores para o nmero de alunos 6, 26, 46, 66, . . . . Como o nmero de alunos
est entre 15 e 30, esse nmero 26 e resulta que temos 26 15 = 9 alunos homens.

131. Um jardim retangular A opo correta (b).


Pelos dados do problema sabemos que
3
3
AD = AB
e
AB = AF.
5
5
3 2
9
3
AD =
AD. A rea do terreno AB AD e a rea do
Logo, AF = AB =
5
5
25
jardim AB AF, portanto a razo entre essas reas
AB AF
AF
9
36
=
=
=
= 36%.
AB AD
AD
25
100

220

OBMEP 2010

Solues do Nvel 2
132. Nmeros decrescentes Observe que 0 <
1 < 32/3 < 32 < 33 , resulta

1
1
1
< <
0 < a < b < c. Como
c
b
a

1
1
1
< 2 < 2/3 < 1.
3
3
3
3

1
1
1
Tambm 1 < 3 < 35 , portanto 1 < 5 3 < 3. Resta observar que 3 = ( )3 , 2 = 32 ,
3
3
3
1 1
1
2/3
=3
e3=
, para estabelecer que
32/3
3
1
3

< 32 < 32/3 <

3<

1
3

133. Os bombons misturados Sejam x o nmero de bombons que Marta ganhou e y o


4
que Carmem ganhou. Sabemos que x + y = 200, x < 100 e x > y. Ento y 100
5
4
e, portanto, x > 100 = 80. Assim, x um inteiro compreendido entre 80 e 100.
5
Como deve ser um mltiplo de 8, s pode ser 88 ou 96. Vamos decidir qual .
4
(a) Se x = 88, ento y = 200 88 = 112. Logo, 88 = x > 112 = 89,6, o que no
5
possvel.
4
(b) Se x = 96, ento y = 200 96 = 104 e 96 = x > 104 = 83,2, o que possvel.
5
Assim, Marta ganhou 96 bombons e Carmem 104.
134. Jantar aos sbado Para simplicar, vamos denotar cada casal por um par de
nmeros, um nmero representando o marido e o outro a mulher. Temos, ento, os
trs pares (1, 2), (3, 4), (5, 6), que no podem ser vizinhos. Podemos considerar o lugar
do marido 1 mesa como sendo xo, j que desconsideramos rotaes na disposio
mesa. Duas disposies possveis so dadas na gura seguinte.
1

............
.................
......
...
....
..
...
..
..
..
..
..
..
..
.
..
.
.
.
.
.
.
.
.
.
.
.
.
.
.
.
.
.
.
.
.
.
..
.
..
.
..
..
..
..
...
..
...
..
......
...
...... ........
...... .....
......

............
.................
......
...
....
..
...
..
..
..
..
..
..
..
.
..
.
.
.
.
.
.
.
.
.
.
.
.
.
.
.
.
.
.
.
.
.
..
.
..
.
..
..
..
..
...
..
...
..
......
...
...... ........
...... .....
......

5
132 546

5
164 523

Essas disposies descrevemos por 132546 e 164523, sempre comeando em 1, xado


numa posio mesa, e prosseguindo no sentido horrio. Assim, o problema se resume
em encontrar todos os nmeros de 6 algarismos distintos que podem ser escritos com
os algarismos 1, 2, 3, 4, 5 e 6, onde:
(a) todos os nmeros comeam com o algarismo 1;
(b) nenhum nmero pode terminar com o algarismo 2;
(c) no podem aparecer juntos 1 e 2, 3 e 4, 5 e 6.
Encontramos os 16 nmeros da tabela seguinte.
OBMEP 2010

221

Solues do Nvel 2
132 546
142 536

132 645
142 635

135 246
145 236

135 264
145 326

135 426
146 235

136 245
146 325

136 254
153 246

136 425
154 236

Logo, a resposta 16 sbados.


135. Expresso com radicais A opo correta (e).
1+

1+

= 1+

= 1 + 2 2 + 2 = 3 + 2 2.

136. Possveis tringulos Como a soma dos comprimentos dos lados menores de um
tringulo deve ser maior que o comprimento do lado maior, devemos ter
a + (a + 2) > a + 5, ou seja, a > 3.
137. Uma diferena A opo correta (a).
Efetuando as operaes indicadas, obtemos

0,4 ( 0,09 1)
20
0,1 20

= (0, 3 1) = 4 + 0,7 = 3,3.

0,5
5
0,4
138. A Terra A frao de terra que cultivada
1

2 1
15 6 5
4
=
=
.
5 3
15
15

Como a terra ocupa 3/10 da superfcie total do globo terrestre, resulta que a rea
4
3
2
2
2
4
8
cultivada

=
, isto ,
=
=
= 8% da superfcie do globo
15 10
25
25
25 4
100
terrestre.
139. Uma frao A gura mostra que M N paralelo a BC, portanto, os tringulos
ABC e AM N so semelhantes e, por isso, seus lados so proporcionais. Usando o lado
4
AM
= . Assim,
dos quadradinhos da grade da gura, obtemos
AB
7
AN
AM
4
=
= .
AC
AB
7
140. Clculo de ngulo A opo correta (c).
Como as retas P Q e RS so paralelas, os ngulos T W S e QT W so complementares.
Assim, QT W = 180 110 = 70 . Por outro lado, sabemos que o tringulo U T V
issceles, portanto, os ngulos em U e em V so iguais. Usando que a soma dos ngulos
internos de um tringulo 180 , vemos que 2T U V = 180 70 = 110 e, portanto,
T U V = 55 . Como os ngulos T U V e QU V so complementares, resulta que
QU V = 180 55 = 125 .
222

OBMEP 2010

Solues do Nvel 2
141. Uma loja de brinquedos Se x denota o desconto em reais e y o nmero total de
peas, ento (13 x) y = 781. Assim, (13 x) e y so divisores de 781 e, como
781 = 11 71, os nicos divisores de 781 so 1, 11, 71 e 781. O divisor 13 x de 781
no pode ser igual a 1, pois sabemos que y 100. A nica opo, ento, 13 x = 11
e y = 71, de modo que a reduo foi de x = R$ 2,00 por unidade.
142. Frao de frao Temos
1

1+
1+

=1+

1+

1
1+
2

=1+

1
3
2

1+

2
3

=1+

1
3
8
=1+ = .
5
5
5
3

143. Potncias de 3 Temos 272 a = (33 )2 a = 36 a = (3a )6 = 26 = 64.


144. Aumento de preo Em reais, o aumento foi de 5,55 5 = 0,55 e, portanto, o
percentual de aumento foi de
0,55 20
11
0,55
=
=
= 11%.
5
5 20
100
145. Roseiras em la possvel plantar as roseiras em 6 las de 5 roseiras cada uma,
conforme mostra a gura.
s

...
.
...
. ..
. ...
. ..
. ...
. ...
. ...
.
. ...
..
..
.
.
..
.
.
....
.
.
. .. .....
. .. .....
. . . ..
. .
...
...
. .
. .
..
. .
..
..
. .
. .
...
..
..
..
..
...
..
..
..
..
..
...
.
..
..
.
.
.
...
..
.
.
..
...
...
..
..
...
.
..
. .
.. .
. .
. .
.........
... ....
. .
. .
...
...
....
....
. .
.
.
. .
...
...
... .
....
........
....
..
..
. .
...
..
. ......
...
. ............. ......
..
. ...... .......
.
...
.
.
...
......
.
......
.
.
.
..
.
..
. ..... ..........
..
.
. ...... .........
.
...
. ...
..
.......
.
..
.......
. ...
.
...
.......
...
.
..
..
.......
..
.
.......
.
...
.......
.
.
....
.
.
.. .
....... .....
.
....... ...
.
. ...... .
.... .
....... . ..
.
....... ...
.
.
.
. .... .
.......
.
. ......
......
..
.
.......
.
.
.....
........
.. .
.
......
..........
.
.
.. .......
.
.
... ........
...
.. .
.
....
.
.......
..
.
... ............
.
.....
......
.
.
...
. .
.....
...
.
..
.
.... .
.... .
.
. .
.
.......................
.......................
.
.
.... .
.... .
.......................
.......................
.
.
....
.
. .....................................
.
...................................
....
.
....
.
.
........
.............
.
.
...................................
................................
....
....

s s
s
s

146. Calculadora diferente Para calcular (2 + 3) (0 + 3) utilizaremos a propriedade


(iii), obtendo (2+3)(0+3) = (20)+(33). Agora, por (ii), temos 20 = 22 = 4
e, por (i), temos 3 3 = 3. Portanto, (2 + 3) (0 + 3) = 4 + 3 = 7 .
Para calcular 1 024 48 vamos usar a mesma estratgia, observando que
1 024 = 976 + 48 e 48 = 0 + 48. Assim,
1 024 48 = (976 + 48) (0 + 48)
= (976 0) + (48 48)
= 2 976 + 48 = 1 952 + 48 = 2 000.
OBMEP 2010

223

Solues do Nvel 2
147. Dois quadrados
Soluo 1: A regio tracejada um trapzio de bases medindo 10 e 4 cm. A altura do
1
trapzio, que a metade da diferena dos lados dos dois quadrados, mede 2 (104) = 3
cm. Assim, a rea procurada mede
1
3 (10 + 4) = 3 7 = 21 cm2 .
2
Soluo 2: A rea do quadrado maior menos a rea do quadrado menor igual a 4
vezes a rea procurada. Assim, a rea procurada mede
100 16
102 42
=
= 25 4 = 21 cm2 .
4
4
FU
FE
148. Paralelismo Sendo IL e EU paralelos, temos
=
. Analogamente, sendo
FL
FI
FN
FI
RE e N I paralelos, temos
=
. Assim,
FR
FE
FI
FE
FN FU
=

= 1.
FR FL
FE
FI
149. Um subconjunto Vamos construir um subconjunto de {1, 2, 3, . . . , 3 000} em que
nenhum elemento seja o dobro do outro. Comeamos incluindo todos os nmeros
mpares, 1, 3, 5, . . . , 2 999. Assim, j temos 1 500 nmeros e nenhum o dobro de algum
outro. Agora podemos acrescentar os nmeros que so o qudruplo de algum nmero
mpar, isto , acrescentar
4 1 , 4 3 , 4 5 , . . . , 4 749 .
4

12

20

2 996

Essa lista tem 749 nmeros e nenhum deles o dobro do outro; alm disso, nenhum
deles o dobro de um nmero mpar.
Logo, nosso conjunto j possui 1 500 + 749 = 2 249 elementos. Basta tomar qualquer
subconjunto com 2 000 elementos para obter um subconjunto de {1, 2, 3, . . . , 3 000} em
que nenhum elemento o dobro do outro.
150. Tringulos retngulos Observemos que os quatro tringulos que aparecem na
gura so tringulos retngulos, dois a dois semelhantes, portanto, seus lados so
proporcionais. Em particular, temos 9/x = y/20, ou seja, 180 = xy. Alm disso, pelo
Teorema de Pitgoras, temos y 2 = x2 + 92 , de modo que
1802 = x2 y 2 = x2 (x2 + 92 ) = x4 + 92 x2 ,
isto , x4 + 92 x2 1802 = 0. Pela frmula de Bhaskara, obtemos

81 94 + 4 1802
9 92 + 4 202
9 41
2
x =
=9
=9
.
2
2
2
Mas x2 > 0, portanto necessariamente x2 = 9 16 e, portanto, como x > 0, a nica
opo x = 12.
224

OBMEP 2010

Solues do Nvel 2
A partir de x = 12, obtemos todas as outras medidas.

y = x2 + 92 = 15 e, pelo Teorema de Pitgoras, obtemos

z = 202 x2 = 16.

Pelo visto, temos

Usando a proporcionalidade v/8 = 9/x, resulta v = 72/x = 6 e, nalmente, pelo

Teorema de Pitgoras, conclumos que w = 82 + v 2 = 10.


151. Uma desigualdade especial A opo correta (c).
Observemos que se x satisfaz a desigualdade, ento x tambm satisfaz a desigualdade.
Assim, os valores que satisfazem a desigualdade formam um conjunto simtrico em
relao origem e, portanto, basta vericar quais x 0 satisfazem x2 < x + 2, ou seja,
(x 2)(x + 1) = x2 x 2 < 0.

Como x + 1 > 0 para x 0, devemos ter x 2 < 0, ou seja, x < 2. Pela simetria
observada, conclumos que 2 < x < 2 a soluo da desigualdade original.
152. Sapo Cururu A cada x saltos do tipo I, o sapo se desloca 10x cm para o Leste e
30x cm para o Norte e, a cada y saltos do tipo II, o sapo se desloca 20y cm para o
Oeste e 40y cm para o Sul. Assim, ao nal de x saltos do tipo I e y do tipo II, o sapo
se deslocou 10x 20y cm para o Leste e 30x 40y cm para o Norte.
(a) Resolvendo
10x 20y = 190
30x 40y = 950

encontramos x = 57 e y = 19. Logo, o sapo dever dar 57 saltos do tipo I e 19


do tipo II, em qualquer ordem, para alcanar um ponto situado a 190 cm para o
Leste e 950 cm para o Norte de sua casa.
(b) Uma vez que o nmero de saltos de cada tipo um nmero inteiro, Cururu s
alcanar o ponto situado a 180 cm para o Leste e 950 cm para o Norte de sua
casa se o sistema
10x 20y = 180
30x 40y = 950
tiver soluo inteira. Mas a soluo desse sistema x = 59 e y = 41/2, que no
inteiro. Portanto, Cururu no conseguir alcanar aquele ponto.

153. Distribuindo algarismos em linhas De acordo com o padro da sequncia, temos


1a linha
2a linha
3a linha
.
.
.
Logo,

0
1 1 0
2 2 2 1 1 0

10a linha 9 9 9 9 9 9 . . . 9 8 . . . 1 1 0
um algarismo 0 em cada linha d 1 10 = 10 algarismos 0 no total;
dois algarismos 1 em nove linhas d 2 9 = 18 algarismos 1 no total;
trs algarismos 2 em oito linhas d 3 8 = 24 algarismos 2 no total;
quatro algarismos 3 em sete linhas d 4 7 = 28 algarismos 3 no total,
OBMEP 2010

225

Solues do Nvel 2
e assim por diante. Portanto, trata-se de descobrir qual o maior dos produtos a
seguir, onde cada um representa quantos algarismos, de 0 a 9, aparecem na sequncia.
1 10 , 2 9 , 3 8 , 4 7 , 5 6 , 6 5 , 7 4 , 8 3 , 9 2 , 10 1
0

Como o maior produto 30, os algarismos mais usados foram 4 e 5, 30 vezes cada um.
154. Ser que existe?
Soluo 1: Se existir esse nmero N, ento
222 . . . 2
2 111 . . . 1
111 . . . 1
=
=
.
2 008
2 1 004
1 004
Logo, N no inteiro, por ser o quociente do nmero mpar 111 . . . 1 pelo nmero par
1 004. Portanto, no existe tal N.
N=

Soluo 2: Fatorando 2 008, obtemos 2 008 = 23 251, portanto, 2 008 divisvel


por 8. Se existisse um inteiro N tal que 2 008 N = 222 . . . 2, teramos, ento, que
8 dividiria 222 . . . 2. Por outro lado, sabemos que um nmero divisvel por 8 se, e
somente se, o nmero formado pelos ltimos trs algarismos for divisvel por 8. Mas
222 = 27 8 + 6 no divisvel por 8. Logo, no existe um nmero N tal que
2 008 N = 222 . . . 2.
155. Conferindo uma desigualdade
Soluo 1: Uma maneira de vericar essa desigualdade comparando cada parcela
1 1 1
1
desta soma, como segue. Comparando as fraes , e com , obtemos
5 6 3
4
3
3
1
1
1
1
1
1
< , portanto, 3 =
<
= 3;
5
4
5
5
4
4
1
1
1
1
< , portanto, 3 =
6
4
6
6

1
1
1
1
< , portanto, 3 =
4
3
4
4

<

1
4

<

1
3

1
;
43

1
.
33

Assim,
1
1
1
1
1
3
3 1 1
3 1 1
1
1
+ 3+ 3 < 3+ 3+ 3 = 3 = < = .
3
4
5
6
4
4
4
4
4 4 4
4 3 3
12
Soluo 2: Uma outra maneira de vericar essa desigualdade testar se
1
1
1
1
+ 3+ 3
> 0.
3
4
5
6
12
Reduzindo ao mesmo denominador, temos
1
24 32 53 33 53 26 33 23 53
1
1
1
=
+ 3+ 3
2
26 5
2 33 2 3
26 33 53
18 000 3 375 1 728 1 000
=
26 33 53
11 897
.
= 6
2 33 53
226

OBMEP 2010

Solues do Nvel 2
Nem preciso efetuar o produto indicado no denominador. Como o numerador
positivo, conclumos que a desigualdade se verica.
156. Parte inteira
(a) Os nmeros 9 e 16 so quadrados perfeitos
e 9 < 12 < 16. Ento,

3 = 9 < 12 < 16 = 4

e, portanto,
12 = 3.

12

(b) Como 12 777 2 < 28 756 < 12 777 3,


temos
28 756
< 3, portanto,
2<
12 777

28 756
= 2.
12 777

(c) Como 2 007 < 2 008, temos 0 <


ou

2 007
2 008

< 1,

28 756
12 777

2 007
2 008

2 007
2 007
= 1.
< 0, portanto,
1 <
2 008
2 008
(d) Como 43 = 64 < 111 < 125 = 53 , temos
3

(5) = 5 = 125 < 111 < 4 = (4) ,


de modo que 5 <

111 < 4 e, portanto,

111

111 = 5.

157. Soma nove


Soluo 1: Vamos dividir em dois casos: nmeros de dois algarismos e nmeros de
trs algarismos. No caso de nmeros de dois algarismos, temos 18, 27, 36, 45, 54, 63,
72, 81 e 90, num total de 9 nmeros. Da mesma maneira, listamos os nmeros de trs
algarismos, como segue:
108, 117, 126, 135, 144, 153, 162, 171, 180
207, 216, 225, 234, 243, 252, 261, 270
306, 315, 324, 333, 342, 351, 360
405, 414, 423, 432, 441, 450
504, 513, 522, 531, 540
603, 612, 621, 630
702, 711, 720
801, 810
900

;
;
;
;
;
;
;
;
;

9 nmeros;
8 nmeros;
7 nmeros;
6 nmeros;
5 nmeros;
4 nmeros;
3 nmeros;
2 nmeros;
1 nmero.

Portanto, temos 9 + 8 + 7 + 6 + 5 + 4 + 3 + 2 + 1 = 45 nmeros de trs algarismos.


Assim, existem 9 + 45 = 54 nmeros entre 10 e 999 cuja soma dos algarismos igual
a 9.
OBMEP 2010

227

Solues do Nvel 2
Soluo 2: No caso de nmeros de dois algarismos, uma vez escolhido o algarismo da
dezena, o algarismo da unidade ca automaticamente denido. Como o algarismo da
dezena pode ser 1, 2, 3, 4, 5, 6, 7, 8 ou 9, temos nove nmeros de dois algarismos tais
que a soma de seus algarismos seja 9.
No caso de nmeros de trs algarismos, denotando por n o algarismo da centena, a soma
dos algarismos da dezena e da unidade 9 n, portanto, temos 9 n + 1 = 10 n
possibilidades de escolha para o algarismo da dezena, que pode ser inclusive 0, e o
algarismo da unidade ca automaticamente denido. Como o algarismo da centena
pode ser 1, 2, 3, 4, 5, 6, 7, 8 ou 9, temos
(101)+(102)+(103)+(104)+(105)+(106)+(107)+(108)+(109) = 45
nmeros de trs algarismos tais que a soma dos seus algarismos 9. Assim, existem
9 + 45 = 54 nmeros entre 10 e 999 cuja soma dos algarismos igual a 9.
158. Retngulos Se a e b denotam o comprimento e a largura do retngulo, temos
a b = 96. Logo, a e b so divisores pares de 96 e, portanto, temos quatro retngulos
satisfazendo as condies dadas, a saber, os retngulos de lados medindo 2 e 48; 4 e
24; 6 e 16 e 8 e 12.
159. Nmero de retas
Soluo 1: Para contar o nmero de retas, dividiremos as retas de acordo com suas
posies.
retas paralelas aos lados dos quadrados: trs horizontais e trs verticais;

retas paralelas s diagonais dos quadrados: 3 + 3 = 6;

outras retas: temos 4 2 = 8 retas, formando uma estrela, como na gura.

228

OBMEP 2010

Solues do Nvel 2
Assim, ao todo, temos 3 + 3 + 6 + 8 = 20 retas.
Soluo 2: Note que o ponto central supruo, pois toda reta que passa por ele
e um outro ponto do reticulado, passa tambm por um terceiro ponto do reticulado.
Logo, o ponto central pode ser eliminado de nossas consideraes. Assim, o problema
ca reduzido a calcular quantas retas so determinadas por dois pontos quaisquer dos
oito pontos do reticulado dado.
u

u
u

u
u

Com esses oito pontos, podem ser formados 8 7 possveis pares de pontos distintos
(8 possibilidades para o primeiro elemento do par e 7 possibilidades para o segundo).
Como a ordem em que o par formado no inui na reta determinada por ele, esse
nmero deve ser dividido por 2, ou seja,
87
= 28.
2
Finalmente, notamos que algumas retas esto sendo contadas trs vezes e, portanto,
devem ser subtradas duas vezes desse nmero 28. So elas as quatro retas que contm os lados do quadrado que delimita o reticulado. Logo, o nmero total de retas
determinadas pelo reticulado
87
2 4 = 20.
2
160. Cubo Um cubo tem seis faces distintas, duas a duas opostas, sendo que as faces
opostas no tm aresta em comum. Temos trs pares de faces opostas, logo trs cores
so sucientes, bastando pintar as faces opostas de uma mesma cor. Por outro lado,
claro que duas cores somente no bastam.
161. rea
Soluo 1: Observemos, primeiro, que a razo entre as reas de dois retngulos que
tm a mesma base igual razo entre suas alturas. De fato, na gura esquerda,
S1

h1

S2

h2

27

18

h1

72

h2

esto representados dois retngulos que tm a mesma base b e alturas h1 e h2 . Suas


reas S1 e S2 so dadas por S1 = b h1 e S2 = b h2 , portanto,
bh1
h1
S1
=
=
.
S2
bh2
h2
OBMEP 2010

229

Solues do Nvel 2
Aplicando essa observao aos dois pares de retngulos dados (ver gura anterior,
direita) e denotando por S a rea do quarto retngulo, temos
h1
1
18
27
=
= ,
=
S
h2
72
4
de modo que S = 27 4 = 108. Assim, a rea do lote que foi dividido mede 27 + 18 +
72 + 108 = 225 km2 .
Soluo 2: Sejam x, y, z e w as medidas dos
retngulos menores, conforme a gura. A rea
procurada

(x + w)(y + z) = xy + xz + wy + wz.

Basta determinar wy, pois


xy = 27, xz = 18 e wz = 72. Mas,

sabido

27

18

72

que

w
wz
72
=
=
=4
x
xz
18
e, portanto, w = 4x. Como xy = 27, segue que wy = 4x 27/x = 4 27 = 108. Assim,
a rea do lote que foi dividido mede
27 + 18 + 72 + 108 = 225 km2 .
162. Inteiro mais prximo
(a) Temos:

19 19
4
1
9
3
+
=1+
+6+ =7+
=7+ .
15
3
15
3
15
5
3
1
Logo, a soma dada est entre 7 e 8. Como > , o nmero inteiro mais prximo
5
2
8.
3
7+ 5

7+

1
2

(b) Temos:
85 43 29 15
+
+
+
=2+
42 21 14
7
1
1
=8+
+
+
42 21

1
1
1
1
+2+
+2+
+2+
42
21
14
7
1
1
1 1 1 1
2
+ =8+
+ + +1 =8+ .
14 7
7 6 3 2
7

Logo, a soma dada est entre 8 e 9. Como


8.
8+ 2
7

8
230

8+

OBMEP 2010

2
7

1
2

< 1 , o nmero inteiro mais prximo


2

Solues do Nvel 2
(c) Temos:

11 1 7 2
30 2
2
+ = + = 3 + .
10 2 5 3
10 3
3
1
2
Logo, a soma dada est entre 3 e 2. Como 3 > 2 , o nmero inteiro mais
prximo 2.
2
3 + 3

3 +

1
2

163. Brincando com nmeros mpares Como cada algarismo mpar, temos:
cinco possibilidades de nmeros com um algarismo: 1, 3, 5, 7 e 9;

para nmeros com dois algarismos, temos cinco possibilidades na casa das unidades
e cinco na casa das dezenas, totalizando 5 5 = 25 possibilidades;

para nmeros com trs algarismos, temos cinco possibilidades na casa das unidades,
cinco na casa das dezenas e cinco na casa das centenas, totalizando 555 = 125
possibilidades.

Assim, Beatriz pode escrever 5 + 25 + 125 = 155 nmeros com todos algarismos sendo
mpares.
164. gua no jarro Inicialmente, o volume de gua no jarro da Maria 1 l =
1 000 ml. Depois de 200 dias, o volume o mesmo, acrescido do que colocado
por Joo e diminudo do que ela tirou para colocar no do Joo, ou seja,
1 000 + 1 2 + 3 4 + + 199 200
= 1 000 + (1 2) + (3 4) + + (199 200)
= 1 000 (1 + + 1) = 900.
100

Logo, depois de 200 dias, Maria ter 900 ml em seu jarro.


165. Formiga no cubo Na gura temos um caminho percorrendo oito arestas que a
formiga pode fazer partindo do vrtice identicado como 1.

Ser possvel ela fazer um caminho passando por nove arestas? Para fazer esse caminho,
ela teria que passar por nove vrtices, pois o vrtice de chegada o mesmo que o de
partida, j que a formiguinha volta ao vrtice inicial.
vrtice
de
partida

vrtice
de
chegada


1 2
3
4 5
6
7 8
9
OBMEP 2010

231

Solues do Nvel 2
Como o cubo s tem oito vrtices, esse passeio no possvel. Logo, o passeio de maior
comprimento percorre oito arestas.
166. Promoo Sejam b e c o nmero de blusas e calas compradas, respectivamente.
Logo, temos 15b + 17c = 143, sendo b e c nmeros inteiros positivos. Observe que
b < 10 e c < 9, pois tanto 15 10 quanto 17 9 so maiores do que 143. A partir
deste ponto, apresentamos duas possibilidades de soluo.
Soluo 1: Temos que 15b = 143 17c, portanto 143 17c um mltiplo de 15, de
modo que 143 17c termina em 0 ou 5. Isso signica que 17c termina em 3 ou 8. Logo,
c = 9 ou c = 4. Como c < 9, a nica soluo c = 4 e, portanto,
b=

143 17 4
= 5.
15

Assim, Joana comprou cinco blusas e quatro calas.


Soluo 2: Temos que
b=

8 2c
143 17c
=9c+
.
15
15

Note que c um nmero inteiro positivo, portanto, 8 2c precisa ser um mltiplo de


15. Se 8 2c 15, c resulta negativo, portanto, 8 2c = 0, ou seja, c = 4. Da obtemos
que b = 5. Assim, Joana comprou cinco blusas e quatro calas.
167. Soma de cubos Temos a identidade do binmio, (x + y)2 = x2 + 2xy + y 2 , e a do
trinmio, (x + y)3 = x3 + 3x2 y + 3xy 2 + y 3 . Substituindo os valores de x + y e x2 + y 2
na identidade do binmio, obtemos 1 = 2 + 2xy e, portanto, xy = 1 . Assim, pela
2
identidade do trinmio,
x3 + y 3 = (x + y)3 3xy(x + y) = 1 3

168. O revezamento em uma corrida Como velocidade =


distncia
, o tempo gasto por Joo foi de
velocidade
t=

1
2

1=

5
.
2

distncia
, ou seja, tempo =
tempo

21
9
9
h= 1+
60 min = 1h45min.
h = 1h +
12
12
12

Logo, Carlos precisa completar a prova em um tempo inferior a


(2h48min) (1h45min) = 1h3min = 63min.
Para isso, sua velocidade v, em km/min, deve satisfazer
v>

21
= t < 63, ou seja,
v

1
60
21
= km/min =
km/h = 20 km/h.
63
3
3

Logo, Carlos deve correr com uma velocidade superior a 20 km/h.


232

OBMEP 2010

Solues do Nvel 2
169. Produtos consecutivos
Soluo 1: Como os produtos so nmeros consecutivos, podemos denot-los por p e
p + 1. Temos, ento,
p2 + p = p(p + 1) = 2 3 5 7 11 13 17 = 510 510.
Resolvendo a equao p2 + p 510 510 = 0, encontramos uma nica raiz positiva,
p = 714. Assim, p + 1 = 715 e, fatorando, obtemos
714 = 2 3 7 17 e 715 = 5 11 13.
Soluo 2: Os nmeros dados so 2, 3, 5, 7, 11, 13 e 17. Se 2 e 5 estiverem no mesmo
grupo, ento um dos produtos termina em 0 e o outro, por ser consecutivo, termina
em 1 ou 9. Os possveis produtos terminados em 1 so 3 7 = 21, 3 17 = 51,
7 13 = 91, 13 17 = 221, 3 7 11 = 231, 3 11 17 = 561 e 7 11 13 = 1 001.
Verica-se que esses grupos no constituem soluo e, analogamente, os terminados em
9. Conclumos que 2 e 5 esto em grupos diferentes. Logo, um produto termina em 5
e o outro em 4 ou 6. Como no possvel formar com os nmeros dados um produto
terminado em 6, necessariamente um dos produtos termina em 4 e o outro em 5. Por
tentativas, obtemos a soluo
714 = 2 3 7 17 e 715 = 5 11 13.
170. Distraindo na la Observe que aquela que gritou os nmeros 9, 18, etc, sempre
gritou mltiplos de 9. O primeiro mltiplo de 3 com quatro algarismos 1 002 e o
primeiro mltiplo de 3 maior do que 2 003 2 004. Logo, Vivi gritou 2 004 e Rosa
1 002. Nenhum desses nmeros mltiplo de 9, portanto, foi Tnia quem gritou 9 e
seus mltiplos.
Quem gritou 3, tambm gritou 12 = 3 + 9, 21 = 3 + 2 9, 30 = 3 + 3 9 e assim por
diante, at 3 + 9k. Da mesma forma, quem grita 6, grita todos os nmeros da forma
6 + 9k. Dividindo por 9, obtemos 2 004 = 6 + 9 222 e 1 002 = 3 + 9 111, portanto,
quem gritou 3 foi Rosa e Vivi gritou 6.
Rosa
3
12
21
.
.
.

Vivi
6
15
24
.
.
.

Tnia
9
18
27
.
.
.

1 002
.
.
.

1 005
.
.
.

1 008
.
.
.

2 001

2 004

2 007

Da mesma forma, dividindo por 9, encontramos que 666 mltiplo de 9 e


888 = 6 + 98 9, portanto Tnia gritou 666 e Vivi gritou 888.
171. Nmero e o dobro Inicialmente note que o dobro de um nmero inteiro par, logo
ele termina em 0, 2, 4, 6 ou 8. No entanto, o nmero procurado no pode terminar em
0, pois nesse caso o seu dobro tambm terminaria em 0, e ambos teriam o algarismo 0
em comum. Portanto, temos os casos a seguir.
OBMEP 2010

233

Solues do Nvel 2
I
1 ... 2
2
......................................
......................................
3 ... 4

II
1 ... 3
2
......................................
......................................
2 ... 6

III
1 ... 4
2
......................................
......................................
2 ... 8
ou
3

IV
1 ... 5
2
......................................
......................................
2 ... 0
ou
3

V
1 ... 6
2
......................................
......................................
3 ... 2

VI
1 ... 7
2
......................................
......................................
2 ... 4
ou
3

VII
1 ... 8
2
......................................
......................................
2 ... 6
ou
3

VIII
1 ... 9
2
......................................
......................................
2 ... 8
ou

Vamos, agora, determinar todas as possibilidades para cada caso, lembrando sempre
que o nmero e seu dobro no podem ter algarismos comuns.
Caso I temos trs possibilidades:
152 2 = 304 ,

182 2 = 364 ,

192 2 = 384 .

Caso II temos duas possibilidades:


143 2 = 286 ,

153 2 = 206 .

Caso III temos trs possibilidades:


134 2 = 268 ,

154 2 = 308 ,

164 2 = 328 .

Caso IV temos trs possibilidades:


135 2 = 270 ,

145 2 = 290 ,

185 2 = 370 .

Caso V temos duas possibilidades:


176 2 = 352 ,

186 2 = 372 .

Caso VI no h nenhuma possibilidade.


Caso VII temos trs possibilidades:
138 2 = 276 ,

148 2 = 296 ,

178 2 = 356 .

Caso VIII temos duas possibilidades:


139 2 = 278 ,

179 2 = 358 .

Assim, temos 3 + 2 + 3 + 2 + 3 + 3 + 2 = 18 solues para esse problema, a saber:


134, 135, 138, 139, 143, 145, 148, 152, 153,
154, 164, 176, 178, 179, 182, 185, 186, 192.

234

OBMEP 2010

Solues do Nvel 2
172. Invertendo os algarismos Devemos contar os nmeros a b de dois algarismos que
tm o algarismo b da unidade maior do que o algarismo a da dezena, ou seja, tais que
b > a. Se a = 1, o algarismo b da unidade pode ser 2, 3, 4, 5, 6, 7, 8 ou 9, portanto, temos
oito possibilidades. Se a = 2, o algarismo b da unidade pode ser 3, 4, 5, 6, 7, 8 ou 9,
portanto, temos sete possibilidades. Continuando dessa maneira, chegamos at a = 8,
quando o algarismo b da unidade s pode ser 9, portanto, temos s uma possibilidade.
Claramente, a no pode ser 0 nem 9. Logo, existem 8 + 7 + 6 + 5 + 4 + 3 + 2 + 1 = 36
nmeros entre 10 e 99 tais que, invertendo a ordem de seus algarismos, obtemos um
nmero maior do que o nmero original.

173. Razo entre segmentos Se o arco P R o dobro do arco RQ, vale a mesma relao
entre os ngulos centrais, ou seja, P OR = 2 ROQ. Como P OR + ROQ = 180 , segue
que
180 = 2 ROQ + ROQ = 3 ROQ,
donde ROQ = 60 . Mas, OR = OQ o raio do crculo, de modo que o tringulo ORQ equiltero. Assim, sua altura RM tambm a mediana, ou seja,
1
OM = M Q. Se r o raio do crculo, ento OM = M Q = r e
2
1
r+2r
PM
P O + OM
= 3,
=
= 1
MQ
MQ
r
2

ou seja, a razo entre P M e M Q 3.


174. Tringulos Vamos supor que a, b e c sejam os comprimentos dos lados do tringulo.
No h perda de generalidade em supor que a b c, de modo que a + b + c 3c.
Como cada lado de um tringulo menor do que a soma dos outros dois, temos que
c < a + b e, portanto, obtemos 2c < a + b + c 3c. Mas, a + b + c = 15, de modo que
2c < 15 3c e, como c um nmero inteiro, 5, 6 ou 7 so as nicas opes para c.

Se c = 5, ento a + b = 10 e temos uma nica possibilidade, a = b = c = 5. Se


c = 6, ento a + b = 9 e temos duas possibilidades para a, 3 ou 4, caso em que,
necessariamente, b 6 ou 5, respectivamente. Se c = 7, ento a + b = 8 e temos quatro
possibilidades para a, 1, 2, 3 ou 4. Nesses casos, necessariamente, b 7, 6, 5 ou 4,
respectivamente. Assim, no total, temos sete desses tringulos.

175. Nmero interessante Suponhamos que N seja um dos nmeros procurados. Como
N e 119 deixam os mesmos restos quando divididos por 2, 3, 4, 5 e 6, temos que
a diferena N 119 entre eles deixa resto zero quando dividido por esses nmeros.
Portanto, N 119 um mltiplo de 2, 3, 4, 5 e 6. Como 60 o mnimo mltiplo
comum desses nmeros, N 119 um mltiplo de 60, ou seja, N 119 = 60k, para
algum inteiro k. Assim, N = 119+60k um nmero interessante para qualquer nmero
inteiro k. Como queremos nmeros distintos de 119 e de trs algarismos, devemos tomar
k positivo e menor do que 15, j que 119+6015 = 1 019 tem quatro algarismos. Assim,
tomamos k de 1 a 14 e obtemos outros 14 nmeros interessantes de trs algarismos, a
saber,
179, 239, 299, 359, 419, 479, 539, 599, 659, 719, 779, 839, 899, 959.

OBMEP 2010

235

Solues do Nvel 2
60
176. Time vencedor O time ganhou 60% das 45 j disputadas, ou seja, 45
= 27
100
partidas. Se ele ganhar mais n partidas, a porcentagem de partidas ganhas ser
27 + n
75
3
no de partidas ganhas
=
= 75% =
= .
o de partidas disputadas
n
45 + n
100
4
Logo, 4 (27 + n) = 3 (45 + n), do que resulta n = 27 como o nmero mnimo de
partidas que o time ainda precisa vencer para atingir 75% de vitrias.
177. Brincando com dados Na tabela seguinte, marcamos com os produtos que so
divisveis por 6.

1
2
3
4
5
6

Logo, temos 15 casos favorveis dentre 36 possibilidades. Assim, a probabilidade de


que o produto seja divisvel por 6 15/36 = 5/12 = 41,7%.
178. Contando solues A equao dada equivalente a xy = 144(x+y) = 144x+144y,
144y
portanto, isolando x, obtemos x =
. Como x e y devem ser inteiros positivos,
y 144
o denominador y 144 deve ser um nmero inteiro positivo, digamos, y 144 = n.
Substituindo essa expresso no valor de x, obtemos
x=

1442
144(n + 144)
= 144 +
.
n
n

Como x deve ser um nmero inteiro, n deve ser um divisor de 1442 . Sendo
1442 = 124 = 28 34 , seus divisores so os nmeros d da forma d = 2a 3b , com
0 a 8 e 0 b 4. Como h 9 valores possveis para a e 5 valores possveis para
b, conclumos que 1442 tem 9 5 = 45 divisores.

Assim, para cada divisor n de 1442 , obtemos uma soluo


1442
, n + 144
(x, y) = 144 +
n

xy
= 144 dada. Portanto, essa equao possui 45 pares de nmeros
x+y
inteiros positivos (x, y) que a satisfazem.
da equao

179. Crculos tangentes Denotemos por r1 , r2 e r3 os raios dos trs crculos. Como os
crculos so tangentes dois a dois, temos

r1 + r2 = 3,
r1 + r3 = 4,

r2 + r3 = 5.

236

OBMEP 2010

Solues do Nvel 2
Substituindo os valores r2 = 3 r1 e r3 = 4 r1 na terceira equao, obtemos
3 r1 + 4 r1 = 5. Da, r1 = 1, r2 = 2 e r3 = 3. Logo, a soma das reas dos
trs crculos (12 + 22 + 32 ) = 14 cm2 .

r3
5 cm

r2

3 cm
4 cm
r1

180. Grupo de amigos Se A a quantidade de dinheiro que Joo recebeu de cada um


de seus amigos, ento ele recebeu um total de 3A. Como ele recebeu, de Jorge, um
quinto do seu dinheiro, ento Jorge tinha 5A. Da mesma maneira, Jos tinha 4A e
Jnio tinha 3A. Assim, os trs amigos tinham, juntos, 5A + 4A + 3A = 12A e a frao
do dinheiro do grupo que cou com Joo foi de (3A)/(12A)=1/4, ou seja, uma quarta
parte.
181. Um trapzio Os tringulos AP B e CP D so semelhantes, pois o ngulo AP B
igual ao ngulo C P D (opostos pelo vrtice) e o ngulo ABD igual ao ngulo B DC
(alternos internos).
A

B
P

Como a razo entre suas reas 4/9, temos que a razo de semelhana entre esses
tringulos 4/9 = 2/3 . Logo,
PB
2
= .
DP
3
Por outro lado, os tringulos CP D e P CB tm a mesma altura em relao s
bases DP e P B, respectivamente. Portanto, a razo entre suas reas igual razo
entre suas bases,
PB
2
rea (P CB)
=
= .
rea (CP D)
DP
3
Como rea (CP D) = 9, segue que a rea do tringulo P CB mede 6 cm2 .
OBMEP 2010

237

Solues do Nvel 2
Observao: O mesmo argumento poderia ser usado para mostrar que tambm a rea
do tringulo ADP mede 6 cm2 . As medidas de 4 e 9 cm2 de rea dadas no enunciado
do problema no desempenham papel especial algum. O argumento exposto acima
prova que num trapzio qualquer os tringulos P CB e ADP tm reas iguais,
mesmo que o trapzio no seja equiltero.
40
182. Vista ruim Seja A o nmero total de alunos da sala. Sabemos que
A no
100
70
40
enxergam bem, portanto,

A usam culos. Assim,


100 100
40
21 100
70

A = 21 ou seja, A =
= 3 25 = 75 .
100 100
74
183. Idade mdia da populao de Campo Verde Se H indica o nmero de homens
e M o de mulheres, ento H/M = 2/3, de modo que M = (3H)/2 e, portanto, a
populao de Campo Verde dada por
3
H +M =H + 2H = 5H.
2

Se a idade mdia dos homens 37 anos, ento


37 = idade mdia dos H homens =

soma das idades de todos homens


,
H

de modo que 37 H a soma das idades de todos os homens. Da mesma forma, 42 M


a soma das idades de todas as mulheres. Segue que a soma das idades de toda a
populao dada por
3
37H + 42M = 37H + 42 2 H = 100H.

Assim, a idade mdia da populao de Campo Verde


37H + 42M
100H
100 2
= 5H =
= 40 anos.
H +M
5
2
184. rea de tringulo Os tringulos ABC e CBD tm bases AC e CD, respectivamente, e a mesma altura h em relao a essas bases.
B

.
.
........
........
.. .
......... . ..
......... ...
. .
...... ... . ..
.
...... .... . ...
. .
. .
...... ...
...... ....
. .
. .
. ..
...... ......
...... .......
.
.
.....
.....
. ...
..
.
...
...
......
.
......
..
.
..
....
.
......
....
......
.
.
.
..
...
..
..
.
.
....
......
....
......
.
.
..
.
..
..
.
..
.
....
......
......
..
....
.
.
.
..
..
...
.
. h
..
.
....
......
......
....
..
.
.
.
..
...
.
.
..
.
..
....
......
......
....
.
.
.
..
...
.
..
.
..
....
......
.
......
....
.
..
.
...
..
.
..
.
....
......
....
.
......
.
.
..
..
.
...
..
.
.
....
......
....
......
.
..
.
.
..
..
.
..
.
.
....
..
......
....
......
.
..
.
.
.
..
....
.....
.
...
.......................................................................................................................................
.......................................................................................................................................

Assim, temos
rea ABC =
238

AC h
2

e rea CBD =

OBMEP 2010

CD h
.
2

Solues do Nvel 2
Logo, a relao entre as reas dada por
rea ABC
=
rea CBD

ACh
2
CDh
2

AC
1,5
15
3
=
=
= .
CD
4 1,5
25
5

Lembrete: A rea de um tringulo a metade do produto de um dos seus lados pela


altura h relativa a este lado, como exemplicado nas duas guras a seguir.
B

..
..
.... ..
.... ...
.... ...
..
....
..
....
....
..
..
....
....
..
..
..
..
..
....
....
..
..
....
....
.
.
..
..
....
....
.
..
.
..
....
....
.
.
.
....
....
.
.
h ..........
....
....
.
.
..
..
....
....
.
..
.
..
....
....
..
.
..
...
..
....
..
...
..
....
..
....
.
.
..
....
....................................................................................
.....................................................................................

...
...
..
.......
........
..
...... ...
...... ....
...... ...
...... ....
...... ......
...... .......
...
....
..
......
......
....
....
......
......
....
....
......
......
....
...
......
......
....
......
....
......
...
....
....
....
......
......
....
....
......
......
....
....
......
......
....
....
......
......
....
......
....
......
.
.
....
....
....
....
......
......
....
....
......
......
....
......
....
......
......................................................
......................................................
...
..

CD h
rea do CBD =
2

rea do ABC =

AC h
2

185. Construindo quadrados perfeitos Sim, ser sempre um quadrado perfeito. De


fato, se n 1, n, n + 1 e n + 2 so quatro inteiros consecutivos, ento seu produto mais
1 um quadrado perfeito, como segue.
(n 1)n(n + 1)(n + 2) + 1 = n(n2 1)(n + 2) + 1
= n(n3 + 2n2 n 2) + 1
= n4 + 2n3 n2 2n + 1
= n4 + 2n3 + (n2 2n2 ) 2n + 1
= (n4 + 2n3 + n2 ) 2n2 2n + 1
= (n2 + n)2 2(n2 + n) + 1
2

= (n2 + n) 1 .

186. Feira de Cincias Sejam x e y o nmero de alunos do ensino fundamental e mdio,


respectivamente, presentes na feira. Sabemos que o nmero daqueles que compraram
um adesivo x/2 do ensino fundamental e y/4 do ensino mdio, portanto, o nmero
daqueles que no compraram um adesivo x/2 do ensino fundamental e 3y/4 do ensino
mdio. Dentre os alunos que no compraram adesivos, os do ensino mdio representam
o dobro dos do ensino fundamental. Logo,
3y
x
=2 ,
4
2

ou seja, x =

3y
4

x
3y
=
.
2
8

Sabendo que o total arrecadado foi de R$ 38,00, estabelecemos que


38 = 0,30

y
3y
y
1,90
x
+ 0,50 = 0,30
+ 0,50 =
y,
2
4
8
4
8

de modo que y = 160 e, como x = 3y/4, segue que x = 120.


OBMEP 2010

239

Solues do Nvel 2
187. Par perfeito Denotemos por n o nmero natural candidato a formar um par
perfeito com 122. Ento devemos ter 122 + n = A2 e 122 n = B 2 , onde A e B
so nmeros naturais. Como B 2 = 2 61 n, os fatores primos 2 e 61 de B 2 devem
aparecer um nmero par de vezes, o que garante que n tem os fatores primos 2 e 61,
ou seja, n = 2 61 m2 = 122 m2 , para algum natural positivo m. Decorre disso que
A2 = 122 + 122 m2 = 122(1 + m2 ).
O menor valor de (1 + m2 ) que satisfaz essa igualdade 1 + m2 = 122, ou seja,
m2 = 121 e m = 11. Consequentemente, n = 122 121 e conclumos que
A2 = 122 + 122 121 = 1222 e B 2 = 122 122 121 = (122 11)2 . Assim, 122
e 122 121 formam um par perfeito.

Observao: Na verdade, 122 121 o menor natural que forma um par perfeito com
122. Ser que existem outros?

188. Um trapzio A resposta correta (d).


Seja P o ponto mdio do segmento DC e tracemos os
segmentos AP e BP . Os trs tringulos assim formados,
ADP, AP B e BP C, so equilteros (porqu?), de
modo que DAP = 60 = P AB. Como o segmento AC
a bissetriz do ngulo P AB (porqu?), conclumos que
P AC = 30 . Assim,

.
........................................
..
..........................................
...
.. .
.. ..
.. ...
.. ...
.. ...
.
.
.. ..
..
..
..
..
.
..
..
..
..
..
.
.
.
..
..
..
..
.
..
..
.
..
..
.
..
..
..
..
..
.
..
..
.
..
..
.
..
.
..
..
..
..
.
.
..
..
..
.
.
..
..
..
..
.
.
.
.
.
..
..
..
..
..
.
.
.
..
..
.
.
.
.. ...
..
..
.. ..
.
.
..
..
.
.
.
..
........................................ .......................................
................................................................................
.
.
..

DAC = DAP + P AC = 60 + 30 = 90 .
189. Mistrio das bolas Seja m o nmero de bolas pretas na primeira urna e n o de bolas
brancas na segunda urna. Inicialmente, Henrique retirou k bolas pretas da primeira
urna e as colocou na segunda urna. Nesse ponto, a situao a seguinte:
na 1a urna temos m k e
pretas

na 2a urna temos

n
brancas

+ k

pretas

Depois, ele retirou k bolas da segunda urna e as colocou na primeira urna. Agora, esse
grupo de k bolas pode ter bolas brancas e pretas. Denotemos por p o nmero de bolas
pretas e por b o de bolas brancas retiradas da 2a urna. Ento k = b + p e
na 1a urna temos m k + p +
pretas

na 2a urna temos

n
brancas

pretas

+ k
pretas

b
brancas

b
brancas

= m k + p+
pretas

brancas

p = n b +k p .
pretas

brancas

pretas

Assim, ele cou com b bolas brancas na primeira urna e k p bolas pretas na segunda
urna. No entanto, k = p+b, ou b = k p. Logo, o nmero de bolas brancas na primeira
urna igual ao nmero de bolas pretas na segunda urna.
240

OBMEP 2010

Solues do Nvel 2
190. Contando a palavra BRASIL Para ler a palavra BRASIL, devemos percorrer
um caminho que comece numa letra B e termine na letra L. Observemos que o caminho a ser percorrido composto sucessivamente de deslocamentos horizontais para a
direita e verticais para baixo. Representemos esses caminhos por sequncias de letras
H (signicando deslocamento horizontal para a direita) e V (signicando deslocamento
vertical para baixo). Vejamos dois exemplos.
(i) Comeamos em B na segunda linha (de cima para baixo) e seguimos o caminho
VHVVV.
(ii) Comeamos em B na quarta linha e seguimos o caminho HVVHH.
Para resolver o problema devemos contar quantos caminhos comeam com B e terminam com L. Para istso, vamos listar esses caminhos, escrevendo Cj para o nmero de
caminhos que comeam com o B da linha j, em que j varia de 1 a 6, como segue.
(1) Primeira linha: VVVVV ; C1 = 1;

(2) segunda: HVVVV, VHVVV, VVHVV, VVVHV, VVVVH ; C2 = 5;

(3) terceira: HHVVV, HVHVV, HVVHV, HVVVH, VHHVV, VHVHV,


VHVVH, VVHHV, VVHVH, VVVHH ; C3 = 10;

(4) quarta: HHHVV, HHVHV, HHVVH, HVHHV, HVHVH, HVVHH,


VHHHV, VHHVH, VHVHH, VVHHH ; C4 = 10;

(5) quinta: HHHHV, HHHVH, HHVHH, HVHHH, VHHHH ; C5 = 5;

(6) sexta: HHHHH ; C6 = 1.

Portanto, a palavra BRASIL aparece


C1 + C2 + C3 + C4 + C5 + C6 = 1 + 5 + 10 + 10 + 5 + 1 = 32
vezes na gura.
Observao: O que signica a simetria C1 = C6 , C2 = C5 e C3 = C4 ?
191. Quais so os nmeros? A equao pode ser escrita na forma x4 y 2 = 71 e,
fatorando x4 y 2 = (x2 y)(x2 + y), na forma
(x2 y)(x2 + y) = 71.
Como x e y so inteiros, cada um dos fatores x2 y e x2 + y tambm um nmero
inteiro, de modo que escrevemos 71 como o produto de dois nmeros inteiros. Como
71 um nmero primo, ele s pode ser escrito como produto de inteiros na forma
1 71 ou 71 1. Assim, temos somente dois casos a considerar, a saber, x2 y = 71
e x2 + y = 1, ou x2 y = 1 e x2 + y = 71. Como x, y so inteiros positivos, temos
x4 = y 2 + 71 72 > 16 = 24 , portanto, x > 2. Em particular, x2 + y = 1 impossvel,
pois implicaria 1 = x2 + y 9 + 1 = 10.
Assim, resta considerar o caso x2 y = 1 e x2 + y = 71. Somando essas duas equaes,
obtemos 2x2 = 72, o que fornece x = 6 e, portanto, y = (6)2 1 = 35. Como x, y
so inteiros positivos, conclumos que a nica soluo x = 6 e y = 35.
OBMEP 2010

241

Solues do Nvel 2
192. No jogo Seja T a quantidade total de dinheiro no jogo. Assim, no incio, os jogadores
possuam
7
Aldo:
T,
18
6
Bernardo:
T,
18
5
Carlos:
T
18
e, no nal, eles possuam
Aldo:
Bernardo:
Carlos:

6
T,
15
5
T,
15
4
T.
15

Para comparar essas fraes, usamos o denominador comum de 18 e 15, a saber, 90.
Desse modo, no incio,
7
35
Aldo:
T =
T,
18
90
6
30
Bernardo:
T =
T,
18
90
25
5
T =
T
Carlos:
18
90
e, no nal,
Aldo:
Bernardo:
Carlos:

6
36
T =
T,
15
90
5
30
T =
T,
15
90
4
24
T =
T.
15
90

1
T, que corresponde a 12 reais, de modo
Logo, foi Aldo quem ganhou um total de
90
1
que
T = 12, ou seja, o total T de dinheiro no incio o jogo foi
90
T = 90 12 = 1 080 reais.
Assim, no nal da partida, os jogadores possuiam, em reais,
Aldo:
Bernardo:
Carlos:

242

36
de 1 080 = 432,
90
30
de 1 080 = 360,
90
24
de 1 080 = 288.
90

OBMEP 2010

Solues do Nvel 2

3
3
193. Um nmero inteiro Denotemos a =
5+2 e b=
5 2. Ento

a3 b 3 =
5+2
52 =4
e
ab =

3
( 5 + 2)( 5 2) = 3 5 4 = 1 = 1,

de modo que M = a b satisfaz M 3 = (a b)3 = a3 b3 3a b(a b) = 4 3 M.

Assim, M 3 + 3M 4 = 0, ou seja, o nmero M raiz do polinmio x3 + 3x 4. Como


o nmero 1 uma raiz desse polinmio, podemos fator-lo e escrever x3 + 3x 4 como
(x 1)(x2 + x + 4). O trinmio x2 + x + 4 tem discriminante negativo, de modo que a
nica raiz real de x3 + 3x 4 1 e, portanto, M = 1. Em particular, M um nmero
inteiro.
194. rea de tringulos
(a) Note que F M C e AM D so ngulos opostos pelo vrtice, de
modo que F M C = AM D. Como M C = M D e os tringulos AM D e F M C so retngulos, estabelecemos que eles
so congruentes. Assim, possuem a mesma rea, donde conclumos que a rea do tringulo ABF igual rea do quadrado
ABCD, que foi dada, medindo 300 cm2 .

(b) Como AD = F C (do item anterior) e M C = M D, segue que os tringulos


AM D, DM F e F M C tm a mesma rea. Por outro lado, a soma das reas
dos dois ltimos a metade da rea do quadrado. Portanto, a rea do tringulo
AF D a metade da rea do quadrado ABCD. Essa rea foi dada, medindo
300 cm2 , logo, a rea do tringulo AF D mede 150 cm2 .
195. Um quadriculado Sejam m e n, respectivamente, o nmero de segmentos ao longo
de dois lados consecutivos do retngulo desenhado por Rosa. Sabemos que o nmero
total de segmentos que so lados de quadrados na diviso de um retngulo em m n
quadrados m(n + 1) + n(m + 1) (prove isso). Assim, como Rosa usou 1 997 segmentos
em seu desenho, temos m(n + 1) + n(m + 1) = 1 997.
Alm disso, um dos lados considerados menor do que ou igual ao outro, digamos,
m n. Nesse caso, obtemos 1 997 = m(n + 1) + n(m + 1) 2m(n + 1).

Como 1 998 > 1 997, segue que 1 998 > 2m(m + 1), ou seja, 999 > m(m + 1),
do que podemos deduzir que 1 m 31. Por outro lado, multiplicando, obtemos
1 997 = mn + m + mn + n = m + n(2m + 1), de modo que n = (1 997 m)/(2m + 1)
e, portanto,
3 994 2m
3 995 (2m + 1)
3 995
2n =
=
=
1.
2m + 1
2m + 1
2m + 1
No entanto, n um inteiro positivo, portanto, 2m + 1 precisa ser um divisor de 3 995.
Como 3 995 = 5 17 47 e 1 m 31, as nicas trs opes so 2m + 1 = 5, 17 ou
47, que fornecem m = 2, m = 8 e m = 23 e os valores correspondentes de n = 399,
n = 117 e n = 42.
OBMEP 2010

243

Solues do Nvel 2
Portanto, Rosa poderia ter desenhado trs conguraes diferentes com os 1 997 segmentos, uma com 2 399 quadrados, outra com 8 117 quadrados e uma terceira,
com 23 42 quadrados. Entretanto, a folha de papel utilizada mede 21 por 29,7 cm
e os segmentos que formam os lados dos quadrados medem 0,5 cm. Assim, as duas
primeiras conguraes no cabem no papel de Rosa e podemos armar que o retngulo que Rosa desenhou consiste em 23 42 quadrados e que, portanto, constitudo
de 966 quadrados.
196. Inteiros de quatro algarismos Temos 1 000 4a2 < 10 000, do que decorre
250 a2 < 2 500. Mas, 152 = 225, 162 = 256 e 502 = 2 500, portanto, como a um
4
nmero natural, obtemos 15 < a < 50. Tambm temos 1 000 a3 < 10 000, do
3
que decorre 750 a3 < 7 500. Mas, 93 = 729, 103 = 1 000, 193 = 6 859 e 203 = 8 000,
portanto, como a um natural, tambm temos 9 < a < 20. Desse modo, as nicas
opes so a = 16, 17, 18 ou 19.
4
Por outro lado, como a3 um nmero inteiro, conclumos que a = 18. De fato, isso
3
pode ser obtido substituindo os quatro possveis valores de a ou, ento, observando
que a3 deve ser um mltiplo de 3 e, consequentemente, que a um mltiplo de 3.
197. Pares positivos Como 501 3x = 3(167 x), a equao dada equivalente a
3
y = (167 x). Como y um inteiro positivo, 167 x deve ser algum mltiplo positivo
5
de 5, ou seja, 167 x = 5k, para algum inteiro positivo k e, portanto, x = 167 5k
ou, ainda, x = 5 33 + 2 5k = 5(33 k) + 2. Como x um inteiro positivo, devemos
ter 1 k 33. Consequentemente, podemos tomar qualquer k = 1, 2, . . . , 33, obtendo
trinta e trs pares de inteiros positivos (x, y) que so solues da equao 3x+57 = 501.
198. Diferena de quadrados A resposta correta (e).
Soluo 1: Observe que o quadrado de um nmero par par e o quadrado de um
nmero mpar mpar. Se os dois nmeros so consecutivos, ento um deles par
e o outro mpar. Portanto, elevando ao quadrado, um dos quadrados par e o
outro mpar. Mas, a diferena entre um nmero par e um nmero mpar sempre
um nmero mpar. Como 2.000 um nmero par, conclumos que no existem dois
nmeros consecutivos tais que a diferena de seus quadrados seja 2 000.
Soluo 2: Seja n um nmero inteiro. Ento (n + 1)2 n2 = n2 + 2n + 1 n2 = 2n + 1,
de modo que a diferena entre o quadrado dos nmeros consecutivos n e n+1 , sempre,
um nmero mpar. Como 2 000 um nmero par, conclumos que no existem dois
nmeros consecutivos tais que a diferena de seus quadrados seja 2 000.
199. Clculo de ngulos Na primeira gura, prolongamos o segmento BC at sua
interseo com o segmento ED, num ponto F. Como os segmentos AB e ED so
paralelos, os ngulos ABF e B F D so alternos internos, portanto, possuem a mesma
medida, ou seja, C F D = 25o . Agora, observe que o ngulo x externo ao tringulo
CDF. Logo, x igual soma dos dois ngulos internos no adjacentes, ou seja,
x = 25o + 55o = 80o .
244

OBMEP 2010

Solues do Nvel 2

A...............................................................................................................................B
.

A..............................................................................................................................B
.

.
.
.

.
..
..
..
..

..
..
..
..
..
160o ......
..
.
.

.
25o......
..
..

.
..
..
..
..
..
..
..
..
..
..
..
..
..
..
C.............. x
.
.
. ..
. . ..
.. ...
..
..
..
..
..
..
..
..
..
..
..
..
..
..
..
.
..
.
..
.
..
..
..
..
..
..
..
..
..
..
..
..
.
..
.
..
..
..
..
55o ......
.
.
..
.
................................................................
...............................................................
.

..
..
..
..
..
..
..
..
..
..
..
..
..
.. C
..
.
x.........................
..
.
..
.
..
..
..
..
..
..
..
..
..
..
..
..
.
..
..
..
..
..
..
..
..
..
..
..
..
..
..
..
.
..
..
..
..
..
150o......
.
.
.
.....................................................................................................................
..................................................................................................................
..

Na segunda gura, novamente prolongamos o segmento BC at sua interseo com


o prolongamento do segmento ED, num ponto F. Como os segmentos AB e EF so
paralelos, os ngulos ABF e DF B so colaterais internos, portanto, suplementares,
ou seja,
DF C = DF B = 180o ABF = 180o 160o = 20o .

Por outro lado, o ngulo C DF igual a 30o , por ser o suplemento do ngulo
E DC = 150o . Agora, observe que o ngulo x externo ao tringulo CDF. Logo, x
igual soma dos dois ngulos internos no adjacentes, ou seja, x = 20o + 30o = 50o .
200. Tabela Como a tabela tem seis colunas, em cada linha escrevemos seis
nmeros consecutivos. Dividindo 1 000
por 6, obtemos
1 000 = 6 166 + 4.

1a linha
2a linha
3a linha
.
.
.
a
167 linha

1
7
13
.
.
.
997

2
8
14
.
.
.
998

3
9
15
.
.
.
999

4
10
16
.
.
.
1 000

5
11
17
.
.
.

6
12
18
.
.
.

Desse modo, para escrever o nmero 1 000 na tabela so necessrias 166 linhas completas (terminando no nmero 6 166 = 996) e mais uma linha com os quatro nmeros
997, 998, 999 e 1 000. Logo, 1 000 est escrito na 167a linha e na quarta coluna.
201. Entre 1 e 2 Se uma frao positiva e menor do que 1, seu numerador deve ser
menor do que seu denominador. Assim, devemos ter
0 < a < 5 e 0 < b < 7.
Como

1
1
1
a + b = (7a + 5b), a condio dada equivale a
5
7
35
1<

7a + 5b
< 2,
35

ou seja, 35 < 7a + 5b < 70.

Desse modo, vemos que o problema consiste em obter todos os inteiros a e b tais que
0 < a < 5,

0 < b < 7 e 35 < 7a + 5b < 70.

Examinemos cada uma das quatro opes de a, de 1 a 4, com correspondentes possibilidades de b, dentre os inteiros 1, 2, 3, 4, 5 e 6.
a = 1. Ento 7a + 5b = 7 + 5b e, de 35 < 7 + 5b < 70, decorre que 28 < 5b < 63,
ou seja, 6 b 12. Como b {1, 2, 3, 4, 5, 6}, conclumos que a nica opo
b = 6.
OBMEP 2010

245

Solues do Nvel 2
a = 2. Ento 7a + 5b = 14 + 5b e, de 35 < 14 + 5b < 70, decorre que 21 < 5b < 56,
ou seja, 5 b 11. Como b {1, 2, 3, 4, 5, 6}, conclumos que as nicas opes
so b = 5 e b = 6.
a = 3. Ento 7a + 5b = 21 + 5b e, de 35 < 21 + 5b < 70, decorre que 14 < 5b < 49,
ou seja, 3 b 9. Como b {1, 2, 3, 4, 5, 6}, conclumos que as nicas opes
so b = 3, 4, 5 ou 6.
a = 4. Ento 7a + 5b = 28 + 5b e, de 35 < 28 + 5b < 70, decorre que 7 < 5b < 42,
ou seja, 2 b 8. Como b {1, 2, 3, 4, 5, 6}, conclumos que as nicas opes
so b = 2, 3, 4, 5 ou 6.
Conclumos exibindo as doze solues a, b na tabela seguinte.
a

2
3
4
5
6

1<
1
5
4
5
4
5
4
5
4
5
4
5

a
5

+
6
7
2
7
3
7
4
7
5
7
6
7

+
+
+
+
+
+

b
7

=
=
=
=
=
=

<2

37
35
38
35
43
35
48
35
53
35
58
35

1<

5
6

3
4
5
6

2
5
2
5
3
5
3
5
3
5
3
5

a
5

+
+
+
+
+
+

+
5
7
6
7
3
7
4
7
5
7
6
7

b
7

=
=
=
=
=
=

<2
39
35
44
35
36
35
41
35
46
35
51
35

202. Triatlon Seja x a velocidade com que Maria nada, em metros por minuto. Ento o
tempo que Maria gasta nadando 800 m dado por 800/x minutos. Sabemos que sua
velocidade na corrida 3 x e, na bicicleta, 2,5 3 x = 7,5 x metros por minuto. Assim,
o tempo total que Maria gasta nas trs etapas
800
4 800
4 000
20 000
800 7,5 + 20.000 + 4 000 2,5
=
+
+
=
x
7,5 x
3x
7,5 x
x
nadando

pedalando

correndo

minutos. Logo, para que ela vena as trs etapas em, no mximo, uma hora e vinte
minutos, ou seja, em 80 min, devemos ter, no mnimo, 80 = 4 800/x, ou seja, x =
4 800/80 = 60 metros por minuto. Segue que 3 x = 180 e 7,5 x = 450 metros por
minuto. Assim, para que Maria termine a prova em, no mximo, 1 hora e 20 minutos,
ela deve desenvolver as seguintes velocidades mnimas:
nadar a uma velocidade mnima de 60 m/min,
pedalar a uma velocidade mnima de 450 m/min e
correr a uma velocidade mnima de 180 m/min.

203. Foto de formatura Os diagramas a seguir representam a situao do problema,


onde os alunos que foram inicialmente retirados esto representados em preto e os
alunos retirados na segunda vez, em cinza.
d d d. . . d d d d d
d d d. . . d d d d t
d d d. . . d d d d d t
d d d. . . d d d d d
d d d. . . d d d d t
d d d. . . d d d d d t
. . .
. . . . .
. . .
. . . . .
. . .
. . . . .
. . .
. . . . .
. . .
. . . . .
. . .
. . . . .
. . . . . .
. . .
. . . . . .
. . .
. . . . . .
. . .
d d d. . . d d d d d
d d d. . . d d d d t
d d d. . . d d d d d t
t t t. . . t
t t t. . . t
246

OBMEP 2010

Solues do Nvel 2
Sejam m e n o nmero de las (linhas horizontais) e de colunas da formao inicial,
respectivamente. Com um aluno de cada uma das m las formada uma nova la,
incompleta: faltam quatro alunos para completar as atuais n 1 colunas, ou seja,
m + 4 = n 1 e, portanto, n = m + 5. Agora temos m las de n 1 alunos, alm de
uma la incompleta, em que faltam quatro alunos.
Tirando um aluno de cada uma das m las completas, ford d d. . . d d d d
mamos um retngulo com uma coluna a menos, portanto
d d d. . . d d d d
. . .
. . . .
preenchemos as atuais trs vagas da nova la. Assim, m = 3
. . .
. . . .
. . .
. . . .
e, portanto, n = 8. O nmero total de alunos na foto dado
d d d. . . d d d d
t t t. . . t t t t
por n m = 3 8 = 24.
204. Circunferncias tangentes
(a) Como as circunferncias de 1 e 3 cm de raio so concntricas, as novas circunferncias tangentes s originais tambm devem ter raio igual a 1 cm.
(b) Os centros das trs circunferncias de 1 cm de raio
mostradas na gura formam um tringulo equiltero de
2 cm de lado. Logo, seus ngulos internos medem 60o .
Como 360/60 = 6, conclumos que podem ser dispostas,
no mximo, seis circunferncias sem sobreposio, nas
condies exigidas.

..
....................
......................
... .
.........
..
.........
.. .....
.. .....
..... ..
..
..... ...
....
....
.
.... ..
....
.
...
.
...
.
...
.
.
...
..
.
.
.
.
....
.
..... ..................
.
.
..
. ... . .................
.
..
. ... ..
..
.
.
..
. ... .
..
.
..
..
....
..
.
..
..
..
.
.
.
..
.
. ... ....
.
... . .........
.
..
..
..
..... . .... . ...
.
.
.
..
...
............ .
.
.
.
........... ....
.
.
..
... . ..
...
.
..
.
.... . . .. ..... .
..
.
... . ... ......
.
.
.
.
.
.
. .....
.
..
.. .
. .......
..
.
.. .
.
. .
.
.
.
....... .....
.
.
...... ................. .
... .
.
.
.
. .. ...
.
.
.
.
.
.
.
.
.
.
.
.
. .....
.
.
.
.
.
.
..
.
.
.
..
.
.
.
.
.
..
.
.
..
...
.
.
..
.
..... ......
.
.
.............
.
.
.....
.
.
..
..
.
..
.
..
..
..
..
..
..
..
..
..
..
..
..
...
...
...
...
....
....
....
.....
....
.....
.....
.......
.....
........
.....................
...................

205. Festa na escola Representando o nmero de docinhos que cada um dos quatro
amigos levou pela inicial de seu nome, temos
A + P + M + F = 90,
1
A + 2 = P 2 = 2M = 2 F.
1
Segue da segunda equao que P = A + 4, M = 2 (A + 2) e F = 2(A + 2). Substituindo
esses valores na primeira equao, obtemos
1
90 = A + (A + 4) + 2 (A + 2) + 2(A + 2) = 1 9A + 9 = 9
2

de modo que

1
2

1
2

A+1 ,

A + 1 = 10, ou seja, A = 18. Assim,

P = 18 + 4 = 22,

M=

18 + 2
= 10 e F = 2(18 + 2) = 40.
2

206. Inao O preo antigo era inferior a 50 reais e sofreu um acrscimo de 20%, com
o que o preo novo ainda um nmero de dois algarismos, que representamos por a b,
onde a o algarismo das dezenas e b o algarismo das unidades, ou seja, a b = 10 a + b.
O preo novo o preo antigo b a com um acrscimo de 20%, ou seja,
10 a + b = a b = (1,2) b a = 1,2(10 b + a) = 12 b + 1,2a,
de modo que 10 a 1,2a = 12 b b, ou seja, 8,8 a = 11 b. Assim,
b=

8,8
4
a = a.
11
5

OBMEP 2010

247

Solues do Nvel 2
Como a e b so algarismos, s podemos ter a = 5 e b = 4 e decorre que o novo preo
R$ 54,00.
207. Gatos no condomnio Sejam x o nmero de famlias que possuem apenas um
ou exatamente cinco gatos e y o nmero de famlias que possuem exatamente trs
gatos. Segue que x + y + x = 29 e, portanto, 2x + y = 29. Como o nmero de gatos
x + 3y + 5x = 6x + 3y, obtemos
nmero de gatos = 6x + 3y = 3(2x + y) = 3 29 = 87.
208. Soma constante Sejam a, b, c, d, e
1 a
ou seja, consideremos a tabela b 9
d e
formar, a saber,
1
b

a
9

a
9

2
c

e f os nmeros que falta preencher na tabela,


2
c . Nas quatro subtabelas 2 2 que podemos
f
9
e

b
d

devemos ter

1+a+b+9=a+2+9+c
1+a+b+9=b+9+d+e

a+2+9+c=9+c+e+f

9
e

c
,
f

b1=c
a+1=d+e
ou seja,

a+2=e+f

Subtraindo a segunda igualdade obtida da terceira, obtemos 1 = f 1, ou f = 1 + d.


A nossa tabela, ento, dada como segue.
1
b
d

2
b1
d+1

a
9
e

Para os nmeros a, b, c, d, e e f temos apenas as opes 3, 4, 5, 6, 7 e 8, sem repetio.


Se a = 3, 4 ou 5, resulta d + e = a + 1 = 4, 5 ou 6, o que impossvel para inteiros
distintos d e e maiores do que 2. Se a = 7, resulta d + e = a + 1 = 8 e poderamos
ter (d, e) = (3, 5) ou (5, 3). Se d = 3, ento e = 5, f = d + 1 = 4 e, necessariamente,
b = 6 ou 8: no entanto, isso impossvel, pois implicaria c = b 1 = 5 = e ou
c = b 1 = 7 = a.

Assim, a no pode ser 3, 4, 5 nem 6, restando, apenas, as alternativas a = 6 ou a = 8.


Usando as relaes a + 1 = d + e e c = b 1, obtemos as duas nicas opes de
preenchimento da tabela dada, como segue.
1
8
4

6
9
3

2
7
5

1
5
6

8
9
3

2
4
7

209. Qual o nmero? Note que 5 E um mltiplo de 5 e no caso, terminado em


A. Como A no pode ser 0, segue que A = 5 e E mpar. Observe que E no pode
ser 1, pois, nesse caso, 4D = 5, o que impossvel para algarismos. Logo, E = 3, 5, 7
ou 9. Analisemos cada uma dessas possibilidades.
248

OBMEP 2010

Solues do Nvel 2
Se E = 3, ento 4D + 1 termina em 5 e, portanto, D = 1 ou D = 6;
se E = 5, ento 4D + 2 par e termina em 5, o que impossvel;
se E = 7, ento 4D + 3 termina em 5 e, portanto, D = 3 ou D = 8;
Se E = 9, ento 4D + 4 par e termina em 5, o que impossvel.
Restam, ento, os quatro casos seguintes, de acordo com (D, E) ser dado por (1, 3),
(6, 3), (3, 7) ou (8, 7).
5B C 37
B C 37
C 37
37
7
......................................
......................................
55555

5B C 63
B C 63
C 63
63
3
......................................
......................................
55555

5B C 13
B C 13
C 13
13
3
......................................
......................................
55555

5B C 87
B C 87
C 87
87
7
......................................
......................................
55555

1o Caso: Se D = 1 e E = 3, ento 3C termina em 5 e, como C denota um algarismo,


a nica opo C = 5 = A, o que no pode ocorrer.
2o Caso: Se D = 6 e E = 3, ento 3C + 2 termina em 5, portanto, 3C termina em
3. Como C denota um algarismo, a nica opo C = 1, resultando 2B = 5, o que
impossvel para um algarismo.
3o Caso: Se D = 3 e E = 7, ento 3C + 1 termina em 5, portanto, 3C termina em 4.
Como C denota um algarismo, a nica opo C = 8, resultando 2B + 2 = 5, o que
impossvel.
4o Caso: Se D = 8 e E = 7, ento 3C + 3 termina em 5, portanto, 3C termina em 2.
Como C denota um algarismo, a nica opo C = 4, resultando 2B + 1 = 5, com o
que B = 2.
Assim, a nica soluo A B C D E = 52 487.
210. Proporo triangular Escolhamos o ponto H do segmento BC de tal modo que
o segmento F H seja paralelo ao segmento AE, como na gura dada. Decorre que os
tringulos AEC e F HC so semelhantes, pois tm lados paralelos.

A
F
G
B

Como F C = 2 AF, decorre, por semelhana, que tambm HC = 2 EH. Por outro lado,
os tringulos BHF e BEG tambm so semelhantes, pois tm lados paralelos.
Dessa semelhana e do fato de G ser o ponto mdio do segmento BF, conclumos que
E o ponto mdio do segmento BH. Assim, BE = EH e, portanto,
EC = EH + HC = EH + 2 EH = 3 EH = 3 BE.
Consequentemente, EC/BE = 3.
OBMEP 2010

249

Solues do Nvel 2
211. Nmeros primos entre si Temos 2000 = 16 125 = 24 53 ,
x y
x2 + y 2
N = 2000
+
= 16 125
y x
xy

16 125
(x2 + y 2 )
xy

um inteiro mpar e x < y so inteiros positivos primos entre si.


Soluo 1: Como x e y so primos entre si, possvel mostrar que xy e x2 + y 2 no
tm fatores primos em comum (prove isso), de modo que xy divide 2 000 e, portanto,
x e y dividem 2 000. Alm disso, 16 deve dividir xy, porque N mpar. Como x e y
so primos entre si, 16 divide x ou y.
1o Caso: Se 16 divide x, ento x = 16. De fato, se x > 16, ento x , no mnimo, 16 5 = 80, pois x divide 2 000; como tambm xy divide 2 000, resultaria que
y 25 < 80 = x, o que no permitido. Logo x = 16 e, como x e y so primos entre
si e y divide 2 000, necessariamente y = 25 ou 125.
2o Caso: Se 16 divide y, uma possibilidade y = 16, quando x s pode ser 1 ou 5,
pois x < y e xy divide 2 000. As outras opes para y so 16 5, 16 25 ou 2 000,
quando a nica opo para x , sempre, 1.
Assim, existem sete solues, a saber,
(16, 25), (16, 125), (1, 16), (5, 16), (1, 80), (1, 400) e (1, 2 000).
Soluo 2: Como N um inteiro mpar, resulta que (24 53 )/(xy) e x2 + y 2 so
inteiros mpares. As opes para xy so 24 , 24 5, 24 52 e 24 53 . Alm disso, x e y
devem ter paridades distintas, para garantir que x2 + y 2 seja mpar. Vamos determinar
x e y para cada uma dessas opes, lembrando que x < y.
xy
24
24 5
24 52
24 53

x
1
1
5
1
24
1
24

y
24
24 5
24
4
2 52
52
24 53
53

Assim, existem sete solues, a saber,


(1, 16), (1, 80), (5, 16), (1, 400), (16, 25), (1, 2 000) e (16, 125).
212. Fique atento Elevando ambos os membros da equao ao quadrado, obtemos x =
x2 4x + 4, que equivalente a x2 5x + 4 = 0. As razes dessa equao segundo
do
grau so = 1 e x = 4. Quando substitumos x = 4 na equao original x = x 2,
x
obtemos 4 = 2, que uma armao verdadeira. Entretanto, quando substitumos

x = 1 naquela equao, obtemos 1 = 1, que falsa. Portanto, a equao dada


possui a nica soluo x = 4.
Observao: O aparecimento da soluo estranha x = 1 deve-se ao fato seguinte: a
armao
a2 = b2 = a = b
250

OBMEP 2010

Solues do Nvel 2
no verdadeira. O que correto a armao
a2 = b 2

a = b.

Desse modo, quando elevamos os dois membros de uma equao ao quadrado, obtemos uma nova equao que pode, eventualmente, conter mais solues que a equao
original. Voc pode ver isso com clareza, por exemplo, nas equaes x = 1 e x2 = 12 .
213. Solues inteiras A equao original pressupe x = 0 e y = 0, portanto, podemos
considerar a equao equivalente
xy = 19(x + y),

com x = 0, y = 0.

Uma vez que estamos procurando solues inteiras e 19 um nmero primo, essa
igualdade implica que x ou y deve ser divisvel por 19. Como a equao simtrica em
relao s variveis x e y, podemos supor que x divisvel por 19, ou seja, x = 19 k,
para algum valor k = 0 inteiro. Assim, a equao original tambm equivalente a
k y = 19 k + y,

com k = 0, y = 0.

Dessa igualdade, obtemos que 19k + y divisvel por k. Uma vez que 19k j divisvel
por k, conclumos que y divisvel por k (prove isso), isto , y = km, para algum valor
m = 0 inteiro. Segue que
k k m = k y = 19 k + y = 19 k + k m = (19 + m)k,
ou seja, k m = 19 + m, que igual a 19 = (k 1) m. Desse modo estabelecemos que os
inteiros m e k1 so divisores do numero primo 19. Como k = 0, segue que k1 = 1,
o que nos deixa com trs opes apenas, como segue.
Se m = 19 e k 1 = 1, ento x = 38 e y = 38;

Se m = 1 e k 1 = 19, ento x = 380 e y = 20;

Se m = 1 e k 1 = 19, ento x = 342 e y = 18.

Desse modo, por simetria, obtemos os nicos cinco pares de nmeros inteiros (x, y) que
so solues da equao dada:
(38, 38), (380, 20), (342, 18), (20, 380) e (18, 342).
214. No ponto de nibus Representemos por M o nmero de meninas e por H o
nmero de meninos que estavam no ponto antes de passar o primeiro nibus. Depois
do embarque das 15 meninas no primeiro nibus, caram M 15 meninas e H meninos
no ponto. Uma vez que, nesse momento, caram dois meninos para cada menina no
ponto, temos H = 2(M 15). No segundo nibus, embarcam 45 meninos e caram
M 15 meninas e H 45 meninos no ponto. Como, nesse momento, caram no ponto
cinco meninas para cada menino, temos M 15 = 5(H 45). Assim, obtemos o sistema
linear
H = 2(M 15)
M 15 = 5(H 45)

Substituindo a primeira equao na segunda, resulta M 15 = 5(2M 30 45) =


10 M 375.
Logo, 9 M = 360, de modo que M = 40 e H = 2(40 15) = 50.
OBMEP 2010

251

Solues do Nvel 2
215. Contorno circular Sejam A, B, C e D os centros dos quatro crculos e M, N, P e
Q os pontos de tangncia entre esses crculos, conforme gura.

Q
D

B
N

Observe que AD = DC = CB = BA = AC = 2a. Logo, os tringulos ABC e


ACD so equilteros e, por isso, seus ngulos internos so iguais a 60o . Portanto,
1
1
ABC = 60o = 360o e DAB = 120o = 360o , o que acarreta que os arcos dos
6
3
contornos internos a esses dois ngulos medem

NM=

1
1
2 a e M Q= 2 a
6
3

5
e os contornos externos por B e A, de traado destacado, medem
2 a e
6
2
2 a, respectivamente. Por simetria, segue que o contorno externo da gura dada
3
tem comprimento igual a
2

2
5
+2
2 a = 6 a.
6
3

216. Um quadriltero especial Como cada diagonal divide o quadriltero em duas


regies de mesma rea, temos
rea (ABD) = rea (BCD) e rea (ABC) = rea (ACD).
Denotemos as reas das quatro regies determinadas pelas diagonais por X, Y, Z e W,
conforme a gura, de modo que
rea (ABD)
rea (BCD)
rea (ABC)
rea (ADC)

=
=
=
=

X + W,
Y + Z,
Z +W e
X + Y.

Assim,

Z X = rea (ABC) rea (ABD)

= rea (ACD) rea (BCD) = X Z

252

B
.
..
........
........
........ ....
........ ... ..
........ ...... .
........ ...... .
........
.
........
.
.
...
.
........
...
........
.
...
........
.
...
........
.
...
........
........
.
...
.
...
........
.
...
........
.
...
........
.
........
...
.
...
........
........
.
...
.
...
........
.
........
...
.
...
A..................................
.
...
.
...
...
.
..
.
...
...
....
.
W
. ....
.
...
...
. .........
.
. .........
.
...
...
.....
.
.
.
.
.....
...
...
.
.
.....
.
.
.
.....
..
.
.
.....
...
.
.
.....
...
.
.
.....
...
.
.
...
.....
.
.
...
.....
.
.
...
.....
.
.
..... ......
Z
.
.
..... ......
.
.
...... E
.
.
.....
.
.
.
.
..
.. ........
.
.
.
.
... .........
.....
...
.
.
.
.
...
.....
...
.
.
.....
X
.
.
...
.....
...
.
.
.....
.
.
...
.....
...
.
.
.....
.
.
...
...
.
.
.....
.....
.
.
...
...
.
.
.....
.....
.
.
...
.
.
...
..... .
..... .
.
...
.
...
....
...
.
...
.
...
.
.
...
........
........
.
...
Y
.
...
.
C
........
...
........
.
....
....
.
..
.
...
........
.
...
........
.
...
........
.
........
...
.
...
........
........
.
...
.
...
........
.
...
........
.
.....
..
...
........
.
.
........
...
...
........
.
.
........
...
...
.
........
. .. ...............
........
. ...
. ...
. ... .......
.................
..............
.
. ...
.....

OBMEP 2010

Solues do Nvel 2
e, portanto, Z = X. Consequentemente, tambm temos Y = W. Seja E o ponto de
corte das diagonais. Como as reas das regies opostas por E so iguais, resulta da
semelhana de tringulos que EA ED = EB EC e EA EB = EC ED.

Dividindo essas duas equaes, obtemos

ED
EB
=
,
EB
ED
portanto, ED = EB. Analogamente, podemos mostrar que EA = EC. Logo, as
diagonais se cortam no ponto mdio e, consequentemente, o quadriltero um paralelogramo. Como os lados medem 10 e 15 cm, o permetro do quadriltero mede
2 10 + 2 15 = 50 cm.
217. Nmero curioso Seja a b = 10 a + b um nmero de dois algarismos a e b que
divisvel pela soma a + b de seus algarismos. Note que, por ser de dois algarismos,
necessariamente a = 0 e que, por ser divisvel pela soma de seus algarismos, tambm
a diferena (10 a + b) (a + b) = 9a divisvel por a + b (prove isso). Assim, basta
atribuir os valores 1, 2, 3, 4, 5, 6, 7, 8 e 9 a a e calcular os valores de b para os quais
a + b divide 9 a. O resultado aparece na tabela.
a
1
2
3
4
5
6
7
8
9

9a
9
18
27
36
45
54
63
72
81

b
0, 2, 8
0, 1, 4, 7
0, 6
0, 2, 5, 8
0, 4
0, 3
0, 2
0, 1, 4
0

Assim, os nmeros que satisfazem a propriedade so


10, 12, 18, 20, 21, 24, 27, 30, 36, 40, 42, 45, 48, 50, 54, 60, 63, 70, 72, 80, 81, 84 e 90,
ou seja, existem 23 nmeros curiosos.
218. Nmero premiado
(a) O maior nmero premiado de seis algarismos distintos precisa comear com 98,
portanto, o nmero procurado da forma 98 c d e f. Por hiptese, temos 9+8+c =
d + e + f. Para que c seja mximo, precisamos que d + e + f seja mximo, e isso
acontece quando d = 7, e = 6 e f = 5. Nesse caso, c = 1 e, consequentemente,
o maior nmero premiado 981 765. Para determinar o menor nmero premiado
de seis algarismos distintos, tentamos um nmero da forma 10 c d e f. No difcil
vericar que 108.234 o menor nmero premiado.
(b) Dado qualquer nmero premiado a b c d e f de seis algarismos distintos, seu par
simtrico d e f a b c tambm premiado e tem seis algarismos distintos; a soma
OBMEP 2010

253

Solues do Nvel 2
desse par simtrico
a b c d e f + d e f a b c = (1000 a b c + d e f ) + (1000 d e f + a b c)
= 1001(a b c + d e f ) = 13 11 7 (a b c + d e f ),
que divisvel por 13. Assim, a soma de todos esses pares simtricos tambm
divisvel por 13. Como a soma de todos esses pares de nmeros premiados simtricos igual soma de todos os nmeros premiados de seis algarismos distintos,
resulta que essa soma divisvel por 13.
Observao: De fato, a soma de todos os nmeros premiados de seis algarismos distintos
tambm divisvel por 7 e por 11.
219. Altura versus lado Sejam ha e hc as alturas relativas aos lados BC = a e AB = c,
respectivamente. Por hiptese, temos que ha a e hc c. Como ha e hc so os
comprimentos das alturas, pelo Teorema de Pitgoras temos ha c e hc a. Um dos
lados considerados maior do que ou igual ao outro, digamos, a c. Das desigualdades
acima, obtemos ha a c ha e, portanto, a = c = ha . Assim, AB perpendicular
a BC e, portanto, o tringulo retngulo issceles. Conclumos que os ngulos do
tringulo medem 45 , 45 e 90 .
220. Fraes egpcias A equao original pressupe x = 0, y = 0 e pede solues inteiras,
positivas e distintas, portanto, podemos considerar a equao equivalente
2xy = 7(x + y),

com x > 0, y > 0 e x = y.

Como 2 e 7 so nmeros primos, segue que 7 divide x ou y. Como a equao simtrica


em x e y, podemos supor que 7 divide x. Ento, x = 7 k, para algum k > 0 inteiro e
decorre que 2 7 k y = 7(7 k + y), ou seja, simplicando, 2 k y = 7 k + y ou, ainda,
(2k 1) y = 7 k = x.
Se 7 dividisse y, teramos y = 7 m, para algum m > 0 inteiro. Nesse caso, teramos
49 2 k m = 2xy = 7(x + y) = 49(k + m), acarretando 2 k m = k + m. Mas, ento
2=

k+m
1
1
= +
1 + 1 = 2,
km
k m

o que signica que k = m = 1 e, portanto, x = 7 = y. Como queremos x = y,


conclumos que 7 no divide y, de modo que 7 divide 2k 1. Tomando k = 4, resulta
x = 28 e
7k
28
y=
=
= 4,
2k 1
7
fornecendo a soluo
1
1
2
+ = .
28 4
7
Observao: A soluo obtida nica. De fato, como 2k 1 , sempre, mpar e 7
divide 2k 1, o mltiplo de 7 que igual a 2k 1 deve ser mpar. Assim, existe algum
inteiro n > 0 tal que
7(2n 1) = 2k 1.
254

OBMEP 2010

Solues do Nvel 2
Isso acarreta que k = 7n 3 e, portanto,
y=

7k
7(7n 3)
7n 3
3(2n 1) + n
n
=
=
=
=3+
.
2k 1
7(2n 1)
2n 1
2n 1
2n 1

Como y deve ser inteiro, conclumos que 2n 1 divide n, de modo que 2n 1 n. No


entanto, n 1 e, portanto, 2n 1 n. A nica possibilidade 2n 1 = n e, portanto,
n = 1. Segue que k = 4 = b e a = 28 do a nica soluo.
221. Tabuleiro de xadrez Um tabuleiro de xadrez um quadrado reticulado de 64
quadradinhos, denominados casas, sendo 32 casas pretas e 32 brancas, posicionados
alternadamente. Uma das peas do xadrez recebe o nome de bispo, havendo um par
deles para cada jogador. Um dos dois bispos de um jogador s se movimenta pelas
casas pretas e o outro s pelas brancas.
Inicialmente, possvel colocar um dos dois bispos
em qualquer uma das 64 casas. Se o bispo estiver
numa casa branca, ento na la em que ele est,
bem como na coluna, temos quatro casas pretas que
no podem ser ocupadas pelo segundo bispo, num
total de oito casas. Assim, o segundo bispo pode ser
colocado em qualquer uma das 32 8 = 24 casas
pretas restantes.
Conclumos, ento, que se um dos bispos ocupar uma
o outro ter 24 casas pretas disposio. Portanto, o
distintas que podem ser obtidas 32 24 = 768.

Nota:
Aqui estamos entendendo que alternando a
posio desses dois bispos no
muda a congurao no tabuleiro de xadrez. Mais precisamente, os bispos tm a mesma cor, isto , pertencem a um
mesmo jogador.

das 32 casas brancas, ento


nmero dessas conguraes

222. Quem menor? Observemos que:


3312 > 3212 = (25 )12 = 260 ;
6310 < 6410 = (26 )10 = 260 ;
1278 < 1288 = (27 )8 = 256 < 260 .
Logo, o maior dos nmeros 3312 . Por outro lado,
127
63

= 2+

1
63

=4+

1
127
=2+
garante que
63
63

1
5
4
+ 2 <4+
<5
63 63
63

e, portanto,

127 4
< 25 < 63.
63
Assim, 1274 < 635 , acarretando 1278 < 6310 . O menor dos trs nmeros dados 1278 .

223. Brincando com nmeros Como queremos encontrar o maior nmero que seja
divisvel pela soma de seus algarismos e tambm menor do que 900, podemos comear
nossa busca dentre os nmeros com o algarismo 8 na casa da centena, j que, no
mnimo, 800 divisvel pela soma 8 + 0 + 0 = 8 de seus algarismos e 899 no tem essa
propriedade. Assim, vamos examinar os nmeros entre 800 e 899.
Queremos, ento, encontrar algarismos b e c tais que 8 + b + c divida
8 b c = 800 + 10b + c. Lembrando que 8 + b + c divide 8 b c = 800 + 10b + c se, e
OBMEP 2010

255

Solues do Nvel 2
somente se, 8 + b + c divide 800 + 10b + c (8 + b + c) = 792 + 9b, basta procurar entre
os divisores de 792 + 9b. Para isso, atribumos valores para b em ordem decrescente, a
partir de 9, at encontrar o maior nmero procurado.
Se b = 9, ento 792 + 9 9 = 873 = 9 97 e esse nmero no possui divisor
8 + 9 + c entre 17 (c = 0) e 26 (c = 9).
Se b = 8, ento 792 + 9 8 = 864 = 25 33 . O maior divisor 8 + b + c desse
nmero entre 16 e 25 24, isto , c = 8.
Logo, o nmero procurado 888.
224. Cortando papis Se na primeira rodada Andr pega n1 pedaos de papel para
cortar cada um deles em sete pedaos, ao nal dessa rodada ele car com 7 n1
pedaos sem cortar, mais 7n1 pedaos cortados, totalizando (7 n1 ) + 7n1 = 7 + 6n1
pedaos de papel. Analogamente, se na segunda rodada Andr pega n2 pedaos de
papel para cortar, ao nal dessa rodada ele car com 7 + 6n1 n2 pedaos que no
foram cortados nessa rodada, mais 7n2 pedaos de papel provenientes dos cortes que
ele fez nessa rodada. Assim, ao nal da segunda rodada, Andr car com
(7 + 6n1 n2 ) + 7n2 = 7 + 6(n1 + n2 ).
Continuando assim, conclui-se que, ao nal de k rodadas, Andr ca com
7 + 6(n1 + n2 + + nk )
pedaos de papel. Ento, para ele car com 2 009 pedaos de papel ao nal de alguma rodada, deveramos ter essa ltima expresso igual a 2 009 ou, equivalentemente,
subtraindo 7 de cada lado, 6(n1 + n2 + + nk ) = 2 002.

No entanto, 2 002 no um mtiplo de 6, de modo que essa equao no admite


soluo. Isso signica que Andr nunca poder car com 2 009 pedaos ao nal de
alguma rodada de sua brincadeira.

225. Um trapzio especial Queremos provar que AE igual a BC. Para isso, suponhamos que AE seja maior do que BC e escolhamos o ponto A sobre AE tal que
EA = BC. Por construo, EA e BC so paralelos, de modo que A BCE um
paralelogramo e, em particular,
A B = CE.

Pela desigualdade triangular, temos


A A + AB > A B.

Logo,

EA + AB + BE = EA + A A + AB + BE
> EA + A B + BE = BC + CE + EB.
Disso decorre que o permetro do tringulo ABE maior do que o permetro do
tringulo BCE, contrrio aos dados do problema.
256

OBMEP 2010

Solues do Nvel 2
Por meio dessa contradio, estabelecemos que, diante das hipteses do problema,
AE no pode ser maior do que BC. Por um processo totalmente anlogo, tambm
podemos estabelecer que, reciprocamente, BC no pode ser maior do que AE, com o
que conclumos que BC = AE. O mesmo raciocnio pode ser utilizado para mostrar
que BC = ED. Assim,
BC = 1 (AE + ED) = 15 cm.
2
A

226. Uma estrela Observe que J EI = B EH.


No tringulo BEH temos
B

20o + 130o + B EH = 180o ,

20

portanto,

J
130

E
I

H
o

J EI = B EH = 30 .

227. Nmero palndromo Um nmero palndromo de quatro algarismos da forma


a b b a, onde a um algarismo entre 1 e 9 e b um algarismo entre 0 e 9. Como o
nmero divisvel por 9, ento a soma 2a + 2b = 2(a + b) de seus algarismos divisvel
por 9, ou seja, a + b divisvel por 9. Como 1 a + b 18, as nicas opes so
a + b = 9 ou 18. Se a + b = 18, necessariamente a = b = 9. Se a + b = 9, temos as nove
solues seguintes.
a=1 e b=8

a=2 e b=7

a=3 e b=6

a=4 e b=5

a=5 e b=4

a=6 e b=3

a=7 e b=2

a=8 e b=1

a=9 e b=0

Assim, existem dez nmeros palndromos de quatro algarismos divisveis por 9, a saber,
1 881, 2 772, 3 663, 4 554, 5 445, 6 336, 7 227, 8 118, 9 009 e 9 999.
228. Multiplicao com letras Como o produto de b por c termina em 1, ento b c
pode ser 21 ou 81 e, portanto, 37 ou 99. A nica possibilidade de escrever o produto
de dois nmeros distintos menores do que 10 21 = 3 7. Assim, temos somente dois
casos possveis.
1o Caso: Se b = 7 e c = 3, deveramos ter
a77
3
7371
7371
= 2457 tem quatro algarismos.
3
2o Caso: Se b = 3 e c = 7, temos
mas isso impossvel, pois,

a33
7
3731
OBMEP 2010

257

Solues do Nvel 2
3731
= 533, necessariamente a = 5.
7
Logo, a nica possibilidade a = 5, b = 3 e c = 7.

e, como

229. Nmeros sortudos


(a) A sequncia de oito nmeros consecutivos de 52 a 59 tem, exatamente, dois nmeros sortudos: 52 e 59. Outro exemplo qualquer sequncia de oito nmeros
que contenha 59 e 61, por exemplo, 55, 56, 57, 58, 59, 60, 61, 62.
(b) Dois exemplos so 994, . . . , 1 005 e 7 994, . . . , 8 005. Existem mais: encontre alguns.
(c) Digamos que uma dcada qualquer sequncia de dez nmeros consecutivos cujo
primeiro termo algum mltiplo de 10. Por exemplo,
10, 11, 12, 13, 14, 15, 16, 17, 18, 19
e
140, 141, 142, 143, 144, 145, 146, 147, 148, 149
so dcadas. Note que qualquer sequncia de sete nmeros consecutivos numa
dcada contm, pelo menos, um nmero sortudo, porque a soma de seus algarismos uma sequncia de sete nmeros consecutivos, um dos quais precisa ser
divisvel por 7. Finalmente, qualquer sequncia de treze nmeros consecutivos
contm pelo menos sete nmeros consecutivos de alguma dcada, que sempre
contm um nmero sortudo. (Examine alguns exemplos para melhor entender
essa justicativa.)
230. Uma sequncia especial Inicialmente escrevemos os primeiros termos dessa sequncia, como segue.
1, 3, 2, 1, 3, 2, 1, 3, 2, . . .

O stimo e o oitavo termos so, respectivamente, iguais ao primeiro e ao segundo. Isso


signica que a sequncia se repete de seis em seis termos. A soma dos seis primeiros
termos 1 + 3 + 2 1 3 2 = 0 e, portanto, a soma dos 96 primeiros termos tambm
0. Assim, a soma dos 100 primeiros termos dessa sequncia igual soma dos quatro
ltimos termos, ou seja, 1 + 3 + 2 1 = 5.

231. Tringulos e ngulos... No tringulo menor, dois ngulos medem 70 e


180 130 = 50 e o terceiro mede
Assim,

180 (50 + 70 ) = 60 .

130
50

= 180 60 = 120 .

70

Agora, no tringulo maior, temos

45 + + 50 = 180 ,
portanto,
= 180 95 = 85 .
258

OBMEP 2010

45

Solues do Nvel 3

Solues do Nvel 3
1. Usando velas A opo correta (d).
Com 43 velas a casa de Joo pode ser iluminada por 43 noites, sobrando 43 tocos de
vela. Como 43 = 4 10 + 3, com esses 43 tocos pode-se guardar 3 tocos e fazer 10
novas velas para iluminar 10 noites. Dessas 10 velas obtemos 10 tocos que, com os 3
que haviam sobrado, do 13 tocos. Como 13 = 4 3 + 1, com esses 13 tocos pode-se
guardar 1 toco e fazer 3 novas velas para iluminar 3 noites. Dessas 3 velas obtemos
3 tocos que, com o que havia sobrado, do 4 tocos, com os quais podemos fazer mais
uma vela. Assim, no total, a casa de Joo pode ser iluminada por 43 + 10 + 3 + 1 = 57
noites.
2. Rodas e bandeiras A opo correta (a).
Os dois discos giram em sentidos opostos; quando um gira no sentido horrio o outro
gira no sentido anti-horrio. Considerando que a engrenagem da esquerda girou um
certo ngulo em um sentido, a engrenagem da direita girou o mesmo ngulo no sentido
oposto, e portanto a bandeirinha cou na posio mostrada na opo (a).
3. Nmero de latas A opo correta (a).
Em cada caixote de madeira de dimenses a b c cabem (a b c)/l3 cubos
de lado l, empilhados regularmente. No caso dos palmitos temos, em centmetros,
a = 60, b = 80, c = 120 e l = 20. Como 60, 80 e 120 so mltiplos de 20, podemos
preencher o caixote, sem deixar espaos, com (6080120)/203 = 72 caixas de papelo
de formato cbico com 20 cm de lado. Logo, em cada caixote cabem 72 8 = 576
latas de palmito.
4. Qual a menor frao? A opo correta (c).
n
Soluo 1: As fraes da forma
, com n inteiro positivo, so
n+1
1
2
3
4
5
,
,
,
,
,....
2
3
4
5
6
n=1

n=2

n=3

n=4

n=5

1
2
3
4
<
<
<
< . . . , ou seja, essa sequncia de fraes
2
3
4
5
crescente. Para comparar cada uma dessas fraes com 7/9, precisamos igualar todos
9
14
7 2
6
7 3
27
28
7
1
=
<
= ,
=
< ,
=
<
=
e
os denominadores, obtendo
2
18
18
9 3
9
9 4
36
36
9
36
35
7
4
7
4
=
>
= . Logo, maior do que e, como a sequncia crescente, a partir
5
45
45
9
5
9
4
7
de , todas as fraes dessa sequncia so maiores do que . Assim, existem apenas
5
9
n
7
1 2 3
trs fraes da forma
que so menores do que , a saber, , e .
n+1
9
2 3 4
Soluo 2: Transformando tudo em nmeros decimais, temos 7/9 = 0,777. . . e 1/2
= 0,5, 2/3 = 0,666. . . , 3/4 = 0,75, 4/5 = 0,8, 5/6 = 0,8333. . . . Logo, a sequncia
crescente e apenas 1/2 = 0,5, 2/3 = 0,666. . . e 3/4 = 0,75 so menores do que 7/9 =
0,777. . . .
Observe que temos

OBMEP 2010

259

Solues do Nvel 3
5. Pistas de corrida A opo correta (c).
Soluo 1: Denotemos por x e y os comprimentos das pistas longa e curta, respectivamente. Numa certa semana, o atleta corre 6(x + 2y) e, na outra, 7(x + y). Como nas
duas semanas ele corre os mesmos 5 000 metros, obtemos 6(x + 2y) = 7(x + y). Logo,
6x + 12y = 7x + 7y e, portanto, 5y = x. Assim, a pista longa cinco vezes maior do
que a pista curta.
Soluo 2: Na semana em que o atleta treinou durante sete dias, ele correu uma pista
longa a mais e cinco pistas curtas a menos do que na semana em que ele treinou apenas
seis dias. Como a distncia corrida foi a mesma nas duas semanas, conclumos que o
comprimento da pista longa igual ao comprimento de cinco pistas curtas.
6. Brincos e brincos A opo correta (c).
Soluo 1: Sabemos que o nmero de mulheres que usam apenas um brinco
0,03 800 = 24. Restam 800 24 = 776 mulheres, das quais 388 usam dois brincos e 388 no usam brincos. Logo, o nmero total de brincos usados por todas as
mulheres 24 + 388 2 = 800.
Soluo 2: Se cada mulher com dois brincos emprestar um de seus brincos a uma das
mulheres que no usam brincos, todas as 800 mulheres estaro com um nico brinco.
Logo, o nmero de brincos igual ao de mulheres, ou seja, 800.
7. Perguntas e respostas A opo correta (e).
A partir da tabela obtemos o nmero de pontos de cada um dos trs participantes.
Ana: 5 12 + (3) 3 + (2) 5 = 60 9 10 = 41
Bento: 5 13 + (3) 7 + (2) 0 = 65 21 = 44

Lucas: 5 12 + (3) 4 + (2) 4 = 60 12 8 = 40

Logo, Bento foi o mais bem classicado, seguido de Ana e, depois de Lucas.
8. Qual a carga? A opo correta (b).
Como o peso de um saco de areia igual ao de oito tijolos e no caminho j h 32
sacos de areia, ele pode carregar ainda 18 sacos de areia, o que equivale a 18 8 = 144
tijolos.
9. Quanto mede a cerca? A opo correta (b).
Entre o terceiro e o sexto poste, temos trs espaos entre postes consecutivos. Logo,
1
a distncia entre dois postes consecutivos 3 3,3 m = 1,1 m e a distncia entre o
primeiro e o ltimo poste de 11 1,1 = 12,1 m.
10. Dzima peridica A opo correta (d).
Soluo 1: Como 1/3 = 0,333 . . . , segue que
0,1333 . . . = 0,333 . . . 0,2 =
260

1
2
1 1
2

= =
.
3 10
3 5
15

OBMEP 2010

Solues do Nvel 3
Soluo 2: Usando simplesmente a regra que fornece a geratriz de uma dzima peridica, tambm podemos obter
0,1333 . . . =

13 1
12
2
=
=
.
90
90
15

11. Valor absoluto A opo correta (e).


Temos: |5| = 5, |3 8| = | 5| = 5 e | 4| = 4. Logo, N = 5 + 5 4 = 6.
12. O peso das frutas A opo correta (b).
A partir das informaes fornecidas pelas trs guras, podemos montar trs equaes
em que, informalmente, denotamos o peso de cada fruta pelo seu prprio nome.
mamo = banana + ma
banana + mamo = 200
banana + 200 = mamo + ma
Somando a primeira com a terceira obtemos, aps cancelamento, 2 ma = 200,
donde ma = 100. Substituindo esse valor na primeira equao, obtemos mamo =
banana + 100 e, substituindo na segunda equao, obtemos 2 banana + 100 = 200,
donde banana = 50. Esses valores fornecem, pela primeira equao, o valor mamo =
150. Assim, a soma dos pesos das frutas 100 + 50 + 150 = 300 gramas.
13. Maratona A opo correta (c).
O comprimento de uma circunferncia de raio r 2 r. Assim, em cada volta Andr
percorre 2 100 m = 200 m. Logo, o nmero de voltas que Andr precisa dar para
completar 42 km = 42.000 m
42 000
210
=
.
200

Agora podemos nalizar o problema de duas maneiras.


1
1
1
<
< , portanto, multiplicando tudo por 210,
1a ) Como 3 < < 4, obtemos
4

3
resulta
210
210
210
<
<
= 70
52,5 =
4

3
e conclumos que Andr deve dar entre 52 e 70 voltas para percorrer os 42 km.
2a ) A aproximao de at a segunda casa decimal 3,14. Da,
210
210

66,88

3,14
e conclumos que Andr deve dar entre 66 e 67 voltas para percorrer os 42 km.
14. Dobrando papel A opo correta (e).
Soluo 1: Denotemos por ABC o tringulo obtido aps dobrar o quadrado original
ao longo das duas diagonais e seja M N o corte pela base mdia nesse tringulo, paralelo
ao lado BC, que um dos lados do quadrado original. A rea do quadrado original
OBMEP 2010

261

Solues do Nvel 3
(BC)2 . Desdobrando-se a folha, vemos que o buraco um quadrado de lado M N e,
1
como M N = BC, sua rea
2
2
1
1
(M N )2 =
BC = (BC)2 .
2
4
Logo, o buraco tem um quarto da rea do quadrado original.

A
N

N
C

Soluo 2: O corte realizado pela base mdia do tringulo, retirando um pequeno


tringulo semelhante ao original, com razo de semelhana 1/2. Assim, a rea do
tringulo retirado um quarto da rea do tringulo original. Abrindo a folha, vemos
essa situao reproduzida quatro vezes, donde o buraco tem um quarto da rea do
quadrado original.
15. Encontre o nmero A opo correta (a).
N N N N N
Para que , , ,
e
sejam nmeros inteiros, N deve ser um mltiplo comum
3 4 5 6
7
de 3, 4, 5, 6 e 7. Como queremos o menor N possvel, ele deve ser o mnimo mltiplo
comum (MMC) de 3, 4, 5, 6 e 7, ou seja,
N = 3 4 5 7 = 420.
16. Equao quadrtica A opo correta (d).
1
Soluo 1: Como 3 e so razes da equao ax2 6x + c = 0, temos 9a 18 + c = 0
3
1
e a 2 + c = 0, ou seja, 9a + c = 18 e a + 9c = 18. Somando essas duas equaes,
9
resulta 10(a + c) = 10a + 10c = 36, ou seja, a + c = 36/10 = 18/5.
Soluo 2: Numa equao ax2 + bx + c = 0 do segundo grau, a soma das razes
1
1
b/a e o produto c/a. Como b = 6, obtemos 10/3 = 3 + = 6/a e 1 = 3 = c/a,
3
3
ou seja, a = c = 9/5. Assim, a + c = 18/5.
17. Cubo A opo correta (d).
Soluo 1: Desenhando o cubo e numerando seus vrtices
de acordo com o enunciado da questo, obtemos uma gura
em que podemos ver que o vrtice 5, por ser diametralmente
oposto, o mais distante do vrtice 6.

OBMEP 2010

3
7

262

5
2

Solues do Nvel 3
Soluo 2: O vrtice 6 est nas faces {1, 2, 6, 7}, {1, 4, 6, 8} e {3, 4, 6, 7}. Como
nessas faces s no aparece o 5, segue que este o vrtice diagonalmente oposto ao 6,
ou seja, o 5 o vrtice mais distante do 6.
18. Time de basquete A opo correta (a).
Basta ler o grco para obter o nmero de pontos de cada aluno. A soma desses pontos
d um total de 7 + 8 + 2 + 11 + 6 + 12 + 1 + 7 = 54 pontos marcados pelo time.
19. O caminho da formiguinha A opo correta (e).
Para cada um dos trs caminhos para ir de A at B, existem trs opes para ir de
B a C. Logo, h um total de 3 3 = 9 possibilidades. Mais geralmente, se fossem m
os caminhos de A at B e n os de B at C, ento o nmero de caminhos que nossa
formiguinha poderia tomar de A at C seria mn; esta armativa um caso particular
do princpio multiplicativo.
20. Operao

A opo correta (a).

Fazendo a = 1 e b = 0 em a

b = a2 ab + b2 , obtemos 1

0 = 12 1 0 + 02 = 1.

21. Indo para a escola Os alunos da escola foram divididos em quatro grupos distintos,
de acordo com o tempo que gastam no trajeto de casa para a escola. Cada uma das
quatro barras do diagrama representa exatamente um desses quatro grupos e cada um
dos alunos dessa escola est em exatamente um desses quatro grupos.
(a) Os alunos que gastam menos de 20 minutos em seu trajeto de casa para a escola
esto representados pela primeira barra, a mais alta, que atinge a marca dos 90.
Logo, 90 alunos gastam menos do que 20 minutos para chegar escola.
(b) O total de alunos na escola a soma dos nmeros representados pelas quatro
barras, portanto, a escola tem um total de 90 + 60 + 10 + 20 = 180 alunos.
(c) Os alunos que gastam mais do que 40 minutos esto repartidos em dois grupos:
os que gastam de 41 a 60 minutos e os que gastam mais do que 60 minutos,
representados pela terceira e quarta barras, as duas mais baixas, uma atingindo
a marca dos 10 e a outra, a marca dos 20. Logo, o total de alunos que gastam
mais do que 40 minutos para chegar escola de 10 + 20 = 30 alunos.
(d) Os alunos que gastam entre 20 e 40 minutos em seu trajeto de casa para a escola
esto representados pela segunda barra, que atinge a marca dos 60. Junto com os
30 alunos que gastam mais do que 40 minutos (item precedente), temos um total
de 60 + 30 = 90 alunos que gastam mais do que 20 minutos para chegar escola.
No primeiro item vimos que 90 alunos gastam menos do que 20 minutos para
chegar escola, que o mesmo nmero dos que levam mais do que 20 minutos, ou
seja, a metade dos alunos da escola que leva mais do que 20 minutos. Conclumos
que no verdade que a maioria dos alunos gasta mais do que 20 minutos para
chegar escola.
22. Campeonato de futebol
(a) Cada uma das seis equipes disputou, com cada uma das outras cinco, exatamente
1
uma partida. Portanto, foram disputadas um total de 2 (6 5) = 15 partidas.
OBMEP 2010

263

Solues do Nvel 3
(b) Cada equipe disputou exatamente 5 partidas. Logo, de x + 1 + 0 = 5 decorre
x = 4. Da mesma forma, para a equipe D temos 1 + 1 + y = 5, portanto y = 3.
O nmero total de gols feitos num campeonato igual ao nmero total de gols
sofridos, ou seja, 6 + 6 + 2 + 3 + 1 + z = 2 + 6 + 6 + 6 + 5 + 3, ou 18 + z = 28,
portanto, z = 10.
23. Poste eltrico Nesta questo utilizamos o Teorema de
Pitgoras. Antes de rever o enunciado desse teorema, lembre
que um tringulo dito retngulo quando um de seus ngulos
reto, ou seja, mede 90 . O lado oposto ao ngulo reto a
hipotenusa e os outros dois lados so os catetos do tringulo
retngulo. Na gura, a hipotenusa a e os catetos so b e c.

Teorema de Pitgoras
C
b

c
A
B
Teorema de Pitgoras. Num tringulo retngulo de hipotenusa a e catetos b e c,
vale a relao a2 = b2 + c2 .

Agora resolvemos a questo.


Para que o poste que perpendicular ao solo, o ngulo em A deve ser reto e, portanto,
o tringulo ABC deve ser retngulo (ver gura). Nesse caso, os dados do problema
do que a hipotenusa mede 2,5 m e os catetos 1,4 m e 2 m. Assim, pelo Teorema de
Pitgoras teramos (2,5)2 = (1,4)2 + 22 .

1,4

2,5
2

Entretanto, (1,4)2 + 22 = 1,96 + 4 = 5,96 e (2,5)2 = 6,25. Logo, essas medidas no


satisfazem o Teorema de Pitgoras e, portanto, o tringulo ABC no retngulo.
Assim, o ngulo em A no reto e, consequentemente, o poste no est perpendicular
ao solo. Conclumos que o professor est certo.
24. Equaes recprocas
(a) Temos y = x +

1
. Usando as expanses do binmio e do trinmio, obtemos
x

1
x
1
y3 = x +
x
y2 = x +

1
1
1
e
= x2 + 2x + 2 = x2 + 2 + 2
x x
x
1
1
1
1
1
+ 3,
= x3 + 3x2 + 3x 2 + 3 = x3 + 3 x +
x
x
x
x
x

portanto,
x2 +
264

1
= y2 2
x2

x3 +

1
1
= y3 3 x +
= y 3 3y.
3
x
x

OBMEP 2010

Solues do Nvel 3
1
1
(b) A equao dada equivalente a x2 + 2 5 x +
+ 8 = 0. Substituindo o
x
x
valor de y e utilizando a identidade do item anterior, obtemos
(y 2 2) 5y + 8 = 0,

ou seja, a equao de segundo grau y 2 5y + 6 = 0, cujas razes so y = 2 e


1
y = 3. Voltando para x, multiplicamos x + = y = 2 e y = 3 por x para obter as
x
equaes quadrticas (x 1)2 = x2 2x + 1 = 0 e x2 3x + 1 = 0, cujas razes so

1
x = 1 e x = 3 5 . Assim, obtivemos todas as trs razes da equao dada.
2
(c) Como x = 0 no raiz da equao dada, podemos dividir tudo por x2 . Desse
modo, encontramos exatamente a equao do item (b), cujas razes j obtivemos.
(d) Como x = 0 no raiz da equao dada, podemos dividir tudo por x3 . Desse
modo, reordenando os termos, obtemos a equao equivalente
1
1
1
x3 + 3 2 x2 + 2 5 x +
+ 12 = 0.
x
x
x
Substituindo o valor de y e utilizando as identidades do item (a), obtemos
y 3 3y 2 y 2 2 5y + 12 = 0,

equivalente equao cbica

y 3 2y 2 8y + 16 = 0.

A forma mais rpida de resolver essa equao ter um pouco de sorte e fatorar
por agrupamento, obtendo, por exemplo,
y 3 2y 2 8y + 16 = y 2 (y 2) 8(y 2) = y 2 8 (y 2),

de modo que as trs razes da equao cbica em y so y = 2 e y = 2 2.

1
Voltando para x, multiplicamos x + = y = 2 e y = 2 2 por x para obter
x

as equaes quadrticas (x 1)2 = x2 2x + = 0, x2 2 + 1 = 0 e


1
2x

2
x + 2 2x + 1 = 0, cujas razes so x = 1, x = 2 1 e x = 2 1. Assim,
obtivemos todas as cinco razes da equao dada.
25. Atirando echas
(a) Os cinco pontos dados esto marcados na gura.
Ordenada
E
C

B
A

abcissa

OBMEP 2010

265

Solues do Nvel 3
(b) No crculo menor temos apenas o ponto A, portanto Manoel acertou apenas uma
vez neste crculo, o que lhe d 300 pontos.
(c) Para calcular o total de pontos, observe que pelo ponto B ele ganha 100 pontos,
por C ele ganha 50 pontos e, por D, 50 pontos. Entretanto, pelo ponto E, ele
no ganha pontos, porque est fora da zona de pontuao. Logo, o nmero total
de pontos que Manoel fez 300 + 100 + 50 + 50 = 500.
26. Festa de aniversrio Como podemos repartir o total de convidados em mesas de
6 ou 7, o nmero de convidados um mltiplo de 6 e de 7. Como o menor mltiplo
comum de 6 e 7 42, podemos ter 42, 84, 126, . . . convidados. Como so menos do
que 120 convidados, s podemos ter 42 ou 84 convidados. Por outro lado, como so
necessrias mais do que 10 mesas, temos mais do que 60 convidados. Logo, descartamos
o 42, e o nmero de convidados s pode ser 84.
27. Medida do cateto O segmento CF, cujo comprimento queremos calcular, um
cateto do tringulo retngulo CDF. O Teorema de Pitgoras, aplicado a esse tringulo, diz que (CD)2 = (CF )2 + (F D)2 = (CF )2 + 242 e, da, tiramos (CF )2 =
(CD)2 242 . Ou seja, para encontrar CF basta conhecer CD. Como os lados opostos de um retngulo (e, mais geralmente, de um paralelogramo) so iguais, temos
CD = AB. Nosso objetivo, ento, passa a ser o clculo de AB. Para isso, olhemos
para o tringulo ABE. Sua rea
1
1
AE BE = (15 BE) = 150,
2
2
donde tiramos BE = 20. O Teorema de Pitgoras aplicado a esse tringulo nos d
(AB)2 = (AE)2 + (BE)2 = 152 + 202 = 625 = 252 , donde AB = 25. Logo, CD =
AB = 25 e, de acordo com nossa observao anterior, obtemos
(CF )2 = (CD)2 242 = 252 242 = (25 + 24)(25 24) = 49.
Assim, CF = 7.
Observe que a soluo independe da medida dos lados AD e BE.
28. Sequncia de Peri Agrupamos a sequncia em blocos numerados consecutivamente, cada bloco formado pelos termos iguais consecutivos, como mostrado a seguir.
1 ,
bloco 1

2, 2 , 3, 3, 3, 4, 4, 4, 4, 5, 5, 5, 5, 5, 1, 1, 1, 1, 1, 1, 2, 2, 2, 2, 2, 2, 2,
bloco 2

bloco 3

bloco 4

bloco 5

bloco 6

bloco 7

3, 3, 3, 3, 3, 3, 3, 3, 4, 4, 4, 4, 4, 4, 4, 4, 4, 5, 5, 5, 5, 5, 5, 5, 5, 5, 5,
bloco 8

bloco 9

bloco 10

1, 1, 1, 1, 1, 1, 1, 1, 1, 1, 1, . . . , k, k, k, k, k, k, k, k, k, k, k, k, k, . . . , k, . . .
bloco 11

bloco n, com k{1,2,3,4,5}

Observe que a numerao de cada bloco coincide com o nmero de termos que ele
contm: o bloco 1 tem um termo, o bloco 2 tem dois termos, o bloco 3 tem trs termos
e assim por diante, at o bloco n, que tem n termos. A posio na sequncia do ltimo
termo de cada bloco obtida somando todos os nmeros de 1 at o nmero atribudo
ao bloco. Por exemplo, como pode ser contado na enumerao acima,
266

OBMEP 2010

Solues do Nvel 3
o ltimo 3 do bloco 8 o 36 termo, pois 1 + 2 + 3 + 4 + 5 + 6 + 7 + 8 = 36.

o ltimo 1 do bloco 11 o 66 termo, pois 1 + 2 + 3 + + 10 + 11 = 66.

Em geral, o ltimo termo do ensimo bloco est na posio 1 + 2 + 3 + + n. Para


calcular o valor desta soma, lembramos que 1, 2, 3, . . . , n uma progresso aritmtica
de razo 1, termo inicial a1 = 1 e ensimo termo an = n. A soma de seus n primeiros
termos , ento,
1 + 2 + 3 + 4 + + n =

n(a1 + an )
n(n + 1)
=
.
2
2

Agora precisamos descobrir em qual bloco se encontra o centsimo termo da sequncia.


Supondo que ele esteja no ensimo bloco, sua posio ser, no mximo, a do ltimo
termo deste bloco. Como ele no estar no bloco n + 1, conclumos que n o menor
1
inteiro tal que 100 2 n(n + 1), ou seja, 200 n(n + 1).

Para determinar esse valor de n, devemos resolver essa inequao e escolher, dentre suas solues, o menor nmero inteiro. Como a expresso bastante simples,
mais fcil resolv-la por tentativa. Fazendo isso, vemos que n = 14. De fato,
13 (13 + 1) = 182 < 200 e 14 (14 + 1) = 210 > 200. Assim, o centsimo termo da
sequncia est no bloco 14. Os nmeros que aparecem nos blocos se repetem de cinco
em cinco, na ordem 1, 2, 3, 4 e 5. Como 14 = 5 2 + 4, o bloco 14 formado pelo
nmero 4. Assim, o centsimo termo da sequncia 4.
Observao: A resoluo acima apresentada da inequao 200 n(n + 1), apesar de
correta, no serviria se o problema pedisse, por exemplo, a determinao do 10 000o
termo da sequncia. Nesse caso, teramos que lidar com a inequao 20 000 n(n+1) e,
claro, achar sua menor soluo inteira por tentativa no parece promissor (a no ser
com muita, muita sorte!). Por isso, vamos resolver a inequao 200 n(n + 1) de uma
maneira que serve em geral.
Comeamos escrevendo 200 n(n + 1) como n2 + n 200 0.
Isso nos leva ao estudo do sinal da
funo quadrtica f (x) = x2 + x 200,
cujo grco est ilustrado na gura. As
razes de f (x) so

1 1 + 800
x1 =
e
2

1 + 1 + 800
x2 =
.
2

14

13

x2

x1

Observe que x1 negativa e x2 , aproximadamente, igual a 13,6. Como f (x) 0 para


x x1 e x x2 , segue que o n que estamos procurando o menor inteiro que maior
do que ou igual a x2 , ou seja, n = 14.
Agora, se quisssemos determinar o 10 000o termo da sequncia, bastaria repetirmos o

1
procedimento acima, encontrando x2 = 2 1+ 1 + 80 000 , que , aproximadamente,
igual a 140,9. Logo, n = 141 e o 10 000o termo da sequncia est no 141o bloco. Como
141 = 28 5 + 1, segue que o 10 000o termo 1.
OBMEP 2010

267

Solues do Nvel 3
29. rea em azulejo A gura dada pode ser decomposta
em quatro guras congruentes gura dada. Para calcular a rea do tringulo sombreado nessa gura, escolhemos como base o lado BC.

E
D
Ento, a altura correspondente AE e, como os azulejos so quadrados com
10 cm de lado, segue que AE = BC = 10 cm. Logo, a rea do tringulo BCE
1
1
base altura = 10 10 = 50 cm2 . Assim, a rea da regio procurada
2
2
4 50 = 200 cm2 .

30. Os cartes de Capitu


Soluo 1: Capitu virou, em primeiro lugar, os 50 cartes pares. Depois disso, caram
na mesa os 50 cartes pares com a face amarela para cima e os 50 cartes mpares com
a face vermelha para cima. Ao virar, em seguida, os mltiplos de 3, ela virou apenas
os mltiplos de 3 mpares, que so 3, 9, 15, 21, 27, 33, 39, 45, 51, 57, 63, 69, 75, 81,
87, 93 e 99. Logo, temos 17 mltiplos de 3 que so mpares e Capitu virou para cima
a face amarela de 50 + 17 = 67 cartes. Assim, sobraram com a face vermelha para
cima 100 67 = 33 cartes.
Observao: Nessa soluo, para determinar a quantidade de mltiplos mpares de 3
menores do que 100 foi suciente escrever esses mltiplos e contar quantos eram. No
entanto, se Capitu tivesse 1 000 cartes (ou mais) esse procedimento seria bastante
trabalhoso, mas, nesse caso, podemos proceder de modo mais geral. Notamos que
os mltiplos mpares de 3 desde 1 at 1 000 formam uma progresso aritmtica, com
primeiro termo a1 = 3, razo r = 6 e o ltimo termo an = 999. Para determinar n
usamos a frmula an = a1 + (n 1)r que, no caso presente, 999 = 3 + (n 1) 6.
Assim, n = 167, ou seja, temos 167 mltiplos mpares de 3 menores do que 1 000.
Soluo 2: Capitu virou, em primeiro lugar, os 50 cartes pares. Depois disso, caram
na mesa os 50 cartes pares com a face amarela para cima e os 50 cartes mpares com
a face vermelha para cima. Ao virar, em seguida, os mltiplos de 3, Capitu procedeu
como segue.
Entre os cartes pares ela virou os que eram tambm mltiplos de 3. Um nmero
que mltiplo de 2 e de 3 tambm mltiplo de 6.
Como
100 = 16 6 + 4, conclumos que Capitu virou 16 cartes entre os cartes pares.
Esses cartes voltaram a car com a face vermelha para cima, cando os outros
34 com a face amarela para cima.
Entre os cartes mpares, como 100 = 33 3 + 1, segue que o nmero total
de cartes (pares e mpares) mltiplos de 3 33. Como vimos acima, entre
estes cartes, 16 so pares, logo 17 so mpares. Assim, Capitu virou 17 cartes
mpares, e esses cartes passaram a ter a face amarela para cima, enquanto que
os outros 33 continuaram com a face vermelha para cima.
31. Enchendo o tanque No que segue, todas as medidas de volume esto dadas em
cm3 .
O volume V do balde dado pela frmula habitual do volume de um cilindro, ou seja,
V = rea da base altura. A base do balde um crculo de 30 cm de dimetro; seu raio,
268

OBMEP 2010

Solues do Nvel 3
ento, mede r = 15 cm e sua rea r2 = 225 cm2 . Logo, V = 48 225 = 10 800 .
A cada viagem, o volume de gua que o homem coloca no balde 4/5 de V e, desse
volume, ele perde 10%. Portanto, resta no balde 90% de 4/5 de V, ou seja,
9
4
18
V =
V = 0,72 V = 0,72 10 800 = 7 776 .
10 5
25
Essa quantidade B = 7 776 de gua a que ele efetivamente coloca no tanque em
cada viagem. O volume de 3/4 do tanque T = 3 300 36 50 = 405 000. Logo, o
4
nmero de baldes necessrios para atingir esse volume
405 000
405 000
625
=
=
.
B
7 776
12
625
625
Usando a aproximao 3,14 para o nmero , obtemos

16,587.
12
12 3,14
Assim, o homem necessitar de 16 baldes, mais 0,587 de um balde, e conclumos que
ele dever fazer 17 viagens.
Observao: Acima usamos uma aproximao para o valor de . importante entender o que isso signica. Como sabemos, um nmero irracional e sua expanso
decimal innita e no peridica. O valor aproximado de , com 31 casas decimais,
3,1415926535897932384626433832795 (o smbolo quer dizer aproximadamente). Por que, ento, no usar 3,142 ou 3,1416 para resolver nosso
problema, em vez de 3,14? Para discutir isso, vamos a um exemplo.

Suponhamos que voc tenha um balde cilndrico com raio da base medindo 1 m e
altura 1 m, e uma caixa de gua de volume de, exatamente 3,141 m3 . O balde deve ser
enchido em uma fonte. Quantas viagens fonte sero necessrias para encher a caixa,
supondo que o volume de gua de cada balde seja integralmente transferido para a
caixa?
Usando a aproximao 3,14, obtemos 3,14 m3 para o volume do balde. Como
volume do tanque
3,141 maior do que 1 (e, claro, menor do que 2), conclumos que
volume do balde
3,14
sero necessrias duas viagens fonte para encher a caixa de gua.

Vamos, agora, usar a aproximao 3,1416. Aqui calculamos o volume do balde


3,141
e obtemos 3,1415 m3 . Ento, volume do tanque 3,1416 menor do que 1, e conclumos,
volume do balde
assim, que basta uma viagem fonte para encher o balde, um resultado bem diferente
do anterior!
Deve car claro com esse exemplo que a escolha inicial de uma aproximao pode
inuenciar fortemente o resultado nal. Nesse caso, dizemos que as condies do
problema so sensveis aproximao. No nosso problema original de encher o tanque,
os dados iniciais no eram sensveis aproximao usada para , o que pode ser
vericado imediatamente repetindo a resoluo dada com 3,142 ou 3,1416.
Em ambos os casos, obtm-se o resultado de 17 viagens.
Em geral, os problemas desse tipo propostos em livros nos ensinos fundamental e mdio
so enunciados de modo pouco sensvel aproximao. Isto justica parcialmente o

uso de = 3,14 bem como o de, por exemplo, 2 = 1,41

(curiosidade: 2 1,4142135623730950488016887242097).

Observamos, tambm, que poucas casas decimais facilitam as contas, em particular


quando no se usam mquinas de calcular. Seria impossvel, na prtica, trabalhar
manualmente com a aproximao de 31 casas que demos para no incio desta conversa.
OBMEP 2010

269

Solues do Nvel 3
O tratamento de problemas de aproximao feito atravs de desigualdades. Infelizmente, tempo e espao no permitem que abordemos esse tpico com mais detalhes no
momento, mas esperamos ter despertado sua curiosidade para o assunto.
32. Fator primo A decomposio de 2 006 em fatores primos 2 006 = 2 17 59.
Assim, o maior fator primo de 2 006 59.
33. Altura de salrio A opo correta (d).
O enunciado diz que 1 real = 275 107 cruzados. O salrio de Joo 640 reais, o que
equivalente a 640 275 107 = 176 000 107 = 176 1010 cruzados. O nmero
de pilhas de cem notas que se pode fazer com essa quantidade de notas de 1 cruzado
176 1010 /102 = 176 108 . Como cada uma destas pilhas tem 1,5 cm de altura, a
altura de todas elas 1,5 176 108 = 264 108 cm.

Agora lembramos que 1 km = 1 000 m = 103 m e que 1 m = 100 cm = 102 cm, donde
1 km = 103 102 = 105 cm. Assim, a pilha de 264 108 cm tem 264 108 /105 =
264 103 = 264 000 km de altura.

34. S bala A opo correta (c).


A primeira bala pode ser de qualquer sabor. Para xar as ideias, suponhamos que
seja de banana. Depois que essa bala retirada, sobram 1 002 + 1 001 balas na caixa
no nosso caso, 1 002 de ma e 1 001 de banana. A probabilidade q de que a segunda
bala seja diferente (no nosso exemplo, de ma) q = 1 002/2 003. A probabilidade p
de que a segunda bala seja igual (no nosso exemplo, de banana) p = 1 001/2 003. A
diferena q p , portanto,
qp=

1 002 1 001
1

=
.
2 003 2 003
2 003

35. Distncia ao centro A opo correta (e).


Os pontos que esto a 6 cm de distncia do ponto P
formam uma circunferncia de centro P e raio R = 6
cm. Se d denota a diagonal do quadrado, do Teorema
de Pitgoras temos

d = 102 + 102 = 2 102 = 10 2 .

L/2 R
D/2

A circunferncia de raio L/2 = 5 cm tangencia o


quadrado em quatro pontos.
A circunferncia de raio D/2 toca o quadrado em quatro pontos, a saber, os vrtices

do quadrado. Temos L = 10, R = 6 e D = 10 2 , portanto

5 < 6 <5 2.
L/2

D/2

(Observe que 1,2 < 2 , 51,2 < 5 2 e, portanto, 6 < 5 2 ). Logo, a circunferncia

de raio R = 6 est entre as duas circunferncias de raios 5 e 5 2 . Assim, ela corta


o quadrado em oito pontos.
270

OBMEP 2010

Solues do Nvel 3
36. Potncias e potncias A opo correta (e).
Soluo 1: Observamos que os termos do lado direito da equao dada podem ser
escritos como potncias de 2. De fato, 4x = (22 )x = 22x e 64 = 26 . Desse modo, a
equao se torna 2(2x ) = 22x +23 . Temos, ento, 2(22x )22x = 26 , donde 22x (21) = 26 ,
ou seja, 22x = 26 . Assim, 2x = 6 e segue que x = 3.
Soluo 2: 4x + 4x = 2(4x ) = 2 22x = 4x + 43 , portanto, 4x = 43 e segue que x = 3.
37. Um raio de luz A opo correta (b).
Vamos acompanhar o trajeto do raio de luz a partir do ponto S. Para isso, lembramos
a propriedade bsica da reexo de um raio de luz num espelho: o ngulo de reexo
igual ao ngulo de incidncia. Por exemplo, na gura dada, os ngulos a e b so iguais,
bem como os ngulos d e e. Observe que, na gura, as paralelas AS e BV so cortadas
pela transversal AB.
Assim,
a = 30 = b,

a + b + c = 180 ,
logo c = 120 e

a
c

c+d = 180 , logo


d = 60 = e.

f
e

30

Como a soma dos ngulos internos do tringulo BCV 180 , segue que f = 90 .
Isso quer dizer que o nosso raio de luz, ao atingir C, ser reetido sobre si mesmo e
far ento o caminho inverso, C B A S. Desse modo, o trajeto completo do
raio ser
S A B C B A S.
O comprimento desse trajeto do raio desde S at retornar a S duas vezes a soma dos
comprimentos dos segmentos AS, AB e BC. dado que AS = 1 m, portanto, resta
calcular AB e BC. Para isso, olhamos para o tringulo ABC. Ele um tringulo
retngulo com ngulos de 30 e 60 . Sabemos que num tal tringulo o cateto oposto
ao ngulo de 30 tem comprimento igual metade do comprimento da hipotenusa
1
(exerccio). No nosso caso, temos BC = AB.
2
Notamos, agora, que os tringulos ABC e CBF so congruentes, pois so tringulos retngulos (f = 90 ) com ngulos iguais (b = 30 ) e um cateto comum (BC), o que
1
1
1
nos mostra que AC = AV = m. Pondo AB = x, temos BC = x e o Teorema
2
2
2
1 2
1 2
2
de Pitgoras, aplicado ao tringulo ABC, nos d x =
+ x . Simplicando,
2
2

3 2 1
obtemos x = , donde x = 1/ 3 = 3 /3.
4
4
Desse modo, obtemos o comprimento do trajeto do raio de luz, como segue.

3
1 3
+
2(SA + AB + BC) = 2 1 +
3
2 3

OBMEP 2010

=2+

3 m.
271

Solues do Nvel 3
38. Diferena de quadrados Usando a fatorao x2 y 2 = (x y)(x + y) com
x = 666 666 666 e y = 333 333 333, vemos que x y = y e x + y = 999 999 999,
portanto,
666 666 6662 333 333 3332 =
=
=
=

333 333 333 999 999 999


333 333 333 (1 000 000 000 1)
333 333 333 000 000 000 333 333 333
333 333 332 666 666 667.

39. Escada de nmero A opo correta (e).


Usando a regra dada, preenchemos as casas vazias a
partir da segunda linha a contar de baixo e obtemos
a gura. Logo,

42
13+x 11+2x

(13 + x) + (11 + 2x) = 42


e, portanto, 24 + 3x = 42, ou seja, x = 6.

5+x x+6

8
3

40. Diferena de potncias A opo correta (c).


Soluo 1: O algarismo nal de 9 867 3 o mesmo que o de 73 = 343, isto , 3. O
algarismo nal de 9 867 2 o mesmo que o de 72 = 49, isto , 9. Se de um nmero
terminado em 3 subtramos outro terminado em 9, o algarismo nal do resultado 4.
Observao: Observe que o algarismo das unidades da diferena 9 867 3 9 867 2 igual
ao algarismo das unidades de (73 72 ).
Soluo 2: n3 n2 = n2 (n 1), com n2 = 9 867 2 terminando em 9 e
n 1 = 9 866 em 6. Como 9 6 = 54, o algarismo nal de n2 (n 1) 4.
41. Parbola girada A opo correta (e).

y
Uma rotao de 180 pode ser visualizada
B=(r,s)
r
como uma meia-volta. Aqui temos uma
A=(m,n)
meia-volta em torno da origem. A gura ilusn
tra o que uma meia-volta faz com as coordenadas dos pontos do plano. Por exemplo, o
-s
-m s
x
m
ponto A o resultado da meia-volta aplicada ao ponto A; em outras palavras, A
-n
A=(-m,-n)
onde o ponto A vai parar aps a meia-volta.
Do mesmo modo, B onde B vai parar aps
-r
B=(-r,-s)
a meia-volta.
fcil ver que na passagem de A para A as coordenadas trocam de sinal. Desse
modo, vemos que uma meia-volta em torno da origem leva um ponto qualquer (x, y)
no ponto (x, y). Assim, (a, b) pertence nova parbola se, e somente se, (a, b)
pertence parbola y = x2 5x + 9 original, ou seja, se b = a2 + 5a + 9 ou, ainda,
b = a2 5a 9. Logo, a equao da nova parbola y = x2 5x 9.
42. Logotipo Seja r o raio dos quatro crculos iguais.
272

OBMEP 2010

Solues do Nvel 3
Ligando os centros A e B de dois desses crculos ao
centro O dos crculos concntricos, obtemos o tringulo OAB, como na gura. Lembrando que a reta
que une os centros de dois crculos tangentes passa pelo
ponto de tangncia, vemos que OA = OB = 1 + r e
AB = 2r, tudo em cm. O tringulo OAB retngulo
em O e o Teorema de Pitgoras d

(2r)2 = (1 + r)2 + (1 + r)2 .


Logo, 4r2 = 2(1 + r)2 , ou 2r2 = (1 + r)2 , do que tiramos r2 2r 1 = 0. Assim,

1
r = 2 8 = 1 2 . Como o raio r positivo, obtemos r = 1 + 2 . Segue que
2

o raio do crculo maior mede 1 + 2r = 3 + 2 2 cm.


43. Padeiro cansado A opo correta (d).
Seja x a quantidade de farinha, em quilos, de que o padeiro dispe. Trabalhando
sozinho, ele usaria x/6 quilos de farinha em uma hora. Trabalhando com seu ajudante,
eles usariam x/2 quilos de farinha em uma hora. Seja t o tempo, em horas, que o
padeiro trabalhou sozinho. Como a farinha acaba em 150 minutos, ou seja, em 2,5
horas (2 horas e 30 minutos), o tempo que ele trabalhou com seu ajudante foi 2,5 t
horas. Logo, a quantidade de farinha gasta durante o tempo que o padeiro trabalhou
sozinho foi de (x/6) t e a quantidade gasta durante o tempo que o padeiro trabalhou
com seu ajudante foi de (x/2) (2,5 t). Como
x t
6

quantidade total
de farinha

quantidade de farinha gasta


pelo padeiro
trabalhando sozinho

x (2,5t)
2

quantidade de farinha gasta


pelo padeiro trabalhando
com o ajudante

1
temos x = 1 x t + 2 x (2,5 t) . A quantidade x de farinha que o padeiro tinha ini6
cialmente era no nula. Logo, podemos dividir ambos os membros por x, encontrando
1
1 = 1 t + 2 (2,5 t) e, portanto, t = 0,75 horas, ou seja, o padeiro trabalhou sozinho
6
durante 45 minutos.

44. Muitas diagonais Num poliedro qualquer, dois vrtices distintos determinam uma
diagonal apenas no caso em que no estejam numa mesma face.
No caso do prisma hexagonal, vemos na gura que o vrtice v no forma uma diagonal
com os vrtices marcados com . Levando o prprio v em conta, vemos que v no forma
uma diagonal com exatamente nove vrtices. Como o prisma tem doze vrtices, segue
que v forma uma diagonal com exatamente 129 = 3 vrtices. O mesmo raciocnio vale
para qualquer vrtice, e conclumos que, de cada vrtice do prisma, partem exatamente
trs diagonais. Como a diagonal que parte de um vrtice v para o vrtice w a mesma
1
que parte de w para v, segue que o nmero de diagonais 12 3 = 18.
2
OBMEP 2010

273

Solues do Nvel 3

X
X
X

X
X

V
X

*
*

X
X

Seja V um vrtice do poliedro. Observando a gura, vemos que V no forma uma


diagonal com exatamente quatorze vrtices, os treze marcados com X e mais o prprio
V. Como o poliedro tem vinte e quatro vrtices no total, sobram 24 14 = 10 vrtices,
com os quais V forma uma diagonal. Logo, o nmero de diagonais desse poliedro
1
24 10 = 120.
2
45. Promoo de sabonete A opo correta (d).
Pela promoo, quem levar 2 unidades paga pelo preo de 1,5 unidade, logo quem levar
4 unidades paga pelo preo de 3 unidades, ou seja, leva quatro e paga trs.
46. Qual o ngulo? A opo correta (b).
Como ABC e DEF so tringulos equilteros, cada um de seus ngulos internos
mede 60 . No tringulo AGD temos
GAD = 180 75 60 = 45

e GDA = 180 65 60 = 55 .

Portanto, AGD = 180 45 55 = 80 . Logo, no tringulo CGH, temos


x + 80 + 60 = 180 , donde x = 40 .
47. Caixa de papelo A opo correta (b).
A gura mostra as dobras que sero
feitas para montar a caixa, que ter as
dimenses seguintes: 20 cm de largura,
15 cm de comprimento e 10 cm de altura. Logo, seu volume ser de
20 15 10 = 3 000 cm3 .
48. Soma de vizinhos A opo correta (b).
Seja x o primeiro termo da sequncia. Como o segundo termo 1 e, a partir do terceiro,
cada termo a soma dos dois anteriores, temos que
o terceiro termo 1 + x;
274

OBMEP 2010

Solues do Nvel 3
o quarto termo 1 + (1 + x) = 2 + x;
o quinto termo (1 + x) + (2 + x) = 3 + 2x;
o sexto termo (2 + x) + (3 + 2x) = 5 + 3x.
Como o quinto termo 2 005, temos 3 + 2x = 2 005, donde x = 1 001. Logo, o sexto
termo da sequncia 5 + 3 1 001 = 3 008.
49. Algarismos crescentes A opo correta (d).
Os nmeros em questo, com

dois algarismos, so 12, 23, 34, 45, 56, 67, 78 e 89 (8 nmeros);


trs algarismos, so 123, 234, 345, 456, 567, 678 e 789 (7 nmeros);
quatro algarismos, so 1 234, 2 345, 3 456, . . . , 6 789 (6 nmeros);
por m, com cinco algarismos, somente 12 345,

num total de 8 + 7 + 6 + 1 = 22 nmeros.


50. Bloco girante A opo correta (b).
De acordo com a gura, podemos concluir que as
dimenses das faces X, Y e Z so 2, 3 e 6 cm2 ,
respectivamente. A seguir, indicaremos os movimentos feitos pelo bloco e as faces que entram em
contato com os quadradinhos em cada etapa. Lembre que giramos o bloco cinco vezes.
As guras a seguir mostram os quadradinhos do tabuleiro que cam em contato com
cada uma das trs faces do bloco, desde a posio inicial at a nal, aps a ltima
rotao.

OBMEP 2010

275

Solues do Nvel 3
Alguns quadradinhos entram em contato com as faces mais de uma vez, conforme
gura a seguir, que mostra todos os quadradinhos que tiveram contato com as faces
do bloco desde a posio inicial at a ltima rotao.

Contando nesta ltima gura, vemos que o bloco esteve em contato com 19 quadradinhos do tabuleiro.
51. Iterando um ponto A opo correta (d).
1
4

x
f (x)

A partir da tabela

2
1

3
3

4
5

5
2

obtemos

f (4) = 5, f (f (4)) = f (5) = 2, f (f (f (4))) = f (f (5)) = f (2) = 1 e


5

f (f (f (f (4)))) = f (f (f (f (4)))) = f (f (f (5))) = f (f (2)) = f (1) = 4.


4 vezes

Como 2 004 mltiplo de 4, segue que f (f (f (f (. . . f (4) . . .)))) = 4. O diagrama a


2 004 vezes

seguir ilustra essa armao.


f

4 5 2 1 4 5 2 1 4 5 2 1 4
4 vezes

8 vezes

12 vezes

... 5 2 1 4
2 004 vezes

52. Esmeralda e o 21 Primeiro vamos listar os nmeros que tm o agrupamento 21


no meio de sua representao decimal.
21, 121, 221, . . . , 921, num total de 10 nmeros.
210, 211, . . . , 219, num total de 10 nmeros.

Tambm devemos contar os agrupamentos 21 obtidos a partir de um par de nmeros


consecutivos tal que o primeiro termina com 2 e o segundo comea com 1, que so os
11 casos seguintes.
12 13, 120 103, 112 113, 122 123, 132 133, 142 143,
152 153, 162 163, 172 173, 182 183, 192 193.

Assim, temos um total de 20 + 11 = 31 agrupamentos 21 nesse nmero.


276

OBMEP 2010

Solues do Nvel 3
53. Muitos fatores A opo correta (d).
Cada um dos fatores uma diferena de quadrados, isto , a2 b2 , em que a = 1 e
b = 1/c2 = (1/c)2 . Usando a fatorao a2 b2 = (a b)(a + b), obtemos
1

1
4

1
9

1
1
1
16
225

1
1
1
1
1 2 1 2 1 2
2
2
3
4
15
1
1
1
1
1
1
1
1
= 1
1+
1
1+
1
1+
1
1+
2
2
3
3
4
4
15
15
14 16
1 16
8
1 3 2 4 3 5
=
=
.
= ...
2 2 3 3 4 4
15 15
2 15
15
= 1

54. Falta um ngulo A opo correta (a).


Os ngulos internos do quadriltero dado so 50 , 180 30 = 150 , e
180 40 = 140 . Como a soma dos ngulos internos de um quadriltero 360 ,
temos 50 + 150 + + 140 = 360 , donde = 20 .
55. Soma de distncias A opo correta (e).
Temos |z x| = 3,7 (1) = 4,7 e |w x| = 9,3 (1) = 10,3. Logo,
|z x| + |w x| = 4,7 + 10,3 = 15.
56. Espiral do Artur A opo correta (d).
A gura mostra que a espiral constituda por segmentos cujos comprimentos formam uma sequncia nita da forma 1, 1, 2, 2, 3, 3, 4, 4, . . . , n, n (se os dois ltimos segmentos da espiral tm o mesmo comprimento) ou, ento, da forma 1, 1, 2, 2, 3, 3, 4, 4, . . . ,
n, n, n + 1 (se os dois ltimos segmentos da espiral tm comprimentos diferentes).

A soma dos k primeiros nmeros naturais dada por


1 + 2 + 3 + + k =
OBMEP 2010

k(k + 1)
2
277

Solues do Nvel 3
e o comprimento total da espiral 4 m = 400 cm. Portanto,
2

n(n + 1)
= 1 + 1 + 2 + 2 + 3 + 3 + + n + n = 400
2

ou, ento,
2

n(n + 1)
+ n + 1 = 1 + 1 + 2 + 2 + + n + n + n + 1 = 400,
2

de modo que n(n + 1) = 400 ou (n + 1)2 = 400.


Entretanto, no existem dois nmeros naturais consecutivos cujo produto seja 400,
isto , a equao n(n + 1) = 400 no tem soluo. Assim, (n + 1)2 = 400, de modo
que n + 1 = 20. Portanto, o ltimo segmento da espiral tem 20 cm e o penltimo
19 cm. Os comprimentos dos segmentos da espiral formam a sequncia de nmeros
1, 1, 2, 2, 3, 3, 4, 4, . . . , 19, 19, 20. Assim, so 19 2 + 1 = 39 segmentos. Como sete j
foram traados, falta traar 32.
57. Quais so os ngulos? A opo correta (d).
Temos B CA = DAC = y (ngulos alternos internos) e B DA = C BD = y (simetria).
Seja O o ponto de interseo das duas diagonais.
Traando o segmento M N paralelo aos lados AD e BC
A.......................................................................................................................................D
. . .
. ......
.
..... .
..... .
. ........ y
.
.....
.
.
.
do retngulo, obtemos C ON = B CA = y e N OD =
..... .
.....
.....
.
.
.
.....
.
.
.
.....
.....
.....
.
.
.
.....
.
.
.
.....
.....
.....
.
.
.
.....
.
.
.
.....
.....
.....
.
.
.
. .. ......
..
.
.
.
.
.
..................................
.................................................................
....
.N
B DA = y. Logo,
M...................................................... .................
.
.
.
.
O

.
.....
.
.
.....
.
.
...
.
.
.....
.
.....
.
.....
.
.
.....
.
.
.
.
.....
.....
.....
.
.
.....
.
.
.
.
.....
.....
.....
.
.....
.
.
. ........
..... .
..... .
. ........
.
....
...............................................................
..
.
................................................................
.....
.
. .

x = C OD = C ON + N OD = 2y.

58. Raiz menor A equao j foi dada na forma fatorada a(x m)(x n) = 0, logo

suas razes so m = 3 5 e n = 5 3 . Devemos apenas decidir qual delas a maior.


Isso pode ser feito de duas maneiras, pelo menos.
2
Podemos elevar m e n ao quadrado, obtendo m= 9 5 = 45 e n2 = 25 3 = 75.

Como 45 < 75, resulta que 3 5 = m = 45 < 75 < n = 5 3 .

Tambm podemos observar que 5 < 2,3 e 1,7 < 3 , portanto,

m < 3 2,3 = 6,9 < 8,5 = 5 1,7 < n.


3 5

6,9

8,5

5 3

59. Comparando reas A opo correta (c).


Os raios dos trs discos menores so 1, 2 e 2 e o do disco
maior 3. Denotemos por b a rea em branco. Ento
v = 1 + 4 + 4 = 9 b
e w = 9 b, ou seja, v = w.
278

OBMEP 2010

Solues do Nvel 3
60. Menor raiz A opo correta (b).
1o Caso: x 1. Nesse caso, x 1 0 e, portanto, |x 1| = x 1. A equao
dada toma a forma (x 1)/x2 = 6, ou 6x2 x + 1 = 0. Essa equao no tem
razes reais porque = (1)2 4 6 1 = 1 24 negativo. Logo, no temos
solues x maiores do que ou iguais a 1.
2o Caso: x < 1. Nesse caso, x 1 < 0 e, portanto, |x 1| = (x 1) = 1 x.
A equao dada toma a forma (1 x)/x2 = 6, ou 6x2 + x 1 = 0. Essa equao
tem as razes

1 1 4 6 (1)
1 25
1 5
x=
=
=
,
26
12
12
1
1
ou seja, x = e x = .
2
3
Com base nesses dois casos, conclumos que nossa equao tem apenas duas solues,
1 1
1
e . Logo, a menor soluo da equao .
2 3
2
61. Toalha redonda A opo correta (d).
Para que a toalha cubra inteiramente a mesa e que tenha o menor
dimetro possvel, o quadrado deve estar inscrito no crculo. A diagonal do quadrado o dimetro do crculo, logo, pelo Teorema de

Pitgoras, temos d2 = 12 + 12 , ou seja, d = 2 .

1
d

62. Solues reais A opo correta (c).


Para que um produto de trs fatores seja negativo, devemos
ter dois fatores positivos e um fator negativo, ou os trs negativos. As possibilidades so as seguintes.
1) (x 1) (x 2) (x 3) .
+

Isso equivale a x > 1, x > 2 e x < 3, ou seja,


2 < x < 3.

2) (x 1) (x 2) (x 3) .
+

Isso equivale a x > 1, x < 2 e x > 3, o que no


possvel. Logo, no pode ocorrer esse caso.

3) (x 1) (x 2) (x 3) .

Isso equivale a x < 1, x > 2 e x > 3, o que no


possvel. Logo, no pode ocorrer esse caso.

OBMEP 2010

279

Solues do Nvel 3
4) (x 1) (x 2) (x 3) .

Isso equivale a x < 1, x < 2 e x < 3, ou seja,


x < 1.

Logo, os nicos reais x satisfazendo (x 1)(x 2)(x 3) < 0 so os reais x tais que
x < 1 ou 2 < x < 3. Assim, a unio de intervalos (0, 1) (2, 3) o conjunto formado
por todos os reais positivos de x tais que (x 1)(x 2)(x 3) < 0.
63. Cossenos crescentes De acordo com a denio de cosseno, temos
cos 25 = 1/(OM ), cos 41 = 1/(ON ) e cos 58 = 1/(OB). Na gura, vemos que
OM < ON < OB. Logo, cos 58 < cos 41 < cos 25 .
64. Central telefnica A opo correta (e).
Existem dois tipos de ramais que podem estar sendo usados. Temos os ramais com
os dois algarismos iguais (00, 11, 22, 33, 44, 55, 66, 77, 88 e 99), num total 10, e
os com
os dois algarismos distintos. Nesse caso, temos 10 9 = 90 nmeros, e metade
deles podem ser usados.
Logo, o maior nmero possvel de ramais em uso 10 + 45 = 55.
65. Horrio de avio Seja d a distncia entre as duas cidades e seja h o horrio de
partida comum do nibus, do trem e do avio. Como distncia = velocidade tempo,
temos d = 100(20h) e d = 300(14h). Logo, 100(20h) = 300(14h), donde
h = 11. Portanto, a distncia entre as duas cidades d = 100 (20 11) = 900 km.
Assim, o avio gasta 1 hora da cidade A cidade B e, portanto, o avio chega s 12
horas.
66. Discos de papelo A opo correta (c).
Lembre que a rea de um crculo de raio r r2 . Se r o raio dos crculos da gura,
ento a rea no aproveitada a rea do quadrado, que dada por 10 10 = 100 cm2 ,
menos a soma das reas dos nove crculos, que 9 r2 cm2 . Ocorre que o raio de
cada crculo r = 5/3 cm, j que os dimetros de trs desses crculos somam um lado
de 10 cm da folha de papelo. Assim, a rea no aproveitada dada por
100 9

5
3

= 100 25 .

Usando a aproximao 3,14, resulta que a rea no aproveitada mede


100 25 100 25 3,14 = 21, 5 cm2 .
280

OBMEP 2010

Solues do Nvel 3
67. Armaes absolutas
(a) | 108| = 108 > 100, verdadeira.

(b) |2 9| = | 7| = 7 = 9 2, verdadeira.

(c) | 6a| = | 6| |a| = 6 |a|, verdadeira.

(d) |5 13| = | 8| = 8 = 8 = 5 13 = |5| |13|, falsa.

(e) |a2 + 5| = a2 + 5, verdadeira, pois a2 + 5 > 0, para qualquer valor de a.

68. Frao radical A opo correta (e).

x
x
Elevando ao quadrado ambos os membros de = 5, obtemos = 25. Assim,
y
y
1 x+y
1
x y
x+y
=
=
+
2y
2
x
2
y y

x
1
1

+ 1 = (25 + 1) = 13.
2
y
2

69. rea de tringulo A opo correta (d).


Os tringulos T KR e GRS so proporcionais por serem tringulos retngulos
RS
GS
com um ngulo agudo igual. Logo, temos
=
. Como GS = T K, segue que
TK R
T
(T K)2 = RS T R = 2 6 = 12, ou seja, T K = 2 3. Tambm
KG = T R + RS = 6 + 2 = 8.
Assim, a rea do tringulo KGR mede

1
1
KG T K =
82 3
2
2
base

= 8 3.

altura

70. Pares de inteiros A opo correta (c).


Temos
13 =

a+ 1
b
=
1
+b
a

ab+1
b
1+ab
a

(ab + 1) a
a
= .
(1 + ab) b
b

Logo, a = 13 b. Como a + b 100, segue que 14 b 100 e, portanto, b 7,14. Como b


inteiro, devemos ter b 7. Logo, os pares so em nmero de sete, a saber,
(13, 1), (26, 2), (39, 3), (52, 4), (65, 5), (78, 6) e (91, 7).
71. Qual a soma? A opo correta (c).
1o Caso: Se x 0, ento |x| = x e, pela primeira equao, temos x + (x) + y = 5,
ou seja, y = 5. Substituindo esse valor na segunda equao, obtemos x = 6, o que no
possvel, pois estamos supondo x 0. Logo, no h soluo nesse caso x 0.
2o Caso: Se y 0, ento |y| = y e, pela segunda equao, temos
x + y y = 6,
OBMEP 2010

281

Solues do Nvel 3
ou seja, x = 6. Substituindo esse valor na primeira equao, obtemos y = 7, o que
no possvel, pois estamos supondo y 0.
3o Caso: Se x > 0 e y < 0, ento |x| = x e |y| = y. Pela primeira equao temos
2x + y = 5 e, pela segunda, x 2y = 6. Multiplicando 2x + y = 5 por 2 e somando com
x 2y = 6, obtemos 5x = 16, de modo que x = 16/5 e segue que y = 5 2x = 7/5.
Assim, x + y = 9/5.
72. Crculo intermedirio A opo correta (a).
A rea do maior crculo 132 = 169 e a do menor 52 = 25 , que tambm a
rea do maior anel. Seja r o raio do crculo intermedirio. Ento, a rea do maior anel
169 r2 . Logo, 169 r2 = 25 , ou seja, r2 = 169 25 = 144 , donde
r2 = 144 e r = 12 cm.
73. Fraes incompletas
(a) Observe que 21 6 = 126. Portanto, o numerador 21 foi multiplicado por 6 para
obter o numerador 126 do primeiro quociente. Logo, o denominador
tambm
foi multiplicado por 6 para dar o denominador 8 do primeiro quociente. O nico
nmero da forma 8 que divisvel por 6 84, e 84 6 = 18. Podemos, ento,
completar as fraes, obtendo
126
21
21 6
=
=
.
84
18 6
18
8 = 0,8 33 . Observe que 33 deve ser
(b) Temos 4/5 = 0,8 e queremos ter
mltiplo de 5, portanto, s pode ser 330 ou 335. Entretanto,
0,8 330 = 264 =
Assim, 33 = 335, com 335 = 5 67, e
maneira de completar a frao,

8.

8 = 268 = 4 67. Logo, s existe uma

2 68
4
268 67
= .
=
335
335 67
5
74. Tringulos impossveis
Figura 1: No est correta, porque a soma dos ngulos internos do tringulo no
180 . De fato, 74 + 42 + 42 = 158 < 180.
Figura 2: No est correta, porque o comprimento dos lados no satisfaz o Teorema de Pitgoras: 182 = 324 = 369 = 144 + 225 = 122 + 152 . Logo, o tringulo
no pode ser retngulo.
Figura 3: No est correta, porque um dos lados de um tringulo no pode ser
menor do que a soma dos outros dois: 15 > 6 + 8.

282

OBMEP 2010

Solues do Nvel 3
75. Razo de reas A opo correta (b).
Como o arco de 60 do crculo I tem o mesmo comprimento que o arco de 45 no crculo
II, conclumos que o raio do crculo I menor do que o do crculo II. Denotemos por r
e R os raios dos crculos I e II, respectivamente.
No crculo I, o comprimento do arco de 60 igual a 1/6
de seu comprimento, ou seja 2 r/6. Analogamente, no
crculo II, o comprimento do arco de 45 igual a 1/8
de seu comprimento, ou seja, 2 R/8. Logo, 2 r/6 =
2 R/8, ou seja, r/R = 6/8 = 3/4. Finalmente, temos
rea do crculo I
r2
r
=
=
2
rea do crculo II
R
R

3
4

9
.
16

76. Inequao errada A opo correta (c).


Nessa questo usaremos as propriedades de desigualdades seguintes. Podemos somar
o mesmo nmero a ambos os membros de uma desigualdade sem alterar seu sentido.
Podemos multiplicar ambos os membros de uma desigualdade por um nmero positivo
sem alterar seu sentido. Assim,
x>y

x + z > y + z (somando z qualquer a ambos os lados)


xz > yz (muliplicando por z > 0 em ambos os lados)

Logo, (a) e (b) esto corretas, pois foi somado z e z a ambos os membros, bem
como (d) e (e), pois ambos os membros foram multiplicados por 1/z 2 e z 2 , ambos
positivos, j que z = 0. A opo (c) falsa, porque z pode ser negativo. Por exemplo,
se x = 5, y = 3 e z = 2, temos 5 > 3 e, no entanto,
5 2 = 10 < 6 = 3 (2) .
xz

yz

77. Equaes geomtricas


(a) |x 5| = 2 signica que a distncia de x a 5 2. Logo, as razes so 3 e 7.
2

(b) |x + 3| = 1 signica que a distncia de x a 3 1. Logo, as razes so 4 e 2.


1

(c) Denotando y = 3x, a equao toma a forma |y 7| = 9, o que equivale a dizer


que a distncia de y a 7 9. Logo, as razes so 2 e 16. Como y = 3x, temos
3x = 2 e 3x = 16, de modo que as razes da equao original so x = 2 e
3
x = 16 .
3
9

9
2

OBMEP 2010

16

283

Solues do Nvel 3
(d) As razes da equao |x + 2| = |x 5| so os nmeros equidistantes de 2 e de
5. No entanto, s pode haver um nico nmero equidistante de dois outros, e que
ca no meio do caminho entre os dois. Como a distncia de 5 a 2 7, o ponto
equidistante deve distar 3,5 de 2 e de 7. Logo, a soluo x = 1,5.
3,5

1,5

3,5

78. Pista circular A opo correta (c).


Vamos marcar os quatro pontos a partir de A. Como a pista
mede 20 km, o comprimento de cada um dos quatro quadrantes 5 km e podemos, ento, marcar os pontos. Como
367 = 18 20 + 7, o carro deu 18 voltas completas e percorreu
mais 7 km a partir de A. Logo, ele passa 2 km de B e para a
1 km de C. Portanto, C o ponto mais prximo.

79. Maior comprimento A opo correta (e).


Note que
AE a hipotenusa de um tringulo de catetos com 5 cm e 9 cm;
CF a hipotenusa de um tringulo de catetos com 2 cm e 4 cm;

AC a hipotenusa de um tringulo de catetos com 3 cm e 4 cm;


FD a hipotenusa de um tringulo de catetos com 2 cm e 9 cm;
CE a hipotenusa de um tringulo de catetos com 2 cm e 5 cm.
Usando o Teorema de Pitgoras calculamos essas hipotenusas.

AE = 52 + 92 = 106 10,3

CF = 22 + 42 = 20 4,47

CF = 32 + 42 = 25 = 5

FD = 22 + 92 = 85 9,22

CE = 22 + 52 = 29 5,39
Como CD = 5 cm, obtemos AE 10,3, CD + CF 5 + 4,47 = 9,47,
AC + CF 5 + 4,47 = 9,47, FD 9,22 e AC + CE 5 + 5,39 = 10,39. Logo,
o maior segmento AC + CE, que mede 10,39 cm.
80. Desigualdade entre inteiros A opo correta (d).
2
1
Se 3x2 < 14, ento 3x2 > 14, ou x2 > 14 = 4 . Como estamos olhando apenas
3
3
2
2
para valores inteiros de x, ento x tambm inteiro. Sendo x2 > 4 , conclumos que
3
x2 , no mnimo, 5. Dentre os nmeros 5, 4, 3, 2, 1, 0, 1, 2, 3 somente quatro,
a saber, 5, 4, 3 e 3 satisfazem x2 5.
284

OBMEP 2010

Solues do Nvel 3
81. Equao cbica A opo correta (d).
Observe que o polinmio cbico dado igual a x 2 007x2 + 2 006x + 2 005 , portanto,
x = 0 uma soluo da equao dada e a opo (a) ca descartada. Como a equao
cbica e x = 0 uma soluo, a opo (e) ca descartada. Agora, para ver se a equao
dada tem uma, duas ou trs solues, s precisamos ver se a equao de segundo grau
2 007x2 + 2 006x + 2 005 = 0 no tem soluo, ou tem uma ou tem duas solues. Mas
o discriminante dessa equao
= 2 0062 4 2 007 2 005 = 2 0062 4(2 006 + 1)(2 006 1)
= 2 0062 4(2 0062 1) = 3 2 0062 + 4 < 0,

de modo que essa equao no possui razes reais. Assim, a equao inicial tem uma
nica raiz real.
Observao: Uma outra maneira (e mais simples) de mostrar que < 0 observar
que 2 006 < 2 007 e 2 006 < 4 2 005, portanto,
2 006 2 006 < 4 2 005 2 007 e 2 0062 4 2 005 2 007 < 0.
82. O perfume de Rosa O volume de um cilindro o produto da rea da base pela
altura. Como o raio da base mede 7 cm, a rea da base 72 e, ento, o volume do
vidro
490
72 10 cm3 = 490 cm3 =
dm3 = 0,49 litros,
1 000
lembrando que 1 000 cm3 = 1 dm3 = 1 litro. Depois de duas semanas, restaram 0,45
litros de perfume, de modo que ela gastou (0,49 0,45) litros. Portanto, a frao que
representa o volume gasto
volume gasto
0,49 0, 45
49 45
=
=
.
volume total
0, 49
49
83. Igualdade com inteiros Como 2n = m2 1 = (m + 1)(m 1), estabelecemos que
m 1 e m + 1 so potncias de 2. Como a diferena de m + 1 e m 1 2, a nica
soluo possvel m 1 = 2 e m + 1 = 22 , donde m = 3. Assim, 2n + 1 = 32 = 9 e
obtemos n = 3. A resposta m = n = 3.
84. O caminho da pulga No primeiro pulo, a pulga percorre 10 1 m, no segundo
2
1 2
pulo, ela percorre 10 2 m, e assim por diante. Depois de 7 pulos, a pulga ter
percorrido
1
1 2
1 3
1 4
1 5
1 6
1 7
+ 10
+ 10
+ 10
+ 10
+ 10
+ 10
2
2
2
2
2
2
2
1 2
1 3
1 4
1 5
1 6
1 7
1
+
+
+
+
+
+
= 10
2
2
2
2
2
2
2
26 + 25 + 24 + 23 + 22 + 2 + 1
127
= 10
9,9.
= 10
27
128
Logo, em 7 dias, ela ter percorrido, aproximadamente 9,9 m. Em geral, depois de n
dias, a pulga ter percorrido
10

10

1
1
+ + n
2
2
OBMEP 2010

metros.
285

Solues do Nvel 3
1
Para calcular a soma acima, note que 2 + + 21 a soma dos n termos de uma
n
progresso geomtrica cujo primeiro termo a1 = 1/2 e cuja razo q = 1/2. A
frmula para essa soma

a1 (1 q n )
1/2(1 1/2n )
1
=
= 1 n.
1q
1 1/2
2

Sn =
Assim,
10

1
1
+ + n
2
2

= 10 1

1
2n

= 10

10
.
2n

Tomando n = 10, obtemos


10

10
1 023
10
= 10
9,99.
= 10
10
2
1 024
1 024

Portanto, ao nal do dcimo dia, a pulga ter percorrido, aproximadamente, 9,99


metros.
A pulga estar a menos de 0,001 m do nal do caminho quando ela j tiver percorrido,
pelo menos, 9,999 = 10 0,001 metros, ou seja, quando
10

10
10 0,0001,
2n

o que equivale a 0,001 10/2n , ou 2n 10/0,001 = 10 000.

Agora, 213 = 210 23 = 1 024 8 = 8 192 < 10 000 < 16 384 = 214 , de modo que
devemos tomar n = 14 e a pulga estar a menos do que 0,001 m do nal do caminho
a partir do dcimo quarto dia.

85. Uma soma alternada A opo correta (d).


A expresso (1)n+1 na denio de Sn tem valor 1 se n for par e tem valor 1 se n
for mpar.
Soluo 1: Associando parcelas consecutivas duas a duas, obtemos uma soma de
vrias parcelas iguais a 1: (1 2) + (3 4) + (5 6) + . Logo,
S1 992 = (1 2) + (3 4) + + (1 991 1 992) = (1) 996 = 996
1 9922=996 parcelas

e
S1 993 = (1 2) + (3 4) + + (1 991 1 992) + 1 993 = 996 + 1 993 = 997.
Assim, S1 992 + S1 193 = 996 + 997 = 1.
Soluo 2: Como
S2n = (1 2) + (3 4) + (5 6) + + 2n (2n + 1) ,
n parcelas iguais a 1

obtemos S2n = n e S2n+1 = S2n +(2n+1) = n+2n+1 = n+1. Assim, S2n +S2n+1 =
1.
286

OBMEP 2010

Solues do Nvel 3
86. O raio da circunferncia A opo correta (c).
2
Soluo 1: Se o raio r, ento o comprimento de um arco de graus
r. Assim,
360
no problema dado, temos que
2 000 m =

2
5
300 r =
r,
360
3

portanto r = 2 000 (3/5) 382,17 m.

Soluo 2: Como a circunferncia tem 360 , um arco de 300 representa 5/6 da circunferncia, portanto, seu comprimento de 2 km 5/6 do comprimento da circunferncia,
isto , (5/6) 2 r = 2 000 m, portanto
r=

1 200
2 000 6
=
382,17 m.
10

87. Quatro passageiros O passageiro que quer car na janela tem trs possveis lugares
para se sentar, o seguinte pode-se sentar em qualquer lugar livre, tendo, portanto, trs
possveis lugares; o seguinte tem dois possveis lugares e o ltimo no tem escolha.
Conclumos que o nmero dessas formas de se sentar 3 3 2 = 18.
88. Os cinco crculos Observemos que qualquer linha que passe pelo centro O do
quadrado ABCD, divide a rea formada pelos crculos C1 , C2 , C3 e C4 pela metade.
Por outro lado, qualquer linha reta que passe pelo centro F do crculo C5 , divide a rea
desse crculo pela metade. Assim, a reta procurada a reta F O.
C1 .......................................................................

..................
......................
2
....
......
....
.....
....
...
....
...
...
...
...
..
...
...
...
..
..
..
..
..
..
..
..
.. ...
..
..
.. .
.
..
..
.
. .
.
. .
.
..
.
.
..
.
.
..
.
.
..
.
.
.
.
.
.
.
.
.
.
.
.
.
.
.
.
.
.
.
.
.
.
.
.
.
.
.
.
.
.
.
..
. ..
.
..
. ......
.
..
.
.
. ..
..... .
.
. ..
.......
.
..
..
..
..
. .
.........
.. ..
.........
..
..
..
..
.
....... .
..
..
....... ...
..
..
..
..
......
......
...
...
..
...
... ............
..
..
...
... ............
...
...
...
....
...
.....
...
...
....
.
......
....
.......
......
....
.......... ..............
....... .......................
.................
....... .......................
...
......
...
...............
. . .......
.
.......
.......
.......
........ ............
.......
.......
.......
.......
.......
..............
.......
.....
....
..
....
.
5 ...........
..
...
...
....
......
...
...
...
......
....
....
...
...
...... .....
..
...
...
..
..
...... .....
...
...
...
...
..
..
..
.......
..
..
..
.......
..
..
..
..
.. .. ............
.. ...
.. ... ............
..
..
.. . .
..
.
.. ..........
. .
............
..
.
.
.
. .
.
.
..
.
. ..
.
.
..
.
......
.
......
..
.
...
... .
.
..
.
.
.
.
....... ..
.
.
.......
.
.
.
..
.
...
.
.
.
.
.
.
.
.
.
.
.......
.......
.
.
...
.
...
.
.
.
.
.
.
.
.
.......
.
.
.
.
.
.......
..
.
.
.
.
. ...........
..
..
.
. ............
.
.
...
...
.
.. ..
.
.
.
. ..
. ..
.
..
.....
. .
. .
.
..... .
....... .
. ..
. ..
.
....... ..
.. ...
.. ...
..
..
.
.
..
..
..
..
..
..
..
..
..
..
..
...
...
...
..
...
...
..
..
..
..
...
..
....
...
...
....
...
...
...
...
. ..
....
...
...
. ..
....
.....
....
...
.....
.....
....
......
....
....
......
......
......................
......................
.....................
..................
..................
.................

Ds

C
s

O
s

C4

C3

89. O tringulo e o quadrado As diagonais do quadrado ABCD


dividem o quadrado em 4 tringulos iguais, portanto, a rea do
tringulo BCE mede uma quarta parte da rea do quadrado, ou
seja,
1 4 = 0,25 cm2 .

B.............................................................................................................................. C
.
.
.
.
.
.
r

. ..
... .. .
. ...
. ...
....... .... .
. ...
....... .... .
.
.
.. .............
.
.
.
.. .
.
.
.. ............
.. ..
.
.
..
.
.
..
..
..
..
.
.
.
..
.
..
..
.
..
...
.
.
.
..
.
..
..
.
..
.
.
.
..
.
.
.
.. ....
.
.. ....
.
.. ...
.
.
.
.
.. ..
.
....
.
.
....
.
.
.
.
.
.
.
.. .
.
.
.. ...
.
.. ....
.
.
.
.
.
.
.
.
..
.. ....
.
..
..
.
..
.
.
.
.
.
..
.
..
.
..
.
..
.
..
..
.
.
..
.
.
.
.
..
.
..
.
..
.
..
.
..
..
.
.
..
.
.
..
. ...
..
..
. ....
.. .
.. .
. ..
.. .
. .
. ..
.. .
.. .
.
. ....
.. .
.
..
.
.. .
...........................................
..
.
...........................................
.
..
..
..

r E

Como o comprimento de BF a metade de BE e CE a altura comum s bases BF


e BE, conclumos que a rea do tringulo CBF necessariamente a metade da rea
do tringulo CBE. Assim, a rea do tringulo CBF 0,125 cm2 .
90. Uma refeio Se S corresponde ao nmero de sanduches e P ao nmero de pratos
de refeio, ento 5S + 7P = 90 e, portanto,
P =

18 S
90 5S
=5
.
7
7
OBMEP 2010

287

Solues do Nvel 3
Como queremos solues inteiras no-negativas P e Q, vemos que 7 deve dividir 18S.
Assim, S s pode ser 4, 11 ou 18 e, nesses casos, P igual a 10, 5 ou 0, respectivamente.
Portanto, temos somente trs formas de fazer a compra sem receber troco, a saber, 4
sanduches e 10 pratos, 11 sanduches e 5 pratos, ou 18 sanduches e nenhum prato.
91. Plano Cartesiano Somando 1 abscissa a do ponto
P = (a, b) transladamos esse ponto uma unidade para
a direita, trocando a por a reetimos esse ponto
pelo eixo y e dividindo a por 2, transladamos esse
ponto metade de sua distncia do eixo x. Analogamente, trocar a ordenada b de P por b 1, ou b 2,
translada P uma ou duas unidades para baixo, trocar
b por b reete o ponto pelo eixo x e trocar b por b/2
translada P para o ponto metade de sua distncia do
eixo y. A gura mostra P junto com os quatro pontos
A = (a + 1, b/2), B = (a/2, b 1), C = (a, b) e
D = (1 a, b 2) no plano cartesiano.

6
3

B
r

D1
r
2

P
r

A
r
2

Cr

2
3

92. Soma dos terminados em 9 A soma das k primeiras parcelas de uma progresso
aritmtica dada por Sk = 1 (a1 + ak ) k, em que a1 e ak = a1 + (k 1) r so o primeiro
2
e ltimo termos, respectivamente, e r a razo. Por exemplo, temos
1 + 2 + 3 + + (n 1) =

1
1
1 + (n 1) (n 1) = n(n 1).
2
2

A soma dada a de uma progresso aritmtica de n parcelas com primeiro termo


a1 = 9 e razo r = 10, de modo que temos an = 9 + (n 1) 10 e, portanto,
Sn =

1
9 + 9 + (n 1) 10 n = 9n + (n 1) 5n = 5n2 + 4n.
2

Como queremos Sn 105 , precisamos encontrar o menor inteiro positivo n tal que
5n2 + 4n 105 ou, equivalentemente, 5n2 + 4n 105 0. Para isso, resolvemos a
equao de segundo grau 5x2 + 4x 105 = 0, obtendo as solues

4 16 + 20 105
,
x=
10

1
e a raiz positiva x1 = 10 4 + 2 000 016 141,02. Como 5x2 + 4x 105 positivo
fora das razes, por ter coeciente dominante 5 > 0, resulta que n = 142 o menor
inteiro positivo n para o qual Sn maior do que 105 .
93. Trs cilindros O volume de um cilindro de raio R e altura h dado por R2 h.
(a) Os trs volumes so V1 = 103 = 1 000 , V2 = 52 10 = 250 e V3 =
52 20 = 500 , portanto, V1 > V3 > V2 .

(b) Como os cilindros V2 e V3 tm o mesmo raio, basta manter o raio do cilindro em


5 cm e a altura entre 10 e 20 cm; por exemplo, h = 15 cm. Nesse caso, o volume
V4 do novo cilindro 52 15 = 375 cm3 .
288

OBMEP 2010

Solues do Nvel 3
(c) Para construir um cilindro de volume V5 entre V1 e V3 , podemos tomar a menor
das duas alturas, que 10 cm, e diminuir o raio do cilindro de maior volume de
10 para 8 cm, obtendo um cilindro de volume V5 = 82 10 = 640 cm3 .
94. Porcentagem de mortalidade A opo correta (a).
15
A proporo de toda a populao que ca doente da enfermidade
e, entre os que
100
8
cam doentes, a proporo dos que morrem
. Assim, a proporo da populao
100
8
15

, o que corresponde a
que morre pela doena
100 100
120
1,2
15 8
=
=
= 1,2%.
1002
10 000
100
95. Agenda de aulas Se a aula da manh segunda ou sexta (em qualquer um dos trs
horrios), ento o dia da aula de tarde pode ser escolhido de trs formas diferentes (em
qualquer um dos dois horrios), portanto, temos 2 3 3 2 = 36 formas diferentes
de escolher o horrio. No caso em que a aula de manh seja no sbado, o dia da aula
da tarde pode ser qualquer dia de segunda a quinta, portanto, temos 3 4 2 = 24
possveis formas de escolher o horrio. Por ltimo, se a aula da manh tera, quarta
ou quinta, ento a aula da tarde s pode ser escolhida de duas formas, portanto, temos
3322 = 36 formas de escolher o horrio. Assim, Eliane pode escolher seu horrio
de 36 + 24 + 36 = 96 formas distintas.
96. Jogo de Cartas A estratgia abaixo permite realizar o jogo com 17 movimentos.
Em cada movimento, o primeiro nmero indica a pilha da qual a carta tomada e o
segundo a pilha em que a carta colocada. Por exemplo, o primeiro movimento (1)
e 4 sobre 2 signica pegar a carta superior da pilha 4 e colocar sobre a pilha 2.
(1) 4 sobre 2
(7) 3 sobre 4
(13) 2 sobre 1

(2) 4 sobre 3
(8) 1 sobre 3
(14) 2 sobre 1

(3) 4 sobre 2
(9) 1 sobre 4
(15) 4 sobre 2

(4) 3 sobre 4
(10) 2 sobre 1
(16) 4 sobre 2

(5) 3 sobre 4
(11) 2 sobre 4
(17) 4 sobre 2

(6) 1 sobre 4
(12) 2 sobre 3

O movimento 2 tambm poderia ser 4 sobre 1, o movimento 4 poderia ser 1 sobre


4, o movimento 5 poderia ser 1 sobre 4, o movimento 6 poderia ser 3 sobre 4. Os
movimentos 4, 5 e 6 poderiam ser permutados em qualquer ordem. Teramos, assim,
pelo menos, seis maneiras de realizar o jogo com 17 movimentos.
Esse jogo poderia ser realizado com um nmero menor de movimentos?
97. Fraes inteiras Como
2 (n2 + 2n + 1) + 8
1
16
2n2 + 4n + 18
=
=
2n + 2 +
,
3n + 3
3
n+1
3
n+1
segue que a expresso entre parnteses deve ser um mltiplo
de 3 e, em particular, n + 1 deve dividir 16. Assim, n pode ser
1, 3, 7 ou 15. Pela tabela ao lado, em cada um desses quatro
casos, ou seja, para n igual a 1, 3, 7 ou 15, o quociente dado
resulta ser um nmero inteiro.

OBMEP 2010

n
1
3
7
15

2n2 + 4n + 18
3n + 3
4
4
6
11
289

Solues do Nvel 3
98. Quatro prefeitos e um crculo O nmero de rodovias igual ao nmero de
pontos que podem ser o centro de um crculo (rodovia) que seja equidistante de quatro
pontos (cidades) dados. Como nenhum crculo passa pelos quatro pontos dados, se
algum crculo for equidistante dos quatro pontos, esse crculo no pode deixar todos os
quatro pontos do lado de dentro ou todos do lado de fora, de modo que deve dividir o
conjunto dos quatro pontos em dois, sem passar por algum deles. Assim, s podemos
ter trs tipos de congurao, de acordo com o nmero de pontos dentro e fora do
crculo. No primeiro, o crculo equidistante deixa trs pontos dentro e um fora; no
segundo, dois dentro e dois fora e, no terceiro, um dentro e dois fora.
Nas guras abaixo esto ilustrados os dois primeiros tipos, em que o crculo contnuo
o equidistante.
.. .. .. .. ..
.. .. .. .. ...
... .
.. ..
.. ...
.. .
.. .
..
..
..
..
..
.
..
..........................
...........................
.
.
..
.
......
....
......
....
.
..
..
.
.
..
....
...
....
....
. ... ..
.. . .
.
.
..
...
.
.. .. .. ... .. ..
.. .....
.... .. . ....
. ..
...
.
.
.
..
. ..
.. ...
..
..
.
.. .. .. .. .... ..
.
.. ..
.
.
.. ..
.
.
..
..
..
.. ..
..
.. .
.
.
.
.
..
.. .
.
..
..
.
.
.
.
.
.
. .. .
. .
.
.
..
..
.
.
. ..
.
. .
.
.
. . ..
.
.
. .
..
.
.
...
.....
.
. .
.
.
. .
.
.
..... ..
..
.
. .
.... ..
.
.
. .
..............
. .
..
.
.............
.
. .
..
.
.
. .
.
.
. .
.
.
.
.. . .
. .
..
.
.
..
.
.
.
.
.
.
.
.
.
.
. .
.
..
.
.. . .
. .
. .
.
.. . .
.
.
.
.
.
. ..
.
.
..
.
.
.
.
.
.
.
.
.
.
.
.
.
.
.
..
.
. ................ ..
.
.
.
. .... ............. ..
.. .. ..
. ..
. ..
. .
.
. .
.........
.
..
.
.. . ..................
.
..
.
.
. ..................
.
.
.. ..
.
...
..
..
.. ....
..
.
.
..
..
..
.
.. .
.
.. .
...
...
..
.
...
. .. .. .. .. ..
. .. .. .. .. .
.
..
...
....
.. .. .
....
....
.
....
..
.....
.....
.
..
.....
.....
......
.......
.
..
.
..
.......................
.....................
..
..
..
..
.
.
..
.. .
.. .
.. ..
.. .
.. ..
.. .. .. .. .. ... ..
.. .. .. .. ..
.. . . .

qs

.. .. .. .. ..
.. .. .. .. .. ... ..
.. ...
.. .
. ..
. ..
.. .
..
.. .
..
..
.
..................
....................
..
.
.
..
.....
........
.....
.......
..
...
....
..
....
.
.
....
..
....
.
..
...
...
.
.
.. .. .. .. .
.. .. ... .. .
...
.
.. .....
..
.
.
.
..
..
..
. .
.. ..
.
.
.. .
..
.
..
.
.
..
.
.
..
..
.. ..
..
.
..
.
.
.
..
.
.
.
..
..
..
.
.. . .
.
.
.
.
.
...........................................
..........................................
.
.
.
.
.
.
.
.
.
.
.
.
.
.
.
.
.
.
.
.
.
.
.
.
.
.
.
.
.
.
.
.
.
.
.
.
.
.
.
.
.
.
.
.
.
.
.
.
.
.
.
.
.
.
.
.
.
.
.
.
.
.
.
.
..
.
..
.
.
.
.
.
.
.
.
.
.
.
.
..
.
.
.
..
.
.
.
.
.
.
.
.
.
.
..
.
.
.
. ..
.
..
.
.
.
.
. .
.
.
.
. ....
.
.
..
.
..
.
. ... .
.
..
....... .
.
..
..
.
...... .
.
.
..
. ..... .
..
...
..
.
..
..
.
.
.. . .. ........ ..
..
.. .. . .........
..
..
.
.. ...
.
.. .... ..
.. .
. .. .. ..
..
.. .
...
. .. .. .. .. ........ ....
..
. .. .. .. .. ........ .....
...
.
.
.
.. .. . .... .
....
....
.
..
.
... ..
.....
.
..
... ....
.....
.... .....
......
.
..
........
.
.... .....
.
.......................
..................
.
..
.. ...
.
..
..
..
..
..
...
..
.. .
.. .
.. .. .......
.. .
.. .........
.. .. .. .. .. ... ..
. .. .. .. .. ..
.. . . .

s
q

Na primeira gura, o centro do crculo equidistante coincide com o centro do crculo


circunscrito ao tringulo formado pelos trs pontos internos. Essa mesma congurao
ocorre no terceiro tipo, em que o centro do crculo equidistante coincide com o centro
do crculo circunscrito ao tringulo formado pelos trs pontos externos. Assim, nesses
dois tipos, o nmero de crculos equidistantes igual ao nmero de tringulos que
podemos formar com trs dentre os quatro pontos, ou seja, quatro.
Na segunda gura, o centro do crculo equidistante est na mediatriz dos dois pontos
internos e, tambm, na mediatriz dos dois pontos externos. Assim, nesse tipo, o nmero
de crculos equidistantes igual ao nmero de maneiras de dividir o conjunto de quatro
pontos em dois conjuntos de dois pontos cada um, ou seja, trs.
Logo, o nmero possvel de projetos de rodovias circulares equidistantes das quatro
cidades 4 + 3 = 7.
99. Fatoriais Queremos a b c = a! + b! + c! com algarismos 0 a, b, c 9. Como
0! = 1! = 1, 2! = 2, 3! = 6 e 4! = 24, algum dos algarismos a, b ou c deve ser maior
do que 4, pois 0! + 1! + 2! + 3! + 4! = 34 s tem dois dgitos. Se algum dos algarismos
a, b ou c for maior do que ou igual a 6, teremos a b c = a! + b! + c! > 6! = 720,
o que acarreta que algum dos algarismos a, b ou c , pelo menos, igual a 7; mas
ento a b c = a! + b! + c! > 7! = 5 040 tem, pelo menos, quatro dgitos, o que uma
impossibilidade.
Assim, algum dentre a, b e c igual a 5 e os demais so menores do que 5. O menor
nmero possvel 5! + 1! + 0! = 120 + 1 + 1 = 122 e o maior nmero possvel
5! + 3! + 4! = 120 + 6 + 24 = 150. Logo, o algarismo a das centenas 1. Se o algarismo
b das dezenas for 5, ento c 4 e
1! + 5! + c! = 1 + 120 + c! = 121 + c! 121 + 4! = 121 + 24 = 145 = 15 c.
290

OBMEP 2010

Solues do Nvel 3
Se o algarismo b das dezenas for 0, 2 ou 3, ento b! igual a 1, 2 ou 6 e, como
necessariamente c = 5, temos que 1! + b! + 5! = 1 + b! + 120 = 121 + b! igual a 122,
123 ou 127, todos diferentes de 1b 5. Resta apenas a opo b = 4 e c = 5. Nesse caso,
efetivamente 1! + 4! + 5! = 1 + 24 + 120 = 145, como queramos. Os trs nmeros
inteiros so a = 1, b = 4 e c = 5.
100. O Riquinho Os 1 000 reais de Riquinho foram repartidos em parcelas crescentes a
partir de 1, de modo que 1 + 2 + 3 + + n 1 000. Como 1 + 2 + 3 + + n a soma
1
dos n primeiros nmeros inteiros a partir de 1, temos 1 + 2 + 3 + + n = 2 (1 + n)n.
1
Assim, queremos encontrar o maior n tal que 2 (1 + n)n = 1 + 2 + 3 + + n 1 000,
ou seja, tal que n2 + n 2 000 0.
Como n2 + n 2 000 igual a 2 000 para n = 0 e o coeciente dominante desse
polinmio 1 > 0, sabemos que os valores de n2 + n 2 000 so negativos para todo
n entre 0 e a raiz positiva do polinmio quadrtico x2 + x 2 000. Pela frmula de
Bhaskara, a raiz positiva dada por

1 + 1 + 8 000
x=
44,22 ,
2

portanto n2 + n > 2 000 para n 45. Assim, Riquinho distribuiu apenas 44 parcelas.
Como Bernardo recebeu a segunda parcela, a quinta parcela (5 = 2+3), a oitava parcela
(8 = 2 + 2 3), e assim por diante, tambm recebeu a ltima, j que 44 = 2 + 14 3,
num total de
1
2 + 5 + 8 + 11 + + 44 = (44 + 2) 15 = 23 15 = 345 reais.
2
Observao: Depois de distribuir as 44 parcelas, ainda sobram
1
1 000 (44 45) = 1 000 990 = 10
2
dos 1 000 reais de Riquinho.
101. Retngulo com dimenses inteiras Sejam a e b os comprimentos dos lados do
retngulo. Supondo a b, temos b2 < a2 + b2 2b2 , pois a2 b2 . As diagonais do

retngulo medem 1 993, portanto, pelo Teorema de Pitgoras, a2 +b2 = 1 993. Assim,
b2 < 1 993 2b2 , ou seja, 996,5 b2 < 1 993. Assim,

31 < 996,5 b < 1 993 < 45 .


Como b um nmero inteiro, 32 b 44. Alm disso, a b e, como a tambm um
nmero inteiro, 1 993 b2 = a2 deve ser um quadrado perfeito.
Para calcular os valores de a2 = 1 993 b2 , com b variando de 32 a 44, calculamos
primeiro 1 993 322 = 969 e observamos que
1 993 332 = 1 993 (32 + 1)2 = (1 993 322 ) (2 32 + 1)
= 969 65 = 904
e, em geral, 1 993 (n + 1)2 = (1 993 n2 ) (2n + 1). Desse modo, fcil obter a
tabela seguinte.
OBMEP 2010

291

Solues do Nvel 3
b
32
39

a2
969
472

b
33
40

a2
904
393

b
34
41

a2
837
312

b
35
42

a2
768
229

b
36
43

a2
697
144

b
37
44

a2
624
57

b
38

a2
549

Como 144 o nico quadrado perfeito da lista, a nica soluo um retngulo de lados
medindo 43 e 12 cm.
102. Mltiplos de 3 e quadrados perfeitos Escrevendo a para um nmero qualquer
da lista, sabemos que a um mltiplo de 3 e que a + 1 um quadrado perfeito, ou
seja, a + 1 = k 2 , para algum inteiro positivo k. Assim, a = k 2 1 = (k 1)(k + 1).

Como a divisvel por 3, ento ou k + 1 ou k 1 divisvel por 3. Logo, k s no


pode ser divisvel por 3 e, portanto, a cada trs inteiros k consecutivos (comeando em
2), dois deles fornecem um nmero da lista: de fato, k = 2 = 1 3 1 d o primeiro
nmero a = 22 1 = 3 da lista; k = 4 = 1 3 + 1 d a = 42 1 = 15, que o segundo;
k = 5 = 2 3 1 d a = 52 1 = 24, que o terceiro; k = 7 = 2 3 + 1 d o quarto
a = 72 1 = 48, e assim por diante. Em geral, uma posio par 2n da lista dada
por k = n 3 + 1, portanto a 2 006a posio par dada por k = 1 003 3 + 1 = 3 010.
Assim, o mltiplo de 3 na 2 006a posio da lista a = 3 0102 1 = 9 060 099.

103. Cinco cartas Simone no precisa virar a carta que tem o nmero 2 porque o outro
lado, vogal ou consoante, de qualquer forma cumpre a condio Se uma carta tem
uma vogal numa face, ento ela tem um nmero par na outra.
................. ................. ................. ................. .................
. .
. .
.
. .
. .
.
. .
. .
.
. .
. .
.
.
. .
. .
.
. .
. .
.
. .
. .
. 2 . . 3 . .M . . A . . E .
. .
. .
.
. .
. .
. .
.
. .
.
.
. .
. .
. .
.
. .
. .
.
. .
. .
. .
. .
.
. .
. .
.
................. ................. ................. ................. .................
.
. .
. .
.
. .
. .
.
Ela tambm no precisa virar a carta com a letra M, j que, do outro lado, o nmero
pode ser par ou mpar que a condio satisfeita. Entretanto, a carta que tem o
nmero 3 precisa ser virada para comprovar que do outro lado tem uma consoante,
bem como as cartas com as letras A e E, para comprovar que do outro lado o nmero
par. Assim, Simone precisa virar somente 3 cartas.
104. O lucro de uma companhia A opo correta (c).
Nos primeiros R$ 1 000,00, a companhia tem lucro de 1 000 6% = 60 reais e, para
os R$ 5 000,00 restantes, tem lucro de 5 000 5% = 250 reais. Logo, o lucro da
companhia nesse dia de R$ 310,00.
105. Sequncia triangular Observe que o 21o termo a soma de 21 nmeros consecutivos. Para descobrir quais so esses nmeros, basta encontrar a primeira parcela do 21o
termo que a soma de 21 nmeros consecutivos. Observamos que, a partir do segundo
termo, a primeira parcela do
2o termo 2
3o termo 4
4o termo 7
5o termo 11
6o termo 16

292

=
=
=
=
=

1 + 1, a do
2 + 1 + 1, a do
3 + 2 + 1 + 1, a do
4 + 3 + 2 + 1 + 1, a do
5 + 4 + 3 + 2 + 1 + 1,

OBMEP 2010

Solues do Nvel 3
e assim por diante. Portanto, podemos ver que a primeira parcela do 21o termo
20 + 19 + + 3 + 2 + 1 + 1 =

21 20
+ 1 = 211
2

e que, portanto, o 21o termo


1
211 + 212 + + 230 + 231 = (211 + 231) 21 = 221 21 = 4 641.
2
106. O jardim octogonal O comprimento total da cerca con.......................
siste na soma dos comprimentos dos contornos da roseira e
.....
....
.....
.....
..
do jardim, ambos utilizando diagonais dos quadradinhos da
.....
....
.
.
...
... ..
.
.
.. ....
..
..
..
.
.
folha quadriculada. O contorno da roseira formado por qua..
.
..
..
..
..
.
.
..
..
.. roseira
..
.
.
.
..
.
..
..
..
..
.
.
..
.
..
tro diagonais e o do jardim por oito diagonais e oito lados.
.. ...
.. ...
..
...
...
....
..
..
.....
Assim, o comprimento total da cerca de oito lados e doze
....
.....
.....
.....
diagonais de quadradinhos. Para descobrir o comprimento
.....................
das diagonais, basta descobrir o comprimento dos lados dos
quadradinhos. Para isso, utilizamos a informao fornecida
relativamente rea do jardim.
Contando o nmero de quadradinhos do jardim, obtemos 24 quadradinhos inteiros
e oito metades, o que igual a 28 quadradinhos inteiros que representam 700 m2
de rea. Desse modo, cada quadradinho mede 700 28 = 25 m2 e, portanto, cada
lado de quadradinho representa 5 m. Pelo Teorema de Pitgoras, a diagonal de cada

=
quadradinho mede d = 52 + 52 5 2 m. Conclumos que o comprimento to
tal da cerca de 8 5 + 12 5 2 = 40 + 60 2 m. Como o prefeito dispe de
R$ 650,00, cada metro dessa cerca pode custar, no mximo,
650
65
65
65
=

=
5,21 reais.
4 + 6 1,414
12,484
40 + 60 2
4+6 2
107. Nmero de caracteres Na 1a linha escrevemos os nmeros de 1 a 9, cada um
seguido de um espao, o que ocupa 18 espaos; sobram 82 espaos. Cada nmero de
dois algarismos mais um espao ocupa trs lugares na linha. Como 82 = 27 3 + 1,
completamos a 1a linha com 27 nmeros de dois algarismos, a partir do nmero 10.
Assim, o ltimo nmero da primeira linha o 36 e sobra um espao. Denotando cada
espao entre nmeros por um trao, podemos representar a 1a linha como segue.
1a linha : 1 2 3 4 5 6 7 8 9 10 36
18

81

Mas 100 = 33 3 + 1, portanto, podemos colocar 33 nmeros de dois algarismos na


segunda linha, cada um seguido de seu respectivo espao, e no nal da linha sobra um
espao.
2a linha : 37 38 69
99

Na 3 linha, colocamos os nmeros de 70 a 99, ocupando 303 = 90 espaos. Ocupamos


os dez espaos restantes com os dois primeiros nmeros de trs algarismos, seguidos de
um espao; no nal da linha, sobram dois espaos.
3a linha : 70 71 99 100 101
90

OBMEP 2010

293

Solues do Nvel 3
A partir da quarta linha, podemos colocar, em cada linha, 1004 = 25 nmeros de trs
algarismos com seus respectivos espaos. De 102 a 999, inclusive, temos 999102+1 =
898 nmeros. Como 898 = 25 35 + 23, ocupamos 35 linhas, desde a 4a at a 38a , com
esses nmeros de trs algarismos e ainda restam 23 desses nmeros, que colocamos na
39a linha, onde ocupam 23 4 = 92 espaos, restando oito espaos, que ocupamos com
1 000, restando trs.
39a linha : 977 999 1 000
92

A partir da 40 linha, podemos colocar, em cada linha, 1005 = 20 nmeros de quatro


algarismos com seus respectivos espaos. Assim, nas 61 linhas restantes, podemos
colocar exatamente 61 20 = 1 220 nmeros de quatro algarismos. Comeando com
1 001, terminamos a 100a linha com o nmero 2 220.
108. A rvore de Emlia Denotemos por gn o nmero de galhos
da rvore depois de n semanas. Como s depois de duas semanas
aparece um galho, g2 = 1. Na terceira semana esse galho produz um
novo galho, ou seja, g3 = 2. O nmero de galhos em uma semana
igual ao nmero de galhos que existiam na semana anterior, mais
os galhos novos.

q q q

.
.
. .
.
.
.
.
.
.
.
.
.
.
.
.
.
.
.
.
.
.
..
.
.
.
.
. .
.
.
. .
.
. ....
. .
. ....
.
. ...
. .
.
. ....
. ...
.
.....
.
.
...
.
.
. .
.
.
.
.
.....
.
.
.....
.
.
.
.
.
.
.
.
.
.
.
.
.
.
.
.
.
.
.
.
.
.
.
.
.
.
.
.
.
..
.
..
.
.
.
.
.. .....
.
... ..... .
.
.
...... .
.
.... .
.
.
.
.
.
.
.
.
.
.
.
.
.
.
.
.
..
.
...
.
..
. .... ....
. .........
.
.
.
. ..
.
.
.
.
.
.
.
.
.
.
.
.
.
.
.
.
.
.
.
.
.

q
q
q

Entretanto, pela regra fornecida, os galhos novos nascem dos galhos que tm pelo
menos duas semanas de idade, isto , na semana n + 1 tem um galho novo para cada
galho que j existia na semana n 1. Assim, temos gn+1 = gn + gn1 , de modo que, a
partir de g2 = 1 e g3 = 2, obtemos
g4
g5
g6
g7
g8

=
=
=
=
=

2+1=3
3+2=5
5+3=8
8 + 5 = 13
13 + 8 = 21

No m de oito semanas, a rvore de Emlia ter um total de 21 galhos.


109. Um teste vocacional
(a) De exatas temos 232 + 112 = 344 alunos e a probabilidade de escolher ao acaso
344
um de exatas
= 0,344.
1 000
(b) Do sexo masculino temos 232 + 116 + 207 = 555 alunos e a probabilidade de
116
= 0,209.
escolher ao acaso um da rea de humanas
555
(c) De biolgicas temos 207+180 = 387 alunos e a probabilidade de escolher ao acaso
180
um do sexo feminino
= 0,465.
387
110. Dois setores circulares Como 60 + 30 = 90 , segue que a rea destacada representa um quarto da rea total do crculo.
Como a rea do crculo mede
20 cm2 , a rea destacada mede 5 cm2 .
294

OBMEP 2010

Solues do Nvel 3
111. Compra de televisores Digamos que Maria tenha encomendado n televisores,
pagando R$ 1 994,00 por televisor. O total pago 1 994 n, e sabemos que nesse mltiplo
de 1 994 no aparecem os algarismos 0, 7, 8 e 9. Para limitar o valor de n, observamos
que
1 994n = 2 000n 6n.
Se 6n < 300, ento o nmero 2 000n 6n tem 7 ou 8 ou 9 no algarismo das centenas
(faa alguns exemplos, lembrando que 2 000n termina com trs zeros). Assim, devemos
ter 6n 300, ou seja, n 50.
Testando n igual a 50, 51, 52, 53, 54 e 55, obtemos 99 700, 101 694, 103 688, 105 682,
107 676 e 109 670, mas n = 56 d 111 664. Assim, o menor nmero de televisores que
Maria pode ter encomendado 56.

112. Distncia entre nmeros Observe que |x y| igual distncia entre os pontos
x e y e, em particular, |x| igual distncia entre x e a origem 0. Pela gura, temos
4 < a < 3 < 2 < b < 1 < c < 0 < 1 < d < 2,
de modo que (a), (b), (c), (d) e (e) so verdadeiras e (f) falsa, pois
|d| = d < 2 < 3 < |a|.
a
4

b
2

d
0

Tambm 1 < |a b| < 3, 1 < |c d| < 3, 0 < |b c| < 2 e 2 < |c a| < 4, o que
acarreta que (g), (j) e (l) so verdadeiras e (h), (i) e (k) so falsas.
113. Cartes premiados O carto de dgitos a b c d premiado se a + b = c + d. Consideremos dois casos. Se o nmero de dgitos c d de um carto premiado for igual ao
nmero de dgitos a b, ento a b c d = a b a b = a b 100 + a b = 101 a b e o nmero
desse carto divisvel por 101. Caso contrrio, c d = a b e, ento h um outro carto
premiado, a saber, c d a b. A soma dos nmeros desses dois cartes
a b c d + c d a b = (a b 100 + c d) + (c d 100 + a b) = 101(a b + c d),
que tambm divisvel por 101. Como todo carto premiado de um desses dois tipos,
a soma dos nmeros de todos os cartes premiados divisvel por 101.
114. O preo da gasolina A opo correta (d).
Soluo 1: O aumento do preo foi de 149,70 29,90 = 119,80 reais, o que corresponde
a
119,80
100% = 400,66%.
29,90
Soluo 2: O preo praticamente passou de 30 para 150, isto , foi multiplicado por 5,
o que equivale ao preo passar de 100 para 500, caracterizando um aumento de 400%.
OBMEP 2010

295

Solues do Nvel 3
115. O tringulo de moedas Supondo que o tringulo esteja formado por n linhas,
foram usadas 1 + 2 + 3 + + n moedas, ou seja,
1
n(n + 1) = 1 + 2 + + n = 480 15 = 465,
2
o que fornece n2 + n 930 = 0. Resolvendo essa equao, obtemos

1 61
1 1 + 4 930
=
.
n=
2
2
Como n = 30 a nica soluo positiva dessa equao, o tringulo tem 30 linhas.
116. Circunferncia e tringulo retngulo Seja r o raio da circunferncia inscrita no
tringulo. Como o tringulo retngulo, fcil vericar que os catetos do tringulo
medem 6+r e 7+r cm, conforme a gura. Pelo Teorema de Pitgoras, temos (6+7)2 =
(r + 6)2 + (r + 7)2 e, desenvolvendo, resulta 169 = 132 = 2(r2 + 13r) + 85, de modo
que r2 + 13r = 42.
Q
Q6
6 ................Q...Q...........
...... ......
...... ......
...
...
...
...
Q.....
..
..
..
Q
.
.
..
.
.
.
. Q
.
.
.
.
.
.
.
.
.
.
Q7
.
.
.
.
.
.
.
.
.
.
Q
.
.
.
.
..
.
..
.
Q
..
..
r ..................... ....................
Q
...............
..............
Q

Assim, a rea do tringulo mede


(r + 6)(r + 7)
r2 + 13r + 42
42 + 42
=
=
2
2
2
2
= 42 cm .

117. Soma de razo

1
1 1 1
1
Como Sn = + + + + n , segue que
2
2 4 8
2

1
1
Sn =
2
2
de modo que

1
1 1 1
+ + + + n
2 4 8
2

1 1
1
+ + + n+1 ,
4 8
2

1
1
1
1
Sn = Sn Sn = n+1 e, portanto,
2
2
2 2
Sn = 1

1
.
2n

Como queremos Sn > 0,99, isso equivale a


1

1
1
,
= Sn > 0,99 = 1 0,01 = 1
n
2
100

ou seja, a 2n > 100. Como 27 = 128 > 100 > 26 = 64, n = 7 o menor n tal que
2n > 100, ou Sn > 0,99.
Observao: Note que, no incio desta resoluo, deduzimos o valor da soma da progresso geomtrica de primeiro termo e razo ambos iguais a 1/2, sem usar a frmula
dessa soma, a saber,
1 1
1
n
1
Sn = 2 2 2 = 1 n .
1
2
1
2
296

OBMEP 2010

Solues do Nvel 3
118. Soma de razes quadradas
(a) Como

r2 = ( 2 + 3)2 = ( 2)2 + 2( 2)( 3) + ( 3)2 = 2 + 2 6 + 3 = 5 + 2 6,

r2 5
segue que r2 5 = 2 6, ou 6 =
.
2
(b) Pelo mesmo argumento,

s2 = ( 215 + 300)2 = 215 + 2 215 300 + 300

= 515 + 10 43 60 = 515 + 10 2 580

> 515 + 10 2 500 = 515 + 500 = 1 015.


119. Duas rodas Enquanto a roda A d 1 200 voltas, a roda B d 1 500 voltas ou,
equivalentemente, a roda A d 4 voltas a cada 5 voltas da roda B. Denotemos por R o
raio da roda A e por r o raio da roda B. O comprimento da roda A 2R e o da roda
B 2r, portanto, o comprimento de 4 voltas da roda A 4 (2R) e o comprimento
de 5 voltas da roda B 5 (2r). Como esses dois comprimentos so iguais, temos
que 4R = 5r. Por outro lado, na gura vemos que 2(r + R) = 9, de modo que
9 = 2(r + R) = 2r + 2

5
9
5
r = 2+
r = r,
4
2
2

e, assim, estabelecemos que r = 2 cm e R = 2,5 cm.


120. Dois divisores A opo correta (c).
O nmero N = 248 1 muito grande mas, mesmo assim, podemos descobrir vrios
de seus divisores. Para isso, utilizamos a igualdade
(x 1) xa1 + xa2 + + x + 1 = xa 1.
Notamos que os divisores de 48 so 1, 2, 3, 4, 6, 8, 12, 24 e 48. Tomando x = 22 e
a = 24 na igualdade acima, estabelecemos que x 1 = 22 1 = 3 um divisor de
(22 )24 1 = 248 1 = N. Analogamente, tomando x = 23 e a = 18, estabelecemos que
x 1 = 23 1 = 7 um outro divisor de (23 )18 1 = 248 1 = N. Procedendo dessa
maneira, vericamos que, para qualquer divisor d de 48, o nmero 2d 1 um divisor
de N. Dessa forma, conclumos que 22 1 = 3, 23 1 = 7, 24 1 = 17, 26 1 = 63,
28 1 = 251, e assim por diante, so divisores de N.
Alm desses, podemos ainda obter outros divisores de N considerando os divisores
pares d = 2n de 48 e usando o produto notvel
2d 1 = 2n

1 = 2n + 1 2n 1 .

Como 2d 1 um divisor de N, tambm 2n + 1 um divisor de N. Por exemplo,


d = 4 = 2 2 fornece o divisor 22 + 1 = 5 de N, d = 6 = 2 3 fornece o divisor
23 + 1 = 9 de N, d = 8 = 2 4 fornece o divisor 24 + 1 = 17 de N, d = 12 = 2 6
fornece o divisor 26 + 1 = 65 de N, e assim por diante.
Note que j obtivemos dois divisores de N = 248 1 entre 60 e 70, a saber, 63 e 65.
OBMEP 2010

297

Solues do Nvel 3
Observao: Com o auxlio de um computador, podemos ver que N , realmente, um
nmero muito grande, j que N = 248 1 = 281 474 976 710 655, e obter sua fatorao,
dada por

N = 248 1 = 32 5 7 13 17 97 241 257 673.


Empregando o argumento exposto na resoluo deste exerccio, possvel encontrar
vrios divisores de nmeros bastante grandes.

121. Rede de estaes O exemplo mostra que podemos conectar pelo menos sete estaes dentro das condies propostas. Comeamos com uma estao particular, e
vamos pensar nela como se fosse a base da rede. Ela pode ser conectada a uma, duas,
ou trs estaes, conforme mostra o primeiro dos dois diagramas a seguir. As estaes
A, B e C tm, ainda, duas linhas no utilizadas, portanto, podem ser conectadas a
duas outras estaes, como no segundo dos dois diagramas a seguir.

....
.......
... ....
.. ...
.
.
.
.
.
.
.
.
.
..
... ....
.........
....
.
.
.
.
.
.
.
.
.
.
.
.
.......
.........
.. ...
..
.
.
.
.
.
.
.
..
... ....
.
............
..... ...
.
.
...
...
....
....
.
.
...
...
....
...
....
...
..
...
...
...
....
....
..........
..........
.........
..
. ..
.
.........
..
..
..
.
..
..
.
.
.
.
.
.
.
.
.
.
.
.
.
.
.
.
..
..
... ....
... ....
.........
.........
....
....

Base

....
....
.........
.........
... ....
... ....
..
..
.
.
.
.
.
.
.
.
.
.
.
.
.
.
.
.
.
.
.
..
..
...
... ........
......... .....
... ..........
... ..........
....
...
.
.
...
...
...
...
... ....... ...
... ......... ....
..... .....
..
..
.
.
.
..
.
.
.
.
.
.
..
... ....
.........
....
.
.
.
.
.
.
.
.
.
.
.
.
.......
.........
.. ...
..
.
.
.
.
.
.
.
..
... ....
.
............
..... ...
.
.
...
...
....
....
.
.
...
...
....
...
....
...
..
...
...
...
....
....
..........
..........
.....
.........
.........
..
. ..
.........
.........
.
.........
..
... ...
..
..
..
.
.
.
..
..
..
..
.
.
.
.
.
.
.
.
.
.
.
.
.
.
.
.
..................
..................
.
.
.
.
.
.
.
...................
...................
.
..
..
.
..
..
.
... ....
... ....
..........
..........
...... ..
...... ..
.........
.........
....
....
.
.
.
.
.
.
.
.
.
.
.
.
.
.
.
.
.
.
.
.
.
.
.
.
.
.
.
.
.
.
.
.
.
.
.
.
.
.
.
.......
.......
........
........
. .
.
... .
... .
.
.
..
..
.
.
.
.
.
.
.
.
.
.
.
.
.
.
.
.
..
..
... ....
... ....
..
.........
.......
....
....

Base

Agora, impossvel acrescentar mais estaes, pois qualquer outra a mais no poderia
ser conectada base satisfazendo as condies do problema. Isso mostra que no
podemos ter mais do que 10 estaes.
Vejamos, agora, se possvel montar uma rede com essas 10 estaes. Observe, no
diagrama acima, que apenas a base conectada a todas as outras estaes (atravs de
um cabo ou de uma conexo via uma estao). As estaes que esto nos extremos
ainda possuem duas linhas no utilizadas, e agora vamos us-las para fechar a rede.
Veja o diagrama a seguir.
298

OBMEP 2010

Solues do Nvel 3
.
...
......
.......
.
.......
. ...........
...... .
...... ..
. ...........
.
......
.
......
.
......
......
.
......
.
.
......
......
.
......
.
.
......
.
......
......
.
.
......
.
......
.
......
.
......
.
.
......
.
.
...... .........
.
.
...... .......
.
.
.
..
.
...
.
.
.
.
...... ..........
.
...... ...........
.
...
...
.
.
......
......
.
.
......
.
......
.
..
...
............
...........
.
.
.
.
.........
.
..........
..
.
.
..
.
.
.
.
.
.
.
.
.
.
.
.
.
.
.
.
.
.
.
.
.
.
.
.
.
.
.
.
.
..
.
.
.
..
....
.
...
..
.
..
.
.
.
.
... ... ..
.
.
.
.
.
......... ...
... .........
....
.
... ..........
.
.
...
.
.
.
...
..
.
.
.
...
..
...
.
...
.
..
... ....... ...
.
.
.
... ......... .....
.
.
..
.
.... ....
.
..
..
.
.
..
.
..
.
.
.
.
.
.
..
.
..
.
.
.
.
..
.
.
.
.
.
.
.
.
.
.
..
.
.
.
..
..
.
... ...
..
.
.
.
.
.........
.
....
..
.
.
..
.
.
.
..
.
.
.
.
.
..
.
.
.
..
.
.
.
..
.
.
.
.
.
..
.
.
.
.
.
.
..
.
.
.
.
..
.
.
.
..
.
.
.
.
.
..
.
.
.........
..
..........
.
..
.
.
.
.
.
.
..
.
.
..
..
.
.
.
.
..
.
.
.
.
.
.
.
.
.
.
..
.
.
..
.
..
.
..
... ...
.
.
.
.
.
.
..
.
............
..... ...
.
..
.
.
.
..
.
...
.
.
...
.
....
....
..
.
.
.
.
...
..
.
.
...
..
.
.
.
....
...
..
....
.
.
...
.
.
.
.
..
...
.
.
.
...
..
....
.
.
..
....
.
.
.
..........
.
..........
.
.
.
.
..
.........
.........
.........
. ..
.
.........
.........
.........
.
.
..
.
.
..
..
..
..
.
.
.
.
..
..
..
..
.
.
.
.
.
.
.
.
.
.
.
.
.
.
.
.
.
.
.
.
.
.
.
..................
..................
.
.
.
.
.
...................
...................
.
.
.
.
.
..
..
..
..
.
.
.
.
... ....
... ....
.
.
..........
..........
...... ..
...... ...
.
.........
.........
..
.
.
....
.
.
.
.
.
.
. ....
.
.
.
.
..
.
.
.
.
..
.
.
.
.
.
.
.
.
.
.
.
.
.
.
.
..
.
.
.
.
.
..
.
.
.
.
.
.
.
.
.
.
.
.
.
.
...
..
.
.
.
.
........
. .....
..
.
.
.
.
.
.......
.
........
.
.
........ .
........
.
.
.. ....
.
.
. ....
........
.
.
........
........
........
.
.
. ...
.... .
.... .
.
.
.
........
.
.
.
........
.......
.......
........
.
........
........
........
.
. .
........
..
.
...
.
........
..
.
..
... . ...............
........ ..
.
................
..
..
.
.......
................................
.
.
..
.
.
.
..
....
.
..
.
.
.
. ...............................
.
.
.
.
.
..
.
..
..
... ...
... ....
.
..
.
....... .
.........
.....
....
.
.
..
.
..
.
.
..
.
..
.
.
....................................................................................................................................
.
.....................................................................................................................................
.
.

Base

122. Bolas brancas e pretas A opo correta (b).


Inicialmente observe que, depois de cada substituio, o nmero de bolas brancas, ou
permanece o mesmo, ou decresce de dois. Logo, o nmero de bolas brancas permanece
par. Por outro lado, cada grupo de bolas removidas que contm pelo menos uma bola
branca substitudo por outro grupo que tambm contm uma bola branca, portanto,
o nmero de bolas brancas nunca zero. Agora observe que a opo (b) a nica
que inclui pelo menos duas bolas brancas, logo ela a opo correta. Um modo de
obter esse resultado remover trs bolas brancas 49 vezes at obter 149 pretas e duas
brancas e, depois, remover uma preta e duas brancas 149 vezes.
123. O cubo Seja a a aresta do cubo que Alice quer construir. Como a rea lateral do
cubo mede 6a2 cm2 , devemos ter 6a2 25 60, isto , a2 250 e, portanto, a < 16.
Com a = 15 temos 4 = 6015 quadrados de lado medindo 15 cm e sobra um retngulo
de 60 por 10 cm. Podemos cortar fora um retngulo de 60 por 2,5 cm e os pedaos
marcados com , de dimenses 15 por 7,5 cm. Assim, na gura, a linha pontilhada
indica dobradura e a linha contnua indica corte; com os dois pedaos de cartolina
marcados com formamos a tampa.

7, 5

124. Um quadrado e um tringulo


Soluo 1: Indiquemos por S1 , S2 e S3 as reas dos tringulos XAB, AY D e
BCZ e por Sq a rea do quadrado ABCD, conforme indicado na gura. Se S
denota a rea do tringulo XY Z, ento S = S1 + S2 + S3 + Sq e como, por hiptese,
Sq = 7/32 S, estabelecemos que
OBMEP 2010

299

Solues do Nvel 3

S1 + S2 + S3
S Sq
Sq
=
=1
S
S
S
25
7
=
.
=1
32
32

.....
......
..... ..
..... ..
..
.....
.....
..
.....
.....
.....
.....
1 .......
....
....
.....
.....
.
.............................................
.
..
..............................................
..
. .
.
.
..
..... .
..... .
. ..
.
.
.
.
.....
.....
. .
.
. ..
.
.
.
.....
.....
.
.
. ..
.
.....
. ...
.
.....
.
.
..
..
.
.
.
.
..
.....
.
.
.....
..
..
.
.
..
.
.
.
.
..
.....
.....
.
.
..
..
..
.
.
.
.
..
.
.
.....
.....
..
.
.
..
..
.
.
.
.
..
.....
.
.
.....
.
.
..
..
..
.
.
q
.
.
.....
.
.
..
.....
.
.
..
..
..
.
.
.
.
.....
.
.
.....
.
.
.
..
..
.
.
.
.
.....
.
.
.....
2
3 ........
.
.
..
..
.
.
.
.
.....
.
.
.....
..
..
.
.
..
.
.
..
.
.
.....
.....
.
.
.
..
.
.
.
.
.....
....................................................................................................................................................
.
.....................................................................................................................................................
...
......
.
.
.

Como XAB e XY Z so tringulos semelhantes, a razo entre suas reas igual


ao quadrado da razo de semelhana, isto ,
XA
XY

S1
=
S

Transladando horizontalmente o tringulo BCZ de modo a justap-lo ao tringulo


ADY, obtemos um tringulo semelhante a XY Z, mas de rea S2 + S3 . Assim,
S2 + S3
=
S

AY
XY

Somando esses dois quocientes obtidos e usando a proporo estabelecida acima, conclumos que
25
S1 + S2 + S3
(XA)2 + (AY )2
(XA)2 + (XY XA)2
=
=
=
32
S
(XY )2
(XY )2
=

(XY )2 + 2(XA)2 2(XY )(XA)


=1+2
(XY )2

XA
XY

XA
XY

ou seja, a razo entre XA e XY procurada satisfaz a equao de segundo grau


2

XA
XY

XA
XY

7
= 0.
32

Usando a frmula de Bhaskara, obtemos dois valores possveis para essa razo, a saber,
7 XA
1
XA
= e
= .
XY
8 XY
8
Soluo 2: Denotemos o comprimento do lado do quadrado ABCD por l, a altura
do tringulo XY Z por H, a altura do tringulo XAB por h e o comprimento do
1
lado Y Z por b. A rea do quadrado l2 e a rea do tringulo XY Z 2 bH. Como
os tringulos XY Z e XAB so semelhantes, temos
X

...
.
...
..... ...
.
..... ...
. .
..... . ...
.....
.
. .
. .
.....
.....
. .
. .
.
.....
.....
.
. ..
.....
. ..
.....
.
.
..
.
....
.
.
........................................
.........................................
.. .
.. ....
.
...
......
...
..... .
.. .
.. .
.
.
. .
. ..
.....
.....
.
.
..
..
.
.
.
. ..
.....
.
. ...
.....
.
.
..
..
.
.
.
. .
.....
.
. ..
.....
.
.
..
..
.
.
..
.
.
..
.....
.
.
.....
.
.
..
..
.
.
..
.
.
..
.....
.
.
.....
..
.
.
..
.
.
..
.
.
.....
..
.....
.
.
..
..
.
.
.
.
..
.....
.
.
.....
..
.
.
..
..
.
.
.
.
..
.....
.
.
.....
..
.
.
..
..
.
.
.
.
..
.....
.
.
.....
..
.
.
..
..
.
.
.
.
.....
..
.
.
.....
..
.
.
..
..
.
.
.
.
.....
.....
.
.
..
..
.
.
..
..
.
.
.
.
.....
.....
.
.
.
.
.
.
.
.
......................................................................................................................................................
.......................................................................................................................................................
...
..
.
.

l
h
XA
=
=
.
b
H
XY
Y

300

OBMEP 2010

Solues do Nvel 3
Hl
1
H2 l
(h + l)2 l
Portanto, b =
, a rea do tringulo XY Z bH =
=
e a razo
h
2
2h
2h
procurada dada por
XA
h
h
1
,
=
=
=
l
XY
H
h+l
1+ h
de modo que resta calcular a razo l/h.
2

Como a razo entre a rea (h+l) l do tringulo XY Z e a rea l2 do quadrado


2h
32/7, obtemos 7(h + l)2 = 64hl. Expandindo e dividindo por h2 , obtemos a equao
quadrtica
l 2
l
7
+ 7 = 0,
50
h
h
que tem solues

50 502 4 49
25 252 72
25 24
l
=
=
=
.
h
14
7
7
XA
tem dois possveis
Assim, l/h tem dois possveis valores, 1/7 e 7 e, portanto,
XY
XA
7 XA
1
valores,
= e
= .
XY
8 XY
8
125. A urna
Soluo 1: Os pares
{1, 2}, {1, 3}, {1, 4}, {1, 5}, {1, 6},
{2, 3}, {2, 4}, {2, 5}, {2, 6}, {3, 4},
{3, 5}, {3, 6}, {4, 5}, {4, 6}, {5, 6}
so os nicos pares de bolas diferentes que podem ser retirados da urna. Logo, podem
ser retirados da urna 5 + 4 + 3 + 2 + 1 = 15 diferentes pares de bolas. Dentre esses,
existem apenas 5 pares de bolas numeradas com nmeros que diferem por uma unidade,
a saber, {1, 2}, {2, 3}, {3, 4}, {4, 5} e {5, 6}. Assim, a probabilidade que um desses pares
5
1
seja retirado
= .
15
3
Soluo 2: Observemos que se extrairmos a primeira bola com um nmero entre
2 e 5, ento dentre as cinco bolas que cam na urna, temos duas possveis bolas que
cumprem a condio do problema, portanto, nesse caso, a probabilidade que a segunda
bola cumpra a condio 2 e a probabilidade que a primeira bola tenha um nmero
5
4
entre 2 e 5 6 . Por outro lado, se a primeira bola extrada for 1 ou 6, s temos uma
bola na urna que cumpre a condio, portanto, nesse caso, a probabilidade para a
escolha da segunda bola 1 e a probabilidade da primeira bola ser 1 ou 6 2 . Assim,
5
6
a probabilidade das bolas serem consecutivas
5
1
4 2 2 1
+ =
= .
6 5 6 5
15
3
126. Soma das razes de uma equao Devemos considerar dois casos. Se x + 1 0,
ento |x + 1| = x + 1 e a equao x2 + 3x + 2 = x + 1, ou seja, x2 + 2x + 1 = 0,
que s possui a soluo x = 1. Se x + 1 < 0, ento |x + 1| = x 1 e a equao
OBMEP 2010

301

Solues do Nvel 3
x2 + 3x + 2 = x 1, ou seja, x2 + 4x + 3 = 0, que tem apenas a soluo x = 3 no
intervalo x < 1. Assim, as nicas solues distintas da equao dada so 1 e 3,
de modo que a soma de todas as razes distintas da equao 4.
127. Produto de trs nmeros Sejam a, b, c, . . . , i, j os nmeros nos 10 crculos, conforme indicado no diagrama.
.....
.........
... ....
..
.
.
.
.
.
.
.
.
.
..
.
..
..........
.........

....
.....
......... ...........
... .... .... ....
..

....
....
.....
......... ........... ...........
... .... .... .... .... ....
..

....
....
....
.....
......... ........... ........... ...........
... .... .... .... .... .... .... ....
..

.
. .
. .
. .
. .
.
. .
.
. .
.
.
.
a ..............b............. ..............c............. ..............d..........................e............. ..............f............ = ..............g..........................h........... ..............i............... ..............j..............
..
..
....
...
...
...
...
...
...
..
.
...
...
..
...
...
..
..

Observe que a, c e f no podem ser zero, pois 0 x = 0, para qualquer x. entretanto,


o produto dos trs nmeros um nmero de quatro algarismos, portanto, a b d < 10 e
os nmeros que aparecem em a b d so 1,2 e 3 ou 1,2 e 4. Observemos que a segunda
opo impossvel de ocorrer, porque o mnimo produto que podemos obter nesse caso
1 23 456 = 10 488 , de modo que a b d = 6 e o produto maior do que 6 000.
Tampouco pode a ser 2 ou 3, porque nesse caso o mnimo valor que tem o produto
2 14 356 = 9 968 e os outros produtos cam maiores do que 10 000. Assim, a = 1.
Continuando essa anlise, chegaremos soluo dada no diagrama.
..........
..........
..
.
..
.
.
.
.
.
.
.
..
.
..
..........
..........

....
.....
......... ...........
... .... .... ....
..

....
....
.....
......... ........... ...........
... .... .... .... .... ....
..

....
....
....
.....
......... ........... ........... ...........
... .... .... .... .... .... .... ....
..

. .
.
.
. .
.
.
. .
. .
.
. .
. .
.
1 ...............2...........................6............ ...............3...........................4...........................5............ = ...............8...........................9...........................7...........................0............
...
...
...
...
...
...
...
...
...
...
...
...
...
...
...
...
...
...

128. rea do tringulo Para determinar a rea de um tringulo, basta conhecer o comprimento de uma base e sua
respectiva altura. Tomando AC como base, a altura corta
AC no ponto H = (1, 0), j que o segmento AC vertical
e o segmento HB horizontal. Assim, a base AC mede 8
unidades e a altura BH relativa a essa base mede 7. Logo,
a rea do tringulo 1 7 8 2 = 28 unidades de rea.
2

y6

A..... = (1, 2)
...
.
..
.

. .....
. ......
.
.....
.
.....
.
.
.....
.
.....
.
.....
.
.....
.
.
.....
.
.....
.
.
.....
.....
.
.
.....
.
...
.
.
.
.
.
...
....
.
.
.
....
....
.
.
....
.
....
.
.
.
.
.
....
.
....
.
.
.
....
....
.
.
.
....
....
.
.
.
....
....
.
.
. ......
. ......
. ..
. ...
. ..
....
..
.
.
.

Ht

B = (8, 0)
x

C = (1, 6)

129. Duas tabelas Vemos que na primeira tabela cada linha uma progresso aritmtica
de razo 3 e cada coluna uma progresso aritmtica de razo 7.
5
12
19
26
33

8
15
22
29
36

11
18
25
32
39

14
21
28
35
42

17
24
31
38
45

39
87
56

Digamos que na segunda tabela a razo das progresses aritmticas das linhas seja a
e a das colunas seja b. Assim, obtemos 39 + 2a + 2b = 87 e 39 2a + 3b = 56.
302

OBMEP 2010

Solues do Nvel 3
392a
392a + b
392a + 2b
56

39a

39

39+a

39+2a
39+2a + b
87

Somando essas duas equaes, resulta 78 + 5b = 143, donde b = 13. Subtraindo as duas
igualdades que anteriormente foram somadas, obtemos 4a b = 31. Segue que a = 11.
Assim, o nmero que estava na posio era 39 + a + 4b = 39 + 11 + 4 13 = 102.
130. A sequncia abc Sabendo que 30 = 2(10 + a), obtemos que a = 5. Assim, b =
2(30 + a) = 2(30 + 5) = 70 e, portanto, c = 2(b + 30) = 2(70 + 30) = 200.
131. Permetro e diagonal A opo correta (b).
Denotando por a e b os comprimentos dos lados do retngulo, temos 2a + 2b = 20, de
modo que a + b = 10. O quadrado do comprimento da diagonal, dado pelo Teorema
de Pitgoras, d2 = a2 + b2 . Mas,
(a + b)2 + (a b)2 = (a2 + 2ab + b2 ) + (a2 2ab + b2 ) = 2a2 + 2b2 = 2d2
1
e (a + b)2 = 100, portanto d2 = 50 2 (a + b)2 . Assim, vemos que o mnimo do

comprimento da diagonal ocorre quando a = b, caso em que d = 50.

132. As idades numa classe Denotemos por a a idade comum dos alunos e por n o
nmero de alunos dessa classe. Temos sete alunos com a 1 anos, dois com a + 2 anos
e os demais, ou seja, n 9 alunos, com a anos. Logo, a soma das idades de todos os
alunos, que 330, pode ser desdobrada em 330 = 7(a1)+2(a+2)+(n9) a = na3,
de modo que na = 330 + 3 = 333 = 9 37.
Como a classe tem mais do que 9 alunos, ento a = 9 e n = 37, ou seja, a classe tem
37 alunos.

133. A mesa redonda O permetro de mesa ampliada


140 + 40 6 140 3,14 + 240 = 679,60 cm.
Se cada convidado precisa de 60 cm de espao, podero sentar-se mesa, no mximo,
679,60
11,3,
60
ou seja, 11 convidados.
134. Brincadeira com sete nmeros
Soluo 1: Os sete nmeros podem ser escritos como
n 3, n 2, n 1, n, n + 1, n + 2, n + 3 .
3n6

3n+6

Observando que 3n 6 + 12 = 3n + 6, estabelecemos que n = 12. Logo, os nmeros


so 9 + 10 + 11 + 12 = 13 + 14 + 15.
OBMEP 2010

303

Solues do Nvel 3
Soluo 2: Sejam n + 1, n + 2, . . . , n + 7 os sete nmeros consecutivos tais que
(n + 1) + + (n + k) = (n + k + 1) + + (n + 7)
para algum k entre 1 e 6. Como todos os nmeros esquerda da igualdade so menores
do que os nmeros direita, existem mais parcelas esquerda, portanto, k = 6, 5 ou
4. Tomando k = 6, obtemos 6n + 1 + 2 + 3 + 4 + 5 + 6 = n + 7, ou seja, 5n + 14 = 0,
que no tem soluo inteira. Tambm k = 5 d 5n + 1 + 2 + 3 + 4 + 5 = n + 6 + n + 7,
ou seja, 3n + 2 = 0, que no tem soluo inteira. Finalmente, com k = 4 obtemos
4n + 1 + 2 + 3 + 4 = 3n + 5 + 6 + 7, portanto, n = 8 e 9 + 10 + 11 + 12 = 13 + 14 + 15
a nica soluo.
135. Um terreno compartilhado Como as reas dos tringulos ABM e ADN so iguais, temos
1
(BM
2

AB) = 1 (N D AD).
2

Como o terreno quadrado, temos AB = AD, de

B ............................................................................................................................. A

M
C

.
.
.
.... . .
.
.... .. .
.
.
.
.... . .
.
.... .. .
.
.
.
.... ... .
.... ... .
.
.
.
....
.
.
....
.
.
.
.
..
....
.
.
..
....
.
.
.
..
.
.
.
.
..
....
..
.
.
....
.
.
.
.
..
.
.
.
.
..
....
.
.
....
..
.
.
.
..
.
.
.
.
..
....
..
....
.
.
.
.
..
.
.
.
.
.
.
..
....
....
.
..
.
..
.
.
. .......
.
..
. .......
.
..
.
.
.
.
...
.
..
..
.....
.
.
.
.
.
.
.
.
.
..
.
.
..
.
.
.
.
.
.
..
..
.
.
.
.
.
.
.
.
.
..
.
.
..
.
.
.
.
..
.
.
..
.
.
.
.
.
.
..
.
.
..
.
.
.
.
.
................. .................................
.
...................................................
.
..
.

modo que BM = DN e, portanto, a gura AM CN simtrica em relao diagonal


AC. Logo, a rea do tringulo ACN a metade da rea do tringulo ADN.
Agora, como esses tringulos tm a mesma altura, resulta DN = 2N C e, por simetria,
BM = 2M C. Conclumos que a distncia ao vrtice C do quadrado dos pontos M e
N deve ser 1/3 do comprimento do lado do quadrado.
136. As duas partculas Denotemos as partculas por A e B e seja v a velocidade da
partcula B. Supondo que A seja a mais rpida, temos que v + 2 a velocidade de A.
Assim, o tempo que B demora para dar uma volta 120/v e o tempo que A demora
120/(v+2). Como esse tempo trs segundos inferior ao de B, temos a equao bsica
120
120
3=
.
v
v+2
Simplicando, isso equivale a v 2 + 2v 80 = 0, cuja raiz positiva
v=

1
2 + 4 + 320 = 1 + 81 = 8.
2

Portanto, a velocidade da partcula mais lenta 8 m/s e a da mais rpida 10 m/s.


137. Queda livre No primeiro segundo, o corpo percorre 4,5 m e, como a distncia
percorrida aumenta 9,8 m a cada segundo em relao ao segundo anterior, o corpo
percorre 4,5 + 9,8 m no segundo segundo, 4,5+29,8 m no terceiro segundo, 4,5+39,8
m no quarto segundo e assim por diante, at o dcimo primeiro segundo, em que o
corpo percorre 4,5 + 10 9,8 = 102,5 m. A distncia total percorrida pelo corpo at
o impacto
4,5 + (4,5 + 9,8) + (4,5 + 2 9,8) + + (4,5 + 10 9,8)
= 4,5 11 + 9,8(1 + 2 + + 10) = 49,5 + 9,8 55 = 588,5 m.
304

OBMEP 2010

Solues do Nvel 3
138. Um caminho triangular Se v representa a velocidade constante com que Janete
caminha, ento v = (1 992 m)/(24 min) = 83 m/min. Janete percorre o outro lado
BC e a hipotenusa CA com a mesma velocidade de v = 83 m/min e gasta 2 horas e
46 minutos, o que igual a 166 min. Assim, BC + AC = 83 166 = 13 778 m.
Mas, pelo Teorema de Pitgoras, a hipotenusa CA do tringulo satisfaz
(CA)2 = (1 922)2 + (BC)2 . Da decorre que
(1 992)2 = (CA)2 (BC)2 = (CA + BC)(CA BC)
= 13 778 (CA BC),

portanto CA BC = 1 9922 /13 778 = 288 m. Assim, CA + BC = 13 778 e


CA BC = 288. Subtraindo, obtemos 2BC = 13 778 288 = 13 490 e, portanto,
BC = 1 13 490 = 6 745 m.
2
139. O preo do feijo A opo correta (a).
Se b denota o preo nal e a o preo inicial de um bem, ento a variao b a e o
aumento percentual
ba
.
a
Observe que os valores intermedirios do bem no alteram a variao do aumento
percentual num certo perodo. Usando apenas os dados de janeiro e de abril da tabela
dada, obtemos os aumentos percentuais do
103,33 65,67
37,66
feijo A:
=
= 0,57 = 57%;
65,67
65,67
36,20
109,50 73,30
=
= 0,49 = 49%;
feijo B:
73,30
73,30
100,00 64,50
35,50
feijo C:
=
= 0,55 = 55%.
64,50
64,50
Portanto, o maior aumento percentual de preo foi o do feijo A e o menor foi o do
feijo B.
140. Interseo de tringulos Quando acrescentamos um novo tringulo a uma gura
constituda de tringulos, ele corta cada um dos lados dos tringulos que j existiam
em, no mximo, dois pontos. Inicialmente, comeando com um s tringulo, no
temos ponto de interseo algum. Acrescentando um segundo tringulo, introduzimos,
no mximo, 23 = 6 pontos de interseo. Do mesmo modo, introduzindo um terceiro
tringulo, introduzimos, no mximo, mais 2 6 = 12 pontos de interseo. Logo, trs
tringulos se intersectam em, no mximo, 6 + 12 = 18 pontos. A gura mostra que
esse caso de 18 pontos de interseo pode acontecer.
.
.
.
.....
.
.....
.
.... .
.
.... .
....
.
....
.
. ........
....
....
..........
.
.
...
.
.... ........... ..
.....
.
....
.
..
................. . ...
.
...
. . .....
.
. .........
............
.. .... .
. .
.. .... ... ...........
. .
. ...
...........................
. .
.................. .......
. ...
. .. ...
.
.
.
......
.
. .....
...........
..
..
........
..
. ..
..
.....
. ..
..
..........
..
. .
..
.... .......
. ..
....
....
..
..
..
..
....
.
....
.
.
....
....
.
...
.
.. .......
....
. .
.. .
... .......
.
....
.
.. .
....
.
.. .
....
.. . ...
.. ......
.... ... .
.
.... ... .
.
.. .....
.. ......
.
...... .
.
....
...... .
.
..... ..
.
.. .
.
..
.
. .
.
........ .
...
.
....
.
.. ........
....
.
.
.
.
....
.
.... .
....
.
..
....
..
.
.... ..
.
.... ...
.
....
.
. ..
.
..
. ..
..
..
.
. ...
.
.
. ......
.
. .......
..
........
..
.
.
.. ......
....
.
.
.
....
.
.
..
..............
.
..
..........................
.
....
.
...
..
..
..............................................
.... ... ..............................
.
..
........
......................
..................
.... .
.
.... .
...................
............
....
.
..
.....
..
.
.....
..
.
.
.
....
.. .
.
..
.
..
....
.
.
......
.
..
. ....
..
.
. ...
.
. ...
.
.
.
.... ...
..
.
.
.... .
.
.
.
.... .
.
.
.....
.
.
.
.
.
.
.
.
. ....
.
.
.........
.
.
.
.
....
.
.. .......
.
.
. ...
.
....
....
. ..
.
.
. ..
.
. .
....
....
.
. .
.
. ..
.
....
.... .
..
.
..
.... .
.
.... .
.
.
....
...
.

s s s

s
s

s s
s

OBMEP 2010

305

Solues do Nvel 3
141. Comparar tringulos De acordo com os dados
do problema, temos
AC
BC
2
AB
=
=
= .
AC
AD
CD
3

B
r

C
r

...............
...................
...........
..
...........
..
..........
.......
..
..
... .
... ..
..
..
..... .......
..... .......
..
..
.
.....
....
.....
....
..
.....
..
.....
....
....
.
..
..
..
....
.....
....
..
.....
.
.
....
....
.....
....
..
..
.....
.....
....
....
..
.....
..
.....
.
....
..
....
..
..
.....
..
....
.....
....
.
.
..
....
.. .........
....
.. .........
.
....
....
.......
.........
.
..
....
.......................................................................................................
.......................................................................................................
...
.
..
..

12

18

27

12

Segue que os tringulos ABC e ACD tm seus lados proporcionais e, portanto,


so semelhantes. Em particular, obtemos B AC = C AD.
142. Queima de velas A opo correta (c).
Como o verdadeiro comprimento das velas irrelevante, podemos estipular qualquer
tamanho para as velas, que a resposta ser, sempre, a mesma. O mais simples supor
que ambas velas tm comprimento igual a uma unidade. Assim, a que queima em 3
horas queima velocidade constante 1/3 (de vela por hora) e a que queima em 4 horas
queima velocidade 1/4 (de vela por hora). Logo, depois de algum tempo (em horas)
t, uma queima t/3 (de vela) e a outra t/4 (de vela), de modo que o que sobra de uma
vela depois de um tempo t 1 t/3 e da outra, 1 t/4. Queremos saber quanto tempo
decorre desde o instante t = 0 at o momento em que o comprimento da vela que
queima mais lentamente o dobro do comprimento da que queima mais rapidamente,
o que equivale a resolver a equao
1

t
2t
t
=2 1
=2 .
4
3
3

2
1
5
12
2
, a nica soluo t =
= 2 horas, o que equivale a 2 horas
Como =
3
4
12
5
5
e 24 minutos. Assim, depois de 2 horas e 24 minutos, o comprimento de uma vela
o dobro do comprimento da outra. Como queremos que isso acontea s 16 horas, as
velas devem ser acesas s 13 horas e 36 minutos.
143. Uma distrao A opo correta (b).
Soluo 1: Seja x o nmero com que Jlia se distraiu. Ela deveria ter obtido 6x mas,
com sua distrao, obteve x/6. Logo, seu erro foi de 6 x x/6 = (35/6)x e, portanto,
(35/6)x
35
em termos percentuais, seu erro foi de
=
0,9722 = 97,22%.
6x
36
Soluo 2: Se N o valor que Jlia deveria ter obtido ento, com seu erro, ela
encontrou N/36, de modo que o erro absoluto cometido foi de N N/36 = (35/36)N
35
e, portanto, o erro relativo foi de
100% = 97,22%.
36
144. Problema de nota Seja c o nmero de problemas resolvidos corretamente e seja e a
soma do nmero de problemas resolvidos incorretamente e de problemas no resolvidos.
Logo, c+e = 80 e 5c3e o nmero de pontos do aluno na avaliao. No caso presente,
c + e = 80
5c 3e = 8
Resolvendo o sistema, encontramos c = 31 e e = 49. Logo, o aluno resolveu 31
problemas corretamente.
306

OBMEP 2010

Solues do Nvel 3
145. Quadrados e tringulos
(a) Os nicos quadrados que no tm nenhum de seus lados paralelos nem reta r,
nem reta s, so os do tipo 1 e os do tipo 2 (ver guras).

Assim, h um total de seis quadrados, sendo quatro do tipo 1 e dois do tipo 2.


(b) Digamos que a distncia vertical ou horizontal entre dois pontos contguos do
reticulado seja igual a uma unidade. O total desses tringulos dezesseis, cada

um deles com catetos iguais a 5 unidades e hipotenusa de 10 unidades. De fato,


cada um dos quadrados do tipo 2, como visto em (a), nos d quatro tringulos,
por diviso ao longo de cada uma das duas diagonais obtendo, assim, oito tringulos. Os oito tringulos restantes so obtidos atravs de uma nica translao
horizontal ou vertical de cada um dos anteriores. Na gura, exemplicamos a
nica translao possvel de um dos quatro tringulos obtidos de um quadrado
do tipo 2.

146. Clculo de reas


(a) A rea hachurada corresponde a um quarto da rea de um crculo de raio r,
1
portanto, a rea hachurada igual a r2 .
4
(b) Observe que a rea da regio marcada com X, que no est hachurada na gura
(a), igual rea do quadrado todo, diminuda da rea da regio hachurada, ou
seja,
1
1
rea da regio marcada com X = r2 r2 = (4 )r2 .
4
4
X

OBMEP 2010

307

Solues do Nvel 3
Voltando ao item (b), a rea da regio hachurada na gura (b) igual rea do
quadrado todo, menos duas vezes a rea da regio marcada com X, ou seja,
igual a
1
1
rea da regio hachurada = r2 2 (4 )r2 = r2 r2 .
4
2
147. Sequncia de algarismos Os nmeros com um algarismo formam os 9 primeiros
termos da sequncia. Os 90 nmeros de dois algarismos formam os 180 termos seguintes.
Depois vm os 2 700 termos correspondentes aos nmeros de trs algarismos, seguidos
pelos 36 000 termos correspondentes aos nmeros de quatro algarismos e nalmente,
os 450 000 termos que so os correspondentes aos nmeros de cinco algarismos. Logo,
enumerando os termos da sequncia, obtemos 488 889 termos.
a1 , . . . , a9 , a10 , . . . , a189 , a190 , . . . , a2 889 , a2 890 , . . . , a38 889 , a38 890 , . . . , a488 889
1 alg

2 algs

3 algs

4 algs

5 algs

Para escrever todos os termos de 1, 2, 3 e 4 algarismos, chegamos 38 889a posio


da sequncia. Logo, o algarismo na 206 788a posio faz parte de um nmero de cinco
algarismos, ou seja, est no bloco
a38 890 , . . . , a488 889 .
5 algs

Esse bloco da forma 10 000, 10 001, . . . 99 999. Para ver quantos nmeros de cinco
algarismos existem desde a posio 38 889 at a posio 206 788, dividimos essa diferena por 5. Assim, 206 788 38 889 = 167 899 e 167 899 = 5 33 579 + 4. Portanto,
precisamos de 33 579 nmeros de cinco algarismos, mais os quatro primeiros algarismos do 33 580o nmero de cinco algarismos, que 43 579, para chegar ao algarismo
na posio 206 788.
Como o quarto algarismo do nmero 43 579 7, temos que o algarismo procurado 7.
148. Soma constante Para resolver este problema, o mais fcil comear dispondo os
nmeros 1, 2, 3, 4, 5, 6, 7, 8 e 9 numa tabela 3 3 de modo que a soma de cada linha,
de cada coluna e de cada diagonal seja 15. Depois basta somar 662 a cada elemento
da tabela, obtendo, por exemplo, a soluo seguinte.
670
663
668

665
667
669

666
671
664

Assim como existem outras maneiras de dispor os nmeros 1, 2, 3, 4, 5, 6, 7, 8 e 9 na


tabela, tambm existem outras solues desse problema.
149. Contando os zeros Inicialmente, vericamos como terminam as potncias de 9,
ou seja, listamos os dois ltimos algarismos, os da dezena e da unidade, das potncias
9n , ordenadamente.
9n

308

Se n for
termina com

0
01

1
09

2
81

3
29

4
61

5
49

6
41

OBMEP 2010

7
69

8
21

9
89

10
01

11
09

12
81

Solues do Nvel 3
Assim, vemos que os dois ltimos algarismos de 910 , 911 e 912 so os mesmos de 90 , 91 e 92 .
A partir 910 , os dois ltimos algarismos das potncias comeam a se repetir, formando uma
sequncia peridica de perodo 10. Como 2 007 = 10 200 + 7 e os dois ltimos algarismos de
910200 so 01, segue que os dois ltimos algarismos de 92 007 so os dois ltimos algarismos
de 97 , ou seja, 69. Ento, os dois ltimos algarismos de 92 007 + 1 so iguais a 69 + 1 = 70.
Assim, existe um nico zero no nal do nmero 92 007 + 1.
150. Crculos dentro do quadrado A resposta desse problema armativa: possvel colocar
um certo nmero de crculos sem superposio dentro de um quadrado de 1 centmetro de
lado, de tal forma que a soma dos raios desses crculos seja maior do que 2 008 centmetros.
Para exibir uma tal congurao, desenhamos linhas paralelas aos lados do quadrado, dividindoo em n2 quadradinhos menores; cada um desses quadradinhos tem lado igual a 1/n. Dentro
de cada um desses quadradinhos, desenhamos um crculo inscrito de raio igual a 1/2n. No
caso particular n = 4, essa construo dada na gura.
2
n = 42 = 16 crculos

lados dos quadradinhos = 1

4
1
raio dos crculos =

soma dos raios = 16 1 = 4 = 2


8
2

Desse modo, a soma dos raios dos n2 crculos igual a n2 1/2n = n/2. Como estamos
interessados no caso desta soma ser maior do que 2 008, devemos ter n/2 > 2 008, ou seja,
n > 4 016. Logo, dividindo o quadrado em 4 0172 quadradinhos (ou mais), a soma dos raios
dos crculos inscritos nos quadradinhos ser maior do que 2 008.
151. Construindo um nmero As condies dadas implicam que os nmeros devem satisfazer
todas as condies seguintes.
(i)
(ii)

... 2

(iii)

... 3

(iv)

1 ...

... 1

... 4

2 ...
3 ...
4 ...

Vamos estudar as possveis posies dos dois algarismos 4 em um nmero de oito algarismos.
De acordo com (iv), existem apenas trs possibilidades:
4
caso A:
4
caso B:
4
4
caso C:
4
4
Em cada um desses casos, existem duas possibilidades de colocar os algarismos 3:
caso A:

caso B:

3 4

caso C:

4
3

3
4

ou

ou
4

ou

3 4

4 3
3

4 3
3

Na tentativa de colocar os algarismos 1 e 2, percebemos que as duas possibilidade do caso B


so impossveis, bem como as primeiras possibilidades dos casos A e C. Os nicos casos que
levam a solues do problema so as segundas possibilidades dos casos A e C, que levam s
duas nicas solues
41 312 432 e 23 421 314.

OBMEP 2010

309

Solues do Nvel 3
152. Nmero na circunferncia Na gura a seguir representamos os nove algarismos escritos
ao redor da circunferncia.

a1
a9

a2

a8

a3

a4

a7
a5

a6

Lendo os algarismos escritos ao redor da circunferncia, de trs em trs, no sentido horrio,


obtemos os seguintes nmeros de trs algarismos cada:
a1 a2 a3 , a2 a3 a4 , a3 a4 a5 , a4 a5 a6 , a5 a6 a7 , a6 a7 a8 , a7 a8 a9 , a8 a9 a1 e a9 a1 a2 .
Para somar esses nmeros usamos o algoritmo da adio, como indicado a seguir.

a1 a2 a3
a2 a3 a4
a3 a4 a5
a4 a5 a6
a5 a6 a7
a6 a7 a8
a7 a8 a9
a8 a9 a1
a9 a1 a2
???????

Analisando esses nove nmeros, notamos que todos tm os algarismos da unidade diferentes.
Logo,
a3 + a4 + a5 + a6 + a7 + a8 + a9 + a1 + a2 = 1 + 2 + 3 + 4 + 5 + 6 + 7 + 8 + 9 = 45 .
Do mesmo modo, esses nove nmeros tambm tm todos os algarismos das dezenas e todos
os algarismos das centenas diferentes. Logo, a soma dos algarismos das dezenas tambm
45 e o mesmo ocorre com os algarismos das centenas. Portanto, para somar basta calcular
45 + 45 10 + 45 100 = 4 995. Assim, a soma dos nove nmeros 4 995.
153. Cada pea em seu lugar A primeira informao certamente falsa, pois se fosse verdadeira, o ouro estaria no cofre 2 ou 3, mas deveria estar no prprio cofre 1, para que a
primeira informao fosse verdadeira. Essa contradio mostra que o ouro no est nem no
cofre 2 nem no cofre 3. Por ser falsa a informao na porta do cofre 1, conclumos que o outro
tambm no est nele. A segunda informao certamente falsa, pois se fosse verdadeira,
o ouro estaria no cofre 2, o que incorreto. Logo, a primeira e a segunda informaes so
falsas. Portanto, o ouro no est no cofre 1, nem no 2 nem no 3, e a prata no est no cofre
1.

310

OBMEP 2010

Solues do Nvel 3
As nicas possibilidades que restam para o ouro so os cofres 4 ou 5. Se o ouro estivesse no
cofre 4,
, , , ouro , ,
2

a informao 4 seria a correta e o nquel estaria na cofre 3. Ento a terceira informao deve
ser falsa e deveramos ter o bronze tambm no cofre 3, o que uma impossibilidade. Logo,
essa possibilidade ca descartada e o ouro deve estar no cofre 5.
, , , , ouro .
2

De fato, com o ouro no cofre 5, a informao 5 a correta e a platina est no cofre cujo
nmero superior, em uma unidade, ao que contm o bronze. Pela armao do cofre 3,
que falsa, teramos o bronze no cofre 3. Logo, a platina est no cofre 4. Como a segunda
armao falsa, a prata no est no cofre 1, s podendo estar no cofre 2. Portanto, temos
a soluo seguinte.
nquel , prata , bronze , platina , ouro .
1

154. Soma de quadrados Como a razo da progresso aritmtica 2, os trs nmeros podem
ser denotados por n 2, n e n + 2. A soma de seus quadrados um nmero de quatro
algarismos iguais, digamos, k k k k, em que k algum inteiro entre 1 e 9, ou seja,
k k k k = (n 2)2 + n2 + (n + 2)2 = 3n2 + 8 .
A partir deste ponto, apresentamos duas possibilidades de soluo.
Soluo 1: Como k k k k 8 = 3n2 um mltiplo de 3, o resto da diviso de k k k k por 3
igual ao resto da diviso de 8 por 3, que 2. Mas k k k k = k 1 111 e o resto da diviso
de 1 111 por 3 1, de modo que o resto da diviso de k por 3 2. Como 1 k 9, s
podemos ter k igual a 2, 5 ou 8. Como na Soluo 1, os casos k = 2 e k = 8 so impossveis e
k = 5 a nica opo, caso em que n = 43 e os trs nmeros procurados so 41, 43 e 45, que
constituem a nica soluo para o problema.
Soluo 2: Como k k k k = 3n2 + 8, obtemos
3n2 = k k k k 8 = (k k k 10 + k) (9 1) = (k k k 10 9) + (k + 1).
Mas kkk 10 9 = k k (k 1) 1 mltiplo de 3, portanto tambm k + 1 um mltiplo de
3. Como 1 k 9, s podemos ter k igual a 2, 5 ou 8. No caso k = 2, obtemos
n2 =

2 222 8
= 738 = 2 369,
3

o que impossvel, pois 738 no um quadrado perfeito. Analogamente, se k = 8, obtemos


n2 =

8 888 8
= 2 960 = 24 5 37,
3

o que, novamente, impossvel, j que esse ltimo nmero no um quadrado perfeito. Resta,
portanto, a ltima opo, k = 5. Nesse caso,
n2 =

5 555 8
= 1 849 = 432 ,
3

portanto, n = 43 e os trs nmeros em progresso aritmtica procurados so 41, 43 e 45, que


constituem a nica soluo para o problema.

OBMEP 2010

311

Solues do Nvel 3
155. Adivinhe o nmero
Soluo 1: Seja x o nmero procurado. Observe que x + 2 divisvel por 3, 4, 5 e 6. O
menor mltiplo comum desses nmeros 60. Logo, x + 2 = 60 e, ento, x = 58.
Soluo 2: Seja x o nmero procurado. O resto da diviso de x por 3 1. Portanto, x
da forma x = 3a + 1, com a 0 inteiro. A seguir, queremos determinar de que forma deve
ser a para que x = 3a + 1 deixe resto 2 na diviso por 4, isto , para que 3a deixe resto 1
na diviso por 4. Qual deve ser o resto da diviso de a por 4? Por um lado, se esse resto for
3, ento a da forma a = 4b + 3, de onde segue que 3a = 12b + 9 = 4 (3b + 2) + 1 deixa,
de fato, resto 1 na diviso por 4. Por outro lado, podemos vericar que qualquer outro resto
no funcionaria. Se, por exemplo, a deixasse resto 2 na diviso por 4, teramos a = 4b + 2 e
3a = 12b + 6 = 4 (3b + 1) + 2 deixaria resto 2 e no 1 na diviso por 4.

Substituindo a = 4b+3 em x = 3a+1, obtemos x = 12b+10. Usamos agora que x deixa resto
3 na diviso por 5. Como 10 mltiplo de 5, 12b tambm deixa resto 3 na diviso por 5. Mas
12b = 10b + 2b e 10b mltiplo de 5. Logo, 2b deixa resto 3 na diviso por 5. Ento, b deixa
resto 4 na diviso por 5. De fato, por um lado, se b = 5c+4, ento 2b = 10c+8 = 5(2c+1)+3
deixa realmente resto 3 na diviso por 5. Por outro lado, como acima, podemos vericar que
4 o nico resto que funciona. Ento, b = 5c + 4 e x = 12b + 10 = 12 (5c + 4) + 10 = 60c + 58.
Conclumos que as solues do problema so os nmeros x da forma x = 60n + 58, com n 0
inteiro. O menor deles, para n = 0, x = 58.
156. Um cdigo Observe que
AOBM EP = AOB 1 000 + M EP e M EP AOB = M EP 1 000 + AOB.
Denotemos AOB = m e M EP = n. Pelos dados do problema, temos
6 AOBM EP = 7 M EP AOB,
donde 6 (1 000 m + n) = 7 (1 000 n + m), ou 6 000 m 7 m = 7 000 n 6 n ou, ainda,
5 993 m = 6 994 n. Dividindo ambos os lados por 13, conclumos que 461 m = 538 n. A
fatorao de 538 em fatores primos 538 = 2 269 e 461 primo. Portanto, 538 e 461 so
primos entre si. Logo, 461 divide n e 538 divide m. Como AOB e M EP so nmeros de trs
algarismos, s podemos ter as solues n = 461, ou n = 822, e m = 538. fcil vericar que
6 538 461 = 3 230 766 = 7 461 538 e que 6 538 822 = 3 232 932 = 5 757 766 = 7 822 538.
Portanto, n = 822 no serve, sendo AOB = 538 e M EP = 461 a nica soluo. Assim, os
algarismos so A = 5, O = 3, B = 8, M = 4, E = 6 e P = 1.
157. Calculando distncias
Soluo 1: Observe que conhecido o ngulo ABD. De fato, o tringulo ABC equiltero,
portanto, ABC = 60 e, como C BD = 90 , obtemos ABD = 150 . Assim, aplicando a Lei
dos Cossenos ao tringulo ABD, resulta

AD2 = 32 + 42 2 3 4 cos 150 = 25 + 24 cos 30 = 25 + 12 3 .

Segue que AD = 25 + 12 3 cm.


Soluo 2: Seja E o ponto sobre a reta BD tal que o tringulo AEB seja retngulo no
vrtice E (veja gura). Nesse tringulo, temos

3
EB
EB
1
AE
AE
= cos 30 =
=
e
= sen 30 =
=
,
2
AB
3
2
AB
3

312

OBMEP 2010

Solues do Nvel 3
portanto,

3 3
EB =
2

3
e AE = ;
2

3 3
em particular, ED = 4 +
.
2

C
3
A

3
E

Aplicando o Teorema de Pitgoras no tringulo AED, obtemos


3
AD = AE + ED =
2
2

Segue que AD =

8+3 3
+
2

= 25 + 12 3.

25 + 12 3 cm.

158. Calculando lados de um tringulo Como o tringulo ABC equiltero, seus ngulos
so todos iguais a 60 . Sobre o lado CB desse tringulo, construmos um novo tringulo
CBP , congruente ao tringulo ABP, tal que P AB = P CB e ABP = C BP (girando o
tringulo ABP no sentido horrio por 60 em torno do ponto B).

C
3 cm

60

P
5 cm
4 cm

3 cm
60

4 cm
60

Note que o ngulo P BP congruente ao ngulo ABC, ou seja, mede 60 . Assim, se traarmos
o segmento P P , temos que o tringulo P BP , que j issceles, pois
P B = BP = 4 cm , de fato, equiltero e, em consequncia, temos que P P = 4 cm.
OBMEP 2010

313

Solues do Nvel 3
C
3 cm

60

5 cm
4 cm

4 cm

3 cm

4 cm

60

60

Denotando o ngulo P P C por a, aplicamos a Lei dos Cossenos ao tringulo CP P e obtemos


52 = 32 + 42 2 3 4 cos a,
ou seja, 25 = 25 24 cos a. Segue que cos a = 0 e, portanto, a = 90 . Dessa forma, estabelecemos C P B = a + 60 = 90 + 60 = 150 .
C
3 cm

60

P
150

5 cm

3 cm

4 cm

4 cm

60

60

Agora, denotando o lado do tringulo ABC por l, aplicamos a Lei dos Cossenos ao tringulo
CBP e obtemos

l2 = 32 + 42 2 3 4 cos 150 = 25 + 12 3.

Segue que 25 + 12 3 cm o comprimento dos lados do tringulo equiltero ABC.


159. Amigo oculto Primeiramente observemos que o nmero de formas de distribuir os presentes
sem nenhuma restrio 5! = 5 4 3 2 1 = 120. Da temos que tirar os casos ruins,
isto , os casos em pelo menos um amigo tirou o seu prprio presente. Esses casos a eliminar
podem ser listados pelo nmero de amigos que tiram seu prprio presente.
Os 5 amigos caram com seus prprios presentes. S h uma possibilidade de acontecer
isso.
Exatamente 4 amigos caram com seus prprios presentes. Isso no possvel.

Exatamente 3 amigos caram com seus prprios presentes. Nessa situao, os outros
dois amigos trocam os presentes. Assim, escolhemos 3 pessoas dentre as 5, isto ,
543
= 10 possibilidades.
32

314

OBMEP 2010

Solues do Nvel 3
Exatamente 2 amigos caram com seus prprios presentes. Nesse caso, escolhemos 2
54
pessoas dentre as 5, isto ,
= 10 possibilidades. Os outros 3 amigos trocam os
2
presentes entre si, dando um total de 10 2 = 20 possibilidades.

Por ltimo, exatamente um amigo cou com seu prprio presente. Nesse caso, escolhemos uma pessoa dentre um total de 5, multiplicando pelo nmero de formas que os
outros amigos no quem com seu presente, que so 9 maneiras. Logo, nessa situao,
temos um total de 5 9 = 45 possibilidades.
Portanto, temos 120 45 20 10 1 = 44 maneiras de distribuir os presentes sem que
algum que com seu prprio presente.
160. Contando solues A equao dada equivalente a xy = 144(x + y) = 144x + 144y,
144y
portanto, isolando x, obtemos x =
. Como x e y devem ser inteiros positivos, o
y 144
denominador y 144 deve ser um nmero inteiro positivo, digamos, y 144 = n. Substituindo
essa expresso no valor de x, obtemos
x=

1442
144(n + 144)
= 144 +
.
n
n

Como x deve ser um nmero inteiro, n deve ser um divisor de 1442 .


Sendo
1442 = 124 = 28 34 , seus divisores so os nmeros d da forma d = 2a 3b , com 0 a 8 e
0 b 4. Como h 9 valores possveis para a e 5 valores possveis para b, conclumos que
1442 tem 9 5 = 45 divisores.
Assim, para cada divisor n de 1442 , obtemos uma soluo
(x, y) = 144 +

1442
, n + 144
n

xy
= 144 dada. Portanto, essa equao possui 45 pares de nmeros inteiros
x+y
positivos (x, y) que a satisfazem.

da equao

161. Determinando uma sequncia Sejam a1 , a2 , . . . , a80 os nmeros dessa sequncia. Para
cada i 1, temos
ai+1 = ai ai+2 ,
ai+2 = ai+1 ai+3 .

Consequentemente, ai+1 = ai ai+1 ai+3 e, como ai+1 = 0, j que o produto dos termos da
sequncia 8 = 0, segue ai ai+3 = 1.

Quaisquer dois nmeros da sequncia, cujos ndices distem 3 um do outro, so tais que o
seu produto igual a 1. Portanto, o produto de seis nmeros consecutivos nessa sequncia ,
sempre, igual a 1. Sendo o produto dos 40 primeiros termos da sequncia igual a 8, conclumos
que o produto dos quatro primeiros termos tambm 8, pois os 36 termos restantes formam
seis grupos de 6 termos consecutivos da sequncia e, em cada grupo desses, o produto igual
a 1. Isto , a1 a2 a3 a4 = 8. Como ai ai+3 = 1, segue que a1 a4 = 1 e, da, a2 a3 = 8.

Temos, tambm, a hiptese de que os 80 termos da sequncia tm produto igual a 8, donde


podemos concluir que a1 a2 = 8, j que os 78 ltimos termos podem ser agrupados em 13
grupos de 6 termos consecutivos, cada um com produto igual a 1, como j vimos.
Ento, de a2 a3 = 8, a1 a2 = 8 e a2 = a1 a3 , segue que a1 a2 a3 = 64 e a3 = 64. Assim,
2
2
a1 = 2, a2 = 4 e a3 = 2.
OBMEP 2010

315

Solues do Nvel 3
Observe, ainda, que a sequncia inteira est, agora, determinada. De fato, temos
1 1 1
1 1 1
2, 4, 2, , , , 2, 4, 2, , , , . . . ,
2 4 2
2 4 2
em que os seis primeiros termos cam se repetindo, sempre na mesma ordem.
162. Construindo uma cerca A soma dos comprimentos dos trs lados (os que no so de
pedra) 140 m.
(a) Se os dois lados vizinhos ao muro de pedra tm 40 m cada um, os dois juntos tm 80 m
e o terceiro lado ter 140 80 = 60 m.

(b) Se o maior dos lados a ser cercado tiver 85 m, ele no pode estar ser vizinho ao muro
de pedras, porque nesse caso esses dois lados mediriam 85 2 = 170 m, que maior do
que 140 m. Logo, ele deveria ser paralelo ao muro de pedra e, nesse caso, cada um dos
outros lados mediria 27,5 m, o que tambm no possvel, j que a cerca composta
de pedaos inteiros de 1 m cada um.
Os dois lados que encostam no muro de pedra podem ter 65 m cada um porque nesse
caso, o outro teria 140 2 65 = 10 m, o que no contraria as condies dadas.
163. Um quadriltero especial Como os tringulos ABC e ACD so retngulos e tm a
mesma hipotenusa AC, pelo Teorema de Pitgoras temos x2 + 112 = y 2 + 72 , onde AB = x
e DC = y. Ento,
(y x)(y + x) = y 2 x2 = 72 = 23 32
e, portanto, y x e y + x so divisores de 72. Para cada fatorao de 72, precisamos resolver
um sistema de duas equaes com duas incgnitas, como na tabela a seguir, e identicar os
casos em que existem solues inteiras.
Fator de 72
y+x yx
72
1
36
2
24
3
28
4
12
6
9
8

Medidas de
x
y
17
19
12
16
3
9
-

Observaes
No h soluo inteira
Possui soluo inteira
No h soluo inteira
Possui soluo inteira
Possui soluo inteira
No h soluo inteira

Assim, h trs solues inteiras para o comprimento dos lados x e y.


164. Trs quadrados Os tringulos retngulos AEB e EHF so congruentes, pois seus
ngulos E BA e F EH so iguais (lados respectivos perpendiculares) e as hipotenusas so
iguais (lados de um quadrado). Ento, AE = F H. Pelo Teorema de Pitgoras,
rea de BEF G = BE 2 = AB 2 + AE 2 = AB 2 + F H 2 = 30 + 20 = 50 cm2 .

165. Bolinha de gude


Soluo 1: Denotemos por x, y e z o nmero de bolinhas que cada um tinha no incio da
partida. Temos

316

OBMEP 2010

Solues do Nvel 3
Incio
1a rodada
2a rodada
3a rodada

1o
x
xyz
2(x y z)
4(x y z)

2o
y
2y
2y 2z (x y z)
2(3y x z)

3o
z
2z
4z
4z 2(x y z) (3y x z)

Como cada um terminou a partida com 64 bolinhas, segue que

4(x y z) = 64

2(3y x z) = 64

4z 2(x y z) (3y x z) = 64

donde

x y z = 16
x + 3y z = 32

x y + 7z = 64

Para resolver o sistema, somamos a primeira com a segunda equaes e a primeira com a
terceira, obtendo
y z = 24;
y + 3z = 40.

Da, obtemos z = 32 e y = 56, portanto, x = 16 + 56 + 32 = 104. Assim, o primeiro jogador


comeou a partida com 104 bolinhas, o segundo, com 56 e o terceiro, com 32.

Soluo 2: Preenchemos a tabela de baixo para cima, isto , do nal para o incio do jogo.
Comeamos com 64 nas trs casas nais.
1o
Incio
Aps a 1a rodada
Aps a 2a rodada
Aps a 3a rodada

2o

3o

64

64

64

Como os dois primeiros jogadores dobraram a quantidade de bolinhas na terceira rodada,


cada um tinha 32 bolinhas e o terceiro jogador deu 32 a cada um deles. Deduzimos que ele
possua 64 + 32 + 32 = 128 bolinhas.
1o
Incio
Aps a 1a rodada
Aps a 2a rodada
Aps a 3a rodada

2o

3o

32
64

32
64

128
64

Quem perdeu a segunda rodada foi o segundo jogador. Logo, a tabela era
1o
Incio
Aps a 1a rodada
Aps a 2a rodada
Aps a 3a rodada

2o

3o

16
32
64

32+16+64=112
32
64

64
128
64

Finalmente,
OBMEP 2010

317

Solues do Nvel 3
Incio
Aps a 1a rodada
Aps a 2a rodada
Aps a 3a rodada

1o
16 + 56 + 32 = 104
16
32
64

2o
56
32 + 16 + 64 = 112
32
64

3o
32
64
128
64

Assim, o primeiro jogador comeou a partida com 104 bolinhas, o segundo, com 56 e o terceiro,
com 32.
166. Uma soma Inicialmente, observe que
1
1
=1 ,
12
2

Assim, temos

S =1
e, portanto, S = 1

1
1
1
=
. Logo,
k (k + 1)
k k+1

1
1 1
= ,
23
2 3

...,

1
1
1
=

.
2 007 2 008
2 007 2 008

1
1
1
1
1 1 1 1 1
+ + + ... +

2 2 3 3 4
2 006 2 007 2 007 2 008
2 007
1
=
.
2 008
2 008

167. Dobrando papel Sejam E e F os pontos de interseo, como na gura. Sejam AB = 2a e


BC = 2b. Ento AM = M B = DN = N C = a e M E = EN = b. Traamos AN e denotamos
por P o ponto de interseo dos segmentos AN e BD. Os segmentos AN e M C so paralelos
(pois AM = N C e AM N C). Como M o ponto mdio de AB e M F AP, temos que F
o ponto mdio do segmento P B. Analogamente, P o ponto mdio do segmento DF e segue
que DP = P F = F B. Por simetria, vericamos que P E = EF e, ento, EF/F B = 1/2. Por
outro lado, rea(M BE) = 1 rea(ABD) = 125 e, como M EF e M BE tm a mesma
4
altura relativamente ao vrtice M e a base do primeiro 1/3 da base do segundo, conclumos
que
1
rea(M EF ) = 125 cm2 .
3
168. Uma rea Os tringulos ABM e ABC tm a mesma altura d em relao s respectivas
bases AM e AC. Como M o ponto mdio de AC, obtemos
1
2
1
2

rea (ABM )
=
rea (ABC)
de modo que
rea (ABM ) =

AM d
1
AM
= ,
=
AC
2
AC d

1
1
rea (ABC) = 100 = 50 cm2 .
2
2
A
a

M
P

318

D
OBMEP 2010

Solues do Nvel 3
Teorema das Bissetrizes Internas

Analogamente,

rea (ABP )
BP
=
.
rea (ABM )
BM

a a

Pelo Teorema das Bissetrizes Internas,


AB
10
2
BP
=
=
= ,
PM
AM
15
3

D
AC =AB
8 8
DC BD

3
portanto, P M = BP. Ento obtemos
2

rea (ABP )
BP
BP
BP
BP
2
=
=
=
= 5
= ,
3
rea (ABM )
BM
BP + P M
5
BP + 2 BP
2 BP
de modo que rea (ABP ) =

2
2
rea (ABM ) = 50 = 20 cm2 .
5
5

169. ltimos algarismos


Soluo 1: Como s queremos saber os dois
ltimos algarismos, basta conhecer as duas ltimas colunas dessa soma (a das dezenas e a das
unidades), ou seja,
8 + 88 2 007 = 8 + . . . 16.
Como 8 + 16 = 24, os dois ltimos algarismos do
nmero so 24.

......
.....
.
.

8 ..........
8 8 ..........
8 .8 8 ..........
. . ........
.
. . .... 2 008
. . ..... parcelas
. . ....
. . ....
8 . . . . . . 8 8 .........
8 . . . . . . 8 8 ..........
88 . . . . . . 8 8 ...................
................................................
................................................

8
8
.
.
.
.
.
.
8
8
8

8..............
8 .........
8 ..........
. ....
. ...
. .... 2 007
. ...... parcelas
. ....
. ...
8 ..........
8 .........
8.................

......................
......................

....................................................
....................................................

Soluo 2: Observemos que os dois ltimos algarismos do nmero dado so iguais aos dois
ltimos algarismos do nmero
2 007

8 + 88 + + 88 = 8 + 2 007 88,
que tambm coincidem com os dois ltimos algarismos de 8 + 7 88 = 624. Logo, os dois
ltimos algarismos do nmero procurado so 24.
170. Idades mltiplas Quando Isabel tem a anos, sua me tem 20 + a anos. Se a divisor
de 20 + a, ento (20 + a)/a = (20/a) + 1 um nmero inteiro e, consequentemente, 20/a
tambm inteiro. Ento, a um divisor de 20 e, portanto, a pode ser 1, 2, 4, 5, 10 ou 20.
Assim, temos um total de 6 vezes em que as idades das duas so mltiplos.
Isabel
Me

1
21

2
22

4
24

5
25

10
30

20
40

Observe que, depois dos 20 anos de Isabel, nunca mais a idade da me ser um mltiplo da
idade de Isabel.
171. Blocos diferentes O volume do cubo 10 10 10 = 1 000 cm3 . O volume V de um
bloco o produto de suas trs medidas, altura (= a), largura (= l) e comprimento (= c).
OBMEP 2010

319

Solues do Nvel 3
Para construir cada bloco, Ana deve usar todos os bloquinhos, portanto, o volume de cada
bloco
V = altura largura comprimento = a l c = 1 000 cm3 .
Assim, precisamos saber de quantas maneiras podemos escrever 1 000 como o produto de trs
nmeros inteiros positivos a, l e c. Para isso, fatoramos 1 000, obtendo a l c = 1 000 = 23 53 .

Soluo 1: Podemos encontrar todos esses nmeros listando as dimenses a, l e c dos blocos.
Sem perda de generalidade, podemos supor que a l c. Ento a3 alc 1 000 e, portanto,
a 10. Logo, a = 1, 2, 4, 5, 8 ou 10. Mas se a = 8, ento lc = 125 = 53 e, como 8 l c, no
h como obter medidas inteiras para l e c. Assim, a s pode ser 1, 2, 4, 5 ou 10. A tabela
mostra as 19 possibilidades para esses blocos.
a

l
1
2
4
5
8
10
20
25
5
10

c
1 000
500
250
200
125
100
50
40
50
25

10

l
2
4
5
10
20
5
8
10

c
250
125
100
50
25
40
25
20

10

10

Soluo 2: Podemos encontrar todos esses nmeros listando as potncias de 2 e 5, sem


esquecer que uma das medidas pode ser 1 (no caso de potncia 0). A tabela mostra as 19
possibilidades para esses blocos.
potncia de 2
3
1, 2

1, 1, 1
3

3
1, 2

3
1, 2

1, 1, 1
1, 2

1, 2
1, 1, 1
1, 1, 1

320

potncia de 5
3

1, 1, 1
1, 2
1, 1, 1

a
1
1
1
1
2
2
1
1
23
5
1
1
2
22
22
2
5
5
25

OBMEP 2010

l
1
23
2
22
22
2
35
2
23 52
5
5
25
2 52
5
25
2 52
22 5
25
25
25

c
53
53
22 53
2 53
53
2 53
52
5
52
23 5
22 52
22 5
22 52
52
5
52
22 5
2 52
25
23

Solues do Nvel 3
172. Quadro negro Inicialmente observe que, de 1 a 77, Joana apagou 11 mltiplos de 7 e 7
mltiplos de 11. Como 77 mltiplo de 7 e de 11, ento ela apagou 11 + 7 1 = 17 nmeros,
sobrando 77 17 = 60 nmeros. Agora, agrupando os 10 000 primeiros nmeros em grupos
de 77 nmeros consecutivos, esse raciocnio se aplica em cada uma das linhas abaixo, isto ,
em cada linha sobraram 60 nmeros.
1a linha: 1,
2, . . . , 77
a linha:
2
78, 79, . . . , 154
3a linha: 155, 158, . . . , 231
.
.
.
.
.
.
.
.
.
.
.
.
Como, 2 008 = 33 60 + 28, sabemos que entre os primeiros 33 77 = 2 541 nmeros,
33 60 = 1 980 nmeros caram sem apagar.
.
.
.
.
.
.
.
.
.
.
.
.
a linha: . . . , . . . , . . . , 2 541.
33
Ainda faltam contar 28 nmeros. Vamos, ento, examinar a 34a linha, que comea com 2 542.
Como os nmeros apagados esto nas colunas 7, 11, 14, 21, 22, 28, 33, 35, etc., e at a 35a
coluna foram apagados oito nmeros, restam 35 8 = 27 nmeros na 34a linha. Logo, depois
de apagados os mltiplos de 7 e de 11 nessa linha, o 28o nmero 2 577. Assim, o nmero na
2 008a posio o 2 577.
173. Conjunto sem mltiplos Inicialmente, observemos que o conjunto {51, 52, 53, . . . , 100}
tem 50 elementos e nenhum de seus elementos mltiplo de outro. Assim, o subconjunto
com o maior nmero de elementos e que satisfaz a propriedade exigida tem, no mnimo, 50
elementos. Para concluir que 50 o maior nmero possvel de elementos de um subconjunto
que satisfaa a propriedade exigida, basta mostrar que todo subconjunto com mais de 50
elementos possui dois nmeros mltiplos. Para isso, denotamos B = {1, 2, 3, . . . , 100} e
dividimos B em 50 subconjuntos disjuntos, considerando os diversos subconjuntos de B cujos
elementos so do tipo nmero mpar 2k , com k natural. Como existem apenas 50 nmeros
mpares entre 1 e 100, obtemos cinquenta subconjuntos dois a dois disjuntos construdos dessa
forma, como segue.
A1 = {1, 2, 4, 8, 16, 32, 64} = B {1 2k | para algum k = 0, 1, 2, 3, . . .};

A2 = {3, 6, 12, 24, 48, 96} = B {3 2k | para algum k = 0, 1, 2, 3, . . .};

A3 = {5, 10, 20, 40, 80} = B {5 2k | para algum k = 0, 1, 2, 3, . . .};


.
.
.
A50 = {99} = B {99 2k | para algum k = 0, 1, 2, 3, . . .}.
Observe que B a unio desses cinquenta subconjuntos, isto ,
B = {1, 2, . . . , 100} = A1 A2 . . . A50 ,

e que, se dois elementos de B estiverem num mesmo subconjunto Ai , ento um deles mltiplo
do outro. Assim, se um subconjunto A de B tiver mais do que 50 elementos, podemos armar
que existem pelo menos dois elementos de A num mesmo subconjunto Ai e, portanto, um
deles mltiplo do outro. Isso prova que 50 o nmero mximo de elementos de qualquer
subconjunto de B que no possua dois elementos tais que um deles seja mltiplo do outro.

OBMEP 2010

321

Solues do Nvel 3
174. Brincando com a calculadora O resultado o mesmo nmero inicial a b c de trs algarismos. De fato, se a b c um nmero de trs algarismos, ento o nmero de seis algarismos
a b c a b c da forma a b c a b c = 1 000 a b c + a b c = 1 001 a b c.
Como 1 001 = 7 11 13, dividindo, sucessivamente, a b c a b c por 7, 11 e 13, obtemos
abcabc
1 001 a b c
=
= a b c.
7 11 13
1 001
175. No galinheiro Sejam x e y, respectivamente, o nmero de galinhas e de pintinhos no
galinheiro.
(a) Temos 4x + 2y = 240, ou seja, 2x + y = 120. Como 8 kg = 8 000 g, temos
160x + 40y 8 000. Assim, 4x + y 200.
200

Em resumo, os nmeros x de galinhas e y de


pintinhos satisfazem
()

2x + y = 120
4x + y 200

r:y = 200- 4x

150
(0,120)
100
50
0

10

20

30

40

50

60

70

s:y = 120 - 2x

(b) A reta 2x + y = 120 corta o eixo Ox em x = 60 e o eixo Oy em y = 120. A reta


4x + y = 200 corta o eixo Ox em x = 50 e o eixo Oy em y = 200. Os grcos dessas
retas esto dadas na gura, em que a desigualdade 4x + y 200 representada pela
regio sombreada. Observe que, na gura, as condies () so representadas pelo
segmento que liga os pontos P e (0, 120). As coordenadas do ponto P so a soluo do
sistema
2x + y = 120
4x + y = 200,
ou seja, x = 40 e y = 40 e, portanto, P = (40, 40).
(c) Temos que 2 20 + 80 = 120 e 4 20 + 80 200. Logo, x = 20 e y = 80 satisfazem as
condies () e, por isso, o galinheiro comporta, sim, 20 galinhas e 80 pintinhos. Agora,
2 30 + 100 = 120, logo, x = 30 e y = 100 no satisfazem as condies () e, por isso,
o galinheiro no comporta 30 galinhas e 100 pintinhos.
(d) O nmero mximo de galinhas 40 e, nesse caso, teremos tambm 40 pintinhos. O
nmero mximo de pintinhos 120 e, nesse caso, teremos 0 galinhas.
176. Um nmero perfeito Se 231 1 um nmero primo, seu nico divisor prprio o nmero
1. Ento, os divisores prprios de 230 (231 1) so
1, 2, 22 , . . . , 229 , 230 , (231 1), 2(231 1), 22 (231 1), . . . 229 (231 1).
A soma S desses divisores
S = [1 + 2 + 22 + + 229 + 230 ] + (231 1)[1 + 2 + 22 + + 229 ] .
Em cada um dos dois colchetes aparece a soma Sn de uma progresso geomtrica de n termos,
sendo o primeiro termo igual a 1 e a razo igual a 2: o primeiro colchete S31 , com 31 termos e

322

OBMEP 2010

Solues do Nvel 3
o segundo S30 , com trinta termos. Usando a frmula da soma dos termos de uma progresso
geomtrica, obtemos
S31 =

231 1
230 1
= 231 1 e S30 =
= 230 1.
21
21

Ento a soma dos divisores prprios de 230 (231 1)


S = (231 1) + (231 1)[230 1] = (231 1)(1 + 230 1) = 230 (231 1).
Logo, essa soma igual a 230 (231 1), como queramos provar.
177. Quinze minutos a mais Sabemos que a velocidade a razo da distncia percorrida
pelo tempo gasto.
Soluo 1: Denotando por t o tempo gasto, em horas, pelo carro mais lento, o que faz a
viagem a uma velocidade de 60 km/h, sabemos que o tempo gasto pelo outro carro de
t 1/4, j que 15 minutos um quarto de hora. Como ambos percorrem a mesma distncia,
segue que 60 t = 70 (t 1/4), portanto, t = 7/4 horas, ou 1 hora e trs quartos de hora.
Logo, a distncia entre as duas cidades 60 7/4 = 105 km.
Soluo 2: Vamos representar por d a distncia, em quilmetros, entre as cidades A e B e
por T o tempo gasto, em horas, pelo carro mais veloz. Como o outro carro gasta 15 minutos
a mais para fazer o mesmo percurso, temos que o tempo gasto pelo carro mais lento igual
a T + 0, 25 horas, pois 15 min = 0,25 h. Como o carro mais veloz anda a 70 km/h, temos
70 = d/T e, como o mais lento anda a 60 km/h, temos 60 = d/(T + 0, 25). Assim,
d = 70 T = 60(T + 0, 25),
ou seja, T = 1,5 h, e a distncia entre as cidades A e B igual a d = 70 1,5 = 105 km.
178. Outros caminhos Qualquer que seja o trajeto de Jlia da sua casa at a escola, se ela
deseja seguir um caminho mais curto, ela deve percorrer exatamente oito quarteires para a
direita e cinco quarteires para cima. Um caminho mais curto ligando a sua casa at a escola
, ento, uma sequncia de travessias de quarteires, sendo oito delas no sentido horizontal
(para a direita) e cinco no sentido vertical (para cima). Assim, para denir um caminho mais
curto, ela precisa apenas decidir em que ordem far essas treze travessias.
Para isso, imaginemos oito cartelas impressas com a letra D e cinco cartelas impressas com a
letra C. Uma permutao qualquer dessas cartelas pode ser interpretada como um caminho
mais curto a ser percorrido por Jlia. Por exemplo, a sequncia de cartelas DDCDCCDDDDCDC dene o caminho indicado na gura dada.

OBMEP 2010

323

Solues do Nvel 3
Para determinar o nmero de maneiras pelas quais podem ser ordenadas essas cartelas, devemos contar de quantas maneiras diferentes se pode colocar cinco cartelas com a letra C
em uma la com treze lugares vagos, sendo os demais oito lugares na la ocupados com as
cartelas com a letra D.
Inicialmente, devemos escolher um dos treze lugares vagos para colocar uma letra C. Colocada
essa letra, sobram doze lugares vagos para a segunda letra C. Colocada essa letra, sobram onze
lugares vagos para a terceira letra, dez lugares para a quarta letra e, nalmente, nove lugares
para a quinta letra C. Agora, uma vez colocadas as cinco letras C, qualquer permutao dessas
letras entre si no altera a distribuio das letras na la. Como a quantidade de permutaes
de cinco objetos 5! = 120, pelo princpio multiplicativo temos que o nmero de maneiras de
ordenar as treze cartelas dado por
13 12 11 10 9
= 1 287,
120
de modo que existem 1 287 caminhos mais curtos diferentes da casa de Jlia at a escola.
179. Escrevendo no tabuleiro Comeando com a letra A, ela pode ser escrita em qualquer
uma das nove casas do tabuleiro. Uma vez escrita a letra A, sobram seis casas nas quais pode
ser escrita a letra B. Uma vez escritas as letras A e B no tabuleiro, sobram trs casas para
a letra C ser escrita. Assim, pelo princpio multiplicativo, existem 9 6 3 = 162 maneiras
diferentes das letras A, B e C serem escritas no tabuleiro.
180. Frao e percentagem A opo correta (d).
Se um nmero x diminudo de 40%, ele passa a valer 60% de x, ou seja, 0,6x. Do mesmo
modo, quando um nmero y diminudo de 60%, ele passa a valer 0,4y. Portanto, a frao
x/y passa a ter o valor
0,6x
6x
x
=
= 1,5 ,
0,4y
4y
y
o que signica que a frao x/y aumentou 50% do seu valor.
181. Tringulos sobrepostos Os pontos A, B, C e D formam o retngulo ABCD.
7

Como as diagonais de um retngulo o dividem em quatro tringulos de mesma rea, a rea


sombreada igual a trs quartos da rea do retngulo ABCD. Assim, a rea sombreada
3
igual a 4 7 4 = 21 cm2 .

Vejamos, agora, o caso da outra gura. Sejam x = DE = CE, y = AE = BE e E o ponto


de interseo dos segmentos AC e BD.

C
x

x
E

A
324

B
OBMEP 2010

Solues do Nvel 3
A rea sombreada a soma das reas dos tringulos ADE e ABC, ou seja,
4x 47
+
= 2x + 14.
2
2
Logo, basta calcular x. Temos que x + y = 7 e, pelo Teorema de Pitgoras aplicado ao
tringulo AED, tambm y 2 = x2 + 42 . Substituindo y = 7 x nessa ltima equao, obtemos
(7 x)2 = x2 + 16, de modo que 49 14x + x2 = x2 + 16, ou seja,
x=

49 16
33
=
.
14
14

Finalmente, a rea sombreada dada por 2

33
131
33
+ 14 =
+ 14 =
cm2 .
14
7
7

182. Dois motoristas Sabemos que a velocidade a razo da distncia percorrida pelo tempo
gasto. Seja d a distncia entre as duas cidades A e B.
O primeiro motorista percorre a distncia de 2d velocidade constante de 80 km/h,
portanto, o tempo total gasto por esse motorista
t=

2d
d
=
horas.
80
40

O segundo motorista percorre a distncia d na ida uma velocidade constante de 90


km/h e, na volta, percorre a mesma distncia d velocidade constante de 70 km/h.
Logo, o tempo gasto na ida e volta
t=

d
d
16 d
8d
+
=
=
horas.
70 90
630
315

Como

8d
8d
d
=
<
,
40
320
315
vericamos que o motorista que viaja velocidade constante de 80 km/h o que gasta menos
tempo no percurso de ida e volta.
183. Soma e inverte Como 0 no o inverso de nmero algum, qualquer sequncia que comece
e termine em 0 deve ser dada por
+1

+1

0 1 1 0.
Uma sequncia dessas a seguinte.
3 +1 5 +1 13 +1 21
1 +1 2 +1 5 +1 8 i
+1
+1
+1
i

0 1 2 3
3
3
3
3
8
8
8
8
8 +1 13 i
21 +1
8 +1 5 i
13 +1 8 +1 3 +1 2



21
21
13
13
13
5
5
5
5
5 +1 3 +1 1 +1 1 i
i
+1
+1
2 1 0.
2
2
2
2
Uma outra sequncia, bem mais curta e simples , simplesmente,
i

+1

+1

0 1 1 0.
184. Carro ex
OBMEP 2010

325

Solues do Nvel 3
(a) Com cada litro de gasolina, que custa R$ 2,49, o carro roda 12,3 quilmetros. Logo, o
2,49
reais. Se o carro zer y quilmetros por litro de
preo do quilmetro rodado de
12,3
1,59
lcool, o preo do quilmetro rodado com lcool de
reais. Para que a utilizao
y
do lcool seja mais vantajosa, nanceiramente, necessrio que
1,59
2,49
<
,
y
12,3

ou seja, que y >

1,59 12,3
= 7,85.
2,49

Assim, o consumo desse carro com lcool deve ser maior do que 7,85 km/l.
(b) Supondo que o consumo do carro seja de x km/l de gasolina e de y km/l de lcool,
queremos saber quando o custo com gasolina maior do que o custo com lcool, isto ,
quando
1,59
2,49
>
,
x
y
o que acarreta 2,49 y > 1,59 x, ou seja, y > 0,64 x, j que x e y so valores positivos.
Um exemplo disso o carro como o do item (a), que consuma 12,3 km/l de gasolina e
8 km/l de lcool. Ou, ento, um carro que faa 10 km/l de gasolina e 7 km/l de lcool.
(c) Com cada litro de gasolina, que custa R$ 2,49, o carro roda x quilmetros. Logo, o
2,49
249
preo de 100 quilmetros rodados de g(x) = 100
=
com gasolina. Com cada
x
x
x
litro de lcool, que custa R$ 1,59, o carro roda + 1 quilmetros. Logo, o preo de 100
2
1,59
318
quilmetros rodados com lcool de a(x) = 100 x
=
.
x+2
2 +1
249
318
= g(x) = a(x) =
, ou seja,
x
x+2
249 (x + 2) = 318 x, cuja soluo x = 7,22 km/l, que deve ser o consumo com
gasolina. Para que o custo seja o mesmo, o consumo do carro com lcool deve ser
7,22
+ 1 = 4,61 km/l.
de
2
x
(e) Supondo que o consumo do carro seja de x km/l de gasolina e de + 1 km/l de lcool,
2
queremos saber quando o custo com lcool menor do que o custo com gasolina, isto
2,49
3,18
<
, o que acarreta 0,69 x < 4,98, ou seja, x < 7,22, j que x um
, quando
x+2
x
valor positivo. Assim, s nanceiramente vantajoso abastecer com lcool se o consumo
com gasolina for menor do que 7,22 km/l. Um exemplo disso um carro que faa 6
km/l de gasolina e, portanto, 4 km/l com lcool: nesse caso, por exemplo, o custo de
100 quilmetros rodados com gasolina de g(6) = R$ 41,50 e com lcool de a(6) = R$
39,75. Observe que, a partir de um consumo de 7,22 km/l de gasolina, nanceiramente
vantajoso abastecer s com gasolina; por exemplo, se o carro zer 10 km/l de gasolina
e, portanto, 6 km/l de lcool, o custo de 100 quilmetros rodados com gasolina de
g(10) = R$ 24,90 e com lcool de a(10) = R$ 26,50.

(d) Para que o custo seja o mesmo, basta ter

Observao: Todos os valores utilizados nessas solues foram arredondados na segunda casa
decimal.
185. Contando tringulos Sejam A, B, . . . , K os onze pontos marcados, como na gura dada.

326

OBMEP 2010

Solues do Nvel 3
K
J
I
H
A

Dividiremos a contagem em trs casos.


(i) Um vrtice A. Nesse caso, um vrtice do tringulo deve estar no conjunto {H, I, J, K}
e o outro vrtice no conjunto {B, C, D, E, F, G}. Como existem quatro escolhas para um
vrtice e seis escolhas para o outro vrtice, a quantidade de tringulos com um vrtice
no ponto A 6 4 = 24.

(ii) Dois vrtices em {B, C, D, E, F, G}. O nmero de possveis escolhas de dois dentre esses
seis pontos
65
6!
2
=
= 15.
C6 =
4!2!
2
O outro vrtice do tringulo qualquer um dos quatro pontos H, I, J ou K. Da, a
quantidade desses tringulos 4 15 = 60.

(iii) Dois vrtices em {H, I, J, K}. O nmero de possveis escolhas de dois dentre esses quatro
pontos
4!
43
2
C4 =
=
= 6.
2!2!
2
Como o outro vrtice pode ser escolhido de seis maneiras diferentes no conjunto {B, C, D,
E, F, G}, resulta que a quantidade desses tringulos 6 6 = 36.
Logo, 24 + 60 + 36 = 120 a quantidade de tringulos cujos vrtices so tomados dentre os
onze pontos da gura.
186. Quadrado perfeito Seja x um nmero de oito algarismos, da forma x = 9999 .
Como o menor desses nmeros 99 990 000 e o maior 99 999 999, temos que
99 990 000 x 99 999 999.
Observemos que 10 0002 = 100 000 000 = 99 999 999 + 1 e que
9 9992 = 10 000 1

= 10 0002 20 000 + 1 = 99 980 001.

Isso mostra que 9 9992 < x < 10 0002 , ou seja, x est compreendido entre dois quadrados
perfeitos consecutivos. Assim, x no pode ser um quadrado perfeito, ou seja, no existe
algum quadrado perfeito da forma 9999 .

n n 1 < 0,01 equivalente a


187. Diferena quase nula A desigualdade

n < 0,01 + n 1. Como os dois lados da desigualdade so nmeros positivos, podemos


elevar ambos membros ao quadrado para obter a desigualdade equivalente

2
2
n < 0,01 + n 1 .

Mas isso equivale a n < 0,012 + 0,02 n 1 + n 1, donde obtemos

n1>

1
1 1002
1 0,012
1002 1
=
.
=
2
0,02
200
100

OBMEP 2010

327

Solues do Nvel 3
Elevando, novamente, ao quadrado os dois membros (no negativos) dessa desigualdade, obtemos
(1002 1)2
1004 2 1002 + 1
1002 1
1
n1>
=
=
+
2
2
200
4 100
4
2 4 1002
ou seja,
1
1
n 1 > 2 500 +
2 40 000
e, nalmente,
1
1
.
n > 2 500 + +
2 40 000
1
Como 2 + 40 1 < 1, temos que o menor nmero inteiro que satisfaz essa ltima desigualdade
000
2 501. Assim, estabelecemos que o menor nmero inteiro positivo que satisfaz a desigualdade
dada o nmero 2 501.
188. Conjunto de Cantor
(a) De acordo com a denio do conjunto de Cantor, temos o desenho seguinte.
C1
C2
C3

1/3

2/3

0 1/9 2/9 1/3

2/3 7/9 8/9 1

(b) 1/3 uma extremidade de C2 , portanto, pertence ao conjunto de Cantor.


3/81 = 1/27 e 1/27 uma extremidade de C4 , portanto, 3/81 pertence ao conjunto
de Cantor. 4/9 est entre 1/3 e 2/3, portanto, est no tero central de C1 , que foi
removido de C2 ; assim, 4/9 no pertence ao conjunto de Cantor. 4/81 est entre 1/27 e
2/27, portanto, est no tero central do primeiro segmento de C3 , que foi removido de
C4 ; assim, 4/81 no pertence ao conjunto de Cantor.
(c) Observe que C1 tem comprimento 1, C2 tem comprimento 2/3, C3 tem comprimento 4/9,
C4 tem comprimento 8/27 e C5 tem comprimento 16/81. Assim, os comprimentos de
C1 , C2 , C3 , . . . , Cn , . . . formam uma progresso geomtrica de razo q = 2/3 e primeiro
termo a1 = 1, como segue.
1,

2
,
3

2
3

2
3

2
3

,... ,

2
3

n1

,...

Em particular, o comprimento de Cn (2/3)n1 .


189. Enchendo uma piscina Como as torneiras A e B despejam gua na piscina com vazo
constante, o volume de gua despejado na piscina de cada uma das torneiras proporcional
ao tempo em que ela ca aberta. Assim, se durante duas horas a torneira A enche 15% do
volume da piscina, ento em 4 horas ela encher 30% do volume da piscina.
Mas, quando as torneiras A e B cam simultaneamente abertas durante quatro horas, elas
conseguem encher 50% do volume da piscina. Da, temos que a torneira B enche 50%30% =
20% do volume da piscina em quatro horas.
Para saber quanto tempo a torneira B deve car aberta para encher os 35% restantes do
volume da piscina, basta utilizar a regra de trs.
horas percentual
4

20%
x

35%

328

OBMEP 2010

Solues do Nvel 3
Logo, a torneira B gastar x =

35 4
= 7 horas para encher os 35% restantes.
20

190. Probabilidade de ser um nmero par


Soluo 1: Sejam a e b os nmeros escritos nas bolas retiradas por Jos e Maria, respectivamente. Existem, ento, nove possibilidades para a e oito possibilidades para b. Desse modo,
existem 9 8 = 72 possibilidades para o nmero a b. Para contar quantos desses nmeros a b
so pares, precisamos analisar separadamente dois casos, como segue.
Ambos nmeros a e b so pares.

O nmero a mpar e o nmero b par.


No primeiro caso, em que a e b so pares, existem quatro possibilidades para a e trs possibilidades para b. Desse modo, existem 4 3 = 12 possibilidades ao todo.

No segundo caso, em que a mpar e b par, existem cinco possibilidades para a e quatro
possibilidades para b. Desse modo, existem 5 4 = 20 possibilidades.
32
4
12 + 20
=
= .
Portanto, a probabilidade de o nmero a b ser par
72
72
9
Soluo 2: A paridade do nmero a ser formado depende da paridade do nmero escrito na
bola a ser retirada por Maria. Dentre os nmeros inteiros de 1 a 9, existem cinco mpares,
1, 3, 5, 7 e 9, e quatro pares, 2, 4, 6 e 8. Portanto, a probabilidade de que o nmero a ser
4
4
formado seja par
= .
5+4
9
191. Mltiplo de 7 N = (n + 6m)(2n + 5m)(3n + 4m) um mltiplo de 7.
Soluo 1: Inicialmente, observemos que, denotando k = n m, temos
N = (n + 6m)(2n + 5m)(3n + 4m)
= (n + 7m m)(2n + 7m 2m)(3n + 7m 3m)

= (n m + 7m) 2(n m) + 7m 3(n m) + 7m

= (k + 7m)(2k + 7m)(3k + 7m).

Como 7 primo e divide N, ento pelo menos um dos trs fatores k + 7m, 2k + 7m ou 3k + 7m
de N mltiplo de 7.
k
k + 7m
= +m inteiro, logo k mltiplo de 7. Segue
(i) Se k +7m mltiplo de 7, ento
7
7
que 2k e 3k tambm so mltiplos de 7 e, portanto, os trs fatores k + 7m, 2k + 7m e
3k + 7m de N so mltiplos de 7. Conclumos que N mltiplo de 73 .
2k + 7m
2k
(ii) Se 2k + 7m mltiplo de 7, ento
=
+ m inteiro, logo 2k mltiplo de 7.
7
7
Como 2 e 7 so primos entre si, segue que k mltiplo de 7, o que leva ao caso anterior
e N resulta ser mltiplo de 73 .
(iii) Se 3k + 7m mltiplo de 7, analogamente conclumos que k mltiplo de 7, o que leva
ao caso anterior e N mltiplo de 73 .
Assim, estabelecemos que N mltiplo de 73 .
Soluo 2: Consideremos os nmeros A = n + 6m, B = 2n + 5m e C = 3n + 4m. Como
o nmero primo 7 divide o produto N = A B C, ento 7 divide pelo menos um desses
fatores. Para concluir que 73 divide N, basta mostrar, portanto, que se 7 divide algum dos
nmeros A, B ou C ento 7 divide cada um deles.
OBMEP 2010

329

Solues do Nvel 3
Suponhamos que 7 divida A. Ento 7 divide 2A. Mas 2A = 2n + 12m = B + 7m. Como 7
tambm divide 7m, segue que 7 divide B. Da mesma forma, como 7 divide A, segue que 7
divide 3A. Mas 3A = 3n + 18m = C + 14m. Como 7 tambm divide 14m, conclumos que 7
divide C.
Suponhamos que 7 divida B. Ento 7 divide 4B. Mas 4B = 8n + 20m = A + 7(n + 2m). Como
7 tambm divide 7(n + 2m), segue que 7 divide A. Como j foi mostrado acima, dividindo A,
7 tambm divide C.
Suponhamos que 7 divida C. Ento 7 divide 5C. Mas 5C = 15n + 20m = A + 7(2n + 2m).
Como 7 tambm divide 7(2n + 2m), segue que 7 divide A. Como j foi mostrado acima,
dividindo A, 7 tambm divide B.
192. Os ngulos 15 e 75 Como DB a diagonal de um quadrado de medindo 1 cm, o
lado
2 = 11 + 12 = 2, ou seja, DB =
Teorema de Pitgoras garante que DB
2 . Recordemos que
sen 60
= 3;
cos 60

3
= sen 30 =
tg 30
.

cos 30
3

1
cos 60 = sen 30 = ;
2

3
= cos 30 =
sen 60
;
2

tg 60 =

(a) O tringulo BCE equiltero, logo seus ngulos internos medem 60 . A partir dessa
informao, obtemos os ngulos assinalados na gura.
1

30

M
60

F
60

30

3
1
CD
=
=
. Como sen 60
, segue que
No tringulo CDF temos
=
DF DF
2

1
3
2
2 3
=
e, portanto, que DF = =
. Ainda no tringulo CDF temos
DF
2
3
3
CF
1
CF
1
CF
=
. Mas cos 60 = , de onde se conclui que =
, ou seja,
cos 60 =
DF
2
2
2 3/3
2 3/3

3
3
. Segue que BF = 1 CF = 1
. Temos, agora,
CF =
3
3

1
FN
FN
3 3

= sen 30 =
=
, de modo que F N =
2
BF
6
1 3/3
sen 60

3
BN
BN
,
= cos 30 =
=
2
BF
1 33

de modo que BN =

Assim, calculamos os trs lados do tringulo DBN, como segue.

DB = 2;

1+ 3
2 3 3 3
+
=
;
DN = DF + F N =
3
6
2

31
BN =
.
2

330

OBMEP 2010

31
.
2

Solues do Nvel 3
(b) No tringulo DBN temos DBN = 45 + 30 = 75 , donde conclumos que
B DN = 15 . Assim, temos

31

BN
6 2
D

2
cos 75 =
=
=
DB
4
2
e
15

1+ 3

6+ 2
DN

2
=
.
cos 15 =
=
3 +1
2
DB
4
2
2
Resta observar que sen 75 = cos 15 , sen 15 = cos 75 e que

sen 75
6+ 2
6 2
sen 15

.
tg 75 =
=
=
e tg 15 =
cos 75
cos 15
6 2
6+ 2

75

3 -1
2

193. Crculos tangentes


(a) Na gura dada esto desenhadas dois crculos concntricos de
raios r e R e um crculo de raio x, simultaneamente tangente
aos dois crculos concntricos. Logo, r + 2x = R, donde
Rr
x=
.
2

R
r

(b) Na gura dada temos dois crculos tangentes de raio x que


tambm so tangentes aos dois crculos concntricos de raios
r e R. Os pontos A, B e C so os centros desses crculos. Para
traar doze crculos de raio x na regio entre os dois crculos
360
concntricos, devemos ter ACB =
= 30 .
12

R
r
C

B
x
x
A

Se T o ponto de tangncia dos crculos de raio x, ento T ponto


mdio do segmento AB e ACT = 15 . Nesse tringulo retngulo
temos
AT
x
C
sen 15 =
=
.
AC
r+x

x
15

r+x

Observe que
sen 15 = sen (45 30 ) = sen 45 cos 30 cos 45 sen 30

3 3
21
6 2
=

=
,
2 2
2 2
4
Rr
, do que
o que coincide com o valor obtido na questo precedente. Mas x =
2

Rr
6 2
conclumos que
=
. Dividindo por r o numerador e o denominador do
R+r
4
membro esquerdo dessa igualdade obtemos
R

1
6 2
r
=
.
R
4
+1
r
OBMEP 2010

331

Solues do Nvel 3
Segue que 4

Observao:
sen /2 =

R
1 =
6 2
r

R
+ 1 e, nalmente,
r

R
4+ 6 2

.
=
r
4 6+ 2

Uma outra maneira de obter o valor de sen 15 utilizar a frmula


(1 cos )/2 do ngulo metade. Para = 30 , obtemos
30
=
sen 15 = sen
2

1 cos 30
=
2

3/2

2
2

Repetindo o argumento apresentado acima com esse valor do seno, obtemos

R
2+ 2 3
=
.
r
2 2 3

bastante curioso e nada evidente, primeira vista, que essas duas expresses envolvendo
radicais sejam iguais:

2+ 2 3
4+ 6 2

=
.
4 6+ 2
2 2 3

194. Mudando a base Num tringulo issceles, a altura relativa base coincide com a mediana.
Traando essa altura no tringulo dado, de base 10, obtemos dois tringulos retngulos com
catetos medindo 5 e h e hipotenusa 13. Pelo Teorema de Pitgoras, temos h2 + 52 = 132 ,

donde h2 = 132 52 = 144 e, portanto, h = 144 = 12. Logo, a rea do tringulo dado
10 12
bh
=
= 60 cm2 .
2
2
Agora colamos os dois tringulos retngulos ao longo do cateto medindo
5, obtendo um tringulo issceles com 12 + 12 = 24 cm de base, lados
de 13 cm e altura relativa base igual a 5 cm. Logo, esse novo tringulo
24 5
issceles tambm tem rea igual a
= 60 cm2 .
2
A=

195. Clube de Matemtica Sejam H e M os nmeros de homens e mulheres, respectivamente,


no clube. Temos duas possibilidades. Se eu sou um menino, temos M = H 1 e, quando
falta um menino, o nmero total de pessoas no clube
M + H 1 = H 1 + H 1 = 2H 2.

3
4 (2H

2), de modo que H = 1. Mas ento, M = 1 1 = 0, o que


Logo, H 1 = M =
no possvel. Assim, necessariamente, eu sou uma menina, e, portanto, M = H + 1 e temos
3
H + 1 = 4 (2H + 1 1), donde H = 2 e M = 3.
196. Uma calculadora diferente
Soluo 1: Para calcular (2 3) + (0 3) utilizamos as propriedades (i), (ii) e (iii), obtendo
(2 3) + (0 3)

(iii)

=
=

(iii)

(i) (ii)

=
=

332

(2 + 0) (3 + 3)
(6 + (4)) (6 + 0)
(6 6) + ((4) 0)

6 + (4) 2
6 8 = 2.

OBMEP 2010

Solues do Nvel 3
Para calcular 1 024 48, observe que 1 024 = 976 + 48. Assim,
1 024 48 = (976 + 48) (0 + 48)
= (976 0) + (48 48)
= 976 2 + 48

= 1 952 + 48 = 2 000.
Soluo 2: Pelas propriedades (i), (ii) e (iii),
a b = ((a b) + b) (0 + b)
= ((a b) 0) + (b b)
= (a b) 2 + b

= 2a 2b b

= 2a b,
para quaisquer inteiros a e b. Assim,

(2 3) + (0 3) = (2 2 3) + (2 0 3) = 1 3 = 2
e
1 024 48 = 2 1 024 48 = 2 048 48 = 2 000.
Observao: Existe uma nica operao sobre os inteiros com as propriedades (i), (ii) e
(iii) do enunciado, a saber, a b = 2a b, como mostramos na segunda soluo. No entanto,
mesmo restringindo o domnio de aos inteiros no negativos, possvel mostrar que uma
operao com as propriedades (i), (ii) e (iii) do enunciado existe, e nica, sendo dada por
a b = 2a b, s que, agora, precisamos nos restringir a nmeros inteiros no negativos a, b
tais que 2a b, para que o resultado da operao ainda seja um nmero inteiro no negativo.
Denotemos o conjunto dos inteiros no negativos por N .
claro que a deduo feita na segunda soluo se aplica somente se a b pois, nesse caso,
a b N . Para provar a existncia e unicidade da operao a b nos inteiros no negativos
tais que 2a b, precisamos de um argumento mais sutil, como segue.

Supondo que a operao a b esteja denida em N sempre que 2a b, dando um resultado em N e satisfazendo as propriedades (i), (ii) e (iii) do enunciado, armamos que
a b = 2a b vale sempre. De fato, dado c N , temos c (2c) = 0, j que podemos cancelar
2c de ambos lados da igualdade
2c = (2c) (2c) = (c + c) (2c + 0) = (c (2c)) + (c 0) = (c (2c)) + 2c.
Agora, dados a, b N , com 2a b e b > a, temos 2a b, b a N e
a b = (2a b) + (b a) (2a b) + (2b 2a)
= (2a b) (2a b) + (b a) (2(b a))

= (2a b) + 0 = 2a b.

Finalmente, para a, b N , com 2a b e a b, isso j foi mostrado na segunda soluo.


Assim, a b = 2a b vale para quaisquer a, b N tais que 2a b.
197. Cercando o globo terrestre Como o raio da Terra muito grande, e foi dado apenas
um acrscimo de 1 m ao comprimento do o ao longo do Equador, parece que a folga entre o
o e o Equador muito pequena. Mais ainda, se trocarmos a Terra por Jpiter ou por uma
OBMEP 2010

333

Solues do Nvel 3
bolinha de gude e realizarmos essa mesma experincia, parece que a altura da folga entre o
o aumentado e o equador dessa esfera tambm muda, sendo que quanto maior a esfera
considerada, menor a folga entre o o e o equador da esfera.
Mostremos que essa ideia intuitiva falsa e que a altura da folga, entre o o e o Equador, sempre de aproximadamente 16 cm, independentemente do raio da esfera em que a experincia
for realizada.
Consideremos a circunferncia de comprimento 2 R de um crculo de raio R e tambm a
circunferncia de comprimento igual a 2 R + 1 de um outro crculo de mesmo centro, de
raio igual a R + h. Assim, h a altura da folga entre as duas circunferncias. Como a
circunferncia de um crculo de raio R + h tem comprimento igual a 2 (R + h), obtemos a
igualdade
h

2 R + 1 = 2 (R + h) = 2 R + 2 h
que, simplicada, fornece 1 = 2 h, ou seja,
h=

1
1

0,16.
2
6,28

Portanto, independentemente do valor de R, a altura da folga obtida com 1 m a mais de o ,


sempre, de aproximadamente 16 cm. Em particular, somente uma formiga capaz de passar
por debaixo desse o.
198. Comprimento de uma corda Sendo AB um dimetro, o tringulo ABC est inscrito
numa semicircunferncia, implicando que esse tringulo retngulo no vrtice C. Pelo Teorema de Pitgoras,
C

BC 2 = AB 2 AC 2 ,

12 cm

ou seja,
BC 2 = 202 122 = 256 = 162 .

20 cm

Assim, obtemos que BC = 16.


199. Dois irmos Sejam x e y

xy
z1

z + 20

as idades atuais dos dois irmos e z a idade do pai. Temos


= 3
= 2[(x 1) + (y 1)] = 2x + 2y 4
= (x + 20) + (y + 20) = x + y + 40

Uma maneira simples de encontrar z multiplicar a terceira equao por 2 e do resultado


subtrair a segunda equao, obtendo 2z + 40 (z 1) = 80 (4), o que implica z = 43.
Usando as duas primeiras equaes podemos calcular, agora, a idade dos dois lhos. Pela
primeira equao, x = y + 3 e, pela segunda, 43 1 = 2x + 2y 4, ou seja, x = 23 y.
Somando essas duas equaes obtidas, encontramos 2x = 26, donde x = 13 e, portanto,
y = 10.

200. Canelonis de ricota Colando os retngulos de massa ao longo do maior lado, Pedro
obtm um cilindro de base circular com 10 cm de comprimento e 16 cm de altura. O volume
que ele, ento, recheia com ricota o volume V = rea da base altura desse cilindro. A
rea da base dada por r2 , onde r denota o raio da base. Vamos, ento, calcular o raio

334

OBMEP 2010

Solues do Nvel 3
sabendo que o permetro da base 10 cm. Temos 2 r = 10, ou seja, r = 5/. Assim, o
volume de ricota para cada caneloni dado, nesse caso, por
V =

52
16 25
400
16 =
=
cm3 .
2

Agora, colando os retngulos de massa ao longo do menor lado, Pedro obtm um cilindro de
base circular com 14 cm de permetro e 12 cm de altura.
O raio da base agora dado por r = 14/2 = 7/.
Assim, o volume de ricota para cada caneloni dado,
nesse caso, por
V =

588
72
12 =
cm3 .
2

Finalmente, para calcular o novo gasto com ricota, usamos uma regra de trs direta.
Volume (cm3 )
Ricota(g)
400

500

588

Segue que
500 588
= 735 g ,
400
de modo que agora Pedro gasta 235 g a mais de ricota por caneloni.
x=

201. Clculo de segmentos O tringulo ABP retngulo com catetos AB = 1 200 e BP =


150 + 350 = 500. Pelo Teorema de Pitgoras, temos
AP 2 = 1 2002 + 5002 = (144 + 25) 104 = 169 104 = (13 102 )2 ,
de modo que AP = 13 102 = 1 300 m. Analogamente, considerando o tringulo retngulo
P CD, temos
DP 2 = 3502 + 1 2002 = (72 + 122 22 )(52 102 ) = 252 502 ,
donde DP = 1 250 m. Os tringulos P CQ e P BA so retngulos com um ngulo em
comum, logo so semelhantes e segue que
PC
CQ
PQ
=
=
.
PA
PB
AB
Substituindo os valores conhecidos, obtemos
PQ
350
CQ
=
=
.
1 300
500
1 200
Assim,
PQ =

350 1 300
350 1 200
= 910 m e CQ =
= 840 m.
500
500

OBMEP 2010

335

Solues do Nvel 3
202. Pr chegar junto! Sabemos que a velocidade a razo da distncia percorrida pelo
tempo gasto. Como as velocidades de Luisa e Ada so constantes, a distncia percorrida por
cada uma proporcional ao tempo decorrido. Logo, se Ada percorre 3 000 120 = 2 880 m
no mesmo tempo em que Luisa percorre 3 000 m, ento Ada percorrer 3 000 m no mesmo
tempo em que Luisa percorrer d m, numa rega de trs direta.
Luisa Ada
3 000 2 880
d
3 000

3 0002
Assim, d =
= 3 125 e Luisa deve partir 125 m antes do ponto A para chegar junto com
2 880
Ada ao ponto B.
203. Um professor enfurecido Quem teve x como nota mensal vai ter um desconto de x%
sobre essa nota, ou seja vai perder
x% de x =

x
x2
x=
.
100
100

x2
. Consideremos essa funo
Logo, uma nota inicial de x, depois do castigo, ca sendo x
100
nota depois do castigo, dada por
x2
.
100
Como as notas mximas e mnimas so 0 e 100, podemos considerar essa funo apenas no
domnio [0, 100], ou seja, para 0 x 100. O grco de f uma parbola com concavidade
para baixo. O valor mnimo dessa funo 0, que ocorre em x = 0 e x = 100 e o valor
mximo ocorre no vrtice, ou seja, no ponto x = 50, que a mdia aritmtica entre as duas
razes 0 e 100 de f.
f (x) = x

(a) A maior nota depois do castigo para os alunos que, antes do castigo, tiraram 50. Essa
nota
502
= 25 .
f (50) = 50
100
(b) A menor nota 0 e ocorre para os alunos que tiraram 0 ou, pasmem, 100 antes do
castigo. De fato, f (0) = f (100) = 0.
(c) Para cada nota maior do que 50 h uma outra nota, menor do que 50, que acaba sendo
igual depois do castigo. De fato, pela simetria da parbola, f (50 n) = f (50 + n), para
cada 0 n 50. Por exemplo, quem tirou 30 acaba com a mesma nota 21 de quem
tirou 70, pois f (30) = 21 = f (70). Assim, procede a reclamao dos alunos que tiraram
notas boas.
204. O percurso de um atleta O Polo Norte da Terra o ponto mais fcil de ser identicado
como soluo: saindo o atleta do Polo Norte, correndo 5 km para o Sul, depois 5 km para o
Leste e nalmente 5 km para o Norte, ele volta novamente para o Polo Norte.
Polo
Norte

A
B

C2
C1
Paralelo com 5 km
Polo
Sul

336

OBMEP 2010

de comprimento

Solues do Nvel 3
y
25

100 x

50

Vamos determinar outros ponto da Terra que satisfazem as hipteses do problema. Consideremos o paralelo (linha paralela ao Equador) de comprimento 5 km. Existem dois deles, um
prximo ao Polo Norte e outro prximo ao Polo Sul. Vamos denotar por C1 o que est mais
prximo do Polo Sul e por C2 o paralelo que est 5 km ao Norte de C1 , distncia essa medida
ao longo de um meridiano. Armamos que qualquer ponto A sobre o paralelo C2 satisfaz
as hipteses do problema. De fato, saindo de A e caminhando 5 km para o Sul, chega-se a
um ponto B do paralelo C1 . Como C1 mede 5 km, saindo de B e caminhando 5 km para o
Leste retorna-se novamente a B. Finalmente, saindo de B e caminhando 5 km para o norte,
retorna-se novamente ao ponto de partida A.
205. reas iguais Seja T a rea do tringulo ABC e denotemos por a e c as reas internas
aos semicrculos de dimetros AB e BC mas externas ao semicrculo de dimetro AC e por
b e d as reas compreendidas entre os catetos do tringulo e o semicrculo de dimetro AC.
Segue que a rea do semicrculo de dimetro AB dada por a + b, portanto,
1 AB 2
a+b=
= AB 2 ,
2
2
8
a rea do semicrculo de dimetro BC dada por c+d, portanto,
1 BC
c+d=
2
2

BC 2
8

a b

e a rea do semicrculo de dimetro AC dada por


b + d + T, portanto,
1 AC
b+d+T =
2
2

C
d
c

AC 2 .
8

AC 2 T. Alm disso, o Teorema de Pitgoras aplicado ao tringulo


8
retngulo ABC fornece AC 2 = AB 2 + BC 2 ou, ento, AB 2 + BC 2 AC 2 = 0. Assim,
e, em particular, b+d =

a + c = (a + b) + (c + d) (b + d) =

AB 2 + BC 2 AC 2 + T = T,
8

ou seja, a soma a + b das reas sombreadas igual rea T do tringulo retngulo ABC.
206. Funo denida por rea
(a) A reta r passa pelo ponto (0, 2), portanto, tem equao dada por y = mx+2. Como essa
reta tambm passa pelo ponto (2, 0), temos 0 = 2m + 2, o que implica m = 1. Assim,
a equao de r y = x + 2. A reta s passa pelo ponto (0, 6), portanto, tem equao
dada por y = mx + 6 e, como tambm passa pelo ponto (3, 0), temos 0 = 3m + 6, o que
implica m = 2. Assim, a equao de s y = 2x + 6.
OBMEP 2010

337

Solues do Nvel 3
(b) Denotemos por A o ponto de encontro das retas r e s, B = (0, 2), O = (0, 0),
D = (3, 0) e por C o ponto de corte da reta s com a reta horizontal por B, como na
gura dada. Por denio, f (0) a soma das reas do tringulo ABC e do trapzio
BODC.
y
s

A
B

y
O

Para determinar as coordenadas de A, igualamos x + 2 = 2x + 6, obtendo


x = 4/3. Substituindo esse valor na equao de r ou s resulta y = 10/3, ou seja,
A = (4/3, 10/3). O ponto C pertence reta s e reta y = 2, portanto, 2x + 6 = 2, ou
seja, x = 2, e obtemos C = (2, 2). A altura do tringulo ABC em relao base BC
1 4
4
h = 10/3 2 = 4/3, portanto, a rea do tringulo ABC igual a 2 = .
2 3
3
3+2
= 5, de modo que
J a rea do trapzio BODC 2
2
f (0) =

19
4
+5=
.
3
3

(c) Observe que f (y) igual a f (0) menos a rea do trapzio de altura y e bases 3 e x,
sendo x a abscissa do ponto da reta s que tem ordenada y, ou seja, satisfaz y = 2x + 6.
1
Assim, x = (6 y)/2 = 3 2 y e a rea desse trapzio dada por
1
3 + 3 2y
1
3+x
y=
y = 3y y 2
2
2
4

e obtemos, para 0 y < 2,


f (y) =

1
1
19
19
3y + y 2 = y 2 3y +
.
3
4
4
3

19
y2
3y +
uma parbola
(c) O grco de f (y) =
4
3
cncava para cima. As coordenadas do vrtice V dessa
3
=6e
parbola so x =
2/4
62
19
19
8
y = f (6) =
36+
= 9 +
= ,
4
3
3
3

x
19
3

4
3
0

6
2

10

8
3

4
19
= , o grco de f, com 0 y < 2,
ou seja, V = 6, 8/3 . Como f (2) = 1 6 +
3
3
o segmento de parbola em linha grossa na gura dada.

338

OBMEP 2010

Solues do Nvel 3
207. PA e PG Os quatro termos de uma progresso aritmtica de razo r podem ser escritos
como
x, x + r, x + 2r, x + 3r.
Assim, os trs nmeros em progresso geomtrica so x, x + 2r, x + 3r. Ento, pela prpria
denio de progresso geomtrica, x + 2r a mdia geomtrica de x e x + 3r, ou seja,
2

x x + 3r = x + 2r .
Segue da que x2 + 3xr = x2 + 4xr + 4r2 e, portanto, xr = 4r2 . O caso r = 0 no
interessante, pois daria origem a progresses constantes. Supondo r = 0, obtemos x = 4r.

Atribuindo valores no-nulos a x, obtemos solues do problema. Por exemplo, para x = 4,


obtemos (r = 1 e) a progresso aritmtica 4, 3, 2, 1 tal que os nmeros 4, 2, 1 formam uma
progresso geomtrica. Note que esse problema tem uma innidade de solues, uma para
cada valor escolhido de x = 0.

208. Plano cartesiano Comecemos examinando alguns casos.


f (1) o nmero de pontos inteiros sobre o segmento que liga (0, 0) ao ponto (1, 4),
portanto, f (1) = 0.
f (2) o nmero de pontos inteiros sobre o segmento que liga (0, 0) ao ponto (2, 3),
portanto, f (2) = 0.
f (3) o nmero de pontos inteiros sobre o segmento que liga (0, 0) ao ponto (3, 6). Como
nesse segmento esto os dois pontos inteiros (1, 2) e (2, 4), segue que f (3) = 2.
6

6
5

.
.
.
.
.
.
.
.
.
.
.
.
.
.
.
.
.
.
.
.
.
.
.
.
.
.
.
.
.
.
.
.
.
.
.
.
.
.
.
.
.

.
.
.
.
.
.
.
.
.
.
.
.
.
.
.
.
.
.
.
.
.
.
.
.
.
.
.
.
.
.
.
.
.
.
.
.
.
.
.
.
.
.
.
.
.
.
.
.
.
.
.

..
..
.
..
.
..
.
..
.
..
.
..
.
..
4
.
..
.
..
.
..
.
..
.
..
.
.
2 ......
.
.
..
.
..
.
..
.
..
.
..
.
.

Vejamos, agora, o caso geral. Note que se um ponto inteiro (x, y) est sobre o segmento que
une (0, 0) a (n, n + 3), sem ser uma das extremidades, ento 0 < x < n e 0 < y < n + 3.
Fixado n, temos dois casos: ou 3 divide n ou 3 no divide n.
1o Caso: 3 divide n. Mostremos que f (n) = 2. De fato, vamos supor que n = 3k, com k
inteiro. Queremos encontrar todos os pontos inteiros do segmento que une a origem (0, 0) ao
ponto (3k, 3k + 3). Seja (x, y) um desses pontos. Ento
x
3k
k
=
=
.
y
3k + 3
k+1
Como a ltima frao acima irredutvel, deduzimos que x um mltiplo de k e, como
0 < x < n = 3k, necessariamente x = k ou x = 2k. Os nicos pontos inteiros so, portanto,
(k, k + 1) e (2k, 2k + 2). Assim, f (n) = 2.
2o Caso: 3 no divide n. Mostremos que f (n) = 0. Vamos precisar do seguinte resultado.
Lema. Se 3 no divide n, ento n e n + 3 so primos entre si.
Demonstrao. Suponhamos que o MDC entre n e n + 3 seja d > 1. Ento d divide n e
n + 3, portanto d divide (n + 3) n = 3. Logo, como d > 1, temos d = 3, o que no possvel,
OBMEP 2010

339

Solues do Nvel 3
porque partimos da hiptese de que 3 no divide n. Assim, o MDC de n e n + 3 1, provando
o lema.
Mostremos, agora, que os nicos pontos inteiros sobre o segmento que une (0, 0) a (n, n + 3)
so as extremidades. De fato, suponhamos que esse segmento contenha algum ponto inteiro
(x, y) diferente das extremidades. Ento 0 < x < n e
n
x
=
.
y
n+3
n
irredutvel, portanto, x mltiplo de n, o que no pode
n+3
acontecer porque 0 < x < n. Assim, conclumos que f (n) = 0.
Mas, pelo lema, a frao

209. Trabalhando com um quadriltero Lembre que


qualquer lado de um tringulo maior do que a diferena e menor do que a soma dos outros dois lados. No
tringulo ADB temos AD AB < BD < AD + AB e no
tringulo CBD temos BC CD < BD < BC + CD.

......
.......
. . .
............. ...
.
............. ...
..
..
..
..
..
..
..
....
...
.....
..
....
...
..
.....
...
..
....
..
....
..
...
...
.....
.....
..
..
.
..
..
..
..
...
.....
..
..
.....
.
.
.
..
..
..
... ........
... ........
.
..
..
..
.. ......
... ........
.
..
.
..
........
..
..........
..
.
..
......
..
.......
.
....
.
.
.
..... ..........................................................................................
.................................................................................................
.....

A .............................................................................
...
....

Substituindo os valores conhecidos, obtemos 9 5 < BD < 5 + 9 e 17 5 < BD < 17 + 5,


ou seja,
4 < BD < 14 e 12 < BD < 22.
Dessas duas desigualdades conclumos que 12 < BD < 14 e, como BD inteiro, s podemos
ter BD = 13.
210. O tringulo de Reuleaux O tringulo de Reuleaux formado por 4 regies, um tringulo
equiltero e trs calotas. Cada calota um sexto de um crculo de raio 1, do qual foi retirado
um tringulo equiltero de lado 1. Pelo Teorema de Pitgoras, a altura desse tringulo
equiltero mede

3
1 2
1
=
,
2
2
portanto, a rea desse tringulo equiltero dada por

1 23
3
=
.
2
4
A rea de um setor circular que um sexto do crculo de raio 1 dada por
rea de cada calota dada pela diferena

.
6
4

1
6

, portanto, a

Assim, a rea do tringulo de Reuleaux mede

3
3
3

+
=
cm2 .
3
6
4
4
2
2
211. Interseo de crculos Seja G o baricentro do tringulo ABC, ou seja, o ponto de
encontro de suas medianas. Como a gura invariante por rotaes de 60 ao redor do ponto
G, vemos que o tringulo XY Z equiltero e que G tambm o seu baricentro.

Calculemos o comprimento L de seu lado em termos do lado a do tringulo ABC e do


raio r dos crculos. Seja CM a altura do tringulo ABC em relao base AB. Como a

340

OBMEP 2010

Solues do Nvel 3

1
altura de um tringulo equiltero de lado a mede 2 a 3 e o baricentro divide a altura em
dois segmentos, um com o dobro do comprimento do outro, obtemos

1
1
1
CM = 1 a 3 , GM = 3 CM = 6 a 3 e CG = 2 CM = 3 a 3 .
2
3
C
CG = 2 GM
G
M

Como AZ = BZ = r, vemos que o ponto Z est na reta mediatriz do segmento AB. Entretanto, essa mediatriz a reta suporte da altura CM do tringulo ABC. Desse modo,
vemos que os pontos C, G, M e Z esto alinhados e que o tringulo M ZB retngulo,
com hipotenusa BZ = r e cateto M B = 1 a.
2
Pelo Teorema de Pitgoras,
MZ =

BZ 2 M B 2 =

1
2

4r2 a2

GZ = GM + M Z =

1
6

a 3+

1
2

4r2 a2 .

A
Z

Como GZ o baricentro do tringulo XY Z, resulta que a altura N Z desse tringulo satisfaz


3
N Z = 2 GZ. Por outro lado, a altura N Z desse tringulo equiltero de lado L dada por

1
1
L 3. Assim, 3 GZ = N Z = 2 L 3 e, portanto,
2
2

1
L = 3 GZ = 1 a + 2 12r2 3a2 .
2
212. Valor mximo Estamos procurando o valor de k para o qual mximo o termo da
sequncia
22
32
k2
(k + 1)2
12
,
,
, ...,
,
,...
1,001 1,0012 1,0013
1,001k 1,001k+1
Queremos comparar os tamanhos de dois termos quaisquer dessa sequncia. O mais simples
comparar um termo qualquer com o seguinte. Ocorre que mais fcil comparar duas fraes
com denominadores positivos iguais. Por exemplo, o terceiro termo maior do que o segundo
porque
22
22 1,001
32
=
<
,
1,0012
1,0013
1,0013
sendo evidente que 22 1,001 = 4,004 < 9 = 32 . Mais geralmente, como 1,001n > 0 para
qualquer n, para vericar se
k2
k 2 1,001
(k + 1)2
=
<
,
1,001k
1,001k+1
1,001k+1
basta vericar se k 2 1,001 < (k + 1)2 . Multiplicando tudo por 1 000, isso equivale a
k(k 2 000) < 1 000. Ora, para 1 k 2 000 temos k(k 2 000) 0 < 1 000 e, para
k 2 001, vale k(k 2 000) > 2 001 > 1 000. Em particular,
2 0012
2 0002
<
1,0012 000
1,0012 001

2 0012
2 0022
>
.
1,0012 001
1,0012 002

Logo, a sequncia dada cresce estritamente com 1 k 2 001 e da decresce estritamente


com k 2 001. Assim, o maior termo dessa sequncia atingido com k = 2 001.
OBMEP 2010

341

Solues do Nvel 3
213. Moedas falsas
(a) Aladim deve retirar de cada saco um nmero diferente de moedas, como segue. Primeiro
retira uma moeda do primeiro saco, depois duas do segundo, da trs do terceiro, e
assim, sucessivamente, at o ltimo saco, do qual retira as dez moedas. Ao todo, foram
retiradas 1 + 2 + 3 + 4 + 5 + 6 + 7 + 8 + 9 + 10 = 55 moedas, que so colocadas na
balana. Se todas essas moedas fossem verdadeiras, pesariam um total de 55 10 = 550
g. Mas, como algumas so falsas, o peso ser menor. Se faltar um grama porque h
somente uma moeda falsa e, portanto, o primeiro saco o procurado. Se faltarem dois
gramas porque h duas moedas falsas e, portanto, o segundo saco o procurado, e
assim sucessivamente.
(b) Vejamos que, em geral, uma tentativa de soluo como a anterior no permite a identicao dos sacos com moedas falsas. Suponhamos que Aladim tenha retirado uma moeda
do primeiro saco, duas moedas do segundo, e assim sucessivamente, at o ltimo saco,
de onde ele retirou dez moedas. Se existissem dois ou mais sacos com moedas falsas, o
procedimento de pesar essas 55 moedas pode ser inconclusivo. Por exemplo, digamos
que na pesagem das 55 moedas faltassem 7 g, ou seja, foram pesadas 7 moedas falsas.
Nesse caso, poderiam existir moedas falsas nos sacos 1 e 6, ou moedas falsas nos sacos
2 e 5, ou moedas falsas nos sacos 1, 2 e 4, etc. Ou seja, procedendo dessa maneira no
possvel identicar quais so os sacos de moedas falsas.
Para resolver esse problema, ele pode proceder de uma outra maneira, como segue.
Primeiro ele retira uma moeda do primeiro saco, depois duas moedas do segundo, da
quatro do terceiro, oito do quarto, dezesseis do quinto saco e assim, sucessivamente, at
o ltimo saco, sempre dobrando, a cada saco, o nmero de moedas retiradas do saco.
Dessa forma so retiradas, ao todo,
1 + 2 + 4 + 8 + 16 + 32 + 64 + 128 + 256 + 512 = 1 023
moedas que, juntas, pesariam 10 230 g, se todas as moedas retiradas fossem verdadeiras.
A diferena entre o peso real obtido na pesagem dessas 1.023 moedas e seu peso ideal de
10 230 g, indica a quantidade de moedas falsas pesadas e em quais dos sacos elas esto.
Vejamos isso atravs de um exemplo. Imaginemos que na pesagem tenham sido obtidos
10 125 g, ou seja, faltaram 10 230 10 125 = 105 g, que correspondem ao nmero
de moedas falsas. Subtraindo, sucessivamente, os nmeros correspondentes s moedas
retiradas de cada saco, comeando sempre do maior nmero menor do que 105, temos
105 64 = 41, 41 32 = 9, 9 8 = 1, ou seja, 105 = 1 + 8 + 32 + 64. Desse resultado,
Aladim pode concluir que foram retiradas 1, 8, 32 e 64 moedas falsas do primeiro, quarto,
sexto e stimo sacos.
Vamos, agora, justicar, de um modo mais formal, o raciocnio desenvolvido no exemplo
numrico. Seja p o peso obtido com a pesagem das 1 023 moedas retiradas. A diferena
10 230p o nmero de moedas falsas retiradas dos sacos. Efetuando divises sucessivas
por 2, pode-se provar que qualquer nmero inteiro positivo se escreve, de maneira
nica, como uma soma de potncias distintas de 2. De fato, isso que fornece a
justicativa terica para a expanso binria, ou seja, em base 2, dos nmeros naturais.
No nosso caso, isso implica que
10 230 p = 1 a0 + 2 a1 + 22 a2 + 23 a3 + + 29 a9 ,
em que cada um dos nmeros a0 , a1 , a2 , a3 , . . . , a9 0 ou 1.
De cada saco foram retiradas quantidades de moedas que so potncias de 2 e cada saco
ou contm somente moedas falsas ou contm somente moedas verdadeiras, isto , em
um mesmo saco no existem os dois tipos de moedas. Da, temos que se algum desses

342

OBMEP 2010

Solues do Nvel 3
nmeros, digamos aj , for 1, ento foram retiradas 2j moedas falsas do saco j + 1; por
outro lado, se o nmero aj for 0, ento foram retiradas 2j moedas verdadeiras do saco
j + 1.
214. Menor inteiro Como q = 2 005 p, queremos
5
p
7
<
< ,
8
2 005 p
8
de onde segue que 5(2 005 p) < 8p e 8p < 7(2 005 p). Logo,
5 2 005
7 2 005
<p<
,
13
15
de modo que 771,15 < p < 935,66. Assim, 772 o menor valor inteiro de p tal que p+q = 2 005
e 5/8 < p/q < 7/8.
215. Mais reas... Observe que a altura h relativa ao lado AB de todos os tringulos ABC
que tm o vrtice C na reta x + y = 7 sempre a mesma, pois a reta x + y = 7 paralela
reta x + y = 3 que passa por A e B. Logo, todos esses tringulos tm a mesma rea, a saber,
1
2

AB h).

Resta, portanto, determinar AB e h. Como AB a hipotenusa de um tringulo retngulo


que tem os dois catetos iguais a 3, segue do Teorema de Pitgoras que

AB = 33 + 32 = 18 = 3 2.
Por outro lado, h a distncia entre as retas paralelas x + y = 3 determinada pelos pontos
A e B e x + y = 7. A reta x = y perpendicular a essas retas paralelas e forma um ngulo
de 45 com o eixo Ox. Sejam M o p da perpendicular reta x + y = 7 traada a partir
de A e D = (7, 0) o ponto de corte da reta x + y = 7 com o eixo Ox. O tringulo AM D
assim formado retngulo issceles, com dois catetos iguais a h e hipotenusa 7 3 = 4. Pelo

Teorema de Pitgoras segue que 42 = h2 + h2 = 2h2 , ou seja, h = 8 = 2 2. Assim, a rea


procurada dada por

1
2 3 2 2 2 = 6.
216. Crculos tangentes Ligando os centros dos trs crculos obtemos o tringulo ABC de
lados AB = 3, AC = 4 e BC = 5. Como 32 + 42 = 52 , esse tringulo retngulo, com
hipotenusa BC.

B
2
1
A 1

B
2

2
3
3

1
A 1

D
2
3
3

Agora construmos o retngulo ADCB com uma cpia congruente ao tringulo ABC e
hipotenusa comum BC, conforme gura. Como DC = AB = 3 e o crculo de centro C
tambm tem raio 3, vemos que o ponto D est no crculo. Finalmente, ligamos o ponto D a
cada um dos vrtices do tringulo ABC e prolongamos esses segmentos at que cortem os
crculos centrados em A, B e C, obtendo os pontos P1 , P2 e P3 , conforme gura.
OBMEP 2010

343

Solues do Nvel 3

P2

P2

P1

P1

P3

P3

Observe que
DP2 = DB + BP2 = CA + BP2 = 4 + 2 = 6,.
DP1 = DA + AP1 = 5 + 1 = 6 e

DP3 = DC + CP3 = 3 + 3 = 6.
Assim, DP1 = DP2 = DP3 = 6 e, portanto, o crculo de centro D e raio 6 passa por P1 , P2
e P3 . Alm disso, como os pontos {D, A, P1 }, {D, B, P2 } e {D, C, P3 } esto alinhados, segue
que o crculo de centro D e raio 6 tangente exteriormente aos crculos dados de centros A, B
e C.
217. Soma nita Observe os possveis produtos x2k1 x2k , 1 2 004 inteiro, so
que
com
k

( 2 1)( 2 1) = 3 2 2, ( 2 + 1)( 2 + 1) = 3 + 2 2 e ( 2 1)( 2 1) = 1. Suponha


+

que a produtos sejam iguais a 3 2 2, b produtos sejam iguais a 3 + 2 2 e 1 002 a b


produtos sejam iguais a 1. A soma dada igual a

a(3 2 2) + b(3 + 2 2) + 1 002 a b = 1 002 + 2a + 2b + 2(b a) 2.


Assim, para que a soma seja inteira, devemos ter a = b. Logo, a soma igual a 1 002 + 4a.
Como a varia de 0 a 501, j que a + b = 2a no pode ser maior do que 1 002, a soma dada s
pode valer 502 valores inteiros distintos.
218. Mltiplos
Soluo 1: Observe que se um inteiro positivo m um mltiplo de um inteiro p, ento
2m p um mltiplo de p. De fato, se m = np, ento 2m p = 2np p = (2n 1)p. Em
particular, tomando m igual a a, a + 1, a + 2 e a + 3, estabelecemos que 2a 5 mltiplo de
5, 2(a + 1) 7 mltiplo de 7, 2(a + 2) 9 mltiplo de 9 e 2(a + 3) 11 mltiplo de 11.
No entanto, sucede que 2a 5 = 2(a + 1) 7 = 2(a + 2) 9 = 2(a + 3) 11, de modo que
2a 5 mltiplo de 5, 7, 9 e 11, donde mltiplo de 5 7 9 11 = 3 465. Assim, o menor
valor possvel de a satisfaz 2a 5 = 3 465, ou seja, a = 1 735.
Soluo 2: Como a mltiplo de 5, temos a = 5i, para algum i inteiro positivo. Mas
a + 1 = 1 + 5i deve ser mltiplo de 7. Por tentativas, vericamos que o menor valor de i para
o qual isso acontece i0 = 4. Para qualquer outro valor de i que torne 1 + 5i mltiplo de 7,
devemos ter (1+5i)(1+5i0 ) = 5(ii0 ) mltiplo de 7 e, portanto, ii0 mltiplo de 7. Logo,
i = i0 + 7j = 4 + 7j e a = 5i = 5(4 + 7j) = 20 + 35j, com j 0 inteiro. De maneira anloga,
a + 2 = 22 + 35j deve ser mltiplo de 9. O menor valor de j para o qual isso acontece j0 = 4,
pois 22 + 35 4 = 162 mltiplo de 9. Para qualquer outro valor de j que torne 22 + 35j
mltiplo de 9, devemos ter (22 + 35j) (22 + 35j0 ) = 35(j j0 ) mltiplo de 9 e, portanto,

344

OBMEP 2010

Solues do Nvel 3
j j0 mltiplo de 9. Logo, j = j0 + 9k = 4 + 9k e a = 20 + 35j = 20 + 35(4 + 9k) = 160 + 315k,
com k 0 inteiro. Finalmente, 11 deve dividir
a + 3 = 163 + 315k = 14 11 + 9 + (28 11 + 7)k
= (14 + 28k) 11 + (9 + 7k).

Temos, portanto, que 11 divide 9 + 7k. Novamente, por tentativas, simples vericar que o
menor menor valor de k para o qual isso acontece k0 = 5. Qualquer outro valor de k para o
qual isso acontea da forma k = 5 + 11l, com l 0 inteiro.

Conclumos que os inteiros positivos a para os quais 5 divide a, 7 divide a + 1, 9 divide a + 2


e 11 divide a + 3 so exatamente os inteiros da forma a = 160 + 315(5 + 11l) = 1 735 + 3 465l,
com l 0 inteiro. Em particular, o menor valor de a 1.735, obtido com l = 0.

219. Equao de duas variveis Observemos que


9xy x2 8y 2 = xy x2 + 8xy 8y 2 = x(y x) + 8y(x y)
= (x y)(8y x)

e que a fatorao de 2005 5 401. Como x e y so inteiros, temos somente quatro opes, a
saber, xy = 5 e 8y x = 401 ou xy = 401 e 8y x = 5. Se xy = 5 e 8y x = 401,
podemos somar essas equaes para obter 7y = 406, ou y = 58 e, portanto, x = 63. Da mesma
forma, se x y = 5 e 8y x = 401, obtemos 7y = 406 e, portanto, y = 58 e x = 63.
Analogamente, se x y = 401 e 8y x = 5, obtemos 7y = 406 e, portanto, y = 58 e
x = 459. As nicas solues so, portanto (63, 58), (63, 58), (459, 58) e (459, 58).
220. Trapzio retngulo Denotemos ABD = B DC = , como na gura dada. Ento temos
BD
CD =
e AD = BD sen , donde
cos
BD
1
CD
= cos =
AD
BD sen
sen cos
=

2
2.
sen 2

Assim, o menor valor da razo CD/AD 2, que ocorre quando sen 2 = 1, ou seja, quando
= 45 .
221. Jogos de futebol Para cada grupo de cinco alunos, existe um nico time formado que os
contm. Logo, contamos
12 11 10 9 8
5
C12 =
= 792
5!
times para cada cinco alunos escolhidos. Por outro lado, em cada time de seis jogadores,
5
temos C6 = 6 modos de escolhermos cinco jogadores, ou seja, existem seis grupos de cinco
jogadores que deram origem ao mesmo time na nossa primeira contagem. Assim, o total de
times formados 792/6 = 132.
222. A soma dos algarismos de um nmero
OBMEP 2010

345

Solues do Nvel 3
(a) imediato que se a um algarismos entre 1 e 9, ento s(a 10k ) = a, j que o nmero
a 10k formado pelo algarismo a seguido de k zeros. Da, temos
a 10k s(a 10k ) = a 10k a = a(10k 1) = a 9 9 = a 9 1 1 .
k noves

k uns

Como todo nmero pode ser decomposto em unidades, dezenas, centenas, etc, isto ,
todo nmero inteiro positivo n pode ser escrito, de maneira nica, na forma
n = a0 + a1 10 + a2 102 + + ak 10k ,
temos que
n s(n) = a1 9 + a2 99 + + ak 9 9 .
k noves

Assim, a diferena n s(n) sempre divisvel por 9.

(b) Seguindo o mesmo raciocnio, temos que ambos s(n) s(s(n)) e s(s(n)) s(s(s(n))) so
divisveis por 9, portanto, n s(s(s(n))) divisvel por 9.
Em particular
2 009 s(s(s(22 009 ))) divisvel por 9 ou, equivalentemente, 22 009 e s(s(s(22 009 )))
2
deixam o mesmo resto quando so divididos por 9.
Como 26 1 = 63 divisvel por 9, ento substituindo x = 26 e m = 334 na identidade
xm 1 = (x 1) xm1 + xm2 + + x2 + x + 1 ,
conclumos que (26 )334 1 = 22 004 1 divisvel por 9 e, portanto, 22 009 25 divisvel
por 9. Como 25 = 32 deixa resto 5 quando dividido por 9, temos que 22 009 deixa resto
5 quando dividido por 9. Por outro lado,
22 009 < (29 )224 < (103 )224 = 10672 .
Assim, 22 009 tem menos do que 672 algarismos e, portanto,
s(22 009 ) < 9 672 = 6.048,

s(s(22 009 )) 5 + 9 + 9 + 9 = 32e

s(s(s(22 009 ))) 2 + 9 = 13.

Como o nico nmero menor do que ou igual a 13 que deixa resto 5 ao ser dividido por
9 5, resulta que
s(s(s(22 009 ))) = 5.

346

OBMEP 2010

Solues dos Desaos

Solues dos Desaos


6 817

+221

51

13 259

1. Cadeia do menor nmero (N2/N3) 265 863 39 260 13 260 1.


2. Qual a metade? (N2/N3)

3. Cada um em seu estado (N1/N2/N3) Bruno ou Amlia (o desao tem duas


solues).
4. Diviso (N1/N2) Resposta: 153.
5. Extra-terrestre (N1/N2) Seise.
6. Que famlia! (N1/N2) 3 meninas e 4 meninos.
7. Siga a pista (N1)

C
8. Cara ou Coroa (N2) Resposta: 12.
9. Contas do papagaio (N1) Resposta: 19.
10. As frias de Toms (N1/N2) Resposta: 16 dias.
11. Maratona de Matemtica (N3) Resposta: 25 e 63, respectivamente.
12. Fraes ignoradas (N1)

OBMEP 2010

347

Solues dos Desaos


13. Caminho de maior total (N2) ???
14. Produtos em linha (N1/N2/N3) casa B.
15. Cdigo Postal (N2/N3) Resposta: 47 679 e 47 779.
16. Anis olmpicos (N1/N2/N3)

17. Partidas de Denise (N2/N3) A primeira, a segunda, a terceira, a quarta e a


oitava.
18. Sete quadrados (N1/N2)

19. Ilha misteriosa (N1/N2/N3) 16 cinzas, 18 marrons e 11 vermelhos.


20. Universo hostil (N3) Resposta: 1 873.
21. O jogo das chas
1
4
7

13
6
5

2
5
8

0
8
16

3
6
9

11
10
3

22. Um sistema Resposta: 23.


23. Constelaes oridas Pelo menos duas solues:

D = 25 169 = 4 225; R = 144 169 = 24 336


D = 49 289 = 14 161; R = 100 144 = 14 400.

348

OBMEP 2010

Solues dos Desaos


24. Dois meses iguais Setembro de 2006.
25. A faixa e o quadrado
................................................................................................................................
..
. ..
. .....
.
. .............................................................................................................................
... ....
.
.
.........
.
.
.
.
.
.
.
. .
.....
.
.
.
. .. .....
. .........
.
.
. ........
.
.
.
.
.....
.....
.
.
.
.
.....
.....
.
.
.
.
.
.
.
.
.....
.....
.
.
.
.
.....
.....
.
.
.
.
.
.
.
.
.....
.....
.
.
.
.
.....
.....
.
.
.
.
.....
.....
.
.
.
.
.....
.....
.
.
.
.
.
.
.
.
.....
.....
.
.
.
.
.....
.....
.
.
.
.
.
.
.
.
.....
.....
.
.
.
.
.....
.....
.
.
.
.
.
.
.
.
.....
.....
.
.
.
.
.....
..... .
.
.
.
.
..... .
.
.
.
..... .
..... .
.
.
..... .
.
.
. ..
... .
.
.
..............................................................................................................................
................................................................................................................................
.
...
.. .

26. Um nmero e seu sxtuplo Resposta: 746 a nica soluo.


27. Oito dentro de um retngulo Pelo menos duas solues:




.
.
.
.
.
.
.
.
.
.
.
.
.
.
.
.
.
.
.
.
.
.
.

.
.
.
.
.
.
.
.
.
.

...........................
..........................



............................
............................

.
.
....
....
.
.
..
 ............. ............. .

.
.
...
..
...
.
..
..
.
.
.
.
...
.
...
.
.
..
....
..
..
...
.
 ........... ..............
..
.
..
.
..
..
..
..
..
..
.
.
.
.







............................
...........................

...
.
.
...
 ........ ........
. .....
...
... .
...
...
... .
. ......
...
.
.
...
.
.
... 
...
... 
.
.
.
...
.
...
.
...
...
...
...
.
...
...
.
...
.
.
..
..
.
.
.
...
...
...
...
.
.
............................
............................



............................
...........................

.
.
.
.
.
.
.
.
.
.

...
.
.
...
 ........ ........
. .....
...
... .
...
...
... .
. ......
...
.
.
...
.
.
... 
...
... 
.
.
.
...
.
...
.
...
...
...
...
.
...
...
.
...
.
.
..
..
.
.
.
...
...
...
...
.
.

.
.
.
.
.
.
.
.
.
.
.
.
.
.
.
.
.
.
.
.
.
.
.

.
.
.
.
..
....
....
.
.
.
..
..
..
 .....
.
.
..
....
..
..
..
....
.
..
..
..
..
.
..
..
....
....
..
.
.
..
..
.
..
.
.. 
..
.
....
..
..
....
..
..
.
 ......... .

.
.
..
..
..
..
..
..
..
.
.





............................
...........................



............................
............................

...........................
..........................



............................
............................



............................
...........................

28. Uma estratgia com um nmero muito grande Resposta: 99 999 585 960.
29. Um nmero surpreendente Resposta: 381 654 729.
30. Qual o erro? Cludia e Bruno.
31. Soma Trs solues:
231468
231468
+
5972

264538
264538
+
9102

273548
273548
+
1602

.................................................
................................................

32. Bolinhas

.................................................
................................................

.................................................
................................................

468908

538178

548698

.
.
....
.....
.. ...
.. . ..
.. . ..
.. . ..
.. . ...
.. . ...
.
.. . ...
.
..
.
..
..
..
..
.
..
.
.
..
.
..
..
.
..
..
.
..
..
..
.
..
.
..
..
.
..
..
..
.
..
..
.
..
..
.
..
..
.
..
..
.
.
.
.
.
..
.........................................
.
.. .
.
.
...
..
.........................................
.
..
.
. .
.
....
.
. ...
.. .
.. .
.
.
.
.. .
.
. ..
.
. ....
.. .
.. .
.
. ....
.. .
.
.
.
.
..
.
.
.
..
.
.
.
..
..
..
.
.
.
..
..
.
.
.
.
..
.
.
.
..
..
.
.
..
.. . ...
.
.
.. . ...
.
.
.
.. . ...
.
.
.. . ..
.
.
.. . ..
.
.
.. . ..
.
.
.
.. ...
.
..
.
.
.
......................
.........................................
.
......................
.
..

t
t

t
t

t
t

33. Um nmero que no divisvel por 5 Resposta: 2 004.


34. Quatro fraes e um inteiro Resposta: 1.
35. O Rei Arthur e o Drago das Trs Cabeas e Trs Caudas Resposta: 5.
OBMEP 2010

349

Solues dos Desaos


36. O passeio do cavalo
A
N
I
T
C

X
S
B
O
J

M
H
W
D
U

R
Y
F
K
P

G
L
Q
V
E

37. As faces do cubo Resposta: 24.


38. Data fatdica Resposta: 17.06.2345
39. Todos com o 2 A opo correta (e).
40. Tortas da vov A opo correta (d).
Vamos examinar cada uma das situaes propostas. Lembre que, no nal, vov recebeu
7 + 6 + 3 2 = 14 docinhos.
(a) Impossvel, porque ela recebeu, no mnimo, 3 2 = 1 docinho de chocolate.

(b) Impossvel, porque ela recebeu, no mnimo, 6 2 = 4 docinhos de cco.


(c) Impossvel, porque 7 2 = 5 > 3.

(d) Possvel, porque Soa pode ter comido um docinho de amora e um de chocolate,
restando 6 de amora, 6 de cco e 2 de chocolate para a vov.
(e) Impossvel, porque 7 no maior do que 6 + 3 2 = 7.
41. Famlia Stimo Os nascimentos ocorreram em seis dias 1o de abril, logo existem
irmos gmeos. Como nesse ano temos dois bolos a mais do que h dois anos, ento
h dois anos o mais jovem ainda no tinha nascido, o penltimo lho tinha acabado
de nascer e os gmeos j tinham nascido. Atualmente, o mais jovem tem um ano e os
gmeos tm x anos, com x 3. Como
1+2+3+4+5+6+x=2
nmero de velas nesse ano

(1 + 2 + 3 + 4 + x 2) ,

nmero de velas 2 anos atrs

temos x = 5. Logo, sero acesas 1 + 2 + 3 + 4 + 2 5 + 6 = 26 velinhas.


42. O salta-cha
(a) A cha 7 salta sobre as chas 8 e 9 formando uma pilha com a cha 10;
(b) a cha 4 salta sobre as chas 5 e 6 formando uma pilha com a cha 8;
(c) a cha 6 salta sobre as chas 3 e 5 formando uma pilha com a cha 2;
(d) a cha 5 salta sobre a pilha (4, 8) formando uma pilha com a cha 9 e
(e) a cha 1 salta sobre a pilha (6, 2) formando uma pilha com a cha 3.
O resultado segue.
350

OBMEP 2010

Solues dos Desaos


.....
......
......
......
......
......... .......... ......... .......... ..........
... ... ... ... ... ... ... ... ... ...
.
. .
. .
. .
..
..
.
..
..
..
. .
.
.
.
.
.
..
.
.
.
.
.
..
.. ... ... ... ... ... ... ... ... ...
....... .. ........... ........... ........... ...........
.......
......
......
......
......
.....
.....
.....
.....
.....
.......... ........... ........... ........... ...........
... .... ... .... ... .... ... .... ... ....
.
.
..
..
..
..
.
.
..
..
..
..
.
.
..
..
..
..
.
..
.. ... ... ... ... ... ... ... ... ...
......... ........... ........... ........... ...........
.......
......
......
......
......

9 10

43. O menor Como 52 = 32 + 42 , temos 52 002 = (32 + 42 )1 001 . Sabemos que, para a > 0
e b > 0,
(a + b)1 001 > a1 001 + b1 001 .
Assim, 52 002 > 32 002 + 42 002 .
44. O maior resultado Estamos procurando o maior valor de (10a + b)/(a + b), onde
a e b representam algarismos, sendo pelo menos um deles diferente de 0. Temos
10a + b
10a + 10b 9b
10a + 10b
9b
9b
=
=

= 10
10 .
a+b
a+b
a+b
a+b
a+b
Logo, se conseguirmos encontrar a e b tais que (10a + b)/(a + b) = 10, teremos o maior
resultado. Note que isso ocorre quando b = 0, ou seja,
20
30
40
50
60
70
80
90
10
=
=
=
=
=
=
=
=
= 10 .
1
2
3
4
5
6
7
8
9
Assim, a resposta 10.
45. Dois mil Observe que os nmeros 189, 8 307 e 99 tm todos peso 18 e que 99
o menor nmero que pesa 18. Para aumentar o peso de um nmero e minimizar o
nmero preciso que o nmero tenha o maior nmero possvel de algarismos 9. Por
outro lado, podemos dizer que o 0 est eliminado dos algarismos a ser considerados,
porque ele aumenta o nmero sem aumentar seu peso.
Temos que 2 000 = 9222+2. Logo, o nmero procurado deve ter 222 algarismos 9 e um
algarismo 2, ou dois algarismos 1. Eliminamos o caso dos nmeros com dois algarismos
1 porque eles tm 224 algarismos, sendo, portanto, maiores do que os nmeros que
possuem o algarismo 2 e tm 223 algarismos. Assim, o nmero procurado tem um 2
seguido de 222 algarismos 9: o nmero 299 . . . 999.
46. No cabeleireiro Seja x o montante inicial no caixa. Esse montante mais o que os
trs clientes pagaram nos dar o caixa zerado.
O primeiro cliente paga x 10 e, depois dele, h x + x 10 = 2x 10 reais no
caixa.
O segundo cliente paga (2x 10) 10 = 2x 20 e, depois dele, h
(2x 10) + (2x 20) = 4x 30 no caixa.

O terceiro cliente paga (4x 30) 10 = 4x 40 e depois dele h


(4x 30) + (4x 40) = 8x 70 no caixa.
Como o caixa est zerado depois do terceiro cliente, 8x 70 = 0, ou seja,
x = 70/8 = 8,75 reais.
OBMEP 2010

351

Solues dos Desaos


47. O macaco e a raposa 2 450 o produto dos nmeros primos 1, 2, 5, 5, 7 e 7.
As trs idades correspondem a uma combinao particular desses nmeros ou de seus
produtos. A raposa no pode descobrir as idades no incio porque pelo menos duas
dessas combinaes tm por soma o dobro de sua idade. De todas as combinaes
possveis, somente 5 + 10 + 49 e 7 + 7 + 50 tm a mesma soma 64.
1a concluso: a raposa tem 32 anos.
Depois da nova dica do macaco, a raposa descobriu as idades porque pode eliminar
uma combinao, a que contm dois nmeros iguais, uma vez que um deles o mais
jovem de todos.
2a concluso: as pessoas tm 5, 10 e 49 anos.
48. Nova sequncia Cada termo da sequncia a soma do termo precedente com os
quadrados de cada um de seus algarismos. De fato,
470 = 425 + 42 + 22 + 52 , 535 = 470 + 42 + 72 + 02 , . . .
Assim, depois de 802, os prximos termos sero 870 e 983.
49. Retngulo quase quadrado A rea um nmero da forma a a b b, onde a e b
representam algarismos, portanto
a a b b = 1100a + 11b = 11(100a + b).
Seja x a largura do terreno. Ento x(x + 1) = 11(100a + b) e deduzimos que x ou x + 1
um mltiplo de 11. Procurar mltiplos de 11 que satisfaam a condio obtida
bastante trabalhoso, por isso, para simplicar, vamos estabelecer quais os valores que
x pode ter, procurando seus valores mnimo e mximo.
Mnimo: a menor rea possvel 1 111, logo x(x + 1) = 1 111 e x > 32.

Mximo: a maior rea possvel 9 999, logo x(x + 1) = 9 999 e x < 100.

Agora testamos todos x entre 32 e 100 tais que x ou x + 1 seja mltiplo de 11 e que
x(x + 1) seja do tipo a a b b. Temos apenas doze opes, como segue.
33 34 = 1 122, 43 44 = 1 892, 44 45 = 1 980, 54 55 = 2 970,
55 56 = 2 970, 65 66 = 4 290, 66 67 = 4 422, 76 77 = 5 852,
77 78 = 6 006, 87 88 = 7 656, 88 89 = 7 832, 99 100 = 9 900.

Encontramos trs possibilidades para as dimenses do terreno, a saber, 33 34, 66 67


ou 99 100 metros.
50. Onde est o erro? Esse deixamos para os alunos!

352

OBMEP 2010

Você também pode gostar